Вы находитесь на странице: 1из 590

TELETRAFFIC ENGINEERING and NETWORK PLANNING

Villy B. Iversen

COM Course 34340


http://www.com.dtu.dk/education/34340/

Technical University of Denmark Building 345v DK2800 Kgs. Lyngby vbi@com.dtu.dk Revised May 11, 2006

ii

Villy Bk Iversen, 2006

iii

PREFACE

This book covers the basic theory of teletrac engineering. The mathematical background required is elementary probability theory. The purpose of the book is to enable engineers to understand ITUT recommendations on trac engineering, evaluate tools and methods, and keep up-to-date with new practices. The book includes the following parts: Introduction: Chapter 1 2, Mathematical background: Chapter 3 6, Telecommunication loss models: Chapter 7 11, Data communication delay models: Chapter 12 14, Measurements: Chapter 15. The purpose of the book is twofold: to serve both as a handbook and as a textbook. Thus the reader should, for example, be able to study chapters on loss models without studying the chapters on the mathematical background rst. The book is based on many years of experience in teaching the subject at the Technical University of Denmark and from ITU training courses in developing countries. Supporting material, such as software, exercises, advanced material, and case studies, is available at: http://www.com.dtu.dk/teletraffic where comments and ideas will also be appreciated.

Villy Bk Iversen April 1, 2006

iv

Notations
a A Ac A B B c C Cn d D E E1,n (A) = E1 E2,n (A) = E2 F g h H(k) I J (z) k K L Lk L m mi mi mr M n N p(i) p{i, t | j, t0 } Oered trac per source Oered trac = Ao Carried trac = Y Lost trac Call congestion Burstiness Constant Trac congestion = load congestion Catalans number Slot size in multi-rate trac Probability of delay or Deterministic arrival or service process Time congestion Erlangs Bformula = Erlangs 1. formula Erlangs Cformula = Erlangs 2. formula Improvement function Number of groups Constant time interval or service time PalmJacobus formula Inverse time congestion I = 1/E Modied Bessel function of order Accessibility = hunting capacity Maximum number of customers in a queueing system Number of links in a telecommuncation network or number of nodes in a queueing network Mean queue length Mean queue length when the queue is greater than zero Random variable for queue length Mean value (average) = m1 ith (non-central) moment ith centrale moment Mean residual life time Poisson arrival process Number of servers (channels) Number of trac streams or trac types State probabilities, time averages Probability for state i at time t given state j at time t0

vi

P (i) q(i) Q(i) Q r R s S t T U v V w W W x X y Y Z i (i) (i) 2

Cumulated state probabilities P (i) = i x= p(x) Relative (non normalised) state probabilities Cumulated values of q(i): Q(i) = i x= q(x) Normalisation constant Reservation parameter (trunk reservation) Mean response time Mean service time Number of trac sources Time instant Random variable for time instant Load function Variance Virtual waiting time Mean waiting time for delayed customers Mean waiting time for all customers Random variable for waiting time Variable Random variable Carried trac per souce Carried trac Peakedness Carried trac per channel Oered trac per idle source Arrival rate for an idle source Palms form factor Lagrange-multiplicator ith cumulant Arrival rate of a Poisson process Total arrival rate to a system Service rate, inverse mean service time State probabilities, arriving customer mean values State probabilities, departing customer mean values Service ratio Variance, = standard deviation Time-out constant or constant time-interval

Contents
1 Introduction to Teletrac Engineering 1.1 Modelling of telecommunication systems . . . . . . . . . . . . . . . . . . . . . 1.1.1 1.1.2 1.1.3 1.1.4 1.2 1.2.1 1.2.2 1.2.3 1.3 1.3.1 1.3.2 1.3.3 1.4 1.5 1.4.1 1.5.1 1.5.2 1.5.3 1.5.4 1.5.5 1.5.6 1.5.7 1.5.8 System structure . . . . . . . . . . . . . . . . . . . . . . . . . . . . . . The operational strategy . . . . . . . . . . . . . . . . . . . . . . . . . Statistical properties of trac . . . . . . . . . . . . . . . . . . . . . . . Models . . . . . . . . . . . . . . . . . . . . . . . . . . . . . . . . . . . System structure . . . . . . . . . . . . . . . . . . . . . . . . . . . . . . User behaviour . . . . . . . . . . . . . . . . . . . . . . . . . . . . . . . Operation strategy . . . . . . . . . . . . . . . . . . . . . . . . . . . . . The telephone network . . . . . . . . . . . . . . . . . . . . . . . . . . . Local Area Networks (LAN) 1 2 3 3 3 4 5 5 7 8 9 9

Conventional telephone systems . . . . . . . . . . . . . . . . . . . . . . . . . .

Communication networks . . . . . . . . . . . . . . . . . . . . . . . . . . . . .

Data networks . . . . . . . . . . . . . . . . . . . . . . . . . . . . . . . 11 . . . . . . . . . . . . . . . . . . . . . . . 12

Mobile communication systems . . . . . . . . . . . . . . . . . . . . . . . . . . 13 Cellular systems . . . . . . . . . . . . . . . . . . . . . . . . . . . . . . 13 Trac engineering in the ITU . . . . . . . . . . . . . . . . . . . . . . . 16 Trac demand characterisation . . . . . . . . . . . . . . . . . . . . . . 17 Grade of Service objectives . . . . . . . . . . . . . . . . . . . . . . . . 23 Trac controls and dimensioning . . . . . . . . . . . . . . . . . . . . . 28 Performance monitoring . . . . . . . . . . . . . . . . . . . . . . . . . . 35 Other recommendations . . . . . . . . . . . . . . . . . . . . . . . . . . 36 Work program for the Study Period 20012004 . . . . . . . . . . . . . 37 Conclusions . . . . . . . . . . . . . . . . . . . . . . . . . . . . . . . . . 38 39 ITU recommendations on trac engineering . . . . . . . . . . . . . . . . . . . 16

2 Trac concepts and grade of service 2.1

Concept of trac and trac unit [erlang] . . . . . . . . . . . . . . . . . . . . 39 vii

viii 2.2 2.3 2.4 2.5

CONTENTS Trac variations and the concept busy hour . . . . . . . . . . . . . . . . . . . 42 The blocking concept . . . . . . . . . . . . . . . . . . . . . . . . . . . . . . . 45 Trac generation and subscribers reaction . . . . . . . . . . . . . . . . . . . . 48 Introduction to Grade-of-Service = GoS . . . . . . . . . . . . . . . . . . . . . 55 2.5.1 2.5.2 2.5.3 2.5.4 Comparison of GoS and QoS . . . . . . . . . . . . . . . . . . . . . . . 56 Special features of QoS . . . . . . . . . . . . . . . . . . . . . . . . . . 57 Network performance . . . . . . . . . . . . . . . . . . . . . . . . . . . 57 Reference congurations . . . . . . . . . . . . . . . . . . . . . . . . . . 58 61

3 Probability Theory and Statistics 3.1 3.1.1 3.1.2 3.1.3 3.1.4 3.1.5 3.2 3.2.1 3.2.2 3.3

Distribution functions . . . . . . . . . . . . . . . . . . . . . . . . . . . . . . . 61 Characterisation of distributions . . . . . . . . . . . . . . . . . . . . . 62 Residual lifetime . . . . . . . . . . . . . . . . . . . . . . . . . . . . . . 64 Load from holding times of duration less than x . . . . . . . . . . . . . 67 Forward recurrence time . . . . . . . . . . . . . . . . . . . . . . . . . . 68 Distribution of the jth largest of k random variables . . . . . . . . . . 69 Random variables in series . . . . . . . . . . . . . . . . . . . . . . . . 70 Random variables in parallel . . . . . . . . . . . . . . . . . . . . . . . 71

Combination of random variables . . . . . . . . . . . . . . . . . . . . . . . . . 70

Stochastic sum . . . . . . . . . . . . . . . . . . . . . . . . . . . . . . . . . . . 72 77

4 Time Interval Distributions 4.1 4.1.1 4.1.2 4.2 4.3 4.4

Exponential distribution . . . . . . . . . . . . . . . . . . . . . . . . . . . . . . 77 Minimum of k exponentially distributed random variables . . . . . . . 79 Combination of exponential distributions . . . . . . . . . . . . . . . . 80

Steep distributions . . . . . . . . . . . . . . . . . . . . . . . . . . . . . . . . . 80 Flat distributions . . . . . . . . . . . . . . . . . . . . . . . . . . . . . . . . . . 82 4.3.1 4.4.1 4.4.2 4.4.3 Hyper-exponential distribution . . . . . . . . . . . . . . . . . . . . . . 83 Polynomial trial . . . . . . . . . . . . . . . . . . . . . . . . . . . . . . 88 Decomposition principles . . . . . . . . . . . . . . . . . . . . . . . . . 88 Importance of Cox distribution . . . . . . . . . . . . . . . . . . . . . . 90 Cox distributions . . . . . . . . . . . . . . . . . . . . . . . . . . . . . . . . . . 84

4.5 4.6

Other time distributions . . . . . . . . . . . . . . . . . . . . . . . . . . . . . . 91 Observations of life-time distribution . . . . . . . . . . . . . . . . . . . . . . . 92 95

5 Arrival Processes 5.1

Description of point processes . . . . . . . . . . . . . . . . . . . . . . . . . . . 95

CONTENTS 5.1.1 5.1.2 5.2 5.2.1 5.2.2 5.2.3 5.3

ix Basic properties of number representation . . . . . . . . . . . . . . . . 97 Basic properties of interval representation . . . . . . . . . . . . . . . . 98 Stationarity (Time homogeneity) . . . . . . . . . . . . . . . . . . . . . 100 Independence . . . . . . . . . . . . . . . . . . . . . . . . . . . . . . . . 100 Simple point process . . . . . . . . . . . . . . . . . . . . . . . . . . . . 101 . . . . . . . . . . . . . . . . . . . . . . . . . . . . . . . . . . 101 105

Characteristics of point process . . . . . . . . . . . . . . . . . . . . . . . . . . 99

Littles theorem

6 The Poisson process 6.1 6.2

Characteristics of the Poisson process . . . . . . . . . . . . . . . . . . . . . . 105 Distributions of the Poisson process . . . . . . . . . . . . . . . . . . . . . . . 106 6.2.1 6.2.2 6.2.3 6.2.4 Exponential distribution . . . . . . . . . . . . . . . . . . . . . . . . . . 107 Erlangk distribution . . . . . . . . . . . . . . . . . . . . . . . . . . . 109 Poisson distribution . . . . . . . . . . . . . . . . . . . . . . . . . . . . 110 Static derivation of the distributions of the Poisson process . . . . . . 113 Palms theorem . . . . . . . . . . . . . . . . . . . . . . . . . . . . . . . 114 Raikovs theorem (Decomposition theorem) . . . . . . . . . . . . . . . 117 Uniform distribution a conditional property . . . . . . . . . . . . . . 117 Interrupted Poisson process (IPP) . . . . . . . . . . . . . . . . . . . . 119 121

6.3

Properties of the Poisson process . . . . . . . . . . . . . . . . . . . . . . . . . 114 6.3.1 6.3.2 6.3.3

6.4

Generalisation of the stationary Poisson process . . . . . . . . . . . . . . . . . 119 6.4.1

7 Erlangs loss system and Bformula 7.1 7.2

Introduction . . . . . . . . . . . . . . . . . . . . . . . . . . . . . . . . . . . . 121 Poisson distribution . . . . . . . . . . . . . . . . . . . . . . . . . . . . . . . . 122 7.2.1 7.2.2 7.2.3 State transition diagram . . . . . . . . . . . . . . . . . . . . . . . . . . 122 Derivation of state probabilities . . . . . . . . . . . . . . . . . . . . . . 124 Trac characteristics of the Poisson distribution . . . . . . . . . . . . 126 State probabilities . . . . . . . . . . . . . . . . . . . . . . . . . . . . . 127 Trac characteristics of Erlangs B-formula . . . . . . . . . . . . . . . 128 Generalisations of Erlangs B-formula . . . . . . . . . . . . . . . . . . 130 Recursion formula . . . . . . . . . . . . . . . . . . . . . . . . . . . . . 135

7.3

Truncated Poisson distribution . . . . . . . . . . . . . . . . . . . . . . . . . . 127 7.3.1 7.3.2 7.3.3

7.4 7.5 7.6

General procedure for state transition diagrams . . . . . . . . . . . . . . . . . 134 7.4.1 Evaluation of Erlangs B-formula . . . . . . . . . . . . . . . . . . . . . . . . . 136 Principles of dimensioning . . . . . . . . . . . . . . . . . . . . . . . . . . . . . 138

x 7.6.1 7.6.2

CONTENTS Dimensioning with xed blocking probability . . . . . . . . . . . . . . 139 Improvement principle (Moes principle) . . . . . . . . . . . . . . . . . 140 145

8 Loss systems with full accessibility 8.1 8.2

Introduction . . . . . . . . . . . . . . . . . . . . . . . . . . . . . . . . . . . . 146 Binomial Distribution . . . . . . . . . . . . . . . . . . . . . . . . . . . . . . . 147 8.2.1 8.2.2 Equilibrium equations . . . . . . . . . . . . . . . . . . . . . . . . . . . 148 Trac characteristics of Binomial trac . . . . . . . . . . . . . . . . . 150 State probabilities . . . . . . . . . . . . . . . . . . . . . . . . . . . . . 153 Trac characteristics of Engset trac . . . . . . . . . . . . . . . . . . 153

8.3

Engset distribution . . . . . . . . . . . . . . . . . . . . . . . . . . . . . . . . . 152 8.3.1 8.3.2

8.4 8.5

Relations between E, B, and C . . . . . . . . . . . . . . . . . . . . . . . . . . 157 Evaluation of Engsets formula . . . . . . . . . . . . . . . . . . . . . . . . . . 158 8.5.1 8.5.2 8.5.3 Recursion formula on n . . . . . . . . . . . . . . . . . . . . . . . . . . 159 Recursion formula on S . . . . . . . . . . . . . . . . . . . . . . . . . . 159 Recursion formula on both n and S . . . . . . . . . . . . . . . . . . . . 160

8.6 8.7

Pascal Distribution (Negative Binomial) . . . . . . . . . . . . . . . . . . . . . 162 Truncated Pascal distribution . . . . . . . . . . . . . . . . . . . . . . . . . . . 163 167

9 Overow theory 9.1 9.2 9.1.1 9.2.1 9.2.2 9.2.3 9.3 9.4 9.3.1 9.4.1 9.4.2 9.4.3 9.5 9.5.1 9.5.2

Overow theory . . . . . . . . . . . . . . . . . . . . . . . . . . . . . . . . . . 168 State probability of overow systems . . . . . . . . . . . . . . . . . . . 169 Preliminary analysis . . . . . . . . . . . . . . . . . . . . . . . . . . . . 172 Numerical aspects . . . . . . . . . . . . . . . . . . . . . . . . . . . . . 173 Parcel blocking probabilities . . . . . . . . . . . . . . . . . . . . . . . . 174 . . . . . . . . . . . . . . . . . . . . . . . . . 176 Trac splitting . . . . . . . . . . . . . . . . . . . . . . . . . . . . . . . 177 BPP trac models . . . . . . . . . . . . . . . . . . . . . . . . . . . . . 179 Sanders method . . . . . . . . . . . . . . . . . . . . . . . . . . . . . . 180 Berkeleys method . . . . . . . . . . . . . . . . . . . . . . . . . . . . . 180 Interrupted Poisson Process . . . . . . . . . . . . . . . . . . . . . . . . 181 Cox2 arrival process . . . . . . . . . . . . . . . . . . . . . . . . . . . 182 185 Equivalent Random Trac method . . . . . . . . . . . . . . . . . . . . . . . . 171

Fredericks & Haywards method

Other methods based on state space . . . . . . . . . . . . . . . . . . . . . . . 179

Methods based on arrival processes . . . . . . . . . . . . . . . . . . . . . . . . 181

10 Multi-Dimensional Loss Systems

CONTENTS

xi

10.1 Multi-dimensional Erlang-B formula . . . . . . . . . . . . . . . . . . . . . . . 185 10.2 Reversible Markov processes . . . . . . . . . . . . . . . . . . . . . . . . . . . . 189 10.3 Multi-Dimensional Loss Systems . . . . . . . . . . . . . . . . . . . . . . . . . 191 10.3.1 Class limitation . . . . . . . . . . . . . . . . . . . . . . . . . . . . . . 191 10.3.2 Generalised trac processes . . . . . . . . . . . . . . . . . . . . . . . . 191 10.3.3 Multi-slot trac . . . . . . . . . . . . . . . . . . . . . . . . . . . . . . 192 10.4 Convolution Algorithm for loss systems . . . . . . . . . . . . . . . . . . . . . 196 10.4.1 The algorithm . . . . . . . . . . . . . . . . . . . . . . . . . . . . . . . 197 10.5 State space based algorithms . . . . . . . . . . . . . . . . . . . . . . . . . . . 203 10.5.1 Fortet & Grandjean (Kaufman & Robert) algorithm . . . . . . . . . . 206 10.5.2 Generalized algorithm . . . . . . . . . . . . . . . . . . . . . . . . . . . 206 10.6 Final remarks . . . . . . . . . . . . . . . . . . . . . . . . . . . . . . . . . . . . 210 11 Dimensioning of telecom networks 211

11.1 Trac matrices . . . . . . . . . . . . . . . . . . . . . . . . . . . . . . . . . . . 211 11.1.1 Kruithofs double factor method . . . . . . . . . . . . . . . . . . . . . 212 11.2 Topologies . . . . . . . . . . . . . . . . . . . . . . . . . . . . . . . . . . . . . 215 11.3 Routing principles . . . . . . . . . . . . . . . . . . . . . . . . . . . . . . . . . 215 11.4 Approximate end-to-end calculations methods . . . . . . . . . . . . . . . . . . 215 11.4.1 Fix-point method . . . . . . . . . . . . . . . . . . . . . . . . . . . . . 215 11.5 Exact end-to-end calculation methods . . . . . . . . . . . . . . . . . . . . . . 216 11.5.1 Convolution algorithm . . . . . . . . . . . . . . . . . . . . . . . . . . . 216 11.6 Load control and service protection . . . . . . . . . . . . . . . . . . . . . . . . 216 11.6.1 Trunk reservation . . . . . . . . . . . . . . . . . . . . . . . . . . . . . 217 11.6.2 Virtual channel protection . . . . . . . . . . . . . . . . . . . . . . . . . 218 11.7 Moes principle . . . . . . . . . . . . . . . . . . . . . . . . . . . . . . . . . . . 218 11.7.1 Balancing marginal costs . . . . . . . . . . . . . . . . . . . . . . . . . 219 11.7.2 Optimum carried trac . . . . . . . . . . . . . . . . . . . . . . . . . . 220 12 Delay Systems 223

12.1 Erlangs delay system M/M/n . . . . . . . . . . . . . . . . . . . . . . . . . . . 223 12.2 Trac characteristics of delay systems . . . . . . . . . . . . . . . . . . . . . . 225 12.2.1 Erlangs C-formula . . . . . . . . . . . . . . . . . . . . . . . . . . . . . 225 12.2.2 Numerical evaluation . . . . . . . . . . . . . . . . . . . . . . . . . . . 226 12.2.3 Mean queue lengths . . . . . . . . . . . . . . . . . . . . . . . . . . . . 227 12.2.4 Mean waiting times . . . . . . . . . . . . . . . . . . . . . . . . . . . . 229

xii

CONTENTS 12.2.5 Improvement functions for M/M/n . . . . . . . . . . . . . . . . . . . . 231 12.3 Moes principle for delay systems . . . . . . . . . . . . . . . . . . . . . . . . . 231 12.4 Waiting time distribution for M/M/n, FCFS . . . . . . . . . . . . . . . . . . 233 12.4.1 Sojourn time for a single server . . . . . . . . . . . . . . . . . . . . . . 235 12.5 Palms machine repair model . . . . . . . . . . . . . . . . . . . . . . . . . . . 236 12.5.1 Terminal systems . . . . . . . . . . . . . . . . . . . . . . . . . . . . . . 238 12.5.2 State probabilities single server . . . . . . . . . . . . . . . . . . . . . 239 12.5.3 Terminal states and trac characteristics . . . . . . . . . . . . . . . . 241 12.5.4 Machinerepair model with n servers . . . . . . . . . . . . . . . . . . . 244 12.6 Optimising the machine-repair model . . . . . . . . . . . . . . . . . . . . . . . 246

13 Applied Queueing Theory

249

13.1 Classication of queueing models . . . . . . . . . . . . . . . . . . . . . . . . . 249 13.1.1 Description of trac and structure . . . . . . . . . . . . . . . . . . . . 249 13.1.2 Queueing strategy: disciplines and organisation . . . . . . . . . . . . . 250 13.1.3 Priority of customers . . . . . . . . . . . . . . . . . . . . . . . . . . . . 252 13.2 General results in the queueing theory . . . . . . . . . . . . . . . . . . . . . . 253 13.3 Pollaczek-Khintchines formula for M/G/1 . . . . . . . . . . . . . . . . . . . . 254 13.3.1 Derivation of Pollaczek-Khintchines formula . . . . . . . . . . . . . . 254 13.3.2 Busy period for M/G/1 . . . . . . . . . . . . . . . . . . . . . . . . . . 255 13.3.3 Waiting time for M/G/1 . . . . . . . . . . . . . . . . . . . . . . . . . . 256 13.3.4 Limited queue length: M/G/1/k . . . . . . . . . . . . . . . . . . . . . 257 13.4 Priority queueing systems: M/G/1 . . . . . . . . . . . . . . . . . . . . . . . . 257 13.4.1 Combination of several classes of customers . . . . . . . . . . . . . . . 258 13.4.2 Work conserving queueing disciplines . . . . . . . . . . . . . . . . . . . 259 13.4.3 Non-preemptive queueing discipline . . . . . . . . . . . . . . . . . . . . 261 13.4.4 SJF-queueing discipline: M/G/1 . . . . . . . . . . . . . . . . . . . . . 263 13.4.5 M/M/n with non-preemptive priority . . . . . . . . . . . . . . . . . . 266 13.4.6 Preemptive-resume queueing discipline . . . . . . . . . . . . . . . . . . 266 13.4.7 M/M/n with preemptive-resume priority . . . . . . . . . . . . . . . . . 268 13.5 Queueing systems with constant holding times . . . . . . . . . . . . . . . . . 268 13.5.1 Historical remarks on M/D/n . . . . . . . . . . . . . . . . . . . . . . . 268 13.5.2 State probabilities of M/D/1 . . . . . . . . . . . . . . . . . . . . . . . 269 13.5.3 Mean waiting times and busy period of M/D/1 . . . . . . . . . . . . . 270 13.5.4 Waiting time distribution: M/D/1, FCFS . . . . . . . . . . . . . . . . 271 13.5.5 State probabilities: M/D/n . . . . . . . . . . . . . . . . . . . . . . . . 273

CONTENTS

xiii

13.5.6 Waiting time distribution: M/D/n, FCFS . . . . . . . . . . . . . . . . 273 13.5.7 Erlang-k arrival process: Ek /D/r . . . . . . . . . . . . . . . . . . . . . 274 13.5.8 Finite queue system: M/D/1/k . . . . . . . . . . . . . . . . . . . . . . 275 13.6 Single server queueing system: GI/G/1 . . . . . . . . . . . . . . . . . . . . . . 276 13.6.1 General results . . . . . . . . . . . . . . . . . . . . . . . . . . . . . . . 277 13.6.2 State probabilities: GI/M/1 . . . . . . . . . . . . . . . . . . . . . . . . 278 13.6.3 Characteristics of GI/M/1 . . . . . . . . . . . . . . . . . . . . . . . . . 279 13.6.4 Waiting time distribution: GI/M/1, FCFS . . . . . . . . . . . . . . . . 280 13.7 Round Robin and Processor-Sharing . . . . . . . . . . . . . . . . . . . . . . . 281 14 Networks of queues 283

14.1 Introduction to queueing networks . . . . . . . . . . . . . . . . . . . . . . . . 283 14.2 Symmetric queueing systems . . . . . . . . . . . . . . . . . . . . . . . . . . . 284 14.3 Jacksons theorem . . . . . . . . . . . . . . . . . . . . . . . . . . . . . . . . . 286 14.3.1 Kleinrocks independence assumption . . . . . . . . . . . . . . . . . . . 289 14.4 Single chain queueing networks . . . . . . . . . . . . . . . . . . . . . . . . . . 289 14.4.1 Convolution algorithm for a closed queueing network . . . . . . . . . . 290 14.4.2 The MVAalgorithm . . . . . . . . . . . . . . . . . . . . . . . . . . . . 294 14.5 BCMP queueing networks . . . . . . . . . . . . . . . . . . . . . . . . . . . . . 297 14.6 Multidimensional queueing networks . . . . . . . . . . . . . . . . . . . . . . . 298 14.6.1 M/M/1 single server queueing system . . . . . . . . . . . . . . . . . . 298 14.6.2 M/M/n queueing system . . . . . . . . . . . . . . . . . . . . . . . . . 301 14.7 Closed queueing networks with multiple chains . . . . . . . . . . . . . . . . . 301 14.7.1 Convolution algorithm . . . . . . . . . . . . . . . . . . . . . . . . . . . 301 14.8 Other algorithms for queueing networks . . . . . . . . . . . . . . . . . . . . . 304 14.9 Complexity . . . . . . . . . . . . . . . . . . . . . . . . . . . . . . . . . . . . . 305 14.10 Optimal capacity allocation . . . . . . . . . . . . . . . . . . . . . . . . . . . . 305 15 Trac measurements 309

15.1 Measuring principles and methods . . . . . . . . . . . . . . . . . . . . . . . . 310 15.1.1 Continuous measurements . . . . . . . . . . . . . . . . . . . . . . . . . 310 15.1.2 Discrete measurements . . . . . . . . . . . . . . . . . . . . . . . . . . . 311 15.2 Theory of sampling . . . . . . . . . . . . . . . . . . . . . . . . . . . . . . . . . 312 15.3 Continuous measurements in an unlimited period . . . . . . . . . . . . . . . . 314 15.4 Scanning method in an unlimited time period . . . . . . . . . . . . . . . . . . 317 15.5 Numerical example . . . . . . . . . . . . . . . . . . . . . . . . . . . . . . . . . 320

0 Bibliography Author index Subject index

CONTENTS 325 333 335

Chapter 1 Introduction to Teletrac Engineering


Teletrac theory is dened as the application of probability theory to the solution of problems concerning planning, performance evaluation, operation, and maintenance of telecommunication systems. More generally, teletrac theory can be viewed as a discipline of planning where the tools (stochastic processes, queueing theory and numerical simulation) are taken from the disciplines of operations research. The term teletrac covers all kinds of data communication trac and telecommunication trac. The theory will primarily be illustrated by examples from telephone and data communication systems. The tools developed are, however, independent of the technology and applicable within other areas such as road trac, air trac, manufacturing and assembly belts, distribution, workshop and storage management, and all kinds of service systems. The objective of teletrac theory can be formulated as follows: to make the trac measurable in well dened units through mathematical models and to derive the relationship between grade-of-service and system capacity in such a way that the theory becomes a tool by which investments can be planned. The task of teletrac theory is to design systems as cost eectively as possible with a predened grade of service when we know the future trac demand and the capacity of system elements. Furthermore, it is the task of teletrac engineering to specify methods for controlling that the actual grade of service is fullling the requirements, and also to specify emergency actions when systems are overloaded or technical faults occur. This requires methods for forecasting the demand (for instance based on trac measurements), methods for calculating the capacity of the systems, and specication of quantitative measures for the grade of service. When applying the theory in practice, a series of decision problems concerning both short term as well as long term arrangements occur. 1

CHAPTER 1. INTRODUCTION TO TELETRAFFIC ENGINEERING

Short term decisions include a.o. the determination of the number of circuits in a trunk group, the number of operators at switching boards, the number of open lanes in the supermarket, and the allocation of priorities to jobs in a computer system. Long term decisions include for example decisions concerning the development and extension of data- and telecommunication networks, the purchase of cable equipment, transmission systems etc. The application of the theory in connection with design of new systems can help in comparing dierent solutions and thus eliminate non-optimal solutions at an early stage without having to build up prototypes.

1.1

Modelling of telecommunication systems

For the analysis of a telecommunication system, a model must be set up to describe the whole (or parts of) the system. This modelling process is fundamental especially for new applications of the teletrac theory; it requires knowledge of both the technical system as well as the mathematical tools and the implementation of the model on a computer. Such a model contains three main elements (Fig. 1.1): the system structure, the operational strategy, and the statistical properties of the trac.

MAN
Stochastic

Traffic
User demands

MACHINE
Deterministic

Structure
Hardware

Strategy
Software

Figure 1.1: Telecommunication systems are complex man/machine systems. The task of teletrac theory is to congure optimal systems from knowledge of user requirements and habits.

1.1. MODELLING OF TELECOMMUNICATION SYSTEMS

1.1.1

System structure

This part is technically determined and it is in principle possible to obtain any level of details in the description, e.g. at component level. Reliability aspects are stochastic as errors occur at random, and they will be dealt with as trac with a high priority. The system structure is given by the physical or logical system which is described in manuals in every detail. In road trac systems, roads, trac signals, roundabouts, etc. make up the structure.

1.1.2

The operational strategy

A given physical system (for instance a roundabout in a road trac system) can be used in dierent ways in order to adapt the trac system to the demand. In road trac, it is implemented with trac rules and strategies which might be dierent for the morning and the evening trac. In a computer, this adaption takes place by means of the operation system and by operator interference. In a telecommunication system, strategies are applied in order to give priority to call attempts and in order to route the trac to the destination. In Stored Program Controlled (SPC) telephone exchanges, the tasks assigned to the central processor are divided into classes with dierent priorities. The highest priority is given to accepted calls followed by new call attempts whereas routine control of equipment has lower priority. The classical telephone systems used wired logic in order to introduce strategies while in modern systems it is done by software, enabling more exible and adaptive strategies.

1.1.3

Statistical properties of trac

User demands are modelled by statistical properties of the trac. Only by measurements on real systems is it possible to validate that the theoretical modelling is in agreement with reality. This process must necessarily be of an iterative nature (Fig. 1.2). A mathematical model is build up from a thorough knowledge of the trac. Properties are then derived from the model and compared to measured data. If they are not in satisfactory accordance with each other, a new iteration of the process must take place. It appears natural to split the description of the trac properties into stochastic processes for arrival of call attempts and processes describing service (holding) times. These two processes are usually assumed to be mutually independent, meaning that the duration of a call is independent of the time the call arrived. Models also exists for describing the behaviour of users (subscribers) experiencing blocking, i.e. they are refused service and may make a new call attempt a little later (repeated call attempts). Fig. 1.3 illustrates the terminology usually applied in the teletrac theory.

CHAPTER 1. INTRODUCTION TO TELETRAFFIC ENGINEERING

Observation Model Deduction Data Verification


Figure 1.2: Teletrac theory is an inductive discipline. From observations of real systems we establish theoretical models, from which we derive parameters, which can be compared with corresponding observations from the real system. If there is agreement, the model has been validated. If not, then we have to elaborate the model further. This scientic way of working is called the research spiral.

Inter-arrival time

Busy
Holding time Idle time

Idle
Arrival time Departure time

Time

Figure 1.3: Illustration of the terminology applied for a trac process. Notice the dierence between time intervals and instants of time. We use the terms arrival and call synonymously. The inter-arrival time, respectively the inter-departure time, are the time intervals between arrivals, respectively departures.

1.2. CONVENTIONAL TELEPHONE SYSTEMS

1.1.4

Models

General requirements to a model are: 1. It must without major diculty be possible to verify the model and it must be possible to determine the model parameters from observed data. 2. It must be feasible to apply the model for practical dimensioning. We are looking for a description of for example the variations observed in the number of ongoing established calls in a telephone exchange, which vary incessantly due to calls being established and terminated. Even though common habits of subscribers imply that daily variations follows a predictable pattern, it is impossible to predict individual call attempts or duration of individual calls. In the description, it is therefore necessary to use statistical methods. We say that call attempt events take place according to a stochastic process, and the inter arrival time between call attempts is described by those probability distributions which characterise the stochastic process. An alternative to a mathematical model is a simulation model or a physical model (prototype). In a computer simulation model it is common to use either collected data directly or to use articial data from statistical distributions. It is however, more resource demanding to work with simulation since the simulation model is not general. Every individual case must be simulated. The development of a physical prototype is even more time and resource consuming than a simulation model. In general mathematical models are therefore preferred but often it is necessary to apply simulation to develop the mathematical model. Sometimes prototypes are developed for ultimate testing.

1.2

Conventional telephone systems

This section gives a short description on what happens when a call arrives to a traditional telephone central. We divide the description into three parts: structure, strategy and trac. It is common practice to distinguish between subscriber exchanges (access switches, local exchanges, LEX) and transit exchanges (TEX) due to the hierarchical structure according to which most national telephone networks are designed. Subscribers are connected to local exchanges or to access switches (concentrators), which are connected to local exchanges. Finally, transit switches are used to interconnect local exchanges or to increase the availability and reliability.

CHAPTER 1. INTRODUCTION TO TELETRAFFIC ENGINEERING

1.2.1

System structure

Here we consider a telephone exchange of the crossbar type. Even though this type is being taken out of service these years, a description of its functionality gives a good illustration on the tasks which need to be solved in a digital exchange. The equipment in a conventional telephone exchange consists of voice paths and control paths. (Fig. 1.4).
Subscriber Stage Group Selector Junctor Subscriber

Voice Paths

Processor

Processor

Control Paths

Processor

Register

Figure 1.4: Fundamental structure of a switching system. The voice paths are occupied during the whole duration of the call (in average three minutes) while the control paths only are occupied during the call establishment phase (in the range 0.1 to 1 s). The number of voice paths is therefore considerable larger than the number of control paths. The voice path is a connection from a given inlet (subscriber) to a given outlet. In a space divided system the voice paths consists of passive component (like relays, diodes or VLSI circuits). In a time division system the voice paths consist of specic time-slots within a frame. The control paths are responsible for establishing the connection. Normally, this happens in a number of stages where each stage is performed by a control device: a microprocessor, or a register. Tasks of the control device are: Identication of the originating subscriber (who wants a connection (inlet)). Reception of the digit information (address, outlet). Search after an idle connection between inlet and outlet. Establishment of the connection. Release of the connection (performed sometimes by the voice path itself).

1.2. CONVENTIONAL TELEPHONE SYSTEMS

In addition the charging of the calls must be taken care of. In conventional exchanges the control path is build up on relays and/or electronic devices and the logical operations are done by wired logic. Changes in the functions require physical changes and they are dicult and expensive In digital exchanges the control devices are processors. The logical functions are carried out by software, and changes are considerable more easy to implement. The restrictions are far less constraining, as well as the complexity of the logical operations compared to the wired logic. Software controlled exchanges are also called SPC-systems (Stored Program Controlled systems).

1.2.2

User behaviour

We consider a conventional telephone system. When an A-subscriber initiates a call, the hook is taken o and the wired pair to the subscriber is short-circuited. This triggers a relay at the exchange. The relay identies the subscriber and a micro processor in the subscriber stage choose an idle cord. The subscriber and the cord is connected through a switching stage. This terminology originates from a the time when a manual operator by means of the cord was connected to the subscriber. A manual operator corresponds to a register. The cord has three outlets. A register is through another switching stage coupled to the cord. Thereby the subscriber is connected to a register (register selector) via the cord. This phase takes less than one second. The register sends the dial tone to the subscriber who dials the desired telephone number of the B-subscriber, which is received and maintained by the register. The duration of this phase depends on the subscriber. A microprocessor analyses the digit information and by means of a group selector establishes a connection through to the desired subscriber. It can be a subscriber at same exchange, at a neighbour exchange or a remote exchange. It is common to distinguish between exchanges to which a direct link exists, and exchanges for which this is not the case. In the latter case a connection must go through an exchange at a higher level in the hierarchy. The digit information is delivered by means of a code transmitter to the code receiver of the desired exchange which then transmits the information to the registers of the exchange. The register has now fullled its obligation and is released so it is idle for the service of other call attempts. The microprocessors work very fast (around 110 ms) and independently of the subscribers. The cord is occupied during the whole duration of the call and takes control of the call when the register is released. It takes care of dierent types of signals (busy, reference etc), pulses for charging, and release of the connection when the call is put down, etc. It happens that a call does not pass on as planned. The subscriber may make an error,

CHAPTER 1. INTRODUCTION TO TELETRAFFIC ENGINEERING

suddenly hang up, etc. Furthermore, the system has a limited capacity. This will be dealt with in Chap. 2. Call attempts towards a subscriber take place in approximately the same way. A code receiver at the exchange of the B-subscriber receives the digits and a connection is set up through the group switching stage and the local switch stage through the B-subscriber with use of the registers of the receiving exchange.

1.2.3

Operation strategy

The voice path normally works as loss systems while the control path works as delay systems (Chap. 2). If there is not both an idle cord as well as an idle register then the subscriber will get no dial tone no matter how long he/she waits. If there is no idle outlet from the exchange to the desired B-subscriber a busy tone will be sent to the calling A-subscriber. Independently of any additional waiting there will not be established any connection. If a microprocessor (or all microprocessors of a specic type when there are several) is busy, then the call will wait until the microprocessor becomes idle. Due to the very short holding time then waiting time will often be so short that the subscribers do not notice anything. If several subscribers are waiting for the same microprocessor, they will normally get service in random order independent of the time of arrival. The way by which control devices of the same type and the cords share the work is often cyclic, such that they get approximately the same number of call attempts. This is an advantage since this ensures the same amount of wear and since a subscriber only rarely will get a defect cord or control path again if the call attempt is repeated. If a control path is occupied more than a given time, a forced disconnection of the call will take place. This makes it impossible for a single call to block vital parts of the exchange, e.g. a register. It is also only possible to generate the ringing tone for a limited duration of time towards a B-subscriber and thus block this telephone a limited time at each call attempt. An exchange must be able to operate and function independently of subscriber behaviour. The cooperation between the dierent parts takes place in accordance to strictly and well dened rules, called protocols, which in conventional systems is determined by the wired logic and in software control systems by software logic. The digital systems (e.g. ISDN = Integrated Services Digital Network, where the whole telephone system is digital from subscriber to subscriber (2 B + D = 2 64 + 16 Kbps per subscriber), ISDN = N-ISDN = Narrowband ISDN) of course operates in a way dierent from the conventional systems described above. However, the fundamental teletrac tools for evaluation are the same in both systems. The same also covers the future broadband systems BISDN which will be based on ATM = Asynchronous Transfer Mode.

1.3. COMMUNICATION NETWORKS

1.3

Communication networks

There exists dierent kinds of communications networks:, telephone networks, telex networks, data networks, Internet, etc. Today the telephone network is dominating and physically other networks will often be integrated in the telephone network. In future digital networks it is the plan to integrate a large number of services into the same network (ISDN, B-ISDN ).

1.3.1

The telephone network

The telephone network has traditionally been build up as a hierarchical system. The individual subscribers are connected to a subscriber switch or sometimes a local exchange (LEX). This part of the network is called the access network. The subscriber switch is connected to a specic main local exchange which again is connected to a transit exchange (TEX) of which there usually is at least one for each area code. The transit exchanges are normally connected into a mesh structure. (Fig. 1.5). These connections between the transit exchanges are called the hierarchical transit network. There exists furthermore connections between two local exchanges (or subscriber switches) belonging to dierent transit exchanges (local exchanges) if the trac demand is sucient to justify it.

Mesh network

Star network

Ring network

Figure 1.5: There are three basic structures of networks: mesh, star and ring. Mesh networks are applicable when there are few large exchanges (upper part of the hierarchy, also named polygon network), whereas star networks are proper when there are many small exchanges (lower part of the hierarchy). Ring networks are applied for example in bre optical systems. A connection between two subscribers in dierent transit areas will normally pass the following exchanges: USER LEX TEX TEX LEX USER The individual transit trunk groups are based on either analogue or digital transmission systems, and multiplexing equipment is often used.

10

CHAPTER 1. INTRODUCTION TO TELETRAFFIC ENGINEERING

Twelve analogue channels of 3 kHz each make up one rst order bearer frequency system (frequency multiplex), while 32 digital channels of 64 Kbps each make up a rst order PCMsystem of 2.048 Mbps (pulse-code-multiplexing, time multiplexing). The 64 Kbps are obtained from a sampling of the analogue signal at a rate of 8 kHz and an amplitude accuracy of 8 bit. Two of the 32 channels in a PCM system are used for signalling and control.
I I

Figure 1.6: In a telecommunication network all exchanges are typically arranged in a threelevel hierarchy. Local-exchanges or subscriber-exchanges (L), to which the subscribers are connected, are connected to main exchanges (T), which again are connected to inter-urban exchanges (I). An inter-urban area thus makes up a star network. The inter-urban exchanges are interconnected in a mesh network. In practice the two network structures are mixed, because direct trunk groups are established between any two exchanges, when there is sucient trac. In the future Danish network there will only be two levels, as T and I will be merged. Due to reliability and security there will almost always exist at least two disjoint paths between any two exchanges and the strategy will be to use the cheapest connections rst. The hierarchy in the Danish digital network is reduced to two levels only. The upper level with transit exchanges consists of a fully connected meshed network while the local exchanges and subscriber switches are connected to two or three dierent transit exchanges due to security and reliability. The telephone network is characterised by the fact that before any two subscribers can communicate a full two-way (duplex) connection must be created, and the connection exists during the whole duration of the communication. This property is referred to as the telephone network being connection oriented as distinct from for example the Internet which is connection-less. Any network applying for example lineswitching or circuitswitching is connection oriented. A packet switching network may be either connection oriented (for example virtual connections in ATM) or connection-less. In the discipline of network planning, the objective is to optimise network structures and trac routing under the consideration of trac demands, service and reliability requirement etc.

1.3. COMMUNICATION NETWORKS

11

Example 1.3.1: VSAT-networks VSAT-networks (Maral, 1995 [76]) are for instance used by multi-national organisations for transmission of speech and data between dierent divisions of news-broadcasting, in catastrophic situations, etc. It can be both point-to point connections and point to multi-point connections (distribution and broadcast). The acronym VSAT stands for Very Small Aperture Terminal (Earth station) which is an antenna with a diameter of 1.61.8 meter. The terminal is cheap and mobile. It is thus possible to bypass the public telephone network. The signals are transmitted from a VSAT terminal via a satellite towards another VSAT terminal. The satellite is in a xed position 35 786 km above equator and the signals therefore experiences a propagation delay of around 125 ms per hop. The available bandwidth is typically partitioned into channels of 64 Kbps, and the connections can be one-way or two-ways. In the simplest version, all terminals transmit directly to all others, and a full mesh network is the result. The available bandwidth can either be assigned in advance (xed assignment) or dynamically assigned (demand assignment). Dynamical assignment gives better utilisation but requires more control. Due to the small parabola (antenna) and an attenuation of typically 200 dB in each direction, it is practically impossible to avoid transmission error, and error correcting codes and possible retransmission schemes are used. A more reliable system is obtained by introducing a main terminal (a hub) with an antenna of 4 to 11 meters in diameter. A communication takes place through the hub. Then both hops (VSAT hub and hub VSAT) become more reliable since the hub is able to receive the weak signals and amplify them such that the receiving VSAT gets a stronger signal. The price to be paid is that the propagation delay now is 500 ms. The hub solution also enables centralised control and monitoring of the system. Since all communication is going through the hub, the network structure constitutes a star topology.

1.3.2

Data networks

Data network are sometimes engineered according to the same principle as the telephone network except that the duration of the connection establishment phase is much shorter. Another kind of data network is given in the so-called packet distribution network, which works according to the store-and-forward principle (see Fig. 1.7). The data to be transmitted are not sent directly from transmitter to receiver in one step but in steps from exchange to exchange. This may create delays since the exchanges which are computers work as delay systems (connection-less transmission). If the packet has a maximum xed length, the network is denoted packet switching (e.g. X.25 protocol). In X.25 a message is segmented into a number of packets which do not necessarily follow the same path through the network. The protocol header of the packet contains a sequence number such that the packets can be arranged in correct order at the receiver. Furthermore error correction codes are used and the correctness of each packet is checked at the receiver. If the packet is correct an acknowledgement is sent back to the preceding node which now can delete its copy of the packet. If the preceding node does not receive

12

CHAPTER 1. INTRODUCTION TO TELETRAFFIC ENGINEERING


HOST HOST 5 2

3 1 4

HOST

HOST

Figure 1.7: Datagram network: Store- and forward principle for a packet switching data network. any acknowledgement within some given time interval a new copy of the packet (or a whole frame of packets) are retransmitted. Finally, there is a control of the whole message from transmitter to receiver. In this way a very reliable transmission is obtained. If the whole message is sent in a single packet, it is denoted messageswitching. Since the exchanges in a data network are computers, it is feasible to apply advanced strategies for trac routing.

1.3.3

Local Area Networks (LAN)

Local area networks are a very specic but also very important type of data network where all users through a computer are attached to the same digital transmission system, e.g. a coaxial cable. Normally, only one user at a time can use the transmission medium and get some data transmitted to another user. Since the transmission system has a large capacity compared to the demand of the individual users, a user experiences the system as if he is the only user. There exist several types of local area networks. Applying adequate strategies for the medium access control (MAC) principle, the assignment of capacity in case of many users competing for transmission is taken care of. There exist two main types of Local Area Networks: CSMA/CD (Ethernet) and token networks. The CSMA/CD (Carrier Sense

1.4. MOBILE COMMUNICATION SYSTEMS

13

Multiple Access/Collision Detection) is the one most widely used. All terminals are all the time listening to the transmission medium and know when it is idle and when it is occupied. At the same time a terminal can see which packets are addressed to the terminal itself and therefore needs to be stored. A terminal wanting to transmit a packet transmit it if the medium is idle. If the medium is occupied the terminal wait a random amount of time before trying again. Due to the nite propagation speed, it is possible that two (or even more) terminals starts transmission within such a short time interval so that two or more messages collide on the medium. This is denoted as a collision. Since all terminals are listening all the time, they can immediately detect that the transmitted information is dierent from what they receive and conclude that a collision has taken place (CD = Collision Detection). The terminals involved immediately stops transmission and try again a random amount of time later (back-o). In local area network of the token type, it is only the terminal presently possessing the token which can transmit information. The token is rotating between the terminals according to predened rules. Local area networks based on the ATM technique are also in operation. Furthermore, wireless LANs are becoming common. The propagation is negligible in local area networks due to small geographical distance between the users. In for example a satellite data network the propagation delay is large compared to the length of the messages and in these applications other strategies than those used in local area networks are used.

1.4

Mobile communication systems

A tremendous expansion is seen these years in mobile communication systems where the transmission medium is either analogue or digital radio channels (wireless) in contrast to the convention cable systems. The electro magnetic frequency spectrum is divided into dierent bands reserved for specic purposes. For mobile communications a subset of these bands are reserved. Each band corresponds to a limited number of radio telephone channels, and it is here the limited resource is located in mobile communication systems. The optimal utilisation of this resource is a main issue in the cellular technology. In the following subsection a representative system is described.

1.4.1

Cellular systems

Structure. When a certain geographical area is to be supplied with mobile telephony, a suitable number of base stations must be put into operation in the area. A base station is an antenna with transmission/receiving equipment or a radio link to a mobile telephone exchange (MTX) which are part of the traditional telephone network. A mobile telephone exchange

14

CHAPTER 1. INTRODUCTION TO TELETRAFFIC ENGINEERING

is common to all the base stations in a given trac area. Radio waves are damped when they propagate in the atmosphere and a base station is therefore only able to cover a limited geographical area which is called a cell (not to be confused with ATMcells). By transmitting the radio waves at adequate power it is possible to adapt the coverage area such that all base stations covers exactly the planned trac area without too much overlapping between neighbour stations. It is not possible to use the same radio frequency in two neighbour base stations but in two base stations without a common border the same frequency can be used thereby allowing the channels to be reused.

1111 0000 1111 0000 1111 0000 111 000 1111 0000 1111 0000 111 000 1111 0000 1111 0000 B 000 A 111 1111 0000 C B C 1111 0000 111 111 000 000 1111 0000 1111 0000 111 111 000 000 1111 0000 111 1111 111 000 C 0000 000 A 1111 B0000A C B 111 111 000 000 1111 0000 1111 0000 111 111 000 000 1111 0000 1111 0000 111 111 000 000 1111 0000 1111 A B 000 A B0000 A C C 111 1111 0000 111 111 000 000 1111 0000 1111 0000 111 111 000 000 1111 0000 1111 111 000 C 0000 000 A 1111 C B B0000A 1111 0000 111 000 000 111 1111 0000 111 1111 0000 111 111 000 000 1111 0000 1111 0000 111 111 000 000 1111 0000 1111 0000 B 000 A B C 0000 C 1111 111 1111 0000 1111 0000 111 000 1111 0000 1111 0000 111 000 1111 0000 1111 0000 A C A B 1111 0000 1111 0000 1111 0000
A

1111 B0000 A C

Figure 1.8: Cellular mobile communication system. By dividing the frequencies into 3 groups (A, B and C) they can be reused as shown. In Fig. 1.8 an example is shown. A certain number of channels per cell corresponding to a given trac volume is thereby made available. The size of the cell will depend on the trac volume. In densely populated areas as major cities the cells will be small while in sparsely populated areas the cells will be large. Channel allocation is a very complex problem. In addition to the restrictions given above, a number of other also exist. For example, there has to be a certain distance (number of channels) between two channels on the same base station (neighbour channel restriction) and to avoid interference also other restrictions exist. Strategy. In mobile telephone systems a database with information about all the subscriber has to exist. Any subscriber is either active or passive corresponding to whether the radio telephone is switched on or o. When the subscriber turns on the phone, it is automatically assigned to a so-called control channel and an identication of the subscriber takes place. The control channel is a radio channel used by the base station for control. The remaining channels are trac channels A call request towards a mobile subscriber (B-subscriber) takes place the following way. The

1.4. MOBILE COMMUNICATION SYSTEMS

15

mobile telephone exchange receives the call from the other subscriber (A-subscriber, xed or mobile). If the B-subscriber is passive (handset switched o) the A-subscriber is informed that the B-subscriber is not available. Is the B-subscriber active, then the number is put out on all control channels in the trac area. The B-subscriber recognises his own number and informs through the control channel in which cell (base station) he is in. If an idle trac channel exists it is allocated and the MTX puts up the call. A call request from a mobile subscriber (A-subscriber) is initiated by the subscriber shifting from the control channel to a trac channel where the call is established. The rst phase with reading in the digits and testing the availability of the B-subscriber is in some cases performed by the control channel (common channel signalling) A subscriber is able to move freely within his own trac area. When moving away from the base station this is detected by the MTX which constantly monitor the signal to noise ratio and the MTX moves the call to another base station and to another trac channel with better quality when this is required. This takes place automatically by cooperation between the MTX and the subscriber equipment normally without being noticed by the subscriber. This operation is called hand over, and of course requires the existence of an idle trac channel in the new cell. Since it is improper to interrupt an existing call, hand-over calls are given higher priorities than new calls. This strategy can be implemented by reserving one or two idle channels for hand-over calls. When a subscriber is leaving its trac area, so-called roaming will take place. The MTX in the new area is from the identity of the subscriber able to locate the home MTX of the subscriber. A message to the home MTX is forwarded with information on the new position. Incoming calls to the subscriber will always go to the home MTX which will then route the call to the new MTX. Outgoing calls will be taken care of the usual way. A widespread digital wireless system is GSM, which can be used throughout Western Europe. The International Telecommunication Union is working towards a global mobile system UPC (Universal Personal Communication), where subscribers can be reached worldwide (IMT2000). Paging systems are primitive one-way systems. DECT, Digital European Cord-less Telephone, is a standard for wireless telephones. They can be applied locally in companies, business centres etc. In the future equipment which can be applied both for DECT and GSM will come up. Here DECT corresponds to a system with very small cells while GSM is a system with larger cells. Satellite communication systems are also being planned in which the satellite station corresponds to a base station. The rst such system Iridium, consisted of 66 satellites such that more than one satellite always were available at any given location within the geographical range of the system. The satellites have orbits only a few hundred kilometres above the Earth. Iridium was unsuccessful, but newer systems such as the Inmarsat system is now in use.

16

CHAPTER 1. INTRODUCTION TO TELETRAFFIC ENGINEERING

1.5

ITU recommendations on trac engineering

The following section is based on ITUT draft Recommendation E.490.1: Overview of Recommendations on trac engineering. See also (Villen, 2002 [99]). The International Telecommunication Union (ITU) is an organisation sponsored by the United Nations for promoting international telecommunications. It has three sectors: the Telecommunication Standardisation Sector (ITUT), the Radio communication Sector (ITUR), and the Telecommunication Development Sector (ITUD). The primary function of the ITUT is to produce international standards for telecommunications. The standards are known as recommendations. Although the original task of ITUT was restricted to facilitate international inter-working, its scope has been extended to cover national networks, and the ITUT recommendations are nowadays widely used as de facto national standards and as references. The aim of most recommendations is to ensure compatible inter-working of telecommunication equipment in a multi-vendor and multi-operator environment. But there are also recommendations that advice on best practices for operating networks. Included in this group are the recommendations on trac engineering. The ITUT is divided into Study Groups. Study Group 2 (SG2) is responsible for Operational Aspects of Service Provision Networks and Performance. Each Study Group is divided into Working Parties. Working Party 3 of Study Group 2 (WP 3/2) is responsible for Trac Engineering.

1.5.1

Trac engineering in the ITU

Although Working Party 3/2 has the overall responsibility for trac engineering, some recommendations on trac engineering or related to it have been (or are being) produced by other Groups. Study Group 7 deals in the X Series with trac engineering for data communication networks, Study Group 11 has produced some recommendations (Q Series) on trac aspects related to system design of digital switches and signalling, and some recommendations of the I Series, prepared by Study Group 13, deal with trac aspects related to network architecture of N- and B-ISDN and IP based networks. Within Study Group 2, Working Party 1 is responsible for the recommendations on routing and Working Party 2 for the Recommendations on network trac management. This section will focus on the recommendations produced by Working Party 3/2. They are in

1.5. ITU RECOMMENDATIONS ON TRAFFIC ENGINEERING

17

the E Series (numbered between E.490 and E.799) and constitute the main body of ITUT recommendations on trac engineering. The Recommendations on trac engineering can be classied according to the four major trac engineering tasks: Trac demand characterisation; Grade of Service (GoS) objectives; Trac controls and dimensioning; Performance monitoring. The interrelation between these four tasks is illustrated in Fig. 1. The initial tasks in trac engineering are to characterise the trac demand and to specify the GoS (or performance) objectives. The results of these two tasks are input for dimensioning network resources and for establishing appropriate trac controls. Finally, performance monitoring is required to check if the GoS objectives have been achieved and is used as a feedback for the overall process. Secs. 1.5.2, 1.5.3, 1.5.4, 1.5.5 describe each of the above four tasks. Each section provides an overall view of the respective task and summarises the related recommendations. Sec. 1.5.6 summarises a few additional Recommendations as their scope do not match the items considered in the classication Sec. 1.5.7 describes the current work program and Sec. 1.5.8 states some conclusions.

1.5.2

Trac demand characterisation

Trac characterisation is done by means of models that approximate the statistical behaviour of network trac in large population of users. Trac models adopt simplifying assumptions concerning the complicated trac processes. Using these models, trac demand is characterised by a limited set of parameters (mean, variance, index of dispersion of counts, etc). Trac modelling basically involves the identication of what simplifying assumptions can be made and what parameters are relevant from viewpoint of of the impact of trac demand on network performance. Trac measurements are conducted to validate these models, with modications being made when needed. Nevertheless, as the models do not need to be modied often, the purpose of trac measurements is usually to estimate the values that the parameters dened in the trac models take at each network segment during each time period. As a complement to trac modelling and trac measurements, trac forecasting is also required given that, for planning and dimensioning purposes, it is not enough to characterise

18

CHAPTER 1. INTRODUCTION TO TELETRAFFIC ENGINEERING


Traffic demand characterisation Grade of Service objectives

QoS requirements Traffic modelling Traffic measurement Endtoend GoS objectives Traffic forecasting Allocation to net work components

Traffic controls and dimensioning

Traffic controls

Dimensioning

Performance monitoring

Performance monitoring

Figure 1.9: Trac engineering tasks. present trac demand, but it is necessary to forecast trac demands for the time period foreseen in the planning process. Thus the ITU recommendations cover these three aspects of trac characterisation: trac modelling, trac measurements, and trac forecasting.

Trac modelling Recommendations on trac modelling are listed in Tab. 1.1. There are no specic recommendations on trac modelling for the classical circuit-switched telephone network. The only service provided by this network is telephony given other services, as fax, do not have a signicant impact on the total trac demand. Every call is based on a single 64 Kbps point-

1.5. ITU RECOMMENDATIONS ON TRAFFIC ENGINEERING

19

to-point bi-directional symmetric connection. Trac is characterised by call rate and mean holding time at each origin-destination pair. Poissonian call arrival process (for rst-choice routes) and negative exponential distribution of the call duration are the only assumptions needed. These assumptions are directly explained in the recommendations on dimensioning. Rec. E.711 E.712 E.713 E.716 E.760 Date 10/92 User demand modelling 10/92 User plane trac modelling 10/92 Control plane trac modelling 10/96 User demand modelling in Broadband-ISDN 03/00 Terminal mobility trac modelling Table 1.1: Recommendations on trac modelling. Title

The problem is much more complex in N- and B-ISDN and in IPbased network. There are more variety of services, each with dierent characteristics, dierent call patterns and dierent QoS requirements. Recommendations E.711 and E.716 explain how a call, in NISDN and BISDN respectively, must be characterised by a set of connection characteristics (or call attributes) and by a call pattern. Some examples of connection characteristics are the following: information transfer mode (circuit-switched or packet switched), communication conguration (point-to-point, multipoint or broadcast), transfer rate, symmetry (uni-directional, bi-directional symmetric or bi-directional asymmetric), QoS requirements, etc. The call pattern is dened in terms of the sequence of events occurred along the call and of the times between these events. It is described by a set of trac variables, which are expressed as statistical variables, that is, as moments or percentiles of distributions of random variables indicating number of events or times between events. The trac variables can be classied into call-level (or connection-level) and packet-level (or transaction-level, in ATM cell-level) trac variables. The call-level trac variables are related to events occurring during the call set-up and release phases. Examples are the mean number of re-attempts in case of non-completion and mean call-holding time. The packet-level trac variables are related to events occurring during the information transfer phase and describe the packet arrival process and the packet length. Recommendation E.716 describes a number of dierent approaches for dening packet-level trac variables. Once each type of call has been modelled, the user demand is characterised, according to E.711 and E.716, by the arrival process of calls of each type. Based on the user demand

20

CHAPTER 1. INTRODUCTION TO TELETRAFFIC ENGINEERING

characterisation made in Recommendations E.711 and E.716, Recommendations E.712 and E.713 explain how to model the trac oered to a group of resources in the user plane and the control plane, respectively. Finally, Recommendation E.760 deals with the problem of trac modelling in mobile networks where not only the trac demand per user is random but also the number of users being served at each moment by a base station or by a local exchange. The recommendation provides methods to estimate trac demand in the coverage area of each base station and mobility models to estimate hand-over and location updating rates.

Trac measurements Recommendations on trac measurements are listed in Tab. 1.2. As indicated in the table, many of them cover both trac and performance measurements. These recommendations can be classied into those on general and operational aspects (E.490, E.491, E.502 and E.503), those on technical aspects (E.500 and E.501) and those specifying measurement requirements for specic networks (E.502, E.505 and E.745). Recommendation E.743 is related to the last ones, in particular to Recommendation E.505. Let us start with the recommendations on general and operational aspects. Recommendation E.490 is an introduction to the series on trac and performance measurements. It contains a survey of all these recommendations and explains the use of measurements for short term (network trac management actions), medium term (maintenance and reconguration) and long term (network extensions). Recommendation E.491 points out the usefulness of trac measurements by destination for network planning purposes and outlines two complementary approaches to obtain them: call detailed records and direct measurements. Recommendations E.504 describes the operational procedures needed to perform measurements: tasks to be made by the operator (for example to dene output routing and scheduling of measured results) and functions to be provided by the system supporting the man-machine interface. Once the measurements have been performed, they have to be analysed. Recommendation E.503 gives an overview of the potential application of the measurements and describes the operational procedures needed for the analysis. Let us now describe Recommendations E.500 and E.501 on general technical aspects. Recommendation E.500 states the principles for trac intensity measurements. The traditional concept of busy hour, which was used in telephone networks, cannot be extended to modern multi-service networks. Thus Recommendation E.500 provides the criteria to choose the length of the read-out period for each application. These criteria can be summarised as

1.5. ITU RECOMMENDATIONS ON TRAFFIC ENGINEERING Rec. Date Title

21

E.490* 06/92 Trac measurement and evaluation - general survey E.491 E.500 E.501 05/97 Trac measurement by destination 11/98 Trac intensity measurement principles 05/97 Estimation of trac oered in the network

E.502* 02/01 Trac measurement requirements for digital telecommunication exchanges E.503* 06/92 Trac measurement data analysis E.504* 11/88 Trac measurement administration E.505* 06/92 Measurements of the performance of common channel signalling network E.743 04/95 Trac measurements for SS No. 7 dimensioning and planning E.745* 03/00 Cell level measurement requirements for the B-ISDN Table 1.2: Recommendations on trac measurements. Recommendations marked * cover both trac and performance measurements.

follows:

a) To be large enough to obtain condent measurements: the average trac intensity in a period (t1 , t2 ) can be considered a random variable with expected value A. The measured trac intensity A(t1 , t2 ) is a sample of this random variable. As t2 t1 increases, A(t1 , t2 ) converges to A. Thus the read-out period length t2 t1 must be large enough such that A(t1 , t2 ) lies within a narrow condence interval about A. An additional reason to choose large read-out periods is that it may not be worth the eort to dimension resources for very short peak trac intervals. b) To be short enough so that the trac intensity process is approximately stationary during the period, i.e. that the actual trac intensity process can be approximated by a stationary trac intensity model. Note that in the case of bursty trac, if a simple trac model (e.g. Poisson) is being used, criterion (b) may lead to an excessively short read-out period incompatible with criterion (a). In these cases alternative models should be used to obtain longer read-out period.

Recommendation E.500 also advises on how to obtain the daily peak trac intensity over the measured read-out periods. It provides the method to derive the normal load and high load trac intensities for each month and, based on them, the yearly representative values (YRV ) for normal and high loads.

22

CHAPTER 1. INTRODUCTION TO TELETRAFFIC ENGINEERING

As oered trac is required for dimensioning while only carried trac is obtained from measurements, Recommendation E.501 provides methods to estimate the trac oered to a circuit group and the origin-destination trac demand based on circuit group measurements. For the trac oered to a circuit group, the recommendation considers both circuit groups with only-path arrangement, and circuit groups belonging to a high-usage/nal circuit group arrangement. The repeated call attempts phenomenon is taken into account in the estimation. Although the recommendation only refers to circuit-switched networks with single-rate connections, some of the methods provided can be extended to other types of networks. Also, even though the problem may be much more complex in multi-service networks, advanced exchanges typically provide, in addition to circuit group trac measurements, other measurements such as the number of total, blocked, completed and successful call attempts per service and per origin-destination pair, which may help to estimate oered trac. The third group of recommendations on measurements includes Recommendations E.502, E.505 and E.745 which specify trac and performance measurement requirements in PSTN and N-ISDN exchanges (E.502), B-ISDN exchanges (E.745) and nodes of SS No. 7 Common Channel Signalling Networks (E.505). Finally, Recommendation E.743 is complementary to E.505. It identies the subset of the measurements specied in Recommendation E.505 that are useful for SS No. 7 dimensioning and planning, and explains how to derive the input required for these purposes from the performed measurements.

Trac forecasting Trac forecasting is necessary both for strategic studies, such as to decide on the introduction of a new service, and for network planning, that is, for the planning of equipment plant investments and circuit provisioning. The Recommendations on trac forecasting are listed in Tab. 1.3. Although the title of the rst two refers to international trac, they also apply to the trac within a country. Recommendations E.506 and E.507 deal with the forecasting of traditional services for which there are historical data. Recommendation E.506 gives guidance on the prerequisites for the forecasting: base data, including not only trac and call data but also economic, social and demographic data are of vital importance. As the data series may be incomplete, strategies are recommended for dealing with missing data. Dierent forecasting approaches are presented: direct methods, based on measured trac in the reference period, versus composite method based on accounting minutes, and top-down versus bottom-up procedures. Recommendation E.507 provides an overview of the existing mathematical techniques for forecasting: curve-tting models, autoregressive models, autoregressive integrated moving average (ARIMA) models, state space models with Kalman ltering, regression models and econometric models. It also describes methods for the evaluation of the forecasting models

1.5. ITU RECOMMENDATIONS ON TRAFFIC ENGINEERING Rec. E.506 E.507 E.508 Date 06/92 Forecasting international trac 11/88 Models for forecasting international trac 10/92 Forecasting new telecommunication services Table 1.3: Recommendations on trac forecasting. Title

23

and for the choice of the most appropriate one in each case, depending on the available data, length of the forecast period, etc. Recommendation E.508 deals with the forecasting of new telecommunication services for which there are no historical data. Techniques such as market research, expert opinion and sectorial econometrics are described. It also advises on how to combine the forecasts obtained from dierent techniques, how to test the forecasts and how to adjust them when the service implementation starts and the rst measurements are taken.

1.5.3

Grade of Service objectives

Grade of Service (GoS) is dened in Recommendations E.600 and E.720 as a number of trac engineering parameters to provide a measure of adequacy of plant under specied conditions; these GoS parameters may be expressed as probability of blocking, probability of delay, etc. Blocking and delay are caused by the fact that the trac handling capacity of a network or of a network component is nite and the demand trac is stochastic by nature. GoS is the trac related part of network performance (NP), dened as the ability of a network or network portion to provide the functions related to communications between users. Network performance does not only cover GoS (also called tracability performance), but also other non-trac related aspects as dependability, transmission and charging performance. NP objectives and in particular GoS objectives are derived from Quality of Service (QoS) requirements, as indicated in Fig. 1.9. QoS is a collective of service performances that determine the degree of satisfaction of a user of a service. QoS parameters are user oriented and are described in network independent terms. NP parameters, while being derived from them, are network oriented, i.e. usable in specifying performance requirements for particular networks. Although they ultimately determine the (user observed) QoS, they do not necessarily describe that quality in a way that is meaningful to users. QoS requirements determine end-to-end GoS objectives. From the end-to-end objectives, a partition yields the GoS objectives for each network stage or network component. This partition depends on the network operator strategy. Thus ITU recommendations only specify

24

CHAPTER 1. INTRODUCTION TO TELETRAFFIC ENGINEERING

the partition and allocation of GoS objectives to the dierent networks that may have to cooperate to establish a call (for example originating national network, international network and terminating national network in an international call). In order to obtain an overview of the network under consideration and to facilitate the partitioning of the GoS, ITU Recommendations provide the so-called reference connections. A reference connection consists of one or more simplied drawings of the path a call (or connection) can take in the network, including appropriate reference points where the interfaces between entities are dened. In some cases a reference point dene an interface between two operators. Recommendations devoted to provide reference connections are listed in Tab. 1.4. Recommendation E.701 provides reference connection for N-ISDN networks, RecomRec. E.701 E.751 E.752 E.755 E.651 Date Title

10/92 Reference connections for trac engineering 02/96 Reference connections for trac engineering of land mobile networks 10/96 Reference connections for trac engineering of maritime and aeronautical systems 02/96 Reference connections for UPT trac performance and GoS 03/00 Reference connections for trac engineering of IP access networks Table 1.4: Recommendations on reference connections.

mendation E.751 for land mobile networks, Recommendation E.752 for maritime and aeronautical systems, Recommendation E.755 for UPT services, and Recommendation E.651 for IPbased networks. In the latter, general reference connections are provided for the end-to-end connections and more detailed ones for the access network in case of HFC (Hybrid Fiber Coax) systems. As an example, Fig. 1.10 (taken from Fig. 6.2 of Recommendation E.651) presents the reference connection for an IPtoPSTN/ISDN or PSTN/ISDNtoIP call. We now apply the philosophy explained above for dening GoS objectives, starting with the elaboration of Recommendation E.720, devoted to N-ISDN. The recommendations on GoS objectives for PSTN, which are generally older, follow a dierent philosophy and can now be considered an exception within the set of GoS recommendations. Let us start this overview with the new recommendations. They are listed in Tab. 1.5. Recommendations E.720 and E.721 are devoted to N-ISDN circuit-switched services. Recommendation E.720 provides general guidelines and Recommendation E.721 provides GoS parameters and target values. The recommended end-to-end GoS parameters are: Pre-selection delay Post-selection delay

1.5. ITU RECOMMENDATIONS ON TRAFFIC ENGINEERING


IP access network PSTN/ISDN gateway

25

CPN

PSTN/ISDN

CPN

a) Direct interworking with PSTN/ISDN

CPN

IP access network

IP core network

PSTN/ISDN gateway

PSTN/ISDN

CPN

b) Interworking with PSTN/ISDN through IP core network

Figure 1.10: IPtoPSTN/ISDN or PSTN/ISDNtoIP reference connection. CPN = Customer Premises Network. Answer signal delay Call release delay Probability of end-to-end blocking After dening these parameters, Recommendation E.721 provides target values for normal and high load as dened in Recommendation E.500. For the delay parameters, target values are given for the mean delay and for the 95 % quantile. For those parameters that are dependent on the length of the connection, dierent sets of target values are recommended for local, toll, and international connections. The recommendation provides reference connections, characterised by a typical range of the number of switching nodes, for the three types of connections. Based on the delay related GoS parameters and target values given in Recommendations E.721, Recommendation E.723 identies GoS parameters and target values for Signalling System # 7 networks. The identied parameters are the delays incurred by the initial address message (IAM) and by the answer message (ANM). Target values consistent with those of Recommendation E.721 are given for local, toll and international connections. The typical number of switching nodes of the reference connections provided in Recommendation E.721 are complemented in Recommendation E.723 with typical number of STPs (signal transfer points). The target values provided in Recommendation E.721 refer to calls not invoking intelligent network (IN) services. Recommendation E.724 species incremental delays that are allowed when they are invoked. Reference topologies are provided for the most relevant service classes, such as database query, call redirection, multiple set-up attempts, etc. Target values of the incremental delay for processing a single IN service are provided for some service

26 Rec. E.720 E.721 E.723 E.724 E.726 E.728 E.770 E.771 E.773 E.774 E.775 E.776 E.671 Date

CHAPTER 1. INTRODUCTION TO TELETRAFFIC ENGINEERING Title

11/98 ISDN grade of service concept 05/99 Network grade of service parameters and target values for circuitswitched services in the evolving ISDN 06/92 Grade-of-service parameters for Signalling System No. 7 networks 02/96 GoS parameters and target GoS objectives for IN Services 03/00 Network grade of service parameters and target values for B-ISDN 03/98 Grade of service parameters for B-ISDN signalling 03/93 Land mobile and xed network interconnection trac grade of service concept 10/96 Network grade of service parameters and target values for circuitswitched land mobile services 10/96 Maritime and aeronautical mobile grade of service concept 10/96 Network grade of service parameters and target values for maritime and aeronautical mobile services 02/96 UPT Grade of service concept 10/96 Network grade of service parameters for UPT 03/00 Post selection delay in PSTN/ISDNs using Internet telephony for a portion of the connection Table 1.5: Recommendations on GoS objectives (except for PSTN).

classes as well as of the total incremental post-selection delay for processing all IN services. Recommendation E.726 is the equivalent of Recommendation E.721 for B-ISDN. As BISDN is a packet-switched network, call-level and packet-level (in this case cell-level) GoS parameters are distinguished. Call-level GoS parameters are analogous to those dened in Recommendation E.721. The end-to-end cell-level GoS parameters are: Cell transfer delay Cell delay variation Severely errored cell block ratio Cell loss ratio Frame transmission delay Frame discard ratio

1.5. ITU RECOMMENDATIONS ON TRAFFIC ENGINEERING

27

While the call-level QoS requirements may be similar for all the services (perhaps with the exception of emergency services), the cell-level QoS requirements may be very dierent depending on the type of service: delay requirements for voice and video services are much more stringent than those for data services. Thus target values for the cell-level must be service dependent. These target values are left for further study in the current issue while target values are provided for the call-level GoS parameters for local, toll and international connections. Recommendation E.728, for B-ISDN signalling, is based on the delay related call-level parameters of Recommendation E.726. Recommendation E.728 in its relation to Recommendation E.726, is analogous to the corresponding relationship between Recommendation E.723 and E.721. In the mobile network series, there are three pairs of recommendations analogous to the E.720/E.721 pair: Recommendations E.770 and E.771 for land mobile networks, Recommendations E.773 and E.774 for maritime and aeronautical systems and Recommendations E.775 and E.776 for UPT services. All these are for circuit-switched services. They analyse the features of the corresponding services that make it necessary to specify less stringent target values for the GoS parameters than those dened in E.721, and dene additional GoS parameters that are specic for these services. For example, in Recommendations E.770 and E.771 on land mobile networks, the reasons for less stringent parameters are: the limitations of the radio interface, the need for the authentication of terminals and of paging of the called user, and the need for interrogating the home and (in case of roaming) visited network databases to obtain the routing number. An additional GoS parameter in land mobile networks is the probability of unsuccessful hand-over. Target values are given for xed-tomobile, mobile-to-xed and mobile-to-mobile calls considering local, toll and international connections. The elaboration of recommendations on GoS parameters and target values for IPbased network has just started. Recommendation E.671 only covers an aspect on which was urgent to give advice. It was to specify target values for the post-selection delay in PSTN/ISDN networks when a portion of the circuit-switched connection is replaced by IP telephony and the users are not aware of this fact. Recommendation E.671 states that the end-to-end delay must in this case be equal to that specied in Recommendation E.721. Let us nish this overview on GoS recommendations with those devoted to the PSTN. They are listed in Tab. 1.6. Recommendations E.540, E.541 and E.543 can be considered the counterpart for PSTN of Recommendation E.721 but organised in a dierent manner, as pointed out previously. They are focused on international connections, as was usual in the old ITU recommendations. Recommendation E.540 species the blocking probability of the international part of an international connection, Recommendation E.541 the endto-end blocking probability of an international connection, and Recommendation E.543 the internal loss probability and delays of an international telephone exchange. A revision of these recommendations is needed to decide if they can be deleted, while extending the scope

28 Rec. E.540 E.541 E.543 E.550 Date

CHAPTER 1. INTRODUCTION TO TELETRAFFIC ENGINEERING Title

11/98 Overall grade of service of the international part of an international connection 11/88 Overall grade of service for international connections (subscriber-to-subscriber) 11/88 Grades of service in digital international telephone exchanges 03/93 Grade of service and new performance criteria under failure conditions in international telephone exchanges Table 1.6: Recommendations on GoS objectives in the PSTN.

of Recommendation E.721 to cover PSTN. The target values specied in all of the GoS recommendations assume that the network and its components are fully operational. On the other hand, the Recommendations on availability deal with the intensity of failures and duration of faults of network components, without considering the fraction of call attempts which is blocked due to the failure. Recommendation E.550 combines the concepts from the elds of both availability and trac congestion, and denes new performance parameters and target values that take into account their joint eects in a telephone exchange.

1.5.4

Trac controls and dimensioning

Once the trac demand has been characterised and the GoS objectives have been established, trac engineering provides a cost ecient design and operation of the network while assuring that the trac demand is carried and GoS objectives are satised. The inputs of trac engineering to the design and operation of networks are network dimensioning and trac controls. Network dimensioning assures that the network has enough resources to support the trac demand. It includes the dimensioning of the physical network elements and also of the logical network elements, such as the virtual paths of an ATM network. Trac controls are also necessary to ensure that the GoS objectives are satised. Among the trac controls we can distinguish: Trac routing: routing patterns describe the route set choices and route selection rules for each origin-destination pair. They may be hierarchical or non-hierarchical, xed or dynamic. Dynamic methods include time-dependent routing methods,in which the routing pattern is altered at a xed time on a pre-planned basis, and state-dependent or event-dependent routing, in which the network automatically alters the routing pat-

1.5. ITU RECOMMENDATIONS ON TRAFFIC ENGINEERING

29

tern based on present network conditions. Recommendations E.170 to E.177 and E.350 to E.353 all deal with routing, are out of the scope of this section. Nevertheless, reference to routing is constantly made in the trac engineering recommendations here presented. On one hand routing design is based on trac engineering considerations: for example, alternative routing schemes are based on cost eciency considerations, dynamic routing methods are based on considerations of robustness under focused overload or failure conditions or regarding trac forecast errors. On the other hand, network dimensioning is done by taking into account routing methods and routing patterns. Network trac management controls: these controls assure that network throughput is maintained under any overload or failure conditions. Trac management controls may be protective or expansive. The protective controls such as code blocking or call gapping assure that the network does not waste resources in processing calls that will be unsuccessful or limit the ow of calls requiring many network resources (overow calls). The expansive controls re-route the trac towards those parts of the network that are not overloaded. Trac management is usually carried out at trac management centres where real-time monitoring of network performance is made through the collection and display of real-time trac and performance data. Controls are usually triggered by an operator on a pre-planned basis (when a special event is foreseen) or in real-time. In the ITUT organisation, network trac management is under the responsibility of WP 2/2. Recommendations E.410 to E.417, dealing with this subject, are out of the scope of this section. Nevertheless, reference to trac management is made in the trac engineering recommendations. For example, measurement requirements specied in the trac and performance measurement recommendations include the real-time measurements required for network trac management. Service protection methods: they are call-level trac controls that control the grade of service for certain streams of trac by means of a discriminatory restriction of the access to circuit groups with little idle capacity. Service protection is used to provide stability in networks with non-hierarchical routing schemes by restricting overow trac to an alternative route that is shared with rst-choice trac. It is also used to balance GoS between trac streams requesting dierent bandwidth or to give priority service to one type of trac. Packet-level trac controls: these controls assure that the packet-level GoS objectives of the accepted calls are satised under any network condition and that a cost-ecient grade of service dierentiation is made between services with dierent packet-level QoS requirements. Signalling and intelligent network (IN) controls: given that these networks are the neural system of the whole network, a key objective in the design and operation of them is to maximise their robustness, that is, their ability to withstand both trac overloads and failures of network elements. It is achieved both by means of redundancy of network elements and by means of a set of congestion and overload controls, as explained in Recommendations E.744 to be described below.

30

CHAPTER 1. INTRODUCTION TO TELETRAFFIC ENGINEERING

Let us classify the recommendations on dimensioning and trac controls into those devoted to circuit-switched networks, to packet-switched networks, and to signalling and INstructured networks.

Circuit-Switched networks Recommendations on trac controls and dimensioning of circuit-switched networks are listed in Tab. 1.7. These recommendations deal with dimensioning and service protection methods taking into account trac routing methods. Rec. E.510 E.520 E.521 E.522 E.524 E.525 E.526 E.527 E.528 E.529 E.731 Date Title

10/45 Determination of the number of circuits in manual operation 11/88 Number of circuits to be provided in automatic and/or semi-automatic operation, without overow facilities 11/88 Calculation of the number of circuits in a group carrying overow trac 11/88 Number of circuits in a high-usage group 05/99 Overow approximations for non-random inputs 06/92 Designing networks to control grade of service 03/93 Dimensioning a circuit group with multi-slot bearer services and no overow inputs 03/00 Dimensioning at a circuit group with multi-slot bearer services and overow trac 02/96 Dimensioning of digital circuit multiplication equipment (DCME) systems 05/97 Network dimensioning using end-to-end GoS objectives 10/92 Methods for dimensioning resources operating in circuit switched mode

Table 1.7: Recommendations on trac controls and dimensioning of circuitswitched networks. Recommendations E.520, E.521, E.522 and E.524 deal with the dimensioning of circuit groups or high-usage/nal group arrangements carrying single-rate (or single-slot) connections. Service protection methods are not considered in these recommendations: Recommendation E.520 deals with methods for dimensioning of only-path circuit groups (Fig. 1.11a). Recommendations E.521 and E.522 provide methods for the dimensioning of simple alternative routing arrangements as the one shown in Fig. 1.11(b), where there only

1.5. ITU RECOMMENDATIONS ON TRAFFIC ENGINEERING

31

Figure 1.11: Examples of circuit group arrangements. exist rst- and second-choice routes, and where the whole trac overowing from a circuit group is oered to the same circuit group. Recommendation E.521 provides methods for dimensioning the nal group satisfying GoS requirements for given sizes of the high-usage circuit groups, and Recommendation E.522 advises on how to dimension high-usage groups to minimise the cost of the whole arrangement. Recommendation E.524 provides overows approximations for non-random inputs which allows for the dimensioning of more complex arrangements (i.e. without the previous mentioned limitations) as that shown in Fig. 1.11 (c). Several approaches are described and compared from the point of view of accuracy and complexity.

Recommendation E.525 introduces service protection methods for networks carrying singlerate connections. It describes the applications and the available methods: split circuit groups, circuit reservation (also called trunk reservation or, in packet-switched networks, bandwidth reservation) and virtual circuits. The recommendations provides methods to evaluate the blocking probability of each trac stream both for only-path circuit groups and for alternative routing arrangements, which allow for the dimensioning of the circuit groups and of the thresholds dening the protection methods. A comparison of the available service protection methods is made from the point of view of eciency, overload protection, robustness and impact of peakedness. Recommendations E.526 and E.527 deal with the dimensioning of circuit groups carrying multi-slot (or multi-rate) connections. Service protection methods are considered in both of them. Recommendation E.526 deals with only-path circuit groups while Recommendation E.527 deals with alternative routing schemes. Tab. 1.8 summarises the items considered in each of the Recommendations mentioned above. Recommendation E.528 deals with the dimensioning of a particular but very important type of circuit group, where Digital Circuit Multiplication Equipment (DCME) is used to achieve statistical multiplexing gain in communications via satellite. This is to save circuits by means of interpolating speech bursts of dierent channels by taking advantage of the silences

32

CHAPTER 1. INTRODUCTION TO TELETRAFFIC ENGINEERING Recommendation Alternative routing Service protection Multi-slot connections E.520 E.521 No No No Yes* No No E.522 E.524 Yes* No No Yes No No E.525 E.526 Yes Yes No No Yes Yes E.527 Yes Yes Yes

Table 1.8: Items considered in the circuit group dimensioning. Recommendations E.520 to E.527. * Only simple arrangements.

existing in a conversation. Dimensioning methods for circuit groups providing integration of trac containing voice, facsimile and voice band data are given. Recommendation E.731 is also devoted to circuit group dimensioning and considers those special features of N-IDSN that may have an impact on trac engineering. Apart from multislot connections and service protection methods, the recommendation studies the impact of attribute negotiation (of attributes aecting either the choice of circuit group or the required number of circuits), of service reservation (reservation of dedicated resources or of resources shared with on demand services) and of point-to-multi-point connections. Recommendation E.529 collects all the dimensioning methods on circuit group or alternative routing arrangement described in previous Recommendations, with a view to giving guidelines for the dimensioning of the whole network using end-to-end GoS objectives. Dimensioning methods for networks with xed, time-dependent, state-dependent or eventdependent trac routing are described. Principles for the decomposition of the networks into blocks that may be considered statistically independent are given, and the iterative procedure required for network optimisation is described.

Packet-Switched networks Recommendations on trac controls and dimensioning of packet-switched networks are listed in Tab. 1.9. They deal with B-ISDN networks using ATM technology, but most of the methods described apply to other packet-switched networks, as for example IPbased networks, in which the admission of connections is controlled. The connection admission control (CAC) establishes a division between the packet-level and the connection-level. When a user request the establishment of a new connection, the CAC decides if the connection can be admitted while satisfying packet-level GoS of both new and existing connections. This decision is usually made by means of allocating resources (typically bandwidth) to each connection and refusing new request when there are insucient resources. Thus:

1.5. ITU RECOMMENDATIONS ON TRAFFIC ENGINEERING Rec. E.735 E.736 E.737 Date Title

33

05/97 Framework for trac control and dimensioning in B-ISDN 05/97 Methods for cell level trac control in B-ISDN 05/97 Dimensioning methods for B-ISDN

Table 1.9: Recommendations on trac controls and dimensioning of packet-switched networks.

From a packet-level perspective: as the CAC assures that packet-level GoS objectives are satised regardless of the rate of connections oered to the network, it makes the packet-level independent from the connection-level oered trac and from the network dimensioning. From a connection-level perspective: as the CAC, in deciding on the acceptance of a connection, takes into account all the packet-level controls implemented, it summarises all the packet-level controls in an amount of resources required by a connection. It makes the connection-level of a packet-switched networks similar to that of a circuitswitched network: the amount of resources required by a connection, called eective or equivalent bandwidth (or, in ATM, equivalent cell rate) is equivalent to the number of slots required by a multi-slot connection in a circuit-switched network. Connectionlevel trac controls and network dimensioning must assure that the connection-level GoS requirements, typically the specied connection blocking probabilities, are satised taking into account the eective bandwidth that has to be allocated to each connection. In practice, this separation between packet-and connection-level is not so complete as described above: the eective bandwidth of a connection depends on the capacity of the physical or logical link in which it is carried (apart from the packet-level trac characteristics of the connection) while, in its turn, the capacity of the links must be dimensioned by taking into account the eective bandwidth of the connections. Thus, an iterative process between connection- and packet-level for network dimensioning is necessary. Recommendation E.735 is the framework for trac control and dimensioning in B-ISDN. It introduces the concepts described above, denes what is a connection and what is a resource, and analyses strategies for logical network conguration. Recommendation E.736 focuses on packet-level. It provides methods for packet-level performance evaluation, proposes possible multiplexing strategies (peak rate allocation, rate envelope multiplexing and statistical rate sharing) and analyses the implications and applications of each of them. Based on this analysis, the recommendation provides methods for packet-level controls. Emphasis is placed on methods for Connection Admission Control and for the integration (or segregation) of services with dierent QoS requirements either by using dedicated resources or by sharing the same resources and implementing loss and/or delay

34

CHAPTER 1. INTRODUCTION TO TELETRAFFIC ENGINEERING

priorities. It also addresses adaptive resource management techniques to control the ow of packets of services with non-stringent delay requirements. Recommendation E.737 provides methods for circuit group and network dimensioning and addresses connection-level trac controls, in particular service protection methods. Trac routing methods are also taken into account. As the eective bandwidth of a connection is modelled as a number of slots of a multi-slot connection, this recommendation is not very dierent from those on circuit-switched network dimensioning. Nevertheless the recommendation deals with some features that are particular of packet-switched networks: the above mentioned iteration between eective bandwidth and network dimensioning; the required bandwidth discretization into multiples of a bandwidth quantisation unit, given that the multi-slot models only deal with integer number of slots; and the implications on the dimensioning of services with dierent packet-level QoS requirements.

Signalling and INStructured Networks The recommendations on trac controls and dimensioning of signalling networks and intelligent networks (IN) are listed in Tab. 1.10. Recommendations E.733 and E.734 deal with dimensioning and Recommendation E.744 with trac controls. Rec. Date Title

E.733 11/98 Methods for dimensioning resources in Signalling System No. 7 networks E.734 10/96 Methods for allocating and dimensioning Intelligent Network (IN) resources E.744 10/96 Trac and congestion control requirements for SS No. 7 and INstructured networks Table 1.10: Recommendations on trac controls and dimensioning of signalling and IN structured networks. Recommendation E.733 provides a methodology for the planning and dimensioning of signalling system No. 7 networks. The methodology takes into account the fact that the eciency of the signalling links should not be the primary consideration, but the performance of the network under failure and trac overload has greater importance. The recommendation describes the reference trac and reference period that, in agreement with Recommendations E.492 and E.500, must be used to dimension the number of signalling links and to ensure that the capacity of network switching elements is not exceeded. It describes the factors for determining a maximum design link utilisation, max , which ensure that the end-to-end delay objectives described in Recommendation E.723 are met. Delays incurred when, due to failures, the link load is 2 max are also taken into account for determining max . Initial values for max being used are described and methods are given for determining the number

1.5. ITU RECOMMENDATIONS ON TRAFFIC ENGINEERING of signalling links and the switching capacity required.

35

Recommendation E.734 deals with resource allocation and dimensioning methods for Intelligent Networks. It discusses the new trac engineering factors to be considered: services with reference period out of the normal working hours, mass calling situations produced by some services, fast implementation of new services with uncertain forecast. The last factor makes it necessary to have the allocation and dimensioning procedures exible enough to provide, as quickly as possible, the resources required as new services are implemented or the user demand changes. The recommendation provides criteria for resource allocation, both for the location of the INspecic elements and for the partitioning of the Intelligent Network functionality (such as service logic) among these elements. It also provides methods for the dimensioning of the IN nodes and of the supporting signalling subnetwork, and discusses the impact on the circuit-switched network dimensioning. Trac and congestion control procedures for SS. No. 7 and INstructured networks are specied in the Q and E.410-series Recommendations. These procedures generally leave key parameter values to be specied as part of the implementation. Given that robustness is a key requirement of signalling and INstructured networks, a proper implementation of these controls is essential. Recommendation E.744 provides guidelines for this implementation, indicating how the control parameters should be chosen in dierent types of networks. The recommendation also advises on requirements to be placed on signalling nodes and IN nodes on the needs for nodelevel overload controls and on how such controls must interrelate with network-level controls. Finally, the recommendation states basic principles to keep dierent systems and controls harmonised in order to allow for various vendor products and network implementations to be interconnected with a high condence the control procedures will work properly.

1.5.5

Performance monitoring

Once the network is operational, continuous monitoring of the GoS is required. Although the network is correctly dimensioned, there are overload and failure situations not considered in the dimensioning where short term (minutes, hours) network trac management actions have to be taken. In situations considered in the dimensioning, trac forecast errors or approximations made in the dimensioning models may lead to a GoS dierent from the one expected. GoS monitoring is needed to detect these problems and to produce feedback for trac characterisation and network design. Depending on the problems detected, network recongurations, changes of the routing patterns or adjustment of trac control parameters can be made in medium term (weeks, months). The urgency of a long term planning of network extensions may also be assessed. Recommendations E.490, E.491, E.502, E.503, E.504, E.505 and E.745, covering both trac and performance measurements, have been described in Sec. 1.5.2, cover both trac and

36

CHAPTER 1. INTRODUCTION TO TELETRAFFIC ENGINEERING

performance measurements. We consider in this section two other Recommendations, E.492 and E.493, listed in Tab. 1.11 which are only related to performance measurements. Rec. E.492 E.493 Date 02/96 Trac reference period 02/96 Grade of Service (GoS) monitoring Title

Table 1.11: Recommendations on performance measurements (for recommendations covering both trac and performance measurements, see Tab. 1.2).

Recommendation E.492 provides the denition of trac reference periods for the purposes of collecting measurements for monitoring Grade-of-Service for networks and network components. This Recommendation is closely related to Recommendation E.500, which denes read-out periods for trac intensity measurements required for network dimensioning. These read-out periods have to be consistent with those used for performance monitoring once the network is operative. Recommendation E.492 also denes the normal and high load periods that are representative of each month. The purpose of these denitions, also consistent with those of Recommendation E.500, is to identify which day and read-out period to use for comparing the monitored GoS to the GoS target values specied for normal and high load. Recommendation E.493 addresses how to perform end-to-end GoS monitoring, taking into account practical limitations. Measurement of blocking or mishandling probabilities is straightforward. However, as direct measurements of end-to-end delays are not feasible in a continuous monitoring, the Recommendation proposes methods to approximate end-to-end delays (mean and 95 % quantile) by means of local measurements autonomously taken in each network element. The proposed methods do not require coordination between network elements to take the measurements. The Recommendation also explains how to apply the proposed methods to the monitoring of each of the connection-level GoS parameters dened in the recommendations on GoS objectives.

1.5.6

Other recommendations

There are a few other Recommendations for which their scope does not match any of the items considered in the classication made here. They are listed in Tab. 1.12. Recommendations E.600 provides a list of trac engineering terms and denitions used throughout the whole set of trac engineering Recommendations. Recommendations E.700 & E.750 are introductory Recommendations to the E.700/749 Series Recommendations on trac engineering for N- and B-ISDN, and to the E.750/799

1.5. ITU RECOMMENDATIONS ON TRAFFIC ENGINEERING Rec. E.523 E.600 E.700 E.750 Date Title

37

11/88 Standard trac proles for international trac streams 03/93 Terms and denitions of trac engineering 10/92 Framework of the E.700-Series Recommendations 03/00 Introduction to the E.750-Series of Recommendations on trac engineering aspects of networks supporting mobile and UPT services

Table 1.12: Recommendations not matching under any of the items considered in the classication made here.

Series Recommendations on trac engineering for mobile networks, respectively. Recommendation E.523 provides standardised 24-hour trac proles for trac streams between countries in dierent relative time locations. This measurement-based information may be useful for those countries where no measurements are available. The proles refer to telephone trac and must not be used for data trac for which the proles may be very dierent.

1.5.7

Work program for the Study Period 20012004

The work in ITU is planned for periods of four years, called study periods. In the past, recommendations developed along a study period were approved and published at the end of the period. At present, working methods are more dynamic: recommendations can be approved and published at any moment, work program prepared for a study period can be updated along the period according to needs. Work program for the 2001-2004 period makes emphasis on trac engineering for Personal Communications, IP Networks and Signalling. Three Questions (i.e. subjects for study) have been dened, one for each topic. The titles of the Questions are: Trac engineering for Personal Communications; Trac engineering for SS7- and IPbased Signalling Networks; Trac engineering for Networks Supporting IP Services. An Expert Group has been formed for each Question. The Expert Group, co-ordinated by a rapporteur, is in charge of elaborating the recommendations related to the Question.

38

CHAPTER 1. INTRODUCTION TO TELETRAFFIC ENGINEERING

1.5.8

Conclusions

An overview of the ITU trac engineering recommendations has been given. A high amount of work of worldwide specialists on trac engineering is behind this extensive set of recommendations. The whole set intends to be a valuable help for engineers in charge of designing and operating telecommunication networks. Nevertheless, the set of trac engineering recommendation can never be a complete set: new technologies, new services, new teletrac methods are continuously appearing and need to be incorporated to the recommendations. Teletrac researchers are encouraged to contribute to the preparation of new recommendations and to the revision of the old ones. The ITU recommendations can be seen as a bridge between the teletrac research activity and the daily trac engineering practice carried out by the operators. An innovative method has a greater chance to be used in practice if it appears in an ITU recommendation. It is thus worth for the researcher to contribute to the ITU in order to extend his ideas. The daily operational practice will also obtain benet from this contribution. Current working methods as for instance the extensive use of E-mail, facilitate informal cooperation of any researcher with the ITU work.

Chapter 2 Trac concepts and grade of service


The costs of a telephone system can be divided into costs which are dependent upon the number of subscribers and costs that are dependent upon the amount of trac in the system. The goal when planning a telecommunication system is to adjust the amount of equipment so that variations in the subscriber demand for calls can be satised without noticeable inconvenience while the costs of the installations are as small as possible. The equipment must be used as eciently as possible. Teletrac engineering deals with optimisation of the structure of the network and adjustment of the amount of equipment that depends upon the amount of trac. In the following some fundamental concepts are introduced and some examples are given to show how the trac behaves in real systems. All examples are from the telecommunication area.

2.1

Concept of trac and trac unit [erlang]

In teletrac theory we usually use the word trac to denote the trac intensity, i.e. trac per time unit. The term trac comes from Italian and means business. According to ITUT (1993 [34]) we have the following denition: Denition of Trac Intensity: The instantaneous trac intensity in a pool of resources is the number of busy resources at a given instant of time. The pool of resources may be a group of servers, e.g. trunk lines. The statistical moments of the trac intensity may be calculated for a given period of time T . For the mean trac 39

40 intensity we get:

CHAPTER 2. TRAFFIC CONCEPTS AND GRADE OF SERVICE

Y (T ) =

1 T

n(t) dt.
0

(2.1)

where n(t) denotes the number of occupied devices at the time t. Carried trac Y = Ac : This is called the trac carried by the group of servers during the time interval T (Fig. 2.1). In applications, the term trac intensity usually has the meaning of average trac intensity.

Number of busy channels 40


. ... . . .... .. . . . . . . .. .. ... . .. ...... . . .. .. . .... . .. . . . . ... .. .. . . . .. . .... . ... ......................... . . . ... . . . . . . .. ... . .... . . . ......... . ...... . . ..................... ........................ . ..... . .... . . . . . .... . . . . . ... . . ... .. . . . . . . . . .... .... ... ... .... .. . . ... .. . . . . . . . .. . . . ... . . . . . . .. . .... .. ... .. . . .. ... .. . . .. . . .. .. . . . ... . . . .. . .. . . .. .. .. .... .. .. .. . . ... . . . .. . ... . . . . . . .... . ........................................... . . . . . . . . .. ... ..... ..... .... ............................................ . . . .. . . ... . ... . . ... . . . . . .. . . . . . . . . .. . .. . . . .. . .. . . . . . .... . . .... .. . ..... . . . . .... . .. . . .. . .... .. . . . . . . .. .. ........................ ... ........................................... ... . ... . . .................. . .. . .. . . . .. . . . . . .. . . . .. . . . . ... . . . . .... . . . . . . .. . .. . ... .. . . . . . . . . . .. . . . . .. ....... . . . . . . . ..... .. . . .. . . . . ... . . . . ..... ... . . . .... . .. . .. . . . .. . . . ... . . . ... .. . ...... ... ... ..... . . . . . ..... . .. . . . ... . . . . . ... .. . ... . . .... ... . . .. .. . ..... . ... . . . . . .. . . . . . ...... .... . . .... . . . .. . . . . . . .. ............................................ . . . ............... .......................... .. . . . .............................................. .............................................. ... . . . .. . . . . . . . . .... ........ ... . ... . ...... .. .. .. . . .. . . . .... . ... .... .... . . . ........ .. .. . . ...... . . . . .. .. . ... . .. ... ... . .

30

n(t)

mean

20

10

................. ...... .......... .. . ... ..

T .........................................

Time

Figure 2.1: The carried trac (intensity) (= number of busy devices) as a function n(t) of time. For dimensioning purposes we use the average trac intensity during a period of time T (mean). The ITU-T recommendation also says that the unit usually used for trac intensity is erlang (symbol E). This name was given to the trac unit in 1946 by CCIF (predecessor to CCITT and to ITU-T), in honour of the Danish mathematician A. K. Erlang (1878-1929), who was the founder of trac theory in telephony. The unit is dimensionless. The total trac carried in a time period T is a trac volume, and it is measured in erlanghours (Eh). It is equal to the sum of all holding times inside the time period. According to the ISO standards the standardised unit should be erlang-seconds, but usually erlang-hours has a more natural order of size). The carried trac can never exceed the number of channels (lines). A channel can at most carry one erlang. The income is often proportional to the carried trac. Oered trac A: In theoretical models the concept oered trac is used; this is the trac which would be carried if no calls were rejected due to lack of capacity, i.e. if the number of

2.1. CONCEPT OF TRAFFIC AND TRAFFIC UNIT [ERLANG]

41

servers were unlimited. The oered trac is a theoretical value and it cannot be measured. It is only possible to estimate the oered trac from the carried trac. Theoretically we operate with two parameters: 1. call intensity , which is the mean number of calls oered per time unit, and 2. mean service time s.

The oered trac is equal to: A = s. (2.2) From this equation it is seen that the unit of trac has no dimension. This denition assumes according to the above denition that there is an unlimited number of servers. If we use the denition for a system with limited capacity we get a denition which depends upon the capacity of the system. The latter denition has been used for many years, for example in the Engset case (Chap. 8), but it is not appropriate, because the oered trac should be independent of the system. Lost or Rejected trac A : The dierence between oered trac and carried trac is equal to the rejected trac. The value of this parameter can be reduced by increasing the capacity of the system.

Example 2.1.1: Denition of trac If the call intensity is 5 calls per minute, and the mean service time is 3 minutes then the oered trac is equal to 15 erlang. The oered trac-volume during a working day of 8 hours is then 120 erlang-hours.

Example 2.1.2: Trac units Earlier other units of trac have been used. The most common which may still be seen are: = Speech-minutes 1 SM = 1/60 Eh. CCS = Hundred call seconds: 1 CCS = 1/36 Eh. This unit is based on a mean holding time of 100 seconds and can still be found, e.g. in USA. EBHC = Equated busy hour calls: 1 EBHC = 1/30 Eh. This unit is based on a mean holding time of 120 seconds. We will soon realize, that erlang is the natural unit for trac intensity because this unit is independent of the time unit chosen. SM

42

CHAPTER 2. TRAFFIC CONCEPTS AND GRADE OF SERVICE

The oered trac is a theoretical parameter used in the theoretical dimensioning formul. However, the only measurable parameter in reality is the carried trac, which often depends upon the actual system. In data transmissions systems we do not talk about service times but about transmission needs. A job can for example be a data packet of s units (e.g. bits or bytes). The capacity of the system , the data signalling speed, is measured in units per second (e.g. bits/second). Then the service time for such a job, i.e. transmission time, is s/ time units (e.g. seconds), i.e. depending on . If on the average jobs are served per time unit, then the utilisation of the system is: s = . (2.3) The observed utilisation will always be inside the interval 0 trac. 1, as it is the carried

Multi-rate trac: If we have calls occupying more than one channel, and calls of type i occupy di channels, then the oered trac expressed in number of busy channels becomes:
N

A=
i=0

i si d i ,

(2.4)

where N is number of trac types, and i and si denotes the arrival rate and mean holding time of type i. Potential trac: In planning and demand models we use the term potential trac, which would equal the oered trac if there were no limitations in the use of the phone because of economics or availability (always a free phone available).

2.2

Trac variations and the concept busy hour

The teletrac varies according to the activity in the society. The trac is generated by single sources, subscribers, who normally make telephone calls independently of each other. A investigation of the trac variations shows that it is partly of a stochastic nature partly of a deterministic nature. Fig. 2.2 shows the variation in the number of calls on a Monday morning. By comparing several days we can recognise a deterministic curve with superposed stochastic variations. During a 24 hours period the trac typically looks as shown in Fig. 2.3. The rst peak is caused by business subscribers at the beginning of the working hours in the morning, possibly calls postponed from the day before. Around 12 oclock it is lunch, and in the afternoon there is a certain activity again.

2.2. TRAFFIC VARIATIONS AND THE CONCEPT BUSY HOUR

43

Calls per minute 160


. . . . . . . . . . . .. .. .. .. .. .. .. ... .. . .. . .. .. . .. . .. . .. . . . . . .. . . .. . . . .. . . . .. . .. . .. . .. . . . . . . .. . . . ... . . . . . . . . .. . . . . . . . . . .. . . . . . . . . . . .. . . . . . . . . . . .. . . . . . . . . . . . . . . .. . . . . . . . . . . .. . . . . . .. .. .. . . . . . . . .. .. .. .. . . . .. . .. .. . . . . .. .. . . .. ... . . ... . .. .. .. .. . . . . . . .. .. .. .. .. . . . . . . . .. ... . . ... . .. .. .. .. . .. . . . . .. . . . . .. .. .. . . . .... ... . . .. .. . . . . . . . .. . . . .. .. ... . . .... ... . . ..... .. ... . . . . .. .. ... . . . . .. . . . . . . . .. . . . . ... . . .. . . .. ... .. ... . .. . . .. .. . . .. . . . . . . . .. . . . . . . .. . .. . . . .. . . . .. . . . .. .. . . .. .. . .. .. . . . .. . .. . .. . . . .. . .. . ... . . . .. .. . . . . . .. . . . . . . . . . . .. . . . . . . . . . . .. . .. . . .. .. . ... .. . .. .. . .. .. . ... .. . . .. . .. . . . .. ... . ... . . . . . . .. .. . . . . . . . . . .. . . . .. . . . ... . . . . . . . . .. .. . . . . ... . . . . .. . . . . . .. .. . . . .... ... . . . . .. . . .. . . . . ... . .. . .. . . . .. ... . . ... .... .. . . . . . . . . .. . . . . . . . . . ... .. . .. . . .. . . . . . . .... .. . .. .. .. . .. . .... ... .. . . . . .. . . .. . . . . .. . .. . .. . .. ... . . . . ... . .... .. . . . . . . . . . .. . . . ...... . . . . . . .. . . . . . . .. . ... .. .. .. . .. . .. . . . . .... .. . . . . .. . . .. ......... . . . . . . . . .. . .. . . . . . .. . . . . .. . . . . . .. . . . . ..... . . .. . . . . . .. . . .. . . . . .. .... ..... .. . . . . . .. . . . . .. . .. .. .. . . .. .. . . .. . . . . .. . . . . .. . . . . . . . .. . .. . .. . . . .. . . .. .. . .. . . . . . . . . . .. . . . . .. .. . . . . . . . . . ... . .. ... . . ... . .. .. . .. . .. . . .. . . .. . . . . . . . . . .. . .. . .. . . . .. . .. . . . . . . . .. . . .. ... . . ... . .. .. . .. .. ... . .. ... . . ... . .. ... .. . .. .. . . . .. . . . . . .. . ... . .. . .. . . . . . .. . .. . . . . . . . ... . .. . . . .. . . . .. .. .. .. . . . . . .. . . . .. . . . .. .. ... .. . .. . .. . ... . .. . .. .. . .. . .. .. .. . . . . .. . . . . .. . . . ... ... ... .. . . . . . . . ... .. .. . .. .. .. . .... .. .. . .. . .. .. .. . . . . .. . . . . . .. . .. . . . . . .. . .. .. . . . . .. . . . .. .... . .. . .. .. .. ... ... . .. .. ..... . . .. . . . . .. . . .. . . . . . . . . .. . ... . . .. . .. . ... . . .. .. . . . ... . . .. . . .... . .... .. .. .. . . . . . .. .. . . .. . . . . . . . . .. . . . . .. . . . . . . .. . . .. . . .. .... .. .. .. .. . .. . . . . . . . .. . . ... . . . . .. .. .. . . . . . . .. .. .... . . . . . . . . .. . .. . . .. . . . . . . . . .. ... . .. .. . ... . .. . . . .. . .. . . . . . . .. . . . . . . . ... . .. ... .. . . . . ... . . . ..... .... .. .. . .. . .. . .. . . . . .. . .. . . .. .. . .. .. . . . .. .. ... . . . . ... .. . . .. . . .. .. . . .. . . . . . .. . .. .. .. . . . . . . . . .... . . . .. .. . . .. .. . .... .. . . . . . .. . .. . . ... .. . . .. . .. . . . .. .... . . . . .. .. . . .. .. . .. . ... . . . . . . .. . . ... . . . . .. .. . .. . . . . . . .. .. . . .. . . . . .. . . ... . . . . . . . . . . . .. .... . . . . . .. . . . . ... . .. . . . . .. . . . .. . .. . . . . .. .... . . . . .. .. . . . . . . ... ... . . ... .. . . .. . .. . .. . . . . . . . . .. . .. .... .. .. ... . . . . . . ... . . .. . . . . .. . . .. . . .. . .. . .. . . .. .. . .. ..... .. .. ... .. . . ... ... . . .. . .. . .. . . ... ..... .. . . . ... .. . . ... ... . .. . . .. . . . .. . .. . . . ... . . .. . . . .. . . . .. . ... . . . .. .. . .... ... ... . . .. . . .. . . . .. . . .. . ... ..... . ... .. . .. .. . . .. . ... .. .. . .. . .. . ... . . . . . .. . . .. .. . . . . ... .. .. . .. . . . . . .. ..... . . . .. . . . . . . .. . . .... .. .. .. . . . . . . . . .. . .. . . . .. . . . . . .. . .. . . . .. . .. .. .. . . . . .. . . .. . .. .. . .. . . . .. .. .. . . . . .. .. . . . .. . .. .. . . . .. . .. .. . .. ... . . . .. . .. . ... .. . . . .... . .. . . .. . . . . ... . .. . . . . ... . . . ... . . . ... . . . .... . . .. . . . . . . . . . . . . . . .. . . . ... . . . .... . . .. . . . .. . . . . . .. . . .. . . .. . . . . . .. . . . . . .. . . .. . . .. .. .... . .. . . . . . . . . .. . .. . . . . .. .. .. ... . .. . ... . .. . . . . . . . . .. . . . .. . . . .. . . . . .. . . . ... . .. ... .. .. ... .. .. .. ... . . . . . . . .. . . . . .. . . . ... . . . . .. .. . . .. .. .. .. . . . . . ..... . . . .. . . . . . . . . . . . . . . . .... . .. . . . . . . .. . ..... . . . .. . . . .. .. . . . .. ... . ... . . . . . . .. .. . . . .. .. . . . .. . . . . .. .. . . . .. .. . . .. .. . . . . .. . . .. .. . .. .. .. . . .. . . . . . . . . . .. .. .. . . ..... . . . . . . . . . . .. . . .. . .. . . . ..... .. . . .. . . .... .. . .. . .. . . .. . . .... .. . . .. . .. . . . . .. . . . . . . .. . .. . . . .. . . .. . . . . . . . . . . . . .

120

80

40

0 8 9 10 11 12 13 Time of day [hour]

Figure 2.2: Number of calls per minute to a switching centre a Monday morning. The regular 24-hour variations are superposed by stochastic variations. (Iversen, 1973 [35]).

100 80 60 40 20 0

Calls per minute

12

16

20 24 Time of day [hour]

Figure 2.3: The mean number of calls per minute to a switching centre taken as an average for periods of 15 minutes during 10 working days (Monday Friday). At the time of the measurements there were no reduced rates outside working hours (Iversen, 1973 [35]).

44

CHAPTER 2. TRAFFIC CONCEPTS AND GRADE OF SERVICE

Around 19 oclock there is a new peak caused by private calls and a possible reduction in rates after 19.30. The mutual size of the peaks depends among other thing upon whether the exchange is located in a typical residential area or in a business area. They also depend upon which type of trac we look at. If we consider the trac between Europa and for USA most calls takes place in the late afternoon because of the time dierence. The variations can further be split up into variation in call intensity and variation in service time. Fig. 2.4 shows variations in the mean service time for occupation times of trunk lines during 24 hours. During business hours it is constant, just below 3 minutes. In the evening it is more than 4 minutes and during the night very small, about one minute. Busy Hour: The highest trac does not occur at same time every day. We dene the concept time consistent busy hour, TCBH as those 60 minutes (determined with an accuracy of 15 minutes) which during a long period on the average has the highest trac. It may therefore some days happen that the trac during the busiest hour is larger than the time consistent busy hour, but on the average over several days, the busy hour trac will be the largest. We also distinguish between busy hour for the total telecommunication system, an exchange, and for a single group of servers, e.g. a trunk group. Certain trunk groups may have a busy hour outside the busy hour for the exchange (for example trunk groups for calls to the USA). In practice, for measurements of trac, dimensioning, and other aspects it is an advantage to have a predetermined welldened busy hour. The deterministic variations in teletrac can be divided into: 24 hours variation (Fig. 2.3 and 2.4). Weekly variations (Fig. 2.5). Normally the highest trac is on Monday, then Friday, Tuesday, Wednesday and Thursday. Saturday and especially Sunday has a very low trac level. A good rule of thumb is that the 24 hour trac is equal to 8 times the busy hour trac (Fig. 2.5), i.e. only one third of capacity in the telephone system is utilised. This is the reason for the reduced rates outside the busy hours. Variation during a year. There is a high trac in the beginning of a month, after a festival season, and after quarterly period begins. If Easter is around the 1st of April then we observe a very high trac just after the holidays. The trac increases year by year due to the development of technology and economics in the society.

Above we have considered traditional voice trac. Other services and trac types have other patterns of variation. In Fig. 2.6 we show the variation in the number of calls per 15 minutes

2.3. THE BLOCKING CONCEPT 300 270 240 210 180 150 120 90 60 30 0 0 4 8 12 16 20 24 Time of day [hour] Mean holding time [s]

45

Figure 2.4: Mean holding time for trunk lines as a function of time of day. (Iversen, 1973 [35]). The measurements exclude local calls. to a modem pool for dial-up Internet calls. The mean holding time as a function of the time of day is shown in Fig. 2.7. Cellular mobile telephony has a dierent prole with maximum late in the afternoon, and the mean holding time is shorter than for wire-line calls. By integrating various forms of trac in the same network we may therefore obtain a higher utilisation of the resources.

2.3

The blocking concept

The telephone system is not dimensioned so that all subscribers can be connected at the same time. Several subscribers are sharing the expensive equipment of the exchanges. The concentration takes place from the subscriber toward the exchange. The equipment which is separate for each subscriber should be made as cheap as possible. In general we expect that about 58 % of the subscribers should be able to make calls at the same time in busy hour (each phone is used 1016 % of the time). For international calls less than 1 % of the subscribers are making calls simultaneously. Thus we exploit statistical

46
. . . . . . .

CHAPTER 2. TRAFFIC CONCEPTS AND GRADE OF SERVICE


. . . . . . . . . . . . . . .

60000

Number of calls per 24 hours


. . . . . . . . . . . . . . . . . . . . . . . . . . . . . . . . . . . . . . . . . . . . . . . . . . . . . . . . . . . . . . . . . . . . . . . . . . . . . . . . . . . . . . .

Number of calls per Busy Hour


. . . . . . . . . . . . . . . . . . . . . . .

7500

50000

. . . .

. . . . . . . . . . . . . . . . . . . .

. . . . . . . . . . . . . . . . . . . .

. . . . . . . . . . . . . . . . . . . .

. . . . . . . . . . . . . . . . . . . .

. . . . . . . . . . . . . . . . . . . . . . . . . . . . . . . . . . . .

. . . . . . . . . . . . . . . . . . . . .

. . . . . . . . . . . . . . . . . . . . . . . . . . . . . . . . . . . . . . . . .

6250
. . . . . . . . . . . . . . . . . . . . . . . . . . . . . . . . . . . . . . . . . . . . . . . . . . . . . . . . . . . . . . . . . . . . . . . . . . . . . . . . . . . . . . . . . . . . . . . . . . . . . . . . . . . . . . . . . . .

40000

. . . . .

5000

30000

. . . .

3750

20000
. . . . . . . .

. . . . . . . . .

2500

10000

. . . .

1250

Sun Mon Tue Wed Thu Fri Sat Sun Mon Tue Wed Thu Fri Sat

Figure 2.5: Number of calls per 24 hours to a switching centre (left scale). The number of calls during busy hour is shown for comparison at the right scale. We notice that the 24hour trac is approximately 8 times the busy hour trac. This factor is called the trac concentration (Iversen, 1973 [35]). multiplexing advantages. Every subscriber should feel that he has unrestricted access to all resources of the telecommunication system even if he is sharing it with many others. The amount of equipment is limited for economical reasons and it is therefore possible that a subscriber cannot establish a call, but has to wait or be blocked (the subscriber for example gets busy tone and has to make a new call attempt). Both are inconvenient to the subscriber. Depending on how the system operates we distinguish between losssystems (e.g. trunk groups) and waiting time systems (e.g. common control units and computer systems) or a mixture of these if the number of waiting positions (buer) is limited. The inconvenience in losssystems due to insucient equipment can be expressed in three ways (network performance measures):

Call congestion B:

The fraction of all call attempts which observes all servers busy (the user-perceived quality-of-service, the nuisance the subscriber experiences).

2.3. THE BLOCKING CONCEPT


14000

47

12000

10000

8000 arrivals 6000 4000 2000

0 0 2 4 6 8 10 12 14 hour of day 16 18 20 22 24

Figure 2.6: Number of calls per 15 minutes to a modem pool of Tele Danmark Internet. Tuesday 1999.01.19.

Time congestion E:

The fraction of time when all servers are busy. Time congestion can for example be measured at the exchange (= virtual congestion).

Trac congestion C: The fraction of the oered trac that is not carried, possibly despite several attempts.

These quantitative measures can forexample be used to establish dimensioning standards for trunk groups. At small congestion values it is possible with a good approximation to handle congestion in the dierent part of the system as mutually independent. The congestion for a certain route is then approximately equal to the sum of the congestion in each link of the route. During the busy hour we normally allow a congestion of a few percentage between two subscribers. The systems cannot manage every situation without inconvenience for the subscribers. The purpose of teletrac theory is to nd relations between quality of service and cost of equipment. The existing equipment should be able to work at maximum capacity during abnormal trac situations (e.g. a burst of phone calls), i.e. the equipment should keep working and make useful connections.

48
1200

CHAPTER 2. TRAFFIC CONCEPTS AND GRADE OF SERVICE

1000

800 service time (sec)

600

400

200

0 0 2 4 6 8 10 12 14 hour of day 16 18 20 22 24

Figure 2.7: Mean holding time in seconds as a function of time of day for calls arriving inside the period considered. Tele Danmark Internet. Tuesday 1999.01.19.

The inconvenience in delaysystems (queueing systems) is measured as a waiting time. Not only the mean waiting time is of interest but also the distribution of the waiting time. It could be that a small delay do not mean any inconvenience, so there may not be a linear relation between inconvenience and waiting time. In telephone systems we often dene an upper limit for the acceptable waiting time. If this limit exceeded then a time-out of the connection will take place (enforced disconnection).

2.4

Trac generation and subscribers reaction

If Subscriber A want to speak to Subscriber B this will either result in a successful call or a failed callattempt. In the latter case A may repeat the call attempt later and thus initiate a series of several callattempts which fail. Call statistics typically looks as shown in Table 2.1, where we have grouped the errors into a few typical classes. We notice that the only error which can be directly inuenced by the operator is technical errors and blocking, and this class usually is small, a few percentages during the Busy Hour. Furthermore, we notice that the number of calls which experience Bbusy depends on the number of A-errors and technical errors & blocking. Therefore, the statistics in Table 2.1 are misleading. To

2.4. TRAFFIC GENERATION AND SUBSCRIBERS REACTION Outcome A-error: Blocking and technical errors: B no answer before A hangs up: B-busy: B-answer = conversation: No conversation: Icountry Dcountry 15 5 10 10 60 % % % % % 20 35 5 20 20 % % % % %

49

40 %

80 %

Table 2.1: Typical outcome of a large number of call attempts during Busy Hour for Industrialised countries, respectively Developing countries.
No answer A A-error Tech. errors and Blocking B-busy B-answer

pn pb pa

ps

Figure 2.8: When calculating the probabilities of events for a certain number of call attempts we have to consider the conditional probabilities. obtain the relevant probabilities, which are shown in Fig. 2.8, we shall only consider the calls arriving at the considered stage when calculating probabilities. Applying the notation in Fig. 2.8 we nd the following probabilities for a call attempts (assuming independence): p{A-error} = pe p{Congestion & tech. errors} = (1 pe ) ps p{Bno answer} = (1 pe ) (1 ps ) pn p{Bbusy} = (1 pe ) (1 ps ) pb p{Banswer} = (1 pe ) (1 ps ) pa (2.5) (2.6) (2.7) (2.8) (2.9)

Using the numbers from Table 2.1 we nd the gures shown in Table 2.2. From this we notice that even if the A-subscriber behaves correctly and the telephone system is perfect, then only 75 %, respectively 45 % of the call attempts result in a conversation. We distinguish between the service time which includes the time from the instant a server is occupied until the server becomes idle again (e.g. both call set-up, duration of the conversation, and termination of the call), and conversation duration, which is the time period where A talks with B. Because of failed callattempts the mean service time is often less than the

50

CHAPTER 2. TRAFFIC CONCEPTS AND GRADE OF SERVICE

I country pe = ps = pn = pb = pa =
15 100 5 85 10 80 10 80 60 80

D country = 15% pe = = 6% ps = = = 13% pn = = 13% pb = 75% pa =


20 100 35 80 5 45 20 45 20 45

= 20% = 44% = 11% = 44% = 44%

Table 2.2: The relevant probabilities for the individual outcomes of the call attempts calculated for Table 2.1 mean call duration if we include all callattempts. Fig. 2.9 shows an example with observed holding times.
Example 2.4.1: Mean holding times We assume that the mean holding time of calls which are interrupted before B-answer (A-error, congestion, technical errors) is 20 seconds and that the mean holding time for calls arriving at the called party (B-subscriber) (no answer, B-busy, B-answer) is 180 seconds. The mean holding time at the A-subscriber then becomes by using the gures in Table 2.1: I country: D country: ma = ma = 20 80 20 + 180 = 148 seconds 100 100

55 45 20 + 180 = 92 seconds 100 100 We thus notice that the mean holding time increases from 148s, respectively 92s, at the A-subscriber to 180s at the B-subscriber. If one call intent implies more repeated call attempts (cf. Example 2.4), then the carried trac may become larger than the oered trac.

If we know the mean service time of the individual phases of a call attempt, then we can calculate the proportion of the call attempts which are lost during the individual phases. This can be exploited to analyse electro-mechanical systems by using SPC-systems to collect data. Each callattempt loads the controlling groups in the exchange (e.g. a computer or a control unit) with an almost constant load whereas the load of the network is proportional to the duration of the call. Because of this many failed callattempts are able to overload the control devices while free capacity is still available in the network. Repeated callattempts are not necessarily caused by errors in the telephone-system. They can also be caused by e.g. a busy Bsubscriber. This problem were treated for the rst time by Fr. Johannsen in Busy published in 1908 (Johannsen, 1908 [52]) . Fig. 2.10 and Fig. 2.11 show some examples from measurements of subscriber behaviour.

2.4. TRAFFIC GENERATION AND SUBSCRIBERS REACTION

51

Number of observations 135164 observations = 142.86 = 3.83 Exponential Hyperexponential 10


3

10

10

10

10

15

20

25

30

35 40 Minutes

Figure 2.9: Frequency function for holding times of trunks in a local switching centre.

52

CHAPTER 2. TRAFFIC CONCEPTS AND GRADE OF SERVICE

Studies of the subscribers response to for example busy tone is of vital importance for the dimensioning of telephone systems. In fact, humanfactors (= subscriberbehaviour) is a part of the teletrac theory which is of great interest. During Busy Hour = 10 16 % of the subscribers are busy using the line for incoming or outgoing calls. Therefore, we would expect that % of the call attempts experience Bbusy. This is, however, wrong, because the subscribers have dierent trac levels. Some subscribers receive no incoming call attempts, whereas others receive more than the average. In fact, it is so that the most busy subscribers on the average receive most call attempts. A-subscribers have an inclination to choose the most busy B-subscribers, and in practice we observe that the probability of B-busy is about 4 , if we take no measures. For residential subscribers it is dicult to improve the situation. But for large business subscribers having a PAX (= PABX) (Private Automatic eXchange) with a group-number a sucient number of lines will eliminate B-busy. Therefore, in industrialised countries the total probability of B-busy becomes of the same order of size as (Table 2.1). For Dcountries the trac is more focused towards individual numbers and often the business subscribers dont benet from group numbering, and therefore we observe a high probability of B-busy (4050 %). At the Ordrup measurements approximately 4% of the call were repeated callattempts. If a subscriber experience blocking or Bbusy there is 70% probability that the call is repeated within an hour. See Table 2.3. Number of observations Attempt no. Success 1 2 3 4 5 >5 Total 56.935 3.252 925 293 139 134 61.678 Continue 75.389 7.512 2.378 951 476 248 Give up 10.942 1.882 502 182 89 114 13.711 p{success} 0.76 0.43 0.39 0.31 0.29 Persistence 0.41 0.56 0.66 0.72 0.74

Table 2.3: An observed sequence of repeated callattempts (national calls, Ordrup measurements). The probability of success decreases with the number of callattempts, while the persistence increases. Here a repeated callattempt is a call repeated to the same Bsubscriber within one hour. A classical example of the importance of the subscribers reaction was seen when Valby gasworks (in Copenhagen) exploded in the mid sixties. The subscribers in Copenhagen generated a lot of callattempts and occupied the controlling devices in the exchanges in the area of Copenhagen. Then subscribers from Esbjerg (western part of Denmark) phoning to Copenhagen had to wait because the dialled numbers could not be transferred to Copenhagen immediately. Therefore the equipment in Esbjerg was kept busy by waiting, and subscribers

2.4. TRAFFIC GENERATION AND SUBSCRIBERS REACTION making local calls in Esbjerg could not complete the call attempts.

53

This is an example of how a overload situation spreads like a chain reaction throughout the network. The more tight a network has been dimensioned, the more likely it is that a chain reaction will occur. An exchange should always be constructed so that it keeps working with full capacity during overload situations. In a modern exchange we have the possibility of giving priority to a group of subscribers in an emergency situation, e.g. doctors and police (preferential trac). In computer systems similar conditions will inuence the performance. For example, if it is dicult to get a free entry to a terminalsystem, the user will be disposed not to log o, but keep the terminal, i.e. increase the service time. If a system works as a waitingtime system, then the mean waiting time will increase with the third order of the mean service time (Chap. 13). Under these conditions the system will be saturated very fast, i.e. be overloaded. In countries with an overloaded telecommunication network (e.g. developing countries) a big percentage of the callattempts will be repeated callattempts.
10
5

10

n = 138543

10

10

10

10

30

60

90

120

150

180 Seconds

Figure 2.10: Histogram for the time interval from occupation of register (dial tone) to B answer for completed calls. The mean value is 13.60 s.

Example 2.4.2: Repeated call attempt This is an example of a simple model of repeated call attempts. Let us introduce the following notation: b = persistence (2.10) B = p{non-completion} (2.11)

54

CHAPTER 2. TRAFFIC CONCEPTS AND GRADE OF SERVICE

The persistence b is the probability that an unsuccessful call attempt is repeated, and p{completion} = (1 B) is the probability that the B-subscriber (called party) answers. For one call intent we get the following history: We get the following probabilities for one call intent:

Attempt No. 0 1 2 3 4 ... Total

p{B-answer} (1 B) (1 B) (B b) (1 B) (B b)2 (1 B) (B b)3 ... (1 B) (1 B b)

p{Continue} 1 Bb (B b)2 (B b)3 (B b)4 ... 1 (1 B b)

p{Give up} B (1 b) B (1 b) (B b) B (1 b) (B b)2 B (1 b) (B b)3 ... B (1 b) (1 B b)

Table 2.4: A single call intent results in a series of call attempts. The distribution of the number of attempts is geometrically distributed.
p{completion} = (1 B) (1 B b) B (1 b) (1 B b) 1 (1 B b) (2.12)

p{non-completion} =

(2.13)

No. of call attempts per call intent = Let us assume the following mean holding times: sc = mean holding time of completed calls sn = 0 = mean holding time of non-completed calls

(2.14)

Then we get the following relations between the trac carried Y and the trac oered A: Y =A A=Y 1B 1Bb 1Bb 1B (2.15)

(2.16)

This is similar to the result given in ITUT Rec. E.502.

In practice, the persistence b and the probability of completion 1 B will depend on the number of times the call has been repeated (cf. Table 2.3). If the unsuccessful calls have

2.5. INTRODUCTION TO GRADE-OF-SERVICE = GOS

55

a positive mean holding time, then the carried trac may become larger than the oered trac.
600

500

n = 7653

400

300

200

100

60

120

180

240

300 Seconds

Figure 2.11: Histogram for all call attempts repeated within 5 minutes, when the called party is busy.

2.5

Introduction to Grade-of-Service = GoS

The following section is based on (Veir, 2001 [98]). A network operator must decide what services the network should deliver to the end user and the level of service quality that the user should experience. This is true for any telecommunications network, whether it is circuitor packet-switched, wired or wireless, optical or copper-based, and it is independent of the transmission technology applied. Further decisions to be made may include the type and layout of the network infrastructure for supporting the services, and the choice of techniques to be used for handling the information transport. These further decisions may be dierent, depending on whether the operator is already present in the market, or is starting service from a greeneld situation (i.e. a situation where there is no legacy network in place to consider). As for the Quality of Service (QoS) concept, it is dened in the ITU-T Recommendation E.800 as: The collective eect of service performance, which determine the degree of satisfaction of a user of the service. The QoS consists of a set of parameters that pertain to the trac performance of the network, but in addition to this, the QoS also includes a lot of other concepts. They can be summarised as:

56

CHAPTER 2. TRAFFIC CONCEPTS AND GRADE OF SERVICE service support performance service operability performance serveability performance and service security performance

The detailed denitions of these terms are given in the E.800. The better service quality an operator chooses to oer to the end user, the better is the chance to win customers and to keep current customers. But a better service quality also means that the network will become more expensive to install and this, normally, also has a bearing to the price of the service. The choice of a particular service quality therefore depends on political decisions by the operator and will not be treated further here. When the quality decision is in place the planning of the network proper can start. This includes the decision of a transport network technology and its topology as well as reliability aspects in case one or more network elements become malfunctioning. It is also at this stage where the routing strategy has to be determined. This is the point in time where it is needed to consider the Grade of Service (GoS). This is dened in the ITU-T Recommendation E.600 as: A number of trac engineering variables to provide a measure of adequacy of a group of ressources under specied conditions. These grade of service variables may be probability of loss, dial tone delay, etc. To this denition the recommendation furthermore supplies the following notes: The parameter values assigned for grade of service variables are called grade of service standards. The values of grade of service parameters achieved under actual conditions are called grade of service results. The key point to solve in the determination of the GoS standards is to apportion individual values to each network element in such a way that the target end-to-end QoS is obtained.

2.5.1

Comparison of GoS and QoS

It is not an easy task to nd the GoS standards needed to support a certain QoS. This is due to the fact that the GoS and QoS concepts have dierent viewpoints. While the QoS views the situation from the customers point of view, the GoS takes the network point of view. We illustrate this by the following example:

Example 2.5.1: Say we want to x the end to end call blocking probability at 1 % in a telephone network. A

2.5. INTRODUCTION TO GRADE-OF-SERVICE = GOS

57

customer will interpret this quantity to mean that he will be able to reach his destinations in 99 out of 100 cases on the average. Fixing this design target, the operator apportioned a certain blocking probability to each of the network elements, which a reference call could meet. In order to make sure that the target is met, the network has to be monitored. But this monitoring normally takes place all over the network and it can only be ensured that the network on the average can meet the target values. If we consider a particular access line its GoS target may well be exceeded, but the average for all access lines does indeed meet the target.

GoS pertains to parameters that can be veried through network performance (the ability of a network or network portion to provide the functions related to communications between users) and the parameters hold only on average for the network. Even if we restrain ourselves only to consider the part of the QoS that is trac related, the example illustrates, that even if the GoS target is fullled this need not be the case for the QoS.

2.5.2

Special features of QoS

Due to the dierent views taken by GoS and QoS a solution to take care of the problem has been proposed. This solution is called a service level agreement (SLA). This is really a contract between a user and a network operator. In this contract it is dened what the parameters in question really mean. It is supposed to be done in such a way, that it will be understood in the same manner by the customer and the network operator. Furthermore the SLA denes, what is to happen in case the terms of the contract are violated. Some operators have chosen to issue an SLA for all customer relationships they have (at least in principle), while others only do it for big customers, who know what the terms in the SLA really mean.

2.5.3

Network performance

As mentioned above the network performance concerns the ability of a network or network portion to provide the functions related to communications between users. In order to establish how a certain network performs, it is necessary to perform measurements and the measurements have to cover all the aspects of the performance parameters (i.e. tracability, dependability, transmission and charging). Furthermore, the network performance aspects in the GoS concept pertains only to the factors related to tracability performance in the QoS terminology. But in the QoS world network performance also includes the following concepts: dependability, transmission performance, and charging correctness.

58

CHAPTER 2. TRAFFIC CONCEPTS AND GRADE OF SERVICE

It is not enough just to perform the measurements. It is also necessary to have an organisation that can do the proper surveillance and can take appropriate action when problems arise. As the network complexity keeps growing so does the number of parameters needed to consider. This means that automated tools will be required in order to make it easier to get an overview of the most important parameters to consider.

2.5.4

Reference congurations

In order to obtain an overview of the network under consideration, it is often useful to produce a so-called reference conguration. This consists of one or more simplied drawing(s) of the path a call (or connection) can take in the network including appropriate reference points, where the interfaces between entities are dened. In some cases the reference points dene an interface between two operators, and it is therefore important to watch carefully what happens at this point. From a GoS perspective the importance of the reference conguration is the partitioning of the GoS as described below. Consider a telephone network with terminals, subscriber switches and transit switches. In the example we ignore the signalling network. Suppose the call can be routed in one of three ways: 1. terminal subscriber switch terminal This is drawn as a reference conguration shown in Fig. 2.12.

S
Ref point A Ref point A

Figure 2.12: Reference conguration for case 1. 2. terminal subscriber switch transit switch subscriber switch terminal This is drawn as a reference conguration shown in Fig. 2.13.

S
Ref point A Ref point B

T
Ref point B

S
Ref point A

Figure 2.13: Reference conguration for case 2.

2.5. INTRODUCTION TO GRADE-OF-SERVICE = GOS

59

3. terminalsubscriber switchtransit switchtransit switchsubscriber switchterminal This is drawn as a reference conguration shown in Fig. 2.14.

S
Ref point A Ref point B

T
Ref point C

T
Ref point B

S
Ref point A

Figure 2.14: Reference conguration for case 3.

Based on a given set of QoS requirements, a set of GoS parameters are selected and dened on an end-to-end basis within the network boundary, for each major service category provided by a network. The selected GoS parameters are specied in such a way that the GoS can be derived at well-dened reference points, i.e. trac signicant points. This is to allow the partitioning of end-to-end GoS objectives to obtain the GoS objectives for each network stage or component, on the basis of some well-dened reference connections. As dened in Recommendation E.600, for trac engineering purposes, a connection is an association of resources providing means for communication between two or more devices in, or attached to, a telecommunication network. There can be dierent types of connections as the number and types of resources in a connection may vary. Therefore, the concept of a reference connection is used to identify representative cases of the dierent types of connections without involving the specics of their actual realizations by dierent physical means. Typically, dierent network segments are involved in the path of a connection. For example, a connection may be local, national, or international. The purposes of reference connections are for clarifying and specifying trac performance issues at various interfaces between different network domains. Each domain may consist of one or more service provider networks. Recommendation I.380/Y.1540 denes performance parameters for IP packet transfer; its companion Draft Recommendation Y.1541 species the corresponding allocations and performance objectives. Recommendation E.651 species reference connections for IP-access networks. Other reference connections are to be specied. From the QoS objectives, a set of end-to-end GoS parameters and their objectives for dierent reference connections are derived. For example, end-to-end connection blocking probability and end-to-end packet transfer delay may be relevant GoS parameters. The GoS objectives should be specied with reference to trac load conditions, such as under normal and high load conditions. The end-to-end GoS objectives are then apportioned to individual resource components of the reference connections for dimensioning purposes. In an operational network, to ensure that the GoS objectives have been met, performance measurements and performance monitoring are required.

60

CHAPTER 2. TRAFFIC CONCEPTS AND GRADE OF SERVICE

In IP-based networks, performance allocation is usually done on a cloud, i.e. the set of routers and links under a single (or collaborative) jurisdictional responsibility, such as an Internet Service Provider, ISP. A cloud is connected to another cloud by a link, i.e. a gateway router in one cloud is connected via a link to a gateway router in another cloud. End-to-end communication between hosts is conducted on a path consisting of a sequence of clouds and interconnecting links. Such a sequence is referred to as a hypothetical reference path for performance allocation purposes.

Chapter 3 Probability Theory and Statistics


All time intervals we consider are non-negative, and therefore they can be expressed by non-negative random variables. A random variable is also called a variate. Time intervals of interests are, for example, service times, duration of congestion (blocking periods, busy periods), waiting times, holding times, CPU-busy times, inter-arrival times, etc. We denote these time durations as lifetimes and their distribution functions as time distributions. In this chapter we review the basic theory of probability and statistics relevant to teletrac theory.

3.1

Distribution functions

A time interval can be described by a random variable T that is characterised by a distribution function F (t):
t

F (t) =
0

dF (u)

for 0 t < , for t < 0 .

(3.1)

F (t) = 0

In (3.1) we integrate from 0 to keep record of a possible discontinuity at t = 0. When we consider waiting time systems, there is often a positive probability to have waiting times equal to zero, i.e. F (0) = 0. On the other hand, when we look at the inter-arrival times, we usually assume F (0) = 0 (Sec. 5.2.3). The probability that the duration of a time interval is less than or equal to t becomes: p(T t) = F (t) . 61

62

CHAPTER 3. PROBABILITY THEORY AND STATISTICS

Sometimes it is easier to consider the complementary distribution function: F c (t) = 1 F (t) . This is also called the survival distribution function. We often assume that F (t) is dierentiable and that the following density function f (t) exists: dF (t) = f (t) dt = p{t < T t + dt}, t 0. (3.2)

Usually, we assume that the service time is independent of the arrival process and that a service time is independent of other service times. Analytically, many calculations can be carried out for any time distribution. In general, we always assume that the mean value exists.

3.1.1

Characterisation of distributions

Time distributions which only assume positive arguments possess some advantageous properties. For the ith non-central moment, which we usually denote the ith moment, it may be shown that Palms identity is valid: E{T i } = mi =
0

ti f (t) dt =

i ti1 {1 F (t)} dt ,

i = 1, 2, . . . . .

(3.3)

Palms identity (3.3), which is valid for life-time distributions (only dened for non-negative arguments), was rst proved in (Palm, 1943,[79]) as follows.
t=0

it

i1

{1 F (t)} dt =

t=0

it

i1

f (x) dx
x=t

dt

x=t x=t x

=
t=0

i ti1 f (x) dx dt dti f (x) dx dti f (x) dx

=
t=0

=
x=0 t=0

=
x=0

xi f (x) dx

= mi . The order of integration can be inverted because the integrand is non-negative. Thus we have proved (3.3). The following simplied proof is correct because we assume that the moments

3.1. DISTRIBUTION FUNCTIONS exist:

63

mi =
t=0

ti f (t) dt

=
i

ti d {1 F (t)} | 0

t=0

= t {1 F (t)}

+
t=0

{1 F (t)} dti

=
t=0

i ti1 {1 F (t)} dt

q.e.d.

Especially, we have the rst two moments under the assumption that they exist:

m1 = m2 =

t f (t) dt =
0 0 0

{1 F (t)}dt , 2t {1 F (t)} dt .

(3.4)

t2 f (t) dt =
0

(3.5)

The mean value (expectation) is the rst moment and often we leave out the index: m = m1 = E{T } . The ith central moment is dened as: E{(T m1 )i } = The variance is the 2nd central moment: 2 = E{(T m1 )2 } . It is easy to show that: 2 = m2 m2 1 m2 = 2 + m2 . 1 A distribution is normally uniquely dened by all its moments. A normalised measure for the irregularity (dispersion) of a distribution is the coecient of variation. It is dened as the ratio between the standard deviation and the mean value: CV = Coecient of Variation = . (3.9) m1 This quantity is dimensionless, and we shall later apply it to characterise discrete distributions (state probabilities). Another measure of irregularity is Palms form factor , which is dened as follows: 2 m2 = 2 =1+ 1. (3.10) m1 m1 or (3.8)
0

(3.6)

(t m1 )i f (t) dt .

(3.7)

64

CHAPTER 3. PROBABILITY THEORY AND STATISTICS

The form factor as well as (/m1 ) are independent of the choice of time scale, and they will appear in many formul in the following. The larger a form factor, the more irregular is the time distribution, and the larger will for example the mean waiting time in a waiting time system be. The form factor takes the minimum value equal to one for constant time intervals ( = 0). To estimate a distribution from observations, we are often satised by knowing the rst two moments (m and or ) as higher order moments require extremely many observations to obtain reliable estimates. Time distributions can also be characterised in other ways. We consider some important ones below.
Example 3.1.1: Exponential distribution For the exponential distribution we get:

m2 =

t2 et dt =

t=0

2 t et dt =

t=0

2 . 2

It may be surprising that the two integrals are identical. The two integrands can, apart from a constant, be transformed to an Erlang3, respectively an Erlang2, density function (4.8), which has the total probability mass one: m2 = 2 2
t=0

(t)2 t 2 dt = 2 e 2

t=0

t et dt =

2 . 2

Example 3.1.2: Constant time interval For a constant time interval of duration h we have: mi = hi .

3.1.2

Residual lifetime

We wish to nd the distribution of the residual life time, given that a certain age x 0 has already been obtained. The conditional distribution F (t + x|x) is dened as follows, assuming p{T > x} > 0 and t 0: p{(T > t + x) (T > x)} p{T > t + x|T > x} = p{T > x} = p{T > t + x} p{T > x} 1 F (t + x) , 1 F (x)

3.1. DISTRIBUTION FUNCTIONS and thus: F (t + x|x) = p{T t + x | T > x} = F (t + x) F (x) , 1 F (x) f (t + x) . 1 F (x)

65

(3.11)

f (t + x|x) =

(3.12)

Fig. 3.1 illustrates these calculations graphically. The mean value m1,r of the residual lifetime can be written as (3.4): m1,r (x) = 1 1 F (x)
t=0

{1 F (t + x)} dt,

x 0.

(3.13)

The Death rate at time x, i.e. the probability, that the considered lifetime terminates within an interval (x, x + dx), under the condition that age x has been achieved, is obtained from (3.11) by letting t = dx: (x) dx = F (x + dx) F (x) 1 F (x) dF (x) . 1 F (x) (3.14)

The conditional density function (x) is also called the hazard function. If this function is given, then F (x) may be obtained as the solution to the following dierential equation: dF (x) + (x) F (x) = (x) , dx which has the following solution (assuming F (0) = 0):
t

(3.15)

F (t) = 1 exp f (t) = (t) exp

(u) du
0 t

(3.16)

(u) du
0

(3.17)

The death rate (t) is constant if and only if the lifetime is exponentially distributed (Chap. 4). This is a fundamental characteristic of the exponential distribution which is called the Markovian property (lack of memory (age)): The probability of terminating is independent of the actual age (history) (Sec. 4.1).

66
. . . .. .. .. . . . . . . . . . . . . .

CHAPTER 3. PROBABILITY THEORY AND STATISTICS f(t)

0.20 0.15 0.10 0.05 0

................... ................... .... .... ..... .... ... . . ... . ... ... ... ... ... ... . ... ... ... . ... . ... . ... . .. .. ... ... .. .. ... . . ... . . .. ... .. ... . . ... .. ... .. ... ... . .. . .. ... ... . .. . .. ... ... . ... .. ... . . . .. ... ... . .. . ... ... .. .. ... ... . . . .. ... ... . . . ... .. ... ... . ... .. . ... . ... . . . .. ... ... . ... . ... .. ... .... . . .... .. . .... . . .... . .. .... .... .... . .. ..... . ..... . . . ..... .. ...... . ....... ....... . .. ......... ........... . . ................. .......................... .. . ........................................ . . .......................... . . . . .

10

12

14

0.25 0.20 0.15 0.10 0.05 0

. . . . . .. . ... . . . .......... ........... ..... ..... .... .... ... ... ... ... ... ... .. .. .. .. .. .. .. .. .. .. .. .. .. .. .. .. .. .. .. .. .. .. .. .. .. .. .. .. .. .. .. .. .. .. .. .. .. .. .. .. .. .. .. .. .. .. ... ... ... ... ... ... ... ... ... ... ... ... ... ... .... ... .... .... .... .... ..... ..... ..... ..... ...... ....... ......... ......... ................ ................... ............................................................................ ........................................................................ . . .

f(t+3| 3)

10

12

14

Figure 3.1: The density function of the residual life time conditioned by a given age x (3.11). The example is based on a Weibull distribution We(2,5) where x = 3 and F (3) = 0.3023. One would expect that the mean residual lifetime m1,r (x) decreases for increasing x, corresponding to that the expected residual lifetime decreases when the age x increases. This is not always the case. For an exponential distribution with form factor = 2 (Sec. 5.1), we have m1,r = m. For steep distributions (1 2) we have m1,r m (Sec. 4.2), whereas for at distributions (2 < ), we have m1,r m (Sec. 4.3).

Example 3.1.3: Waiting-time distribution Let us consider a queueing system with innite queue where no customers are blocked. The waiting time distribution Ws (t) for a random customer usually has a positive probability mass (atom) at t = 0, because some of the customers get service immediately without any delay. We thus have Ws (0) > 0. The waiting time distribution W+ (t) for customers having positive waiting times then becomes (3.11): Ws (t) Ws (0) W+ (t) = , 1 Ws (0)

3.1. DISTRIBUTION FUNCTIONS

67

or if we denote the probability of a positive waiting time {1 Ws (0)} by D (probability of delay): D {1 W+ (t)} = 1 Ws (t) . For the density function we have (3.11): D w+ (t) = ws (t) . For mean values we get: Dw =W , (3.20) where the mean value for all customers is denoted by W , and the mean value for the delayed customers is denoted by w. These formul are valid for any queueing system with innite queue. (3.19) (3.18)

3.1.3

Load from holding times of duration less than x

So far we have attached the same importance to all lifetimes independently of their duration. The importance of a lifetime is often proportional to its duration, for example when we consider the load of queueing system, charging of CPU-times, telephone conversations etc. If we allocate a weight factor to a life time proportional to its duration, then the average weight of all time intervals (of course) becomes equal to the mean value:

m=
0

t f (t) dt ,

(3.21)

where f (t) dt is the probability of an observation within the interval (t, t + dt), and t is the weight of this observation. In a trac process we are interested in calculating the proportion of the total trac which is due to holding times of duration less than x:
x

t f (t) dt x =
0

(3.22)

(This is the same as the proportion of the mean value which is due to contributions from lifetimes less than x). Often relatively few service times make up a relatively large proportion of the total load. From Fig. 3.2 we see that if the form factor is 5, then 75% of the service times only contribute with 30% of the total load (Vilfred Paretos rule). This fact can be utilised to give priority to short tasks without delaying the longer tasks very much (Chap. 13).

68 Relative load 1.0 0.9 0.8 0.7 0.6 0.5 0.4 0.3 0.2 0.1 0.0

CHAPTER 3. PROBABILITY THEORY AND STATISTICS

. . . . .. .. .. . .. .. .. .. .. . . . . . . . . . . . . . . . . . . . . . . . . . . . . . . . . . . . . . . . . . . . . .. . . . . . . . . . . . . . .. . . . . . . . . . . .. . . . .. . . . . .. . . . . .. . . . . . . . .. . . .. . . . . . . . . . . . . . . . .. . . . . .. . .. . . . .. .. . . .. .. . .. .. .. . . .. .. . . .. . .. . . .. .. .. . .. .. .. .. .. .. . . .. .. .. . .. .. .. .. .. .. .. .. .. .. .. . .. .. .. .. .. .. .. .. .. .. .. .. .. .. .. .. .. . . . .. .. .. .. .. .. .. .. .. .. .. .. .. .. .. .. .. .. .. . .. .. .. .. .. .. ... .. .. .. .. .. .. ... .. ... ... ... ... ... . ... ... ... ... .. .. ... ... .. .. ... ... ... ... ... ... ... ... ... ... . . .. ... ... ... ... ... ... ... ... ... ... .. . .... ... ... ... ... ... .... .... ... .... .. .. .. ... .... ... .... .... .... .... .... .... .... .... .... .... . ... ..... .... .... ..... .... ..... .... .... ..... .... ..... ..... ..... .. .. ..... ..... ...... ..... ..... ...... ..... ...... ..... ...... .. ... ....... ...... ...... ............... ....... ............... ..... . ..... ..... ........ ....... . ........ ......... ........... ......... ........... ........... ....... ........... ....... . .................... . ....................... ......................................... . ... ......................................... ........

=2

=5

0.0

0.1

0.2

0.3

0.4

0.5

0.6

0.7

0.8

0.9

1.0

Percentile/100 Figure 3.2: Example of the relative trac load from holding times shorter than a given value given by the percentile of the holding time distribution (3.22). Here = 2 corresponds to an exponential distribution and = 5 corresponds to a Pareto-distribution. We note that the 10% largest holding times contributes with 33%, respectively 47%, of the load (cf. customer averages and time averages in Chap. 5).

3.1.4

Forward recurrence time

The residual lifetime from a random point of time is called the forward recurrence time. In this section we shall derive some important formul. To formulate the problem we consider an example. We wish to investigate the lifetime distribution of cars and ask car-owners chosen at random about the age of their car. As the point of time is chosen at random, then the probability of choosing a car is proportional to the total lifetime of the car. The distribution of the future residual lifetime will then be identical with the already achieved lifetime. By choosing a sample in this way, the probability of choosing a car is proportional to the lifetime of the car, i.e. we will preferably choose cars with longer lifetimes (length-biased sampling). The probability of choosing a car having a total lifetime x is given by (cf. moment

3.1. DISTRIBUTION FUNCTIONS distribution in statistics) (cf. the derivation of (3.22)): x f (x) dx . m

69

As we consider a random point of time, the distribution of the remaining lifetime will be uniformly distributed in (0, x ]: f (t| x) = 1 , x

0 < t x.

Then the density function of the remaining lifetime at a random point of time is as follows: v(t) =
t

1 x f (x) dx , x m (3.23)

v(t) =

1 F (t) . m

where F (t) is the distribution function of the total lifetime and m is the mean value. By applying the identity (3.3), we note that the ith moment of v(t) is given by the (i + 1)th moment of f (t):

mi,v =

ti v(t) dt 1 F (t) dt m
0

=
0

ti

1 1 i+1 m

(i + 1) ti {1 F (t)} dt , (3.24)

mi,v =

1 1 mi+1,f . i+1 m

We obtain the mean value:

m , (3.25) 2 where is the form factor of the lifetime distribution. These formul are also valid for discrete time distributions. m1,v =

3.1.5

Distribution of the jth largest of k random variables

Let us assume that k random variables {T1 , T2 , . . . , Tk } are independent and identically distributed with distribution function F (t). The distribution of the jth largest variable will

70 then be given by:

CHAPTER 3. PROBABILITY THEORY AND STATISTICS

j1

p{jth largest t} =
i=0

k i

{1 F (t)}i F (t)ki .

(3.26)

as at most j 1 variables may be larger than t. The smallest one (or kth largest, j = k) has the distribution function: Fmin (t) = 1 {1 F (t)}k , (3.27) and the largest one (j = 1) has the distribution function: Fmax (t) = F (t)k . (3.28)

If the random variables has individual distribution functions Fi (t), we get an expression more complex than (3.26). For the smallest and the largest we get:
k

Fmin (t) = 1
i=1 k

{1 Fi (t)} ,

(3.29)

Fmax (t) =
i=1

Fi (t) .

(3.30)

3.2

Combination of random variables

We can combine lifetimes by putting them in series or in parallel or by a combination of the two.

3.2.1

Random variables in series

A linking in series of k independent time intervals corresponds to addition of k independent random variables, i.e. convolution of the random variables.
2 If we denote the mean value and the variance of the ith time interval by m1,i , i , respectively, then the sum of the random variables has the following mean value and variance: k

m = m1 =
i=1 k

m1,i ,

(3.31)

=
i=1

2 i .

(3.32)

3.2. COMBINATION OF RANDOM VARIABLES

71

In general, we should add the so-called cumulants, and the rst three cumulants are identical with the rst three central moments. The distribution function of the sum is obtained by the convolution: F (t) = F1 (t) F2 (t) Fk (t) , where is the convolution operator (Sec. 6.2.2). (3.33)

Example 3.2.1: Binomial distribution and Bernoulli trials Let the probability of success in a trial (e.g. throwing a dice) be equal to p and the probability of failure thus equal to 1 p. The number of successes in a single trial will then be given by the Bernoulli distribution: 1p , i = 0, p1 (i) = (3.34) p, i = 1. If we in total make S trials, then the distribution of number of successes is Binomial distributed: pS (i) = S i p (1p)Si , i (3.35)

which therefore is obtainable by convolving S Bernoulli distributions. If we make one additional trial, then the distribution of the total number of successes is obtained by convolution of the Binomial distribution (3.35) and the Bernoulli distribution (3.34): pS+1 (i) = pS (i) p1 (0) + pS (i1) p1 (1) = S i S p (1p)Si (1p) + pi1 (1p)Si+1 p i i1 S i + S i1 pi (1p)Si+1

S +1 i p (1p)Si+1 , i

q.e.d.

3.2.2

Random variables in parallel

By the weighting of independent random variables, where the ith variable appears with weight factor pi , where pi = 1 ,
i=1

72

CHAPTER 3. PROBABILITY THEORY AND STATISTICS

2 and has mean value m1,i and variance i , the random variable of the sum has the mean value and variance as follows:

m =
i=1

pi m1,i ,

(3.36)

2 =
i=1

2 pi (i + m2 ) m2 . 1,i

(3.37)

In this case we must weight the non-central moments. For the jth moment we have mj =
i=1

pi mj,i ,

(3.38)

where mj,i is the jth non-central moment of the distribution of the ith interval. The distribution function is as follows: F (t) =
i=1

pi Fi (t) .

(3.39)

A similar formula is valid for the density function: f (t) =


i=1

pi fi (t) .

A weighted sum of distributions is called a compound distribution.

3.3

Stochastic sum

By a stochastic sum we understand the sum of a stochastic number of random variables (Feller, 1950 [27]). Let us consider a trunk group without congestion, where the arrival process and the holding times are stochastically independent. If we consider a xed time interval T , then the number of arrivals is a random variable N. In the following N is characterised by: N: density function p(i) , mean value m1,n ,
2 variance n ,

(3.40)

Arriving call number i has the holding time Ti . All Ti have the same distribution, and each arrival (request) will contribute with a certain number of time units (the holding times) which

3.3. STOCHASTIC SUM is a random variable characterised by: T : density function f (t) , mean value m1,t ,
2 variance t ,

73

(3.41)

The total trac volume generated by all arrivals (requests) arriving within the considered time interval T is then a random variable itself: ST = T1 + T2 + + TN . (3.42)

p1 p2 p3

T1 T1 T1 T2 T2 T3

pi

T1

T2

Ti

Figure 3.3: A stochastic sum may be interpreted as a series/parallel combination of random variable.

In the following we assume that Ti and N are stochastically independent. This will be fullled when the congestion is zero. The following derivations are valid for both discrete and continuous random variables (summation is replaced by integration or vice versa). The stochastic sum becomes a combination of random variables in series and parallel as shown in Fig. 3.3 and dealt with in Sec. 3.2. For a given branch i we nd (Fig. 3.3): m1,i = i m1,t ,
2 2 i = i t , 2 m2,i = i t + (i m1,t )2 .

(3.43) (3.44) (3.45)

74

CHAPTER 3. PROBABILITY THEORY AND STATISTICS

By summation over all possible values (branches) i we get:

m1,s =
i=1

p(i) m1,i

=
i=1

p(i) i m1,t , (3.46)

m1,s = m1,t m1,n ,

m2,s =
i=1

p(i) m2,i

=
i=1

2 p(i) {i t + (i m1,t )2 } ,

2 m2,s = m1,n t + m2 m2,n , 1,t 2 2 s = m1,n t + m2 (m2,n m2 ) , 1,t 1,n 2 2 2 s = m1,n t + m2 n . 1,t

(3.47)

(3.48)

We notice there are two contributions to the total variance: one term because the number 2 of calls is a random variable (n ), and a term because the duration of the calls is a random 2 variable (t ).
Example 3.3.1: Special case 1: N = n = constant (m = n) m1,s = n m1,t ,
2 2 s = t n .

(3.49)

This corresponds to counting the number of calls at the same time as we measure the trac volume so that we can estimate the mean holding time.

Example 3.3.2: Special case 2: T = t = constant (m = t) m1,s = m1,n t ,


2 2 s = t 2 n .

(3.50)

If we change the scale from 1 to m1,t , then the mean value has to be multiplied by m1,t and the variance by m2 . The mean value m1,t = 1 corresponds to counting the number of calls, i.e. a 1,t problem of counting.

3.3. STOCHASTIC SUM

75

Example 3.3.3: Stochastic sum As a non-teletrac example N may denote the number of rain showers during one month and Ti may denote the precipitation due to the ith shower. ST is then a random variable describing the total precipitation during a month. N may also for a given time interval denote the number of accidents registered by an insurance company and Ti denotes the compensation for the ith accident. ST then is the total amount paid by the company for the considered period.

76

CHAPTER 3. PROBABILITY THEORY AND STATISTICS

Chapter 4 Time Interval Distributions


The exponential distribution is the most important time distribution within teletrac theory. This time distribution is dealt with in Sec. 4.1. Combining exponential distributed time intervals in series, we get a class of distributions called Erlang distributions (Sec. 4.2). Combining them in parallel, we obtain hyperexponential distribution (Sec. 4.3). Combining exponential distributions both in series and in parallel, possibly with feedback, we obtain phase-type distributions, which is a class of general distributions. One important subclass of phase-type distributions is Coxian-distributions (Sec. 4.4). We note that an arbitrary distribution can be expressed by a Coxdistribution which can be used in analytical models in a relatively simple way. Finally, we also deal with other time distributions which are employed in teletrac theory (Sec. 4.5). Some examples of observations of life times are presented in Sec. 4.6.

4.1

Exponential distribution

In teletrac theory this distribution is also called the negative exponential distribution. It has already been mentioned in Sec. 3.1.2 and it will appear again in Sec. 6.2.1. In principle, we may use any distribution function with nonnegative values to model a life time. However, the exponential distribution has some unique characteristics which make this distribution qualied for both analytical and practical uses. The exponential distribution plays a key role among all lifetime distributions. 77

78

CHAPTER 4. TIME INTERVAL DISTRIBUTIONS

This distribution is characterised by a single parameter, the intensity or rate : F (t) = 1 et , f (t) = et , The gamma function is dened by:

> 0, > 0,

t 0, t 0.

(4.1) (4.2)

(n + 1) =
0

tn et dt = n! .

(4.3)

We replace t by t and get the th moment: m = ! , m1 m2 2 = = = = 1 , 2 , 2 1 , 2 2, (4.4)

Mean value m = Second moment: Variance: Form factor:


.. ..................... .................... .. . . ...

.. . .. ..................... .................... ... .

Figure 4.1: In phase diagrams an exponentially distributed time interval is shown as a box with the intensity. The box thus means that a customer arriving to the box is delayed an exponentially distributed time interval before leaving the box. The exponential distribution is very suitable for describing physical time intervals (Fig. 6.2). The most fundamental characteristic of the exponential distribution is its lack of memory. The distribution of the residual time of a telephone conversation is independent of the actual duration of the conversation, and it is equal to the distribution of the total life-time (3.11): f (t + x|x) = e(t+x) ex

= et = f (t) . If we remove the probability mass of the interval (0, x) from the density function and normalise the residual mass in (x, ) to unity, then the new density function becomes congruent

4.1. EXPONENTIAL DISTRIBUTION

79

with the original density function. The only continuous distribution function having this property is the exponential distribution, whereas the geometric distribution is the only discrete distribution having this property. An example with the Weibull distribution where this property is not valid is shown in Fig. 3.1. For k = 1 the Weibull distribution becomes identical with the exponential distribution. Therefore, the mean value of the residual life-time is m1,r = m, and the probability of observing a lifetime in the interval (t, t + dt), given that it occurs after t, is given by p{t < X t + dt|X > t} = f (t) dt 1 F (t) (4.5)

= dt . Thus it depends only upon and dt, but it is independent of the actual age t.

4.1.1

Minimum of k exponentially distributed random variables

We assume that two random variables X1 and X2 are mutually independent and exponentially distributed with intensities 1 and 2 , respectively. A new random variable X is dened as: X = min {X1 , X2 } . The distribution function of X is: p{X t} = 1 e(1 +2 )t . (4.6)

This distribution function itself is also an exponential distribution with intensity (1 + 2 ). Under the assumption that the rst (smallest) event happens within the time interval t, t+dt, then the probability that the random variable X1 is realized rst (i.e. takes places in this interval and the other takes place later) is given by: p{X1 < X2 | t} = P {t < X1 t + dt} P {X2 > t} P {t < X t + dt} 1 e1 t dt e2 t (1 + 2 ) e(1 +2 )t dt 1 , 1 + 2 (4.7)

i.e. independent of t. Thus we do not need to integrate over all values of t. These results can be generalised to k variables and make up the basic principle of the simulation technique called the roulette method, a Monte Carlo simulation methodology.

80

CHAPTER 4. TIME INTERVAL DISTRIBUTIONS

4.1.2

Combination of exponential distributions

If one exponential distribution (i.e. one parameter) cannot describe the time intervals in sucient detail, then we may have to use a combination of two or more exponential distributions. Conny Palm introduced two classes of distributions: steep and at. A steep distribution corresponds to a set of stochastic independent exponential distributions in series (Fig. 4.2), and a at distribution corresponds to exponential distributions in parallel (Fig. 4.4). This structure naturally corresponds to the shaping of trac processes in telecommunication and data networks. By the combination of steep and at distribution, we may obtain an arbitrary good approximation for any distribution function (see Fig. 4.7 and Sec. 4.4). The diagrams in Figs. 4.2 & 4.4 are called phase-diagrams.

4.2

Steep distributions

.................. ................. .... ..... ...... ... .. . .. ...

1

................................... ....... ........... ............ .. ........... ........... ........ ... . .. ...

2

.................................. ................. ...... .......... .... ..... ................... ... .. .. .. .. .

. . . . . . . . . . . . . . . . . . .... . . . . . . . . . . . . . . . . . ... . . .... . .       .........................................................

.. . .

k

.................. ................. . ..... ....... ... ... .. .. ..

Figure 4.2: By combining k exponential distributions in series we get a steep distribution ( 2). If all k distributions are identical (i = ), then we get an Erlangk distribution.

Steep distributions are also called hypoexponential distributions or generalised Erlang distributions with a form factor in the interval 1 < 2. This generalised distribution function is obtained by convolving k exponential distributions (Fig. 4.2). Here we only consider the case where all k exponential distributions are identical. Then we obtain the following density function which is called the Erlang-k distribution: (t)k1 et , (k 1)!

f (t) =

> 0,

t 0,

k = 1, 2, . . . .

(4.8)

F (t) =
j=k

(t)j t e j!
k1

(4.9)

= 1
j=0

(t)j t e j!

(cf. Sec. 6.1) .

(4.10)

4.2. STEEP DISTRIBUTIONS 3 50 .. ..

81

.. .. . . . . . . . . . . . . . . . . . . . . . . . . . . . . . . . . . . . . . . . . . . . . . . . . . . . . . . . . . . . . . . . . . . . . . . . . . . . . . . . . . . . . . . . . . . . . . . . . . . . . . . . . . . . . . . . . . . . . . . . . . . . . . . . . . . . . . . . . . . . . . . . . . . . . . . . . . . . . . . . . . . . . . . . . . . . . . . . . . . . . . . . . . . . . . . . . . . . . . . . . . . . . . . . . . . . . . . . . . . . . . . . . . . . . . . . . . . . . . . . . . . . . . . . . . . . . . . . . . . . . . . . . . . . . . . . . . . . . . . . . . . . . . . . . . . . . . . . . . . . . . . . . . . . . . . . . . . . . . . .. . . .. .. ........ . .. ... .... .. . . .... . . .... .. .... . ...... .. . . . . ... .. ... . . .. .. .. . . . .. . .. .. .. . . .. . .. . .. .. .. .. . . . . .. .. .. .. .. . . .. . .. . .. ... . .. . .. . ... . .. . .. ... . . .. . . . ... . .. . .. .. ... . . . ... . .. . . .. . .. . .. .. ................. .. . ... ...... . . .......... . ..... . . ..... . . .. . . ... .. . .. ..... .... .. .. . .. . ... . .... .. . ... ..... .. . . .. . . .. .... .. . . ..... . ... .. . ....... .. .. .. . .... .. ...... . .... .. . .. .. ........ . . .. .... . ... . . .... . . . ... . . .. . .. ... .... . .. . . ... .... . .. . . . .... . . .... . . .. . .... . . .... . . . .. .... . . .... .. . .... . .... .. . .. . .. .... . . .. .... . .... . . . . ...... .. .... . .. . .. . .... . . .... . .... . . . . .. ..... . .. . . ..... . .... ..... . . .. .... .. . . . .. ..... . .... . . .. ..... .. . . . ..... . ..... . . . ..... . ..... .... . . . . ....... .... .... . . . .. . . .... .. . . . . ..... .. ....... . . . . . . ... . ..... ... . . ........ . ........ . . . . ..... ....... . .. . . .. .. .... . .. . . ...... . . .... . . . . ......... . . .. .. .. . . . . . . .......... ............ . . .. ... ....... .. . . . .. . .. . . ...................... ... . ....... . . ... . . . .. . . . ......... . ....... .. . ... ... . . .. . . . .. ..................... . . . .. ..... . ............ .. . . . ..... .. .. . ....... .. .. . . ...... .. . .......... . . .................. . ..... . .. . .. . ...... ................ . ..... .. . .. ....... .................................. ... . .. . ..... . .. . .. .. . .. . . .. . .. . .. .. . . ....... .. .. . .. .. ........ .. . .. . .. . . .. . .. .. . .. . .. .. .. . . .. . .......... . . ... . .. . .. .. . .. . . . .. ............ ............................................ .... . .. .. .. .. . ...................... . .......................... ... .. . .......... . .. .. . .. . .. ............................... .. . ....................................... ............... .. . .. . .. .. . .. . .. .. . .. . ... . . .. . .. . .. .. . .. ... .. .. . .. . .. .. . .. . .. .. . .. . . .. .. .......................................................

Erlangk distributions

5 2 1

Figure 4.3: Erlangk distributions with mean value equal to one. The case k = 1 corresponds to an exponential distribution (density functions). The following moments can be found by using (3.31) and (3.32): m = 2 = k , k , 2 2 1 =1+ , m2 k 1
i

(4.11) (4.12)

= 1+ The ith non-central moment is:

(4.13)

(i + k 1)! mi = (k 1)!

(4.14)

The density function is derived in Sec. 6.2.2. The mean residual lifetime m1,r (x) for x 0

82 will be less than the mean value:

CHAPTER 4. TIME INTERVAL DISTRIBUTIONS

m1,r (x) m,

x 0.

With this distribution we have two parameters (, k) available to be estimated from observations. The mean value is often kept xed. To study the inuence of the parameter k in the distribution function, we normalise all Erlangk distributions to the same mean value as the Erlang1 distribution, i.e. the exponential distribution with mean 1/, by replacing t by kt or by k: (kt)k1 kt f (t) dt = k dt , (4.15) e (k 1)! m = 2 = 1 , 1 , k2 1 . k (4.16) (4.17) (4.18)

= 1+

Notice that the form factor is independent of time scale. The density function (4.15) is illustrated in Fig. 4.3 for dierent values of k with = 1. The case k = 1 corresponds to the exponential distribution. When k we get a constant time interval ( = 1). By solving f (t) = 0 we nd the maximum value at: t = k1 . k (4.19)

The so-called steep distributions are named so because the distribution functions increase quicker from 0 to 1 than the exponential distribution do. In teletrac theory we sometimes use the name Erlangdistribution for the truncated Poisson distribution (Sec. 7.3).

4.3

Flat distributions

The general distribution function is in this case a weighted sum of exponential distributions (compound distribution) with a form factor 2:

F (t) =
0

1 et dW () ,

> 0,

t 0,

(4.20)

where the weight function may be discrete or continuous (Stieltjes integral). This distribution class corresponds to a parallel combination of the exponential distributions (Fig. 4.4). The density function is called complete monotone due to the alternating signs (Palm, 1957 [82]): (1) f () (t) 0 . (4.21)

4.3. FLAT DISTRIBUTIONS


.... .. ................................................................. .. ................................................................. . .. . .. . ... .... .. ... .. .. . .. .. .. . .. .. .. . .. .. .. . .. .. .. .... .. . .. .. ................................................................. ................................................................. . .. . ... . ... .. ..... ... .. . .. ... . ... . . ... .. ... . . .. ... ... .. .. ... . .. ... ... . ... .. ... . .... .. ..... .. ..... ....... ... .... .. . .. ................................... ................................... .. . .. ... ... .. .. .. .. .. .. .. .. .. .. .. .. .. .. .. .. .. .. .. .. .. .. .. .. .. .. .. .. . .. .... .... ... . ... .. ................................................................. ................................................................. .. .. .. ...

83 p1 1
.. .. .. .. .. .. .. .. .. .. .. .. .. .. .. .. .. .. .. .. .. .. .. .. ... ... .. ... .. ... ... .. ... .. ... ... .. .. ... ... ... .. ... .. ... ... . ... ... ... ... .. ... . ... .. ... . ... .. ... . ... . ... . .... ... ................................... ................................... . .... . .. ... .. .. .. .. .. .. .. .. .. .. .. .. .. .. .. .. .. .. .. .. .. .. . .. .. .. .. .. .. .. ..

p2

2 . . .

. . .

pk

k i=1

pi = 1

Figure 4.4: By combining k exponential distributions in parallel and choosing branch number i with the probability pi , we get a hyperexponential distribution, which is a at distribution ( 2). The mean residual life-time m1,r (x) for all x 0 is larger than the mean value: m1,r (x) m, x 0. (4.22)

4.3.1

Hyper-exponential distribution

In this case, W () is discrete. Suppose we have the following given values: 1 , 2 , . . . , k , and that W () has the positive increases: p1 , p2 , . . . , pk ,

where
k

pi = 1 .
i=1

(4.23)

For all other values W () is constant. In this case (4.20) becomes:


k

F (t) = 1
i=1

pi ei t ,

t 0.

(4.24)

84

CHAPTER 4. TIME INTERVAL DISTRIBUTIONS

The mean values and form factor may be found from (3.36) and (3.37) (i = m1,i = 1/i ):
k

m1 =
i=1

pi , i
k

(4.25)
2

= 2
i=1

pi 2 i

i=1

pi i

2.

(4.26)

If k = 1 or all i are equal, we get the exponential distribution. This class of distributions is called hyperexponential distributions and can be obtained by combining k exponential distributions in parallel, where the probability of choosing the ith distribution is given by pi . The distribution is called at because its distribution function increases more slowly from 0 to 1 than the exponential distribution does. In practice, it is dicult to estimate more than one or two parameters. The most important case is for n = 2 (p1 = p, p2 = 1 p): F (t) = 1 p e1 t (1 p) e2 t . (4.27)

Statistical problems arise even when we deal with three parameters. So for practical applications we usually choose i = 2pi and thus reduce the number of parameters to only two: F (t) = 1 pe2pt (1 p)e2(1p)t . (4.28) The mean value and form factor becomes: m = = 1 , 1 . 2p(1 p) (4.29)

For this choice of parameters the two branches have the same contribution to the mean value. Fig. 4.5 illustrates an example.

4.4

Cox distributions

By combining the steep and at distributions we obtain a general class of distributions (phase type distributions) which can be described with exponential phase in both series and parallel (e.g. a k matrix). To analyse a model with this kind of distributions, we can apply the

4.4. COX DISTRIBUTIONS

85

10000

Number of observations

57055 observations m = 171.85 s form factor = 3.30 1000

100

10

10

15

20

25 Time

Figure 4.5: Density (frequency) function for holding times observed on lines in a local exchange during busy hours.

86

CHAPTER 4. TIME INTERVAL DISTRIBUTIONS

(1 p0 ) q1 (1 p1 ) q2 (1 p2 ) 1 1 2

qk (1 pk )

Figure 4.6: A Coxdistribution is a generalised Erlangdistribution having exponential distributions in both parallel and series. The phase-diagram is equivalent to Fig. 4.7.

.. .. . .......................................... ......................................... .. .... .. ........................................ ........................................ .... .. ........................................ .................................. ..... ... . . . . .. . . .. .. . . . . . . . . . . . . . . . . . . . . . . . . . . . . . . . . . . . . . . . . . . . . . . . . . . . . . . . . . . . . . . . . . . . . . . . . . . . . . . . . . . . . . . . . . . . . . . . . . . . . . . . . . . . . . . . . . . . . . . . . . . . . . . . . . . . . . . . . . . . . . . . . . . . . . . . . . . . . . . . . . . . . . . . . . . . . . . . . . . . . . . . . . . . . . . . . . . . . . . . . . . . . . . . . . . . . . . . . . . . . . . . . . . . . . . . . . ... ... ... . . ... ... ... . . .. .. .. . .. . .. .. . .. .. .. .. . . ...................................................................................................................................................... .. .. . .. . . . . . . ........................................................ ................................................................ .......................... . ... . ... . ... . .. .. ..

p0

p1

p2

pk1 .............................................................................................

1 p0

1 p1

1 p2

. . . . . . . . . . . . . . . . . . . . . . . . . . . . . . . . . . . . . . . . . . . . . . . . . . . . . . . . . . . . . . . . . . . . . . . . . . . . . . . . . . . . . . . . . . . . . . . . . . . . . . . . . . . . . . . . . . . . . . . . . . . . . . . . . . . . . . . . . . . . . . . . . . . . . . . ... ... . ... ... . .. .. . . . .. .. . . .. .. . . . . .... .... .. .. ....................................................................................................... ...................................................................................................... . ... . . .. ..

1 pk1

Figure 4.7: The phase diagram of a Cox distribution, cf. Fig. 4.6.

4.4. COX DISTRIBUTIONS

87

theory of Markov processes, for which we have powerful tools as the phase-method. In the more general case we can allow for loop back between the phases. We shall only consider Cox-distributions as shown in Fig. 4.6 (Cox, 1955 [17]). These also appear under the name of Branching Erlang distribution (Erlang distribution with branches). The mean value and variance of this Cox distribution (Fig. 4.7) are found from the formulae in Sec. 3.2 for random variables in series and parallel as shown in g. 4.6:
k i

m1 =
i=1

qi (1 pi )
j=1

1 j

(4.30)

where qi = p0 p1 p2 pi1 . (4.31) The term qi (1 pi ) is the probability of jumping out after being in ith phase. It can be shown thet the mean value can be expressed by the simple form:
k

m1 =
i=1

qi = i

m1,i ,
i=1

(4.32)

where m1,i = qi /i is the ith phase related mean value. The second moment becomes:
k

m2 =
i=1 k

{qi (1 pi ) m2,i } ,
i i 2

=
i=1

qi (1 pi )

j=1

1 + 2 j

j=1

1 j

, (4.33)

2 where m2,i is obtained from (3.8): m2,i = 2,i + m2 . It can be shown that this can be written 1,i as: k i qi 1 m2 = 2 . (4.34) i i=1 j=1 j

From this we get the variance (3.8): 2 = m2 m2 . 1 The addition of two Coxdistributed random variables yields another Cox-distributed variable, i.e. this class is closed under the operation of addition. The distribution function of a Cox distribution can be written as a sum of exponential functions:
k

1 F (t) =
i=1

ci ei t ,

(4.35)

88 where

CHAPTER 4. TIME INTERVAL DISTRIBUTIONS

0
i=1

ci 1 ,

and < ci < + .

4.4.1

Polynomial trial

The following properties are of importance for later applications. If we consider a point of time chosen at random within a Coxdistributed time interval, then the probability that this point is within phase i is given by: mi , m i = 1, 2, . . . , k . (4.36)

If we repeat this experiment y (independently) times, then the probability that phase i is observed yi times is given by multinomial distribution (= polynomial distribution): p{y | y1, y2 , . . . , yk } = where y=
i=1

y y1 y2 . . . yk

m1,1 m
k

y1

m1,2 m

y2

m1,k m

yk

(4.37)

yi , y! . y1 ! y2 ! yk !

and

y y1 y2 . . . yk

(4.38)

These (4.38) are called the multinomial coecients. By the property of lack of memory of the exponential distributions (phases) we have full information about the residual life-time, when we know the number of the actual phase.

4.4.2

Decomposition principles

Phasediagrams are a useful tool for analysing Cox distributions. The following is a fundamental characteristic of the exponential distribution (Iversen & Nielsen, 1985 [41]): Theorem 4.1 An exponential distribution with intensity can be decomposed into a twophase Cox distribution, where the rst phase has an intensity > and the second phase intensity (cf. Fig. 4.8).

4.4. COX DISTRIBUTIONS

89

According to Theorem 4.1 a hyperexponential distribution with phases is equivalent to a Cox distribution with the same number of phases. The case = 2 is shown in Fig. 4.10. We have another property of Cox distributions (Iversen & Nielsen, 1985 [41]): Theorem 4.2 The phases in any Cox distribution can be ordered such as i i+1 . Theorem 4.1 shows that an exponential distribution is equivalent to a homogeneous Cox distribution (homogeneous: same intensities in all phases) with intensity m and an innite number of phases (Fig. 4.8). We notice that the branching probabilities are constant. Fig. 4.9 corresponds to a weighted sum of Erlangk distributions where the weighting factors are geometrically distributed.
.. . .. ..................... ................... .. ..

.... .. ..................... .................... ... .

.... .. ..................... .................... ... .

.... .. ........................................ ....................................... . .. .. . . . . . . . . . . . . . . . . . . . . . . . . . . . . . . . . . . . . . . . . . . . . . . . . . . . . . . . . . . . . . . . . . . . . . . . . . . . . . . . . . . . . . . . . . . . . . . . . . . . . . . . . . . . . . . . . . . . . . . . . . . . . . . . . . . . . . . . . . . . . . . . . . . . . . . . . . . . . .. ... ... .. ... . .. .. . . .. .. . . .. .. . . . ............................................................................. .. . . ........................................................................... . .. . ...

Figure 4.8: An exponential distribution with rate is equivalent to the shown Cox distribution (Theorem 4.1). 1p 1p 1p ............................................................................................... 1p

.. . ..................... ................... .. . . ..

.... .. ....................................... ....................................... ... . ........................................ ....................................... .. .. . . .. ... . . . . . . . . . . . . . . . . . . . . . . . . . . . . . . . . . . . . . . . . . . . . . . . . . . . . . . . . . . . . . . . . . . . . . . . . . . . . . . . . . . . . . . . . . . . . . . . . . . . . . . . . . . . . . . . . . . . . . . . . . . . . . . . . . . . . . . . . . . . . . . . . . . . . . . . . . .. ... ... ... ... . .. ... . .. . .. .. .. . . . ........................................................................................... .. . . . ............................................................... ......................... . ..... .. . . .. . .. . .. .

p=

..... . ............................................... ................... .......................... . .. . . . . . . . . . . . . . . . . . . . . . . . . . . . . . . . . . . . . . . . . . . . . . . . . . . . . . . . . . . . . . . . . . . . . . . . . . . . . . . . . . . . . . . . . . . . . . . . . . . . . . . . . . . . . . . . . . . . . . . . . . . . . . . . . . . . . . . . . . . . . . . . . . . . . . . .. ... ... ... ... . .. ... . .. .. . . . .. . . . . . . ... ... .. .. .............................................................................................................. ............................................................................................................. . .... . .. . .. .

Figure 4.9: An exponential distribution with rate is by successive decomposition transformed into a compound distribution of homogeneous Erlangk distributions with rates > , where the weighting factors follows a geometric distribution (quotient p = /).

By using phase diagrams it is easy to see that any exponential time interval () can be decomposed into phase-type distributions (i ), where i . Referring to Fig. 4.11 we

90

CHAPTER 4. TIME INTERVAL DISTRIBUTIONS 2 ) 1 ......... ....................................................................................................... ................... .................................................................................. . q = (1 p1 )(1 2 1

.... .. ................................. ................................. .. ..

.................... ................... . . . . . . . . . . . . . . . . . . . . . . . . . . . . . . . . . . . . . . . . . . . . . . . . . . . . . . . . . . . . . . . . . . . . . . . . . . . . . . . . . . . . . . . . . . . . . . . . . . . . . . . . . . . . . . . . . . . . . . . . . .. ... ... .. ... . .. .. . .. .. . .. . . . . ............................................................................................................................................................... . . . .............................................................................................................................................................. . . .. . ....

p = p1 + (1 p1 )

Figure 4.10: A hyperexponential distribution with two phases (1 > 2 , p2 = 1 p1 ) can be transformed into a Cox2 distribution (cf. Fig. 4.4). notice that the rate out of the macro-state (dashed box) is independent of the micro state. When the number of phases k is nite and there is no feedback the nal phase must have rate . 1p1 1pi

... .. ....................................... ....................................... ... .

1
. . . . . . . . . . . . . . . . . . . . . . . . . . . . . . . . . . . . . . . . . . . . . . . . . . . . . . ... ... .. . .. .. . . .

... .. ........................... .... .... ..................... ..... ... ... ... .

.... .. ............................ .......................... ... .

i
. . . . . . . . . . . . . . . . . . . . . . . . . . . . . . . . . . . . . . . . . . . . . . . . . . . . . ... ... .. .. . .. . . . .

.... .. ........................... .... .... .......................... ... ... ... .

.... .. ........................... .......................... .. ..

k =
. . . . . . . . . . . . . . . . . . . . . . . . . . . . . . . . . . . . . . . . . . . . . . . . . . . . . ... ... .. . .. . .. . . .

p1

pi

pk = 1

Figure 4.11: This phase-type distribution is equivalent to a single exponential when pi i = . Thus i as 0 < pi 1.

4.4.3

Importance of Cox distribution

Cox distributions have attracted a lot of attention during recent years. They are of great importance due to the following properties: a. Cox distribution can be analysed using the method of phases. b. One can approximate an arbitrary distribution arbitrarily well with a Cox distribution. If a property is valid for a Cox distribution, then it is valid for any distribution of practical interest.

By using Cox distributions we can with elementary methods obtain results which previously required very advanced mathematics.

4.5. OTHER TIME DISTRIBUTIONS

91

In the connection with practical applications of the theory, we have used the methods to estimate the parameters of Cox distribution. In general there are 2 k parameters in an unsolved statistical problem. Normally, we may choose a special Cox distribution (e.g. Erlangk or hyperexponential distribution) and approximate the rst moment. By numerical simulation on computers using the Roulette method, we automatically obtain the observations of the time intervals as Cox distribution with the same intensities in all phases.

4.5

Other time distributions

In principle, every distribution which has nonnegative values, may be used as a time distribution to describe the time intervals. But in practice, one may work primarily with the above mentioned distributions. We suppose the parameter k in Erlang-k distribution (4.8) takes non-negative real values and obtain the gamma distribution: f (t) = 1 (t)k1 et , (k) > 0, t 0. (4.39)

The mean value and variance are given in (4.11) and (4.12). A distribution also known in teletrac theory is the Weibull distribution: F (t) = 1 e(t) ,
k

t 0,

k > 0,

> 0.

(4.40)

This distribution has a time-dependent death intensity (3.14): dF (t) e(t) k (t)k1 dt = (t) = 1 F (t) e(t)k = k (t)k1 . (4.41)
k

The distribution has its origin in the reliability theory. For k = 1 we get the exponential distribution. The Pareto distribution is given by: F (t) = 1 (1 + 0 t)
1+ 0

(4.42)

92

CHAPTER 4. TIME INTERVAL DISTRIBUTIONS

The mean value and form factor are as follows: 1 m1 = , = 2 , 0 0 < 0 < . (4.43)

Note that the variance does not exist for 0 . Letting 0 0 (4.42) becomes an exponential distribution. If the intensity of a Poisson process is gamma distributed, then the inter-arrival times are Pareto-distributed. Later, we will deal with a set of discrete distributions, which also describes the lifetime, such as geometrical distribution, Pascal distribution, Binomial distribution, Westerberg distribution, etc. In practice, the parameters of distributions are not always stationary. The service (holding) times can be physically correlated with the state of the system. In manmachine systems the service time changes because of busyness (decreases) or tiredness (increases). In the same way, electromechanical systems work more slowly during periods of high load because the voltage decreases. For some distributions which are widely applied in the queueing theory, we have the following abbreviated notations (cf. Sec. 13.1): M Ek Hn D Cox G Exponential distribution (Markov), Erlang-k distribution, Hyper-exponential distribution of order n, Constant (Deterministic), Cox distribution, General = arbitrary distribution.

4.6

Observations of life-time distribution

Fig. 4.5 shows an example of observed holding times from a local telephone exchange. The holding time consists of both signalling time and, if the call is answered, conversation time. Fig. 6.2 shows observation and interarrival times of incoming calls to a transit telephone exchange during one hour. From its very beginning, the teletrac theory has been characterised by a strong interaction between theory and practice, and there has been excellent possibilities to carry out measurements. Erlang (1920, [11]) reports a measurement where 2461 conversation times were recorded in a telephone exchange in Copenhagen in 1916. Palm (1943 [79]) analysed the eld of trac

4.6. OBSERVATIONS OF LIFE-TIME DISTRIBUTION

93

measurements, both theoretically and practically, and implemented extensive measurements in Sweden. By the use of computer technology a large amount of data can be collected. The rst stored program controlled by a mini-computer measurement is described in (Iversen, 1973 [35]). The importance of using discrete values of time when observing values is dealt with in Chapter 15. Bolotin (1994, [7]) has measured and modelled telecommunication holding times. Numerous measurements on computer systems have been carried out. Where in telephone systems we seldom have a form factor greater than 6, we observe form factors greater than 100 in data trac. This is the case for example for data transmission, where we send either a few characters or a large quantity of data. To describe these data we use heavy-tailed distributions. A distribution is heavy-tailed in strict sense if the tail of the distribution function behaves as a power law, i.e. as 1 F (t) t , 0 < 2 . The Pareto distribution (4.42) is heavy-tailed in strict sense. Sometimes distributions with a tail heavier than the exponential distribution are classied as heavy-tailed. Examples are hyper-exponential, Weibull, and log-normal distributions. More recent extensive measurements have been performed and modelled using self-similar trac models (Jerkins & al., 1999 [51]). These subjects are dealt with in more advanced chapters.

94

CHAPTER 4. TIME INTERVAL DISTRIBUTIONS

Chapter 5 Arrival Processes


Arrival processes, such as telephone calls arriving to an exchange are described mathematically as stochastic point processes. For a point process, we have to be able to distinguish two arrivals from each other. Informations concerning the single arrival (e.g. service time, number of customers) are ignored. Such information can only be used to determine whether an arrival belongs to the process or not. The mathematical theory for point process was founded and developed by the Swede Conny Palm during the 1940es. This theory has been widely applied in many subjects. It was mathematically rened by Khintchine ([63], 1968), and has been made widely applicable in many textbooks.

5.1

Description of point processes

In the following we only consider simple point processes, i.e. we exclude multiple arrivals as for example twin arrivals. For telephone calls this may be realized by a choosing sucient detailed time scale. Consider call arrival times where the ith call arrives at time Ti : 0 = T0 < T1 < T2 < . . . < Ti < Ti+1 < . . . . The rst observation takes place at time T0 = 0. The number of calls in the half open interval [0, t[ is denoted as Nt . Here Nt is a random variable with continuous time parameters and discrete space. When t increases, Nt never decreases. 95 (5.1)

96

CHAPTER 5. ARRIVAL PROCESSES

120 110 100 90 80 70 60 50 40 30 20 10 0

Accumulated number of calls

10

20

30

40

50

60 Time [s]

Figure 5.1: The call arrival process at the incoming lines of a transit exchange. The time distance between two successive arrivals is: Xi = Ti Ti1 , i = 1, 2, . . . . (5.2)

This is called the inter-arrival time, and the distribution of this process is called the interarrival time distribution. Corresponding to the two random variables Nt and Xi , the two processes can be characterised in two ways: 1. Number representation Nt : time interval t is kept constant, and we observe the random variable Nt for the number of calls in t. 2. Interval representation Ti : number of arriving calls is kept constant, and we observe the random variable Ti for the time interval until there has been n arrivals (especially T1 = X1 ). The fundamental relationship between the two representations is given by the following simple relation: Nt < n, if and only if (5.3) Tn = n Xi t, n = 1, 2, . . . i=1 This is expressed by Feller-Jensens identity: p {Nt < n} = p {Tn t} , n = 1, 2, . . . (5.4)

5.1. DESCRIPTION OF POINT PROCESSES

97

Analysis of point process can be based on both of these representations. In principle they are equivalent. Interval representation corresponds to the usual time series analysis. If we for example let i = 1, we obtain call averages, i.e. statistics based on call arrivals. Number representation has no parallel in time series analysis. The statistics we obtain are calculated per time unit and we get time averages (cf. the dierence between call congestion and time congestion). The statistics of interests when studying point processes can be classied according to the two representations.

5.1.1

Basic properties of number representation

There are two properties which are of theoretical interest: 1. The total number of arrivals in interval [t1 , t2 [ is equal to Nt2 Nt1 . The average number of calls in the same interval is called the renewal function H: H (t1 , t2 ) = E {Nt2 Nt1 } . 2. The density of arriving calls at time t (time average) is: t = lim Nt+t Nt = Nt . t0 t (5.6) (5.5)

We assume that t exists and is nite. We may interpret t as the intensity by which arrivals occur at time t (cf. Sec. 3.1.2). For simple point processes, we have: p {Nt+t Nt 2} = o(t) , p {Nt+t Nt = 1} = t t + o(t) , p {Nt+t Nt = 0} = 1 t t + o(t) , where by denition: o(t) = 0. t0 t lim (5.10) (5.7) (5.8) (5.9)

3. Index of Dispersion for Counts, IDC. To describe second order properties of the number representation we use the index of dispersion for counts, IDC. This describes the variations of the arrival process during a time interval t and is dened as: IDC = Var{Nt } . E{Nt } (5.11)

98

CHAPTER 5. ARRIVAL PROCESSES By dividing the time interval t into x intervals of duration t/x and observing the number of events during these intervals we obtain an estimate of IDC(t). For the Poisson process IDC becomes equal to one. IDC is equal to peakedness, which we later introduce to characterise the number of busy channels in a trac process (7.7).

5.1.2

Basic properties of interval representation

4. The distribution f (t) of time intervals Xi (5.2) (and by convolving the distribution by itself i1 times the distribution of the time until the ith arrival). Fi (t) = p {Xi t} , E {Xi } = m1,i . (5.12) (5.13)

The mean value is a call average. A renewal process is a point process, where sequential inter-arrival times are stochastic independent to each other and have the same distribution (except for X1 ), i.e. m1,i = mi . (IID = Identically and Independently Distributed). 5. The distribution V (t) of the time interval from a random epoch until the rst arrival occurs. The mean value of V (t) is a time average, which is calculated per time unit. 6. Index of Dispersion for Intervals, IDI. To describe second order properties for the interval representation we use the Index of Dispersion for Intervals, IDI. This is dened as: IDI = Var{Xi } , E{Xi }2 (5.14)

where Xi is the inter-arrival time. For the Poisson process, which has exponentially distributed service times, IDI becomes equal to one. IDI is equal to Palms form factor minus one (3.10). In general, IDI is more dicult to obtain from observations than IDC, and more sensitive to the accuracy of measurements and smoothing of the trac process. The digital technology is more suitable for observation of IDC, whereas it complicates the observation of IDI (Chap. 15). Which of the two representations one should use in practice really depends on the actual case. This can be illustrated by the following examples.
Example 5.1.1: Measuring principles Measures of teletrac performance are carried out by one of the two basic principles as follows: 1. Passive measures. Measuring equipment records at regular time intervals the number of arrivals since the last recording. This corresponds to the scanning method, which is suitable for computers. This corresponds to the number representation where the time interval is xed.

5.2. CHARACTERISTICS OF POINT PROCESS

99

2. Active measures. Measuring equipment records an event at the instant it takes place. We keep the number of events xed and observe the measuring interval. Examples are recording instruments. This corresponds to the interval representation, where we obtain statistics for each single call.

Example 5.1.2: Test calls Investigation of the trac quality. In practice this is done in two ways: 1. The trac quality is estimated by collecting statistics of the outcome of test calls made to specic (dummy-) subscribers. The calls are generated during busy hour independently of the actual trac. The test equipment records the number of blocked calls etc. The obtained statistics corresponds to time averages of the performance measure. Unfortunately, this method increases the oered load on the system. Theoretically, the obtained performance measures will dier from the correct values. 2. The test equipments collect data from call number N, 2N, 3N, . . ., where for example N = 1000. The trac process is unchanged, and the performance statistics is a call average.

Example 5.1.3: Call statistics A subscriber evaluates the quality by the fraction of calls which are blocked, i.e. call average. The operator evaluates the quality by the proportion of time when all trunks are busy, i.e. time average. The two types of average values (time/call) are often mixed up, resulting in apparently conicting statement.

Example 5.1.4: Called party busy (B-Busy) At a telephone exchange 10% of the subscribers are busy, but 20% of the call attempts are blocked due to B-busy (called party busy). This phenomenon can be explained by the fact that half of the subscribers are passive (i.e. make no call attempts and receive no calls), whereas 20% of the remaining subscribers are busy. G. Lind (1976 [73]) analysed the problem under the assumption that each subscriber on the average has the same number of incoming and outgoing calls. If mean value and form factor of the distribution of trac per subscriber is b and , respectively, then the probability that a call attempts get B-busy is b .

5.2

Characteristics of point process

Above we have discussed a very general structure for point processes. For specic applications we have to introduce further properties. Below we only consider number representation, but we could do the same based on the interval representation.

100

CHAPTER 5. ARRIVAL PROCESSES

5.2.1

Stationarity (Time homogeneity)

This property can be described as, regardless of the position on the time axis, then the probability distributions describing the point process are independent of the instant of time. The following denition is useful in practice: Denition: For an arbitrary t2 > 0 and every k 0, the probability that there are k arrivals in [t1 , t1 + t2 [ is independent of t1 , i.e. for all t, k we have: p {Nt1 +t2 Nt1 = k} = p {Nt1 +t2 +t Nt1 +t = k} . There are many other denitions of stationarity, some stronger, some weaker. Stationarity can also be dened by interval representation by requiring all Xi to be independent and identically distributed (IID). A weaker denition is that all rst and second order moments (e.g. the mean value and variance) of a point process must be invariant with respect to time shifts. Erlang introduced the concept of statistical equilibrium, which requires that the derivatives of the process with respect to time are zero. (5.15)

5.2.2

Independence

This property can be expressed as the requirement that the future evolution of the process only depends upon the present state. Denition: The probability that k events (k is integer and 0) take place in [t1 , t1 + t2 [ is independent of events before time t1 p {Nt2 Nt1 = k|Nt1 Nt0 = n} = p {Nt2 Nt1 = k} (5.16)

If this holds for all t, then the process is a Markov process: the future evolution only depends on the present state, but is independent of how this has been obtained. This is the lack of memory property. If this property only holds for certain time points (e.g. arrival times), these points are called equilibrium points or regeneration points. The process then has a limited memory, and we only need to keep record of the past back the the latest regeneration point.
Example 5.2.1: Equilibrium points = regeneration points Examples of point process with equilibrium points. a) Poisson process is (as we will see in next chapter) memoryless, and all points of the time axes are equilibrium points. b) A scanning process, where scannings occur at a regular cycle, has limited memory. The latest scanning instant has full information about the scanning process, and therefore all scanning points are equilibrium points.

5.3. LITTLES THEOREM

101

c) If we superpose the above-mentioned Poisson process and scanning process (for instance by investigating the arrival processes in a computer system), the only equilibrium points in the compound process are the scanning instants. d) Consider a queueing system with Poisson arrival process, constant service time and single server. The number of queueing positions can be nite or innite. Let a point process be dened by the time instants when service starts. All time intervals when the system is idle, will be equilibrium points. During periods, where the system is busy, the time points for accept of new calls for service depends on the instant when the rst call of the busy period started service.

5.2.3

Simple point process

We have already mentioned (5.7) that we exclude processes with multiple arrivals. Denition: A point process is called simple, if the probability that there are more than one event at a given point is zero: p {Nt+t Nt 2} = o(t) . (5.17)

With interval representation, the inter-arrival time distribution must not have a probability mass (atom) at zero, i.e. the distribution is continuous at zero (3.1): F (0+) = 0 (5.18)

Example 5.2.2: Multiple events Time points of trac accidents will form a simple process. Number of damaged cars or dead people will be a non-simple point process with multiple events.

5.3

Littles theorem

This is the only general result that is valid for all queueing systems. It was rst published by Little (1961 [75]). The proof below was shown by applying the theory of stochastic process in (Eilon, 1969 [24]). We consider a queueing system, where customers arrive according to a stochastic process. Customers enter the system at a random time and wait to get service, after being served they leave the system. In Fig. 5.2, both arrival and departure processes are considered as stochastic processes with cumulated number of customers as ordinate.

102

CHAPTER 5. ARRIVAL PROCESSES

We consider a time space T and assume that the system is in statistic equilibrium at initial time t = 0. We use the following notation (Fig. 5.2): N(T ) = A(T ) = number of arrivals in period T . the total service times of all customers in the period T = the shadowed area between curves = the carried trac volume.
N (T ) T A(T ) N (T ) A(T ) T

(T )

= the average call intensity in the period T . = mean holding time in system per call in the period T . = the average number of calls in the system in the period T .

W (T ) = L(T ) =

We have the important relation among these variables: L(T ) = A(T ) W (T ) N(T ) = = (T ) W (T ) T T (5.19)

If the limits of = limT (T ) and W = limT W (T ) exist, then the limiting value of L(T ) also exists and it becomes: L=W (Littles theorem). (5.20)

This simple formula is valid for all general queueing system. The proof had been rened during the years. We shall use this formula in Chaps. 1214.

Example 5.3.1: Littles formula If we only consider the waiting positions, the formula shows: The mean queue length is equal to call intensity multiplied by the mean waiting time. If we only consider the servers, the formula shows: The carried trac is equal to arrival intensity multiplied by mean service time (A = y s = /). This corresponds to the denition of oered trac in Sec. 2.1.

5.3. LITTLES THEOREM

103

Number of events 9 8 7 6 5 4 3 2 1 0 0
. . . . . . . . . . . . . . . . . . . . . . . . . . . . . . . . . . . . . . . . . . . . . . . . . . . . . . . . . . . . . . . . . . . . . . . . . . . . . . . . . . . . . . . . . . . . . . . . . . . . . . . . . . . . . . . . . . . . . . . . . . . . . . . . . . . . . . . . . . . . . . . . . . . . . . . . . . . . . . . . . . . . . . . . . . . . . . . . . . . . . . . . . . . . . . . . . . . . . . . . . . . . . . . . . . . . . . . . . . . . . . . . . . . . . . . . . . . . . . . . . . . . . . . . . . . . . .. . . .. . . . . . . . . . . . . . . . . . . . . . . . . . . . . . . . . . . . . . . . . . . . . . . . . . . . . . . . . . . . . . . . . . . . . . . . . . . . . . . . . . . . . . . . . . . . . . . . . . . . . . . . . . . . . . . . . . . . . . . . . . . . . . . . . . . . . . . . . . . . . . . . . . . . . . . . . . . . . . . . . . . . . . . . . . . . . . . . . . . . . . . . . . . . . . .

Arrival process......................... . . . . ...


. . . . . . . . . . . . . . . . . . . . . . . . . . . . . . . . . . . . . . . . . . . . . . . . . . . . . . . . . . . . . . . . . . . . . . . . . . . . . . . . . . . . . . . .. . .. . . . . .. . . . . . . . . . . . . . . . . . . . . . . . . . . . . . . . . . . . . . . . . . . . . . . . . . . . . . . . . . . . .. ............ .... ...... .

. .

Departure process

Time

Figure 5.2: A queueing system with arrival and departure of customers. The vertical distance between the two curves is equal to the actual number of customers being served. The customers in general dont depart in the the same order as they arrive, so the horizontal distance between the curves dont describe the actual time in the system of a customer.

104

CHAPTER 5. ARRIVAL PROCESSES

Chapter 6 The Poisson process


The Poisson process is the most important point process. Later we will realize that its role among point processes is as fundamental as the role of the Normal distribution among statistical distributions. By the central limit theorem we obtain the Normal distribution when adding random variables. In a similar way we obtain the exponential distribution when superposing stochastic point processes. Most other applied point processes are generalisations or modications of the Poisson process. This process gives a surprisingly good description of many reallife processes. This is because it is the most random process. The more complex a process is, the better it will in general be modelled by a Poisson process. Due to its great importance in practice, we shall study the Poisson process in detail in this chapter. First (Sec. 6.2) we base our study on a physical model with main emphasis upon the distributions associated to the process, and then we shall consider some important properties of the Poisson process (Sec. 6.3). Finally, in Sec. 6.4 we consider the interrupted Poisson process as an example of generalisation.

6.1

Characteristics of the Poisson process

The fundamental properties of the Poisson process are dened in Sec. 5.2: a. Stationary, b. Independent at all time instants (epochs), and c. Simple. (b) and (c) are fundamental properties, whereas (a) is unnecessary. Thus we may allow a Poisson process to have a timedependent intensity. From the above properties we may derive 105

106

CHAPTER 6. THE POISSON PROCESS

other properties that are sucient for dening the Poisson process. The two most important ones are: Number representation: The number of events within a time interval of xed length is Poisson distributed. Therefore, the process is named the Poisson process. Interval representation: The time distance Xi (5.2) between consecutive events is exponentially distributed. In this case using (4.8) and (4.10) FellerJensens identity (5.4) shows the fundamental relationship between the cumulated Poisson distribution and the Erlang distribution (Sec. 6.2.2):
n1

j=0

(t)j t e = j!

x=t

(x)n1 ex dx = 1 F (t) . (n 1)!

(6.1)

This formula can also be obtained by repeated partial integration.

6.2

Distributions of the Poisson process

In this section we consider the Poisson process in a dynamical and physical way (Fry, 1928 [30]) & (Jensen, 1954 [11]). The derivations are based on a simple physical model and concentrate on the probability distributions associated with the Poisson process. The physical model is as follows: Events (arrivals) are placed at random on the real axis in such a way that every event is placed independently of all other events. So we put the events uniformly and independently on the real axes. The average density is chosen as events (arrivals) per time unit. If we consider the axis as a time axis, then on the average we shall have arrivals per time unit. The probability that a given arrival pattern occurs within a time interval is independent of the location of the interval on the time axis.
.. ........................................................................................................................................................................................................................ ............................................ .......................................................................................................................................................................... . . . .. . . .... . . . . . . . . . . .. .. . ................................. ................................. ... ................................. ........ ........................ .. . ........... .......... ... ........... ....... ... .... ... ... ... .. .. .. . .. .

t1

t2

Time

Figure 6.1: When deriving the Poisson process, we consider arrivals within two non overlapping time intervals of duration t1 and t2 , respectively. Let p(, t) denote the probability that events occur within a time interval of duration t. The mathematical formulation of the above model is as follows: 1. Independence: If t1 and t2 are two nonoverlapping intervals (Fig. 6.1), we have because of the independence assumption: p (0, t1 ) p (0, t2 ) = p (0, t1 + t2 ) . (6.2)

6.2. DISTRIBUTIONS OF THE POISSON PROCESS 2. The mean value of the time interval between two successive arrivals is 1/ (3.4):

107

p(0, t) dt =
0

1 ,

0<

1 < .

(6.3)

Here p(0, t) is the probability that there are no arrivals within the time interval (0, t), which is identical to the probability that the time until the rst event is larger than t (the complementary distribution). The mean value (6.3) is obtained directly from (3.4). Formula (6.3) can also be interpreted as the area under the curve p(0, t), which is a neverincreasing function decreasing from 1 to 0. 3. We notice that (6.2) implies that the event no arrivals within the interval of length 0 is sure to take place: p(0, 0) = 1 . (6.4) 4. We also notice that (6.3) implies that the probability of no arrivals within a time interval of length is zero and never takes place: p(0, ) = 0 . (6.5)

6.2.1

Exponential distribution

The fundamental step in the following derivation of the Poisson distribution is to derive p(0, t) which is the probability of no arrivals within a time interval of length t, i.e. the probability that the rst arrival appears later than t. We will show that {1 p(0, t)} is an exponential distribution (cf. Sec. 4.1). From (6.2) we have: ln p (0, t1 ) + ln p (0, t2 ) = ln p (0, t1 + t2 ) . Letting ln p(0, t) = f (t), (6.6) can be written as: f (t1 ) + f (t2 ) = f (t1 + t2 ) . By dierentiation with respect to e.g. t2 we have: f (t2 ) = ft2 (t1 + t2 ) . From this we notice that f (t) must be a constant and therefore: f (t) = a + b t . By inserting (6.8) into (6.7), we obtain a = 0. Therefore p(0, t) has the form: p(0, t) = ebt . (6.8) (6.7) (6.6)

108 From (6.3) we obtain b : 1 = or:

CHAPTER 6. THE POISSON PROCESS

p(0, t) dt =
0 0

1 ebt dt = , b

b = . Thus on the basis of item (1) and (2) above we have shown that: p(0, t) = et . (6.9)

If we consider p(0, t) as the probability that the next event arrives later than t, then the time until next arrival is exponentially distributed (Sec. 4.1): 1 p(0, t) = F (t) = 1 et , F (t) = f (t) = et , We have the following mean value and variance (4.4): m1 = 2 = 1 , 1 . 2 (6.12) > 0, > 0, t 0, t 0. (6.10) (6.11)

The probability that the next arrival appears within the interval (t, t + dt) may be written as: f (t) dt = et dt = p(0, t) dt , (6.13)

i.e. the probability that an arrival appears within the interval (t, t + dt) is equal to dt, independent of t and proportional to dt (3.17). Because is independent of the actual age t, the exponential distribution has no memory (cf. Secs. 4.1 & 3.1.2). The process has no age. The parameter is called the intensity or rate of both the exponential distribution and of the related Poisson process and it corresponds to the intensity in (5.6). The exponential distribution is in general a very good model of call inter-arrival times when the trac is generated by human beings (Fig. 6.2).

6.2. DISTRIBUTIONS OF THE POISSON PROCESS Number of observations 2000 1000 = Theory 500 200 100 50 20 10 5 2 1 0 4 8 12 16 20 Interarrival time [scan=0.2s] 5916 Observations

109

Figure 6.2: Interarrival time distribution of calls at a transit exchange. The theoretical values are based on the assumption of exponentially distributed interarrival times. Due to the measuring principle (scanning method) the continuous exponential distribution is transformed into a discrete Westerberg distribution (15.14) (2 -test = 18.86 with 19 degrees of freedom, percentile = 53).

6.2.2

Erlangk distribution

From the above we notice that the time until exactly k arrivals have appeared is a sum of k IID (independently and identically distributed) exponentially distributed random variables. The distribution of this sum is an Erlangk distribution (Sec. 4.2) and the density is given by (4.8): gk (t) dt = (t)k1 t e dt , (k 1)! > 0, t 0, k = 1, 2, . . . . (6.14)

For k = 1 we of course get the exponential distribution. The distribution gk+1(t), k > 0, is obtained by convolving gk (t) and g1 (t). If we assume that the expression (6.14) is valid for

110 gk (t), then we have by convolution:


t

CHAPTER 6. THE POISSON PROCESS

gk+1 (t) =

0 t

gk (tx) g1 (x) dx {(t x)}k1 (tx) x e e dt (k 1)!


t 0

=
0

k+1 t e (k 1)!

(t x)k1 dx

(t)k t e . k!

As the expression is valid for k = 1, we have by induction shown that it is valid for any k. The Erlang-k distribution is, from a statistical point of view, a special gamma-distribution. The mean value and the variance are obtained from (6.12): m1 = 2 = k , k , 2 1 . k (6.15)

= 1+

Example 6.2.1: Call statistics from an SPC-system (cf. Example 5.1.2) Let calls arrive to a stored programcontrolled telephone exchange (SPCsystem) according to a Poisson process. The exchange automatically collects full information about every 1000th call. The inter-arrival times between two registrations will then be Erlang1000 distributed and have the form factor = 1.001, i.e. the registrations will take place very regularly.

6.2.3

Poisson distribution

We shall now show that the number of arrivals in an interval of xed length t is Poisson distributed with mean value t. When we know the above-mentioned exponential distribution and the Erlang distribution, the derivation of the Poisson distribution is only a matter of applying simple combinatorics. The proof can be carried through by induction. We want to derive p(i, t) = probability of i arrivals within a time interval t. Let us assume that: (t)i1 t p(i 1, t) = e , > 0 , i = 1, 2, . . . (i 1)!

6.2. DISTRIBUTIONS OF THE POISSON PROCESS

111

This is correct for i = 0 (6.9). The interval (0, t) is divided into three nonoverlapping intervals (0, t1 ) , (t1 , t1 + dt1 ) and (t1 + dt1 , t). From the earlier independence assumption we know that events within an interval are independent of events in the other intervals, because the intervals are nonoverlapping. By choosing t1 so that the last arrival within (0, t) appears in (t1 , t1 + dt1 ), then the probability p(i, t) is obtained by the integrating over all possible values of t1 as a product of the following three probabilities: a) The probability that (i 1) arrivals occur within the time interval (0, t1 ): p (i 1, t1 ) = (t1 )i1 t1 e , (i 1)! 0 t1 t .

b) The probability that there is just one arrival within the time interval from t1 to t1 + dt1 : dt1 . c) The probability that no arrivals occur from t1 + dt1 to t: e(tt1 ) . The product of the rst two probabilities is the probability that the ith arrival appears in (t1 , t1 + dt1 ), i.e. the Erlang distribution from the previous section. By integration we have:
t

p(i, t) =
0

(t1 )i1 t1 e dt1 e(tt1 ) (i 1)!


t 0

i et = (i 1)! p(i, t) = (t)i t e . i!

t1i1 dt1 ,

(6.16)

This is the Poisson distribution which we thus have obtained from (6.9) by induction. The mean value and variance are: m1 = t , 2 = t . (6.17) (6.18)

The Poisson distribution is in general a very good model for the number of calls in a telecommunication system (Fig. 6.3) or jobs in a computer system.
Example 6.2.2: Slotted Aloha Satellite System Let us consider a digital satellite communication system with constant packet length h. The satellite

112 Number of observations 150

CHAPTER 6. THE POISSON PROCESS

125

900 observations = 6.39 calls/s = theory

100

75

50

25

0 0 2 4 6 8 10 12 14 16 18 Number of calls/s

Figure 6.3: Number of Internet dial-up calls per second. The theoretical values are based on the assumption of a Poisson distribution. A statistical test accepts the hypothesis of a Poisson distribution.
is in a geostationary position about 36.000 km above equator, so the round trip delay is about 280 ms. The time axes is divided into slots of xed duration corresponding to the packet length h. The individual terminal (earth station) transmits packets so that they are synchronised with the time slots. All packets generated during a time slot are transmitted in the next time-slot. The transmission of a packet is only correct if it is the only packet being transmitted in a time slot. If more packets are transmitted simultaneously, we have a collision and all packets are lost and must be retransmitted. All earth stations receive all packets and can thus decide whether a packet is transmitted correctly. Due to the time delay, the earth stations transmit packets independently. If the total arrival process is a Poisson process (rate ), then we get a Poisson distributed number of packets in each time slot. p(i) = The probability of a correct transmission is: p(1) = h eh . (6.20) (h)i h . e i! (6.19)

This corresponds to the proportion of the time axes which is utilised eectively. This function, which is shown in Fig. 6.4, has an optimum for h = 1, as the derivative with respect to h is zero

6.2. DISTRIBUTIONS OF THE POISSON PROCESS


for this value: ph (1) = eh (1 h) , Max{p(1)} = e1 = 0.3679 .

113

(6.21) (6.22)

We thus have a maximum utilisation of the channel equal to 0.3679, when on the average we transmit one packet per time slot. A similar result holds when there is a limited number of terminals and the number of packets per time slot is Binomially distributed.

Carried trac 1.0 0.9 0.8 0.7 0.6 0.5 0.4 0.3 0.2 0.1
.............................................................................................................................................. ................................................................................................................................................ .. . .. . .. . .. . .. . .. . .. . .. . .. . .. .. .. . .. . .. . .. . .. . .. .. .. . .. .. .. . .. . .. . .. . .. . .. . .. . .. . .. . .. . . . .. .. . .. . .. . .. . .. . .. .. .. ............................. ............................... . .......... ......... ......... ........ .. ... .. ........ ........ ...... . .. ....... ..... ....... ..... . ....... ....... .... .... .. .. .. ...... ...... . ... .. ... ...... ....... . . ...... . ...... ... .. ... . ...... ...... . ... .. ... ...... ...... . . ...... . ...... .. .... ... ....... ....... . ... ...... .. .... ....... ....... . .. ....... .. .. . ....... ....... . .. ........ .. ... ....... ....... . .. ........ ............................... .. ... .................................... ........ ........ ....... ....... .... .... .. ....... .... ..... . .... ...... ...... . . ... ...... ......... ...... . ...... ....... ...... ....... ....... ....... . .. ....... ..... .. ....... .. ........ .... ........ .... ........ ........ .. ......... ... ... ......... ... . ........... ........... ... ... ............. ............. .. .. .................. ................... ... . . ............................ ............................. .. .. ........................ .................... . .. .

Ideal

Slotted Aloha

Simple Aloha

0.0 0.00 0.25 0.50 0.75 1.00 1.25 1.50 1.75 2.00 2.25 2.50 2.75 3.00 Oered trac Figure 6.4: The carried trac in a slotted Aloha system has a maximum (example 6.2.2). The Simple Aloha protocol is dealt with in example 7.2.1.

6.2.4

Static derivation of the distributions of the Poisson process

As it is known from statistics, these distributions can also be derived from the Binomial process by letting the number of trials n (e.g. throws of a die) increase to innity and at the same time letting the probability of success in a single trial p converge to zero in such a way that the average number of successes n p is constant. This approach is static and does not stress the fundamental properties of the Poisson process which has a dynamic independent existence. But it shows the relationship between the two processes as illustrated in Table 6.1.

114

CHAPTER 6. THE POISSON PROCESS

The exponential distribution is the only continuous distribution with lack of memory, and the geometrical distribution is the only discrete distribution with lack of memory. For example, the next outcome of a throw of a die is independent of the previous outcome. The distributions of the two processes are shown in Table 6.1.

6.3

Properties of the Poisson process

In this section we shall show some fundamental properties of the Poisson process. From the physical model in Sec. 6.2 we have seen that the Poisson process is the most random point process that may be found (maximum disorder process). It yields a good description of physical processes when many dierent factors are behind the total process.

6.3.1

Palms theorem (Superposition theorem)

The fundamental properties of the Poisson process among all other point processes were rst discussed by the Swede Conny Palm. He showed that the exponential distribution plays the same role for stochastic point processes (e.g. interarrival time distributions), where point processes are superposed, as the Normal distribution does when stochastic variables are added up (the central limit theorem). process 1
.. . .. ................................................................................................................................................................................................................................................ .............................................................................................................................................................................................................................................. ... ..

process 2

.. . .. ................................................................................................................................................................................................................................................ .............................................................................................................................................................................................................................................. ... ..

. . . process n

. . .

.. . .. ................................................................................................................................................................................................................................................ .............................................................................................................................................................................................................................................. ... ..

Total process

.. . .. ................................................................................................................................................................................................................................................ .............................................................................................. ................................................................................................................................................ . ... .. . . . . .. .. . ... .. ... . . . . . . .

Time

Random point of time

Figure 6.5: By superposition of n point processes we obtain under certain assumptions a process which locally is a Poisson process.

6.3. PROPERTIES OF THE POISSON PROCESS

115

BINOMIAL PROCESS Discrete time Probability of success:

p,

0<p<1

POISSON PROCESS Continuous time Intensity of succes:

>0

Number of attempts since previous success or since a random attempt to get a success GEOMETRIC DISTRIBUTION p(n) = p (1 p)n1 , m1 = 1 , p n = 1, 2, . . . 2 = 1p p2

Interval between two successes or from a random point until next success EXPONENTIAL DISTRIBUTION f (t) = et , m1 = 1 , t0 2 = 1 2

Number of attempts to get k successes PASCAL = NEGATIVE BINOMIAL DISTR. p(n|k) = k , p n1 k p (1 p)nk , n k k1 2 = k(1 p) p2

Time interval until kth success ERLANGK DISTRIBUTION f (t|k) = k , (t)k1 et , (k 1)! 2 = k 2 t0

m1 =

m1 =

Number of successes in n attempts BINOMIAL DISTRIBUTION p(x| n) = m1 = p n , n x p (1 p)nx , x x = 0, 1, . . .

Number of successes in a time interval t POISSON DISTRIBUTION f (x| t) = (t)x t , e x! t0 2 = t

2 = p n (1p)

m1 = t ,

Table 6.1: Correspondence between the distributions of the Binomial process and the Poisson process. A success corresponds to an event or an arrival in a point process. Mean value = m1 , variance = 2 . For the geometric distribution we may start with a zero class. The mean value is then reduced by one whereas the variance is the same.

116

CHAPTER 6. THE POISSON PROCESS

Theorem 6.1 Palms theorem: by superposition of many independent point processes the resulting total process will locally be a Poisson process. The termlocally means that we consider time intervals which are so short that each process contributes at most with one event during this interval. This is a natural requirement since no process may dominate the total process (similar conditions are assumed for the central limit theorem). The theorem is valid only for simple point processes. If we consider a random point of time in a certain process, then the time until the next arrival is given by (3.23). We superpose n processes into one total process. By appropriate choice of the time unit the mean distance between arrivals in the total process is kept constant, independent of n. The time from a random point of time to the next event in the total process is then given by (3.23): n t p{T t} = 1 1 Vi . (6.23) n i=1 If all sub-processes are identical, we get: p{T t} = 1 1 V From (3.23) and (5.18) we nd (letting = 1):
t0

t n

(6.24)

lim v(t) = 1 ,
t

and thus: V (t) =


0

1 dt = t .

(6.25)

Therefore, we get from (6.24) by letting the number of sub-processes increase to innity: p{T t} = lim t 1 1 n
n

= 1 et .

(6.26)

which is the exponential distribution. We have thus shown that by superposition of identical processes we locally get a Poisson process. In a similar way we may superpose non-identical processes and obtain a Poisson process locally.
Example 6.3.1: Life-time of a route in an ad-hoc network A route in a network consists of a number of links connecting the end-points of the route (Chap. 11). In an ad-hoc network links exist for a limited time period. The life-time of a route is therefore the time until the rst link is disconnected. From Palms theorem we see that the life-time of the route tends to be exponentially distributed.

6.3. PROPERTIES OF THE POISSON PROCESS

117

6.3.2

Raikovs theorem (Decomposition theorem)

A similar theorem, the decomposition theorem, is valid when we split a point process into sub-processes, when this is done in a random way. If there are n times fewer events in a sub-process, then it is natural to reduce the time axes with a factor n. Theorem 6.2 Raikovs theorem: by a random decomposition of a point process into subprocesses, the individual sub-process converges to a Poisson process, when the probability that an event belongs to the sub-process tends to zero. This is also seen from the following general result. If we generate a sub-process by random splitting of a point process choosing an event with probability p, then the sub-process has the formfactor p : p = 2 + p ( 2) , where is the form factor of the original process. In addition to superposition and decomposition (merge and split, or join and fork), we can make another operation on a point process, namely translation (displacement) of the individual events. When this translation for every event is a random variable, independent of all other events, an arbitrary point process will converge to a Poisson process. As concerns point processes occurring in reallife, we may, according to the above, expect that they are Poisson processes when a suciently large number of independent conditions for having an event are fullled. This is why the Poisson process is a good description of for instance the arrival processes to a local exchange from all local subscribers. As an example of limitations in Palms theorem (Theorem 6.1) it can be shown that the superposition of two independent processes yields an exact Poisson process only if both sub processes are Poisson processes.

6.3.3

Uniform distribution a conditional property

In Sec. 6.2 we have seen that a uniform distribution in a very large interval corresponds to a Poisson process. The inverse property is also valid: Theorem 6.3 If for a Poisson process we have n arrivals within an interval of duration t, then these arrivals are uniformly distributed within this interval. The length of this interval can itself be a random variable if it is independent of the Poisson process. This is for example the case in trac measurements with variable measuring intervals (Chap. 15). This can be shown both from the Poisson distribution (number representation) and from the exponential distribution (interval presentation).

118

CHAPTER 6. THE POISSON PROCESS

Channels

Total traffic [erlang]

Overflow link

Primary link

ON OFF
Figure 6.6: Overow system with Poisson arrival process (intensity ). Normally, calls arrive to the primary group. During periods when all n trunks in the primary group are busy, all calls are oered to the overow group.

6.4. GENERALISATION OF THE STATIONARY POISSON PROCESS

119

6.4

Generalisation of the stationary Poisson process

The Poisson process has been generalised in many ways. In this section we only consider the interrupted Poisson process, but further generalisations are MMPP (Markov Modulated Poisson Processes) and MAP (Markov Arrival Processes).

6.4.1

Interrupted Poisson process (IPP)

Due to its lack of memory the Poisson process is very easy to apply. In some cases, however, the Poisson process is not sucient to describe a real arrival process as it has only one parameter. Kuczura (1973 [70]) proposed a generalisation which has been widely used. The idea of generalisation comes from the overow problem (Fig. 6.6 & Sec. 9.2). Customers arriving at the system will rst try to be served by a primary system with limited capacity (n servers). If the primary system is busy, then the arriving customers will be served by the overow system. Arriving customers are routed to the overow system only when the
.......... ........... ... .. ... .. .. . .. . . . . .... .. . . .......................................................................... .. . . . .. . .. .................. .. . .. ................... . . .. . .................................................................... .... . . .. . .. . . .. .. . .. .. .. . ... . . ... .. ... ........... .. . ......... .. .. .. .. .. .. .. .. .. .. ... ... .. .. . .. . . . . .. . .. . . . . .. .. . . . . . . . .. . . .. . . . . . . .. . . .. . . . . . . .. . .. . . . ......... ......... . . . . . . . . ........................................................... ........................................................... . .. .. . . . . . .. . .. . . . . .. . .. . . . . .. .. . . . . .. . . .. . . . . .. . . .. . . . .. . .. . . . . .. .. .. .. .. .. .. .. ... .. ... .. .. ... .. .. .. .......... .. .. ........... . . .. .. .. ... .. ... .. .. .. .. . .. .. . .. . . .................... .. ................... . ......................................................................... . .... .. .. . .. . .. . .. . ... . ........................................................................... . . . .. . .. .. . ... ... .. .. .......... .........

on

IPP arrival process

Poisson process

Switch

Arrivals ignored

Figure 6.7: Illustration of the interrupted Poisson process (IPP) (cf. Fig. (6.6)). The position of the switch is controlled by a two-state Markov process. primary system is busy. During the busy periods customers arrive at the overow system according to the Poisson process with intensity . During the non-busy periods no calls arrive to the overow system, i.e. the arrival intensity is zero. Thus we can consider the arrival process to the overow system as a Poisson process which is either On or O (Fig. 6.7). As a simplied model to describe these On (O ) intervals, Kuczura used exponentially distributed time intervals with intensity ( ). He showed that this corresponds to hyper-exponentially distributed interarrival times to the overow link, which are illustrated by a phasediagram in Fig 6.8. It can be shown that the parameters are related as follows: = p 1 + (1 p)2 , = 1 2 , + + = 1 + 2 . (6.28)

120
... .. .. ................................. ................................... .. ... ... ... ... ... ... ... ... . . ... ... ... ... ... ... ... ... ... ... .. .. ... ... ... ... .. . .................................... ................................... .... . . .. . .. ... ... ... ... ... ... ... ... ... ... ... ... ... ... ... ... ... ... ... ... ... ... ... ................................... .................................. .. .. .. . ...

CHAPTER 6. THE POISSON PROCESS 1


.................................. .................................. ... ... ... ... ... ... ... ... ... ... ... ... ... ... ... ... ... ... ... ... ... ... ... ... .... .................................. . . .. . .................................. . . ... ... ... ... ... ... . . ... ... . . ... ... ... ... ... ... ... ... ... ... . . ... ... ... ... .................................. ..................................

1p

Figure 6.8: The interrupted Poisson process is equivalent to a hyperexponential arrival process (6.28). Because a hyperexponential distribution with two phases can be transformed into a Cox2 distribution (Sec. 4.4.2), the IPP arrival process is a Cox-2 arrival processes as shown in Fig. 4.10. We have three parameters available, whereas the Poisson process has only one parameter. This makes it more exible for modelling empirical data.

Chapter 7 Erlangs loss system and Bformula


In this and the following chapters we consider the classical teletrac theory developed by Erlang (Denmark), Engset (Norway) and Fry & Molina (USA). It has successfully been applied for more than 80 years. In this chapter we consider the fundamental Erlang-B formula. In Sec. 7.1 we put forward the assumptions for the model. Sec. 7.2 deals with the case with innite capacity, which results in a Poisson distributed number of busy channels. In Sec. 7.3 we consider a limited number of channels and obtain the truncated Poisson distribution and Erlangs B-formula. In Sec. 7.4 we describe a standard procedure (cook book) for dealing with state transition diagrams. This is the key to classical teletrac theory. We also derive an accurate recursive formula for numerical evaluation of Erlangs B-formula in Sec. 7.5. Finally, in Sec. 7.6 we study the basic principles of dimensioning, where we balance the GradeofService (GoS) and the costs of the system.

7.1

Introduction

Erlangs B-formula is based on the following model, described by the three elements structure, strategy, and trac(Fig. 1.1): a. Structure: We consider a system of n identical channels (servers, trunks, slots) working in parallel. This is called a homogeneous group. b. Strategy: A call arriving at the system is accepted for service if at least one channel is idle. Every call needs one and only one channel. We say the group has full accessibility. Often the term full availability is used, but this terminology will only be used in connection with reliability aspect. If all channels are busy the system is congested and a call attempt is blocked. It The blocked (= rejected, lost, congested) call attempt disappears without any after-eect as it may be accepted by an alternative route. This strategy is the most important one and has been applied with success for many years. 121

122

CHAPTER 7. ERLANGS LOSS SYSTEM AND BFORMULA It is called Erlangs loss model or the Lost Calls Cleared = LCCmodel.

c. Trac: In the following we assume that the service times are exponentially distributed with intensity (corresponding to a mean value 1/), and that the arrival process is a Poisson process with rate . This type of trac is called Pure Chance Trac type One, PCT-I. The trac process then becomes a pure birth and death process, a simple Markov process which is easy to deal with mathematically. Denition of oered trac: We dene the oered trac as the trac carried when the number of channels (the capacity) is innite (2.2). In Erlangs loss model with Poisson arrival process this denition of oered trac is equivalent to the average number of call attempts per mean holding time: A= We consider two cases: 1. n = : Poisson distribution (Sec. 7.2), 2. n < : Truncated Poisson distribution (Sec. 7.3). We shall later see that this model is insensitive to the holding time distribution, i.e. only the mean holding time is of importance for the state probabilities. The type of distribution has no importance for the state probabilities. Performancemeasures: The most important grade-of-service measures for loss systems are time congestion E, call congestion B, and trac (load) congestion C. They are all equal for Erlangs loss model because of the Poisson arrival process (PASTAproperty, Sec. 6.3). 1 = . (7.1)

7.2

Poisson distribution

We assume the arrival process is a Poisson process and that the holding times are exponentially distributed, i.e. we consider PCT-I trac. The number of channels is assumed to be innite, so we never observe congestion (blocking).

7.2.1

State transition diagram

We dene the state of the system, [ i ], as the number of busy channels i (i = 0, 1, 2, . . .). In Fig. 7.1 all states of the system are shown as circles, and the rates by which the trac process changes from one state to another state are shown upon the arcs of arrows between the states. As the process is simple (Sec. 5.1), we only have transitions to neighbouring states. If we assume the system is in statistical equilibrium, then the system will be in state [ i ] the

7.2. POISSON DISTRIBUTION


........... ... .......... ............... . .................. ..... .. . .... ... ....... ...... ... ... .... .. ......... . ... .... .... . .... .. .. ......... .... . .. ..... .... ........ . . .. .... .. .. .. .. .. .. .. .. .. . . . . . . . . . . . . . . . . . . . . . . . . . . . . . . . . . .. .. . . .. .. . .. .. ... . . .. .. ... .. . ... .... . ................ ............... ..... .. ... ......... ...... ... ..... .... ... ....... ...... ... . . ................ .............. ............... ..............

123
. .... ........... .... .......... . ............... . ......... ........ .................. ..... ......... ..... .. . ... ....... ...... .... ... ........ ..... ... .... .. ......... . ... .... ........... ..... .... .... .. . .. .... ... . .. .. ... . ... ... . .. ... .. .. .. .. .. .. .. .. .. . . . . . . . . . . . . . . . . . . . . . . . . . . . . . . . . . . . . .. .. . . .. .. . .. . . .. . .. .. .. . . .. . .... .... . ....... ....... ..... . ........ ...... ........ .... ... .. .. .... ... . ...... ...... ... . ...... ...... ... ... ..... . ... ..... ... ..... ............... ............... .............. .............. ............... ..............

i1

(i1)

(i+1)

Figure 7.1: The Poisson distribution. State transition diagram for a system with innitely many channels, Poisson arrival process (), and exponentially distributed holding times (). proportion of time p(i), where p(i) is the probability of observing the system in state [ i ] at a random point of time, i.e. a time average. When the process is in state [ i ] it will jump to state [ i+1] times per time unit and to state [ i1] i times per time unit. Of course, the process will leave state [ i ] at the moment there is a state transition. The future development of the trac process only depends upon the present state, not upon how the process came to this state (the Markov-property). The equations describing the states of the system under the assumption of statistical equilibrium can be set up in two ways, which both are based on the principle of global balance: a. Node equations In statistical equilibrium the number of transitions per time unit into state [ i ] equals the number of transitions out of state [ i ]. The equilibrium state probability p(i) denotes the proportion of time (total time per time unit) the process is in state [ i ]. The average number of jumps from state [ 0 ] to state [ 1 ] is p(0) per time unit, and the average number of jumps from state [ 1 ] to state [ 0 ] is p(1) per time unit. For state [ i ] we get the following equilibrium or balance equation: p(0) = p(1) , p(i1) + (i + 1) p(i+1) = ( + i ) p(i) , i = 0, i > 0. (7.2) (7.3)

The node equations are always applicable, also for state transition diagrams in several dimensions, which we consider in later chapters. b. Cut equations In many cases we may exploit a simple structure of the state transition diagram. If we put a ctitious cut for example between the states [ i1 ] and [ i ] (corresponding to a global cut around the states [ 0 ], [ 1 ], . . . [ i1] ), then in statistical equilibrium the trac process changes from state [ i1 ] to [ i ] the same number of times as it changes from state [ i ] to [ i1 ]. In statistical equilibrium we thus have per time unit: p(i1) = i p(i) , i = 1, 2, . . . . (7.4)

Cut equations are primarily used for one-dimensional state transition diagrams, whereas node equations are applicable to any diagram.

124

CHAPTER 7. ERLANGS LOSS SYSTEM AND BFORMULA

As the system always will be in some state, we have the normalisation restriction:

p(i) = 1 ,
i=0

p(i) 0 .

(7.5)

We notice that node equations (7.3) involve three state probabilities, whereas the cut equations (7.4) only involve two. Therefore, it is easier to solve the cut equations. Loss system will always be able to enter statistical equilibrium if the arrival process is independent of the state of the system. We shall not consider the mathematical conditions for statistical equilibrium in this chapter.

7.2.2

Derivation of state probabilities

For one-dimensional state transition diagrams the application of cut equations is the most appropriate approach. From Fig. 7.1 we get the following balance equations:

p(0) = p(1) , p(1) = 2 p(2) , ... ...

p(i2) = (i 1) p(i1) , p(i1) = i p(i) , p(i) = (i + 1) p(i+1) , ... ... .

Expressing all state probabilities by p(0) yields, when we introduce the oered trac A = / :

7.2. POISSON DISTRIBUTION

125

p(0) p(1) p(2) ... p(i1) = p(i) = = A p(1) 2 ...

= = =

p(0) , A p(0) , A2 p(0) , 2 ...

Ai1 A p(i2) = p(0) , i1 (i 1)! A p(i1) i A p(i) i+1 ... = = Ai p(0) , i! Ai+1 p(0) , (i + 1)! ...

p(i+1) = ...

The normalisation constraint (7.5) implies:

1 =
j=0

p(j) A2 Ai ++ + ... 2! i!

= p(0) 1 + A + = p(0) eA , p(0) = eA ,

and thus the Poisson distribution: Ai A e , i!

p(i) =

i = 0, 1, 2, . . . .

(7.6)

The number of busy channels at a random point of time is thus Poisson distributed with both mean value (6.17) and variance (6.18) equal to A. We have earlier shown that the number of calls in a xed time interval also is Poisson distributed (6.16). Thus the Poisson distribution is valid both in time and in space. We would, of course, obtain the same solution by using node equations.

126

CHAPTER 7. ERLANGS LOSS SYSTEM AND BFORMULA

7.2.3

Trac characteristics of the Poisson distribution

From a dimensioning point of view, the system with unlimited capacity is not very interesting. We summarise the important trac characteristics of the loss system: Time congestion: Call congestion: Carried trac: Lost trac: Trac congestion: E = 0, B = 0,

Y A

=
i=1

i p(i) = A ,

= A Y = 0,

C = 0.

Carried trac by the ith trunk assuming sequential hunting is given later in (7.14). Peakedness Z is dened as the ratio between variance and mean value of the distribution of state probabilities (cf. IDC, Index of Dispersion of Counts). For the Poisson distribution we nd (6.17) & (6.18): 2 Z= = 1. (7.7) m1 The peakedness has dimension [number of channels] and is dierent from the coecient of variation which has no dimension (3.9). Duration of state [ i ]: In state [ i ] the process has the total intensity ( + i ) away from the state. Therefore, the time until the rst transition (state transition to either i + 1 or i 1) is exponentially distributed (Sec. 4.1.1): fi (t) = ( + i )e( + i ) t , t 0.

Example 7.2.1: Simple Aloha protocol In example 6.2.2 we considered the slotted Aloha protocol, where the time axes was divided into time slots. We now consider the same protocol in continuous time. We assume that packets arrive according to a Poisson process and that they are of constant length h. The system corresponds to the trac case resulting in a Poisson distribution which also is valid for constant holding times (Sec. 7.2). The state probabilities are given by the Poisson distribution (7.6) where A = h. A packet is only transmitted correctly if (a) the system is in state [ 0 ] at the arrival time and (b) no other packets arrive during the service time h. We nd: pcorrect = p(0) e h = e2 A . The trac transmitted correctly thus becomes: Acorrect = A pcorrect = A e2 A .

7.3. TRUNCATED POISSON DISTRIBUTION

127

This is the proportion of the time axis which is utilised eciently. It has an optimum for h = A = 1/2, where the derivative with respect to A equals zero: Acorrect = e2 A (1 2 A) , A 1 max{Acorrect } = = 0.1839 . 2e (7.8)

We thus obtain a maximum utilisation equal to 0.1839 when we oer 0.5 erlang. This is half the value we obtained for a slotted system by synchronising the satellite transmitters. The models are compared in Fig. 6.4.

7.3

Truncated Poisson distribution

We still assume Pure Chance Trac Type I (PCT-I) as in Sec. 7.2. The number of channels is now limited so that n is nite. The number of states becomes n+1, and the state transition diagram is shown in Fig. 7.2.

Figure 7.2: The truncated Poisson distribution. State transition diagram for a system with a limited number of channels (n), Poisson arrival process (), and exponential service times ().

7.3.1

State probabilities

We get similar cut equations as for the Poisson case, but the state space is limited to {0, 1, . . . , n} and the normalisation condition (7.5) now becomes:
n

p(0) =
j=0

Aj j!

We get the so-called truncated Poisson distribution (Erlangs rst formula): Ai p(i) = n i! j , A j! j=0 0 i n.

(7.9)

128

CHAPTER 7. ERLANGS LOSS SYSTEM AND BFORMULA

The name truncated means cut o and is due to the fact that the solution may be interpreted as a conditional Poisson distribution p(i | i n). This is easily seen by multiplying both numerator and denominator by eA .

7.3.2

Trac characteristics of Erlangs B-formula

Knowing the state probabilities we are able to nd performance measures dened by state probabilities. Time congestion: The probability that all n channels are busy at a random point of time is equal to the proportion of time all channels are busy (time average). This is obtained from (7.9) for i = n: An n! En (A) = p(n) = . (7.10) An A2 ++ 1+A+ 2! n! This is Erlangs famous B-formula (1917, [11]). It is denoted by En (A) = E1,n (A), where index one refers to the alternative name Erlangs rst formula. Call congestion: The probability that a random call will be lost is equal to the proportion of call attempts blocked. If we consider one time unit, we nd B = Bn (A): B= p(n)
n

= p(n) = En (A) .

(7.11)

p()
=0

Carried trac: If we use the cut equation between states [ i1 ] and [ i ] we get:
n n

Y Y

=
i=1

i p(i) =
i=1

p(i1) = A {1 p(n)} , (7.12)

= A {1 En (A)} ,

where A is the oered trac. The carried trac will be less than both A and n. Lost trac: A = A Y = A En (A) . Trac congestion: C= AY = En (A) . A

7.3. TRUNCATED POISSON DISTRIBUTION

129

We thus have E = B = C because the call intensity is independent of the state. This is the PASTAproperty which is valid for all systems with Poisson arrival processes: Poisson Arrivals See Time Averages. In all other cases at least two of the three congestion measures are dierent. Erlangs B-formula is shown graphically in Fig. 7.3 for some selected values of the parameters. Trac carried by the ith channel (the utilisation ai ) : 1. Random hunting: In this case all channels carry the same trac on the average. The total carried trac is independent of the hunting strategy and we nd the utilisation: ai = a = A {1 En (A)} Y = . n n (7.13)

This function is shown in Fig. 7.4, and we observe that for a given congestion E we obtain the highest utilisation for large channel groups (economy of scale). 2. Ordered hunting = sequential hunting: The trac carried by channel i is the dierence between the trac lost from i1 channels and the trac lost from i channels: ai = A {Ei1 (A) Ei (A)} . (7.14)

It should be noticed that the trac carried by channel i is independent of the total number of channels. Thus channels after channel i have no inuence upon the trac carried by channel i. There is no feed-back. Improvement function: This denotes the increase in carried trac when the number of channels is increased by one from n to n + 1: Fn (A) = Yn+1 Yn , = A{1 En+1 } A{1 En } , Fn (A) = A {En (A) En+1 (A)} = an+1 . We have: 0 Fn (A) 1 . The improvement function Fn (A) is tabulated in Moes Principle (Arne Jensen, 1950 [50]) and shown in Fig. 7.5. In Sec. 7.6.2 we consider the application of this principle for optimal economic dimensioning. Peakedness: (7.15) (7.16)

130

CHAPTER 7. ERLANGS LOSS SYSTEM AND BFORMULA

This is dened as the ratio between the variance and the mean value of the distribution of the number of busy channels, cf. IDC (5.11). For the truncated Poisson distribution by using (7.14) it can be shown that: 2 (7.17) = 1 A {En1 (A) En (A)} = 1 an , m The dimension is [number of channels]. In a group with ordered hunting we may thus estimate the peakedness from the trac carried by the last channel. Z= Duration of state [ i ]: The total intensity for leaving state [ i ] is constant and equal to ( + i ), and therefore the duration of the time in state [ i ] (sojourn time) is exponentially distributed with density function: f (t) = ( + i ) e( + i ) t , 0 i < n ,
i

fn (t) = (n ) e(n ) t ,

i = n.

(7.18)

7.3.3

Generalisations of Erlangs B-formula

The literature on the Bformula is very extensive. Here we only mention a couple of important properties. Insensitivity: It can be shown that Erlangs B-formula, which above is derived under the assumption of exponentially distributed holding times, is valid for arbitrary holding time distributions. The state probabilities for both the Poisson distribution (7.6) and the truncated Poisson distribution (7.9) only depend on the holding time distribution through the mean value which is included in the oered trac A. It can be shown that all classical loss systems with full accessibility are insensitive to the holding time distribution. The fundamental assumption for the validity of Erlangs B-formula is thus a Poisson arrival process. According to Palms theorem this is fullled when the trac is originated by many independent sources. This is fullled in ordinary telephone systems under normal trac conditions. The formula is thus very robust. The combined arrival process and service time process are described by a single parameter A. This explains the wide application of the B-formula both in the past and today. Continuous number of channels: Erlangs Bformula can mathematically be generalised to non-integral number of channels (including a negative number of channels). This is useful when for instance we want to nd the number of channels n for a given oered trac A and blocking probability E. In Chap. 9 we will also use this for dealing with overow trac.

7.3. TRUNCATED POISSON DISTRIBUTION

131

Blocking probability E (A) 1.0


....................... .......................... ............................... .............................. ........................ ........................ ................... .................. ....... ........ ................ ............. ............ ... ...... ......... .......... .................... ................... ... ...... .......... ................. ................. ........ ........ ............... .............. ....... ....... ............. ............. ...... ...... ........... ........... ...... .......... ...... .......... . ... ... ........ ......... ..... ..... ......... ........ .... .... ........ ........ .... .... ....... ....... ...... .... ...... .... ... ...... ... ...... .. . ... ... ..... ... ..... ... ..... ... ..... ... ..... ..... ... ... .. . ... ..... ...... ....... ... .... ......... .... ... ......... . .. . ... ........ ........ .. .... .. ... .... .... ........ .. .... .. ....... ... ....... . . ...... ...... ... .. ... .. .. .... ....... ... ... .. ...... ...... .. ... ... ...... ...... .. .. ... ... .. ... ...... . ... ..... .. ..... ... .. ... .... .. .. .. ..... ..... ... ... . ..... ..... .. ... ... ..... . ..... .. .. .... ... .... . . .. .... . .... .. .. .. . .. .. .. .... .... . . .... .. .... .. . . .... .... .. . .. . . .. .... . . .. ... . .. .... . .. ... . .. . .. ... ... . . . ... ... .. . . ... ... . . .. . ... ... .... ... . . . ..... ... ..... . ... .. . .. .. ..... ..... . . . .. ... . ... ..... ... . . . ... ..... .. ..... . ... . . . .... ..... . . ... .. . ... .... .... . . . ... .... ... .. .... . . . .... .... . ... . ... . . . .. .... .. . . . . ... .... .... . .. . . .. . . . . ... ... .... . . .... . .. . . .. .... . .... . . .. ... . . ... .. . . . . .. .... .. .... . . . . .. .... . .. .... . . . . . ... . ... . .. . .. . ... . ... . . . .. ... . ... .. . . . ... . . ... .. . . .. ... ... . . . . .. ... ... . . .. . .. . . . . .. ... . . .. ... . . ... . .. ... . . .. . . ... . ... . . .. . . .. ... ... . . . . .. ... . . ... .. . . ... . . . ... . .. . . . ... ... . . .. .. . . ... . ... . . . .. . . ... .. ... . . . . . ... . .. . ... . . . . . .. ... . .. . ... . . ... . . . .. . ... . .. . . . . .. ... . . ... .. . . . . . ... . .. ... . . . . .. . . . . .. .. . .. . . . .. . . .. .. . . . .. . . . . .. . .. . .. . . . . . .. . .. .. . . . .. . . . .. . .. . . . .. . .. . . . . . . .. . . . .. . . . . . . . . .. ... .. ... . . . . . . .. ... ... .. . . . . . . . . .... . . . ... . . . .. .. ... . ... . . . . . .. ... . . . .. ... . . . .. ... . . . .. .. . . . ... ... . . . . . . ... .. ... . . . .. . . ... . ... . . . .. . . . .. ... ... . . . . . .. . ... .. . . ... . . . . .. . . . . .. . . . ... .. ... . . . . . ... . ... .. . . . .. . . ... . ... . . . .. . . . .. ... ... . . . . . . .. ... . . . .. ... . . . . . . . . .. ... . . . ... .. . . . . ... . . .. ... . . .. . . ... . . ... . . .. . . .. . . ... ... . . . . . .. . ... . .. ... . . . . . . . . . .. . ... .. . . . ... . . . ... . . . .. ... . . .. . . . ... . ... . .. . . . .. . . . ... ... . . . . . . .. ... . . .. . ... . . . . . . . .. . ... .. . . ... . . . . ... . . . ... .. . .. . . . . ... . ... . . . .. .. . . . ... ... . . . . . . .. ... . . . .. ... . . . . . . . . .. ... . .. ... . . . . . ... . . . .. ... .. . . . . . . ... ... . . . .. . . .. . ... ... . . . . . . .. ... . .. ... . . . .. . . . .. . . . .. ... . .. . . ... . . ... . . . ... .. .. . . .. . . .. . .... . .. . . .. .. . . ... ... .. . . . . . .. .... .. . . .... .. . . ... .... . . . ... .. .. ... . .. .... . . .. . ... ... .... .... .. .. .. ... ..... ... . ..... .. .. .. . .. ...... .. .... ...... .. .. .. .... .. ....... .... ... ........ . . ... ...... .. ................................................ ................................................................. ... ........ ... ............................................................................................................ .

n 1 2

0.8

10

0.6

0.4

20

0.2

0.0

12

16

20

24 28 Oered trac A

Figure 7.3: Blocking probability En (A) as a function of the oered trac A for various values of the number of channels n (7.9).

132

CHAPTER 7. ERLANGS LOSS SYSTEM AND BFORMULA

Figure 7.4: The average utilisation per channel a (7.13) as a function of the number of channels n for given values of the congestion E.

7.3. TRUNCATED POISSON DISTRIBUTION

133

Improvement function F1 (A) = a+1 1.0


........... ........... ........... .......... .......... .......... ........ ........ ........ ....... ....... ....... ...... ...... .... ...... .... ...... .... .... ..... ..... .... ..... .... ..... .... ..... .... ..... .... ..... .... .... .... .... ... ... .... .... ... ... .... ... .... ... .... ... .... ... ... .... .... ... ... ... ... .... ... ... ... .. ... .. ... ... ... ... .. ... .. ... ... ... ... .. .. ... .. ... .. .. ... .. .. ... .. ... .. ... .. .. .. ... ... .. .. .. ... .. ... .. .. ... .. .. ... .. .. ... .. ... .. .. ... .. ... .. .. ... .. .. .. .. .. .. .. .. .. .. .. .. .. .. .. .. .. .. .. .. .. . .. .. .. .. .. .. .. .. .. .. .. .. . .. .. .. .. . .. .. .. . . .. .. .. .. .. .. .. .. .. .. . . .. . .. .. . .. . . .. . . .. . . .. . . .. . .. . .. . . . .. . . .. . .. .. . . . . . .. . .. . .. .. . . . . . .. .. . . .. .. . . . . . .. . .. . .. . .. . . . . .. . .. .. . . .. . . . . .. . .. . .. . . .. . . .. . . .. . .. . . .. . . .. . . .. . . .. . .. . . .. . . .. . . .. . . . .. . . .. . . .. . . . . . .. . .. .. . . . . . . . .. . . .. . . . . . . . . .. . . .. .. . . .. . . . . .. . . . .. . . . . . . . .. . . . .. . . . . . . . . .. . . . .. . . . . . . . .. . .. . .. . . .. . . . . . . . . . .. . . . . .. . . . . . . . . . . . .. . . .. . . . . . . . . . . .. . . . . .. . . . . . . . . . . . . .. . . . . .. . . . . . . . . . .. . . . . .. . . . . . . . . . . . . .. . . . . .. . . . . . . . . . . .. . . . . .. . . . . . . . . . . . .. . . .. . . . . . . . . . . . .. . . . . .. . . . . . . . . . . . .. . . . . .. . . . . . . . . . . . .. . . . .. . . . . . . . . . . .. . . . . .. . . . . . . . . . . . . .. . . . . .. . . . . . . . . . . .. . . . . .. . . . . . . . . . . .. . . . . .. . . . . . . . . . . . . .. . . . . .. . . . . . . . . . . .. . . . .. . . . . . . . . . . . .. . . . . .. . . . . . . . . . . . .. . . . .. . . . . . . . . . . . .. . . . . .. . . . . . . . . . . . .. . . . . .. . . . . . . . . . . . .. . . . . .. . . . . . . . . . . .. . . . .. . . . . . . . . .. . . .. . .. . . .. . . . . . . . . . .. . . . .. . . . . . . . . . .. .. . . . .. .. . . . . . . . . . .. .. . . . . .. . . . . . .. . .. . . . . .. . . . .. . . . . . . .. . . . .. . .. . . . .. . . . . .. . . .. . . .. . . . .. . . . .. . . . .. . . .. . . . .. . . .. . . .. . . . . .. . . .. . . . .. . . .. . . . . .. . . . .. .. . . . .. . . . . .. . . .. .. .. . .. . .. . . . . .. .. . . . .. .. . . . . .. . .. . .. .. . .. . .. . . . .. . . .. . .. . .. . .. .. . . . . . . . .. . .. . .. . .. . . . . .. . .. . .. . .. . .. . .. .. . .. .. . .. . . .. . .. .. . .. .. . .. .. . .. . . .. . .. ... .. .. . .. . . ... .. ... .. .. .. .. .. ... .. ... .. .. .. .. .. ... .. ... .. .. .. .. ... ... ... .. ... .. ... .... .. ... .. ... .. .... ... ... .... ... .... ... ... ..... ... ..... .... ... .... ... ...... ... .... ...... .... ..... ...... ..... . . . . .. ..... . . ..... . .. . . . . . . .... . . . . .. ....................................................................................................................................................................................................................................................................... .............................................................................................................................................................................................. ........................................................................ .... .. . . . . .. . . . . .... . . .. . ............. ..... . . . . .

0.8

20

0.6

10

0.4

0.2

0.0

12

16

20 24 28 Number of channels n

Figure 7.5: Improvement function Fn (A) (7.16) of Erlangs Bformula. By sequential hunting Fn (A) equals the trac an+1 carried on channel number (n + 1).

134

CHAPTER 7. ERLANGS LOSS SYSTEM AND BFORMULA

7.4

General procedure for state transition diagrams

The most important tool in teletrac theory is formulation and solution of models by means of state transition diagrams. From the previous sections we identify the following standard procedure for dealing with state transition diagrams. It consists of a number of steps and is formulated in general terms. The procedure is also applicable for multi-dimensional state transition diagrams, which we consider later. We always go through the following steps: a. Construction of the state transition diagram. Dene the states of the system in an unique way, Draw the states as circles, Consider the states one at a time and draw all possible arrows for transitions away from the state due to the arrival process (new arrival or phase shift in the arrival process), the departure (service) process (the service time terminates or shifts phase). In this way we obtain the complete state transition diagram. b. Set up the equations describing the system. If the conditions for statistical equilibrium are fullled, the steady state equations can be obtained from: node equations (general), cut equations. c. Solve the balance equations assuming statistical equilibrium. Express all state probabilities by for example the probability of state [ 0 ], p(0). Find p(0) by normalisation. d. Calculate the performance measures expressed by the state probabilities. In practise, we let the non-normalised value of the state probability q(0) equal to one, and then calculate the relative values q(i), (i = 1, 2, . . .). By normalising we then nd: p(i) = where Qn =
=0

q(i) , Qn

i = 0, 1, . . . , n ,
n

(7.19)

q() .

(7.20)

The time congestion becomes: p(n) = q(n) Qn1 =1 . Qn Qn (7.21)

7.4. GENERAL PROCEDURE FOR STATE TRANSITION DIAGRAMS

135

7.4.1

Recursion formula

If q(i) becomes very large (e.g. 1010 ), then we may multiply all q(i) by the same constant (e.g. 1010 ) as we know that all probabilities are within the interval [0, 1]. In this way we avoid numerical problems. If q(i) becomes very small, then we may truncate the state space as the density function of p(i) often will be bell-shaped (unimodal) and therefore has a maximum. In many cases we are theoretically able to control the error introduced by truncating the state space (Stepanov, 1989 [94]). We may normalise after every step which implies more calculations, but ensures a high accuracy. Let the normalized state probabilities for at system with x 1 channels be given by: Px1 = {px1 (x1), px1 (x2), . . . , px1 (0)} , x = 1, 2, . . . , (7.22) where index (x1) indicates that it is state probabilities for a system with (x1) channels. Let us assume we have the following recursion for qx (x) given by some function of previous state probabilities: qx (x) = f {px1 (x1), px1 (x2), . . . , px1 (0)} , x = 1, 2, . . . , (7.23)

where qx (x) will be a relative state probability. Assuming we know the normalised state probabilities for (x1) channels (7.22) we want to nd the normalised state probabilities for a system with x channels. The relative values of state probabilities do not change when we increase number of channels by one, so we get: qx (i) = px1 (i) , i = 0, 1, 2, . . . , x1 , (7.24)

f {px1(x1), px1 (x2), . . . , px1 x(0)} , i = x .

The new normalisation constant becomes:


x

Qx =
i=0

qx (i) = 1 + qx (x) ,

as we in the previous step normalised the sum of state probabilities ranging from 0 to x1 so they add to one. We thus get: px1 (i) , 1 + q (x) x px (i) = qx (x) , 1 + qx (x) i = 0, 1, 2, . . . , x 1 , (7.25) i = x.

The initial value for the recursion is given by p0 (0) = 1. The recursion algorithm thus start with this value and nd the state probabilities of a system with one channel more by (7.24) and (7.25). The recursion is numerically very stable because we in (7.25) divide with a number greater than one.

136

CHAPTER 7. ERLANGS LOSS SYSTEM AND BFORMULA

Let us consider a simple birth & death process with arrival rate i and departure rate i in state i. Then qx (x) only depends on the previous state probability. By using the cut equation we get the following recursion formula: qx (x) = x1 px1 (x 1) . x (7.26)

The time congestion for x channels is Ex (A) = px (x). Inserting (7.26) into (7.25) we get a simple recursive formula for the time congestion:
x1 Ex1 qx (x) x Ex = = , 1 + qx (x) 1 + x1 Ex1 x

E0 = 1 .

(7.27)

1 Introducing the inverse time congestion probability Ix = Ex we get: x Ix = 1 + Ix1 , I0 = 1 . x1

(7.28)

This is a general recursion formula for calculating time congestion for all systems with state dependent arrival rates i and homogeneous servers.
Example 7.4.1: Calculating probabilities of the Poisson distribution If we want to calculate the Poisson distribution (7.6) for very large mean values m1 = A = /, then it is advantageously to let q(m) = 1, where m is equal to the integral part of (m1 + 1). The relative values of q(i) for both decreasing values (i = m1, m2, . . . , 0) and for increasing values (i = m+1, m+2, . . .) will then be decreasing, and we may stop the calculations when for example q(i) < 1020 and nally normalise q(i). In practice there will be no problems by normalising the probabilities. A more strict approach is to use the above recursion formula.

7.5

Evaluation of Erlangs B-formula

For numerical calculations the formula (7.10) is not very appropriate, since both n! and An increase quickly so that overow in the computer will occur. If we apply (7.27), then we get the recursion formula: A Ex1 (A) Ex (A) = (7.29) , E0 (A) = 1 . x + A Ex1 (A) From a numerical point of view, the linear form (7.28) is the most stable: x Ix (A) = 1 + Ix1 (A) , I0 (A) = 1 , (7.30) A where In (A) = 1/En (A). This recursion formula is exact, and even for large values of (n, A) there are no round o errors. It is the basic formula for numerous tables of the Erlang Bformula, i.a. the classical table (Palm, 1947 [81]). For very large values of n there are more ecient algorithms. Notice that a recursive formula, which is accurate for increasing index, usually is inaccurate for decreasing index, and vice versa.

7.5. EVALUATION OF ERLANGS B-FORMULA

137

Example 7.5.1: Erlangs loss system We consider an Erlang-B loss system with n = 6 channels, arrival rate = 2 calls per time unit, and departure rate = 1 departure per time unit, so that the oered trac is A = 2 erlang. If we denote the non-normalised relative state probabilities by q(i), we get by setting up the state transition diagram the values shown in the following table:

i 0 1 2 3 4 5 6 Total

(i) 2 2 2 2 2 2 2

(i) 0 1 2 3 4 5 6

q(i) 1.0000 2.0000 2.0000 1.3333 0.6667 0.2667 0.0889 7.3556

p(i) 0.1360 0.2719 0.2719 0.1813 0.0906 0.0363 0.0121 1.0000

i p(i) 0.0000 0.2719 0.5438 0.5438 0.3625 0.1813 0.0725 1.9758

(i) p(i) 0.2719 0.5438 0.5438 0.3625 0.1813 0.0725 0.0242 2.0000

We obtain the following blocking probabilities:

Time congestion: Trac congestion:

E6 (2) = p(6) = 0.0121 . C6 (2) = AY 2 1.9758 = = 0.0121 . A 2


6

Call congestion:

B6 (2) = {(6) p(6)}


i=0

(i) p(i)

0.0242 = 0.0121 . 2.0000

We notice that E = B = C due to the PASTAproperty.

138

CHAPTER 7. ERLANGS LOSS SYSTEM AND BFORMULA

By applying the recursion formula (7.29) we of course obtain the same results: E0 (2) = 1 , E1 (2) = 21 2 = , 1+21 3 2 2 3 2+2 2 2 5 3+2
2 3

E2 (2) =

2 , 5 4 , 19

E3 (2) =

2 5

E4 (2) =

4 2 19 2 4 = 21 , 4 + 2 19 2 2 21 4 2 = 109 , 5 + 2 21 4 2 109 4 4 = 331 = 0.0121 . 6 + 2 109

E5 (2) =

E6 (2) =

Example 7.5.2: Calculation of E (A) for large x By recursive application of (7.30) we nd: Ix (A) = 1 + x x! x (x1) + + x , + 2 A A A

which of course is the inverse blocking probability of the B-formula. For large values of x and A this formula can be applied for fast calculation of the B-formula, because we can truncate the sum when the terms become very small.

7.6

Principles of dimensioning

When dimensioning service systems we have to balance grade-of-service requirements against economic restrictions. In this chapter we shall see how this can be done on a rational basis. In telecommunication systems there are several measures to characterise the service provided. The most extensive measure is Quality-of-Service (QoS), comprising all aspects of a connection as voice quality, delay, loss, reliability etc. We consider a subset of these, Grade-of-Service (GoS) or network performance, which only includes aspects related to the capacity of the network.

7.6. PRINCIPLES OF DIMENSIONING

139

By the publication of Erlangs formul there was already before 1920 a functional relationship between number of channels, oered trac, and grade-of-service (blocking probability) and thus a measure for the quality of the trac. At that time there were direct connections between all exchanges in the Copenhagen area which resulted in many small trunk groups. If Erlangs B-formula were applied with a xed blocking probability for dimensioning these groups, then the utilisation would become poor. Kai Moe (1893-1949), who was chief engineer in the Copenhagen Telephone Company, made some quantitative economic evaluations and published several papers, where he introduced marginal considerations, as they are known today in mathematical economics. Similar considerations were later done by P.A. Samuelson in his famous book, rst published in 1947. On the basis of Moes works the fundamental principles of dimensioning are formulated for telecommunication systems in Moes Principle (Jensen, 1950 [50]).

7.6.1

Dimensioning with xed blocking probability

For proper operation, a loss system should be dimensioned for a low blocking probability. In practice the number of channels n should be chosen so that E1,n (A) is about 1% to avoid overload due to many non-completed and repeated call attempts which both load the system and are a nuisance to subscribers (Cf. Bbusy [52]). n A (E = 1%) a F1 (A) E [%] a F1 (A1 ) 1 2 5 10 20 0.596 0.052 3.640 0.696 0.173 50 0.750 0.099 5.848 0.856 0.405 100 84.064 0.832 0.147 8.077 0.927 0.617

0.010 0.153 1.361 4.461 12.031 37.901 0.010 0.076 0.269 0.442 0.000 0.001 0.011 0.027 1.198 1.396 1.903 2.575 0.012 0.090 0.320 0.522 0.000 0.002 0.023 0.072

A1 = 1.2 A 0.012 0.183 1.633 5.353 14.437 45.482 100.877

Table 7.1: Upper part: For a xed value of the blocking probability E = 1% n trunks can be oered the trac A. The average utilisation of the trunks is a, and the improvement function is F1,n (A) (7.16). Lower part: The values of E, a and F1,n (A) are obtained for an overload of 20%. Tab. 7.1 shows the oered trac for a xed blocking probability E = 1% for some values of n. The table also gives the average utilisation of channels, which is highest for large groups. If we increase the oered trac by 20 % to A1 = 1.2 A, we notice that the blocking probability increases for all n, but most for large values of n.

140

CHAPTER 7. ERLANGS LOSS SYSTEM AND BFORMULA

From Tab. 7.1 two features are observed: a. The utilisation a per channel is, for a given blocking probability, highest in large groups (Fig. 7.4). At a blocking probability E = 1 % a single channel can at most be used 36 seconds per hour on the average! b. Large channel groups are more sensitive to a given percentage overload than small channel groups. This is explained by the low utilisation of small groups, which therefore have a higher spare capacity (elasticity). Thus two conicting factors are of importance when dimensioning a channel group: we may choose among a high sensitivity to overload or a low utilisation of the channels.

7.6.2

Improvement principle (Moes principle)

As mentioned in Sec. 7.6.1 a xed blocking probability results in a low utilisation (bad economy) of small channel groups. If we replace the requirement of a xed blocking probability with an economic requirement, then the improvement function F1,n (A) (7.16) should take a xed value so that the extension of a group with one additional channel increases the carried trac by the same amount for all groups. In Tab. 7.2 we show the congestion for some values of n and an improvement value F = 0.05. We notice from the table that the utilisation of small groups becomes better corresponding to a high increase of the blocking probability. On the other hand the congestion in large groups decreases to a smaller value. See also Fig. 7.7. If therefore we have a telephone system with trunk group size and trac values as given in the table, then we cannot increase the carried trac by rearranging the channels among the groups. This service criteria will therefore in comparison with xed blocking in Sec. 7.6.1 allocate more channels to large groups and fewer channels to small groups, which is the trend we were looking for. The improvement function is equal to the dierence quotient of the carried trac with respect to number of channels n. When dimensioning according to the improvement principle we thus choose an operating point on the curve of the carried trac as a function of the number of channels where the slope is the same for all groups (A/n = constant). A marginal increase of the number of channels increases the carried trac with the same amount for all groups. It is easy to set up a simple economical model for determination of F1,n (A). Let us consider a certain time interval (e.g. a time unit). Denote the income per carried erlang per time unit by g. The cost of a cable with n channels is assumed to be a linear function: cn = c0 + c n . (7.31)

7.6. PRINCIPLES OF DIMENSIONING n A (F = 0.05) a E1 (A) [%] A1 = 1.2 A E {%} a F1 (A1 ) 1 0.271 0.213 21.29 0.325 24.51 0.245 0.067 2 5 10 20 0.593 0.97 3.55 0.693 0.169 50 35.80 0.713 0.47 42.96 3.73 0.827 0.294 100 78.73 0.785 0.29 94.476 4.62 0.901 0.452

141

0.607 2.009 4.991 11.98 0.272 0.387 0.490 10.28 13.30 3.72 6.32 1.82 4.28

0.728 2.411 5.989 14.38 0.316 0.452 0.573 0.074 0.093 0.120

Table 7.2: For a xed value of the improvement function we have calculated the same values as in table 7.1. The total costs for a given number of channels is then (a) cost of cable and (b) cost due to lost trac (missing income): Cn = g A E1,n (A) + c0 + c n , (7.32)

Here A is the oered trac, i.e. the potential trac demand on the group considered. The costs due to lost trac will decrease with increasing n, whereas the expenses due to cable increase with n. The total costs may have a minimum for a certain value of n. In practice n is an integer, and we look for a value of n, for which we have (cf. Fig. 7.6): Cn1 > Cn As E1,n (A) = En (A) we get: A {En1 (A) En (A)} > or: F1,n1 (A) > FB F1,n (A) , where: FB = cost per extra channel c = . g income per extra channel (7.34) (7.35) c A {En (A) En+1 (A)} , g (7.33) and Cn Cn+1 .

FB is called the improvement value. We notice that c0 does not appear in the condition for minimum. It determines whether it is protable to carry trac at all. We must require that for some positive value of n we have: g A {1 En (A)} > c0 + c n . (7.36)

Fig. 7.7 shows blocking probabilities for some values of FB . We notice that the economic demand for prot results in a certain improvement value. In practice we choose FB partly independent of the cost function.

142 25 Costs

CHAPTER 7. ERLANGS LOSS SYSTEM AND BFORMULA


. .. ... ... . . .... .... .... .... .. ... .. ... .. .. .. .. .. .. .. ... .. ... .. .. .. ... .. ... .. .. .. .. .. ... .. .. .. .. .. . .. .. .. .. ... .. ... .. .. ... .. ... .. .. ... .. .. ... ... .. .. .. ... .. ... .. . .. .. .... .. .. .... .. .. ... ... .. .. .. ... .. .. .. .. .. ... ... .. .. . ... .. .. ... . .. . .. .. .. .. ... ... .. .. .. .. .. .. ... ... .. .. .. .. .. .. ... ... .. .. .. .. ... ... .. .. .. .. .. ... ... .. . .. .. . .. .. ... .. ... ... . .. ... .. .. .. .. ... .... .. .. .. ... ... .. .. .. .. ... .. .. .. .. ... ... ... ... . .. .. .. ... ... ... ... .. ... ... ... .. .. ... ... ... ... .. .. ... ... ... ... .. ... ... .. ... ... .. ... .. ... ... .. ... ... ... ... ... .. ... . ... ... ... ... .. ... .. .. .. . ... .... .... .... ...... ... .. .... .... ... ..... .. . ... .... ...... .... ...... .. ... .. ... ..... ..... .... ... .... ... ... ... .. ....................... ..... .. ...................... ..... ... .... ... .. . .. .. .. ... . ... . ... ... .. . . .. ... . ...... .. .. . . .. . . .... .. .. . ... .. ... .. ... . ... . . .. . . .. ... . ... .. .. ... ... . .. . ... ... .. . . ... ... .. .. ... ... . . .. .. . ... . ... . . . .. . .. ... ... .. ... .. ..... . . ..... .. . ..... . .. ... .. ... ... . . . .. ... . ... .. . . . . ... .. ... .. .. ... .. . .. ... ... . . ... .. ... . .. . . ... .. ... ... ... .. . . . . .. ... . ... .. . . . ... ... . .. ... .. ... . .. ... ... ... ... .. . ... ... . . .. . .. ... . ... .. . ... .. .. . ... . .. ... . .. ... .. . . . . .. ... .. . ... .. .. . ... ... . . . .. .. .... . .. .. ..... .. .. .... ... .... .. . ... . ... ... . ... ... . ... ... . ... . . ... ... ... ... . .. .. . ...... ... . ....... ... . ...... . . . ... ... . ..... ... . ... ............ ............. . ... ... . ...................................................... . .................................................... . ... . ...

20

Total costs

15

Blocked trac

10

Cable

10

20

30

40

50 60 No. of trunks n

Figure 7.6: The total costs are composed of costs for cable and lost income due to blocked trac (7.32). Minimum of the total costs are obtained when (7.33) is fullled, i.e. when the two cost functions have the same slope with opposite signs (dierence quotient). (FB = 0.35, A = 25 erlang). Minimum is obtained for n = 30 trunks. In Denmark the following values have been used: FB = 0.35 for primary trunk groups. FB = 0.20 for service protecting primary groups. FB = 0.05 for groups with no alternative route. (7.37)

7.6. PRINCIPLES OF DIMENSIONING

143

15

Blocking probability E [%]

10

.. . . .. . . .. . . .. . . .. . . .. . . .. . . .. . . .. . . .. . . . .. . . . .. . . .. . . .. . . .. . . .. . . .. . . .. . . . .. . . .. . . . .. . . .. . . .. . . .. . . .. . . .. . . .. . . .. . . .. . . . . .. . . .. . . .. . . .. . . .. . . .. . . .. . . .. . . .. . . .. . . .. . . .. . . .. . . .. . . . . .. . . .. . . . .. . . .. . . .. . . .. . . .. . . .. . . . .. . . .. . . .. . . .. . . .. . . .. . . .. . . . . .. . . .. . . .. . . .. . . .. . . .. . . . .. . . .. . . .. . . .. . . .. . . .. . . . . .. . . .. . . .. . . .. . . . .. . . . . .. . . .. . . .. . . .. . . . .. . . .. . . .. . . .. . . .. . . .. . . . . . .. . . .. . . . .. . .. .. . .. .. . .. . .. . . .. . . .. .. . .. . .. . .. . .. .. . .. .. . .. . .. . .. . .. .. . . . . . . . .. . . . .. . . . . . . . .. . . . .. . . . . . . . .. . . .. . . . . . . . .. . . . .. . . . . . . . .. . . .. . . . . . . . .. . . . .. . . . . . . .. .. . . . . . . .. . .. . . . . . . .. . . . . .. . . . . . . .. . . .. . . . . ... . . . ... . . . . . ... . . . ... . . . . . . ... . ... . . . . . . . . ... . ... . . . . . . ... . . . ... . . . . ... . . ... . . . . . . ... . ... . . . . . .. ... ... . . .. . . ... ... . . .. . . .... .. .... . . . . . . .... .... .. . . .. . . .... .... . . . . .. .... . . .. .... . . . . .... .... .. . . .. . . ..... ..... . . .. . . ..... .. ..... . . . . .. ..... . ..... . .. . . . ..... . ...... . . .. .. . ...... . ...... . . . . .. ...... ...... . .. . . . ....... .. ....... . . ... . . ....... . ... ........ . . . ... ........ . ........ . ... . . ......... . ... . ......... . ... . . .......... . ... .......... . . . ... ........... . . ........... ... . . ... . . ........... ............ .. ... . .. ... ............. . ............. .... . .... . .............. .. .............. . .. .... . ................ .... ................ . ..... . .. ............. ..... ......... .. . . ..... ..... . .. . .. ...... . ...... . .. ...... . .. ...... . .. . ....... .. ....... . . ... ........ ... ......... . . ......... . ... .......... ... . ........... .. ........... ... .. ... ............. ............. .... .... .. ............... .. ................ .... .... ................. ................. .. .. ..... ..... ................... ..................... ...... ... ...... ........................ ... .......................... ........ ....... .............................. ......................... ... ... ......... .... ......... ... ... ............ ............ .... .... ................ ................ ...... ...... .................... .................... ........ ........ ............................. ............................. ............ ............ ............................................. ............................................. ................. .................. ............................................. ............................................ ............................... ................................ .................................................................... ....................................................................... .............................................................. .........................................................

FB

0.35

0.20 0.10 0.05

10

20

30

40

50

60

70

80 90 100 Oered trac A

Figure 7.7: When dimensioning with a xed value of the improvement value FB the blocking probabilities for small values of the oered trac become large (cf. Tab. 7.2).

144

CHAPTER 7. ERLANGS LOSS SYSTEM AND BFORMULA

Chapter 8 Loss systems with full accessibility


In this chapter we generalise Erlangs classical loss system to state-dependent Poisson-arrival processes, which include the so-called BPP-trac models: Binomial case: Engsets model, Poisson case: Erlangs model, and Pascal (Negative Binomial) case: PalmWallstrms model. o These models are all insensitive to the service time distribution. Engset and Pascal models are even insensitive to the distribution of the idle time of sources. After an introduction in Sec. 8.1 we go through the basic classical theory. In Sec. 8.2 we consider the Binomial case, where the number of sources S (subscribers, customers, jobs) is limited and the number of channels n always is sucient (S n). This system is dealt with by balance equations in the same way as the Poisson case (Sec. 7.2). We consider the strategy Lost-Calls-Cleared (LCC). In Sec. 8.3 we restrict the number of channels so that it becomes less than the number of sources (n < S). We may then experience blocking and we obtain the truncated Binomial distribution, which also is called the Engset distribution. The probability of time congestion E is given by Engsets formula. With a limited number of sources, time congestion, call congestion, and trac congestion dier, and the PASTAproperty is replaced by the general arrival theorem, which tells that the state probabilities of the system observed by a customer (call average) is equal to the state probability of the system without this customer (time average). Engsets formula is computed numerically by a formula recursive in the number of channels n derived in the same way as for Erlangs B-formula. Also formul recursive in number of sources S and in both n & S are derived. In Sec. 8.6 we consider the Negative Binomial case, also called the Pascal case, where the arrival intensity increases linearly with the state of the system. If the number of channels is limited, then we get the truncated Negative Binomial distribution (Sec. 8.7). 145

146

CHAPTER 8. LOSS SYSTEMS WITH FULL ACCESSIBILITY S sources n channels

Figure 8.1: A full accessible loss system with S sources, which generates trac to n channels. The system is shown by a so-called chicko-gram. The beak of a source symbolises a selector which points upon the channels (servers) among which the source may choose.

8.1

Introduction

We consider a system with same structure (full accessibility group) and strategy (Lost-CallsCleared) as in Chap. 7, but with more general trac processes. In the following we assume the service times are exponentially distributed with intensity (mean value 1/); the trac process then becomes a birth & death process, a special Markov process, which is easy to deal with mathematically. Usually we dene the state of the system as the number of busy channels. All processes considered in Chapter 7 and 8 are insensitive to the service time distribution, i.e. only the mean service time is of importance for the state probabilities. The service time distribution itself has no inuence. Denition of oered trac: In Sec. 2.1 we dene the oered trac A as the trac carried when the number of servers is unlimited, and this denition is used for both the Engset-case and the Pascal-case. The oered trac is thus independent of the number of servers. Only for stationary renewal processes as the Poisson arrival process in the Erlang case this denition is equivalent to the average number of calls attempts per mean service time. In Engset and Pascal cases the arrival processes are not renewal processes as the mean interarrival time depends on the actual state. Peakedness is dened as the ratio between variance and mean value of the state probabilities. For the oered trac the peakedness is considered for an innite number of channels. We consider the following arrival processes, where the rst case already has been dealt with in Chap. 7: 1. Erlang-case (P Poisson-case): The arrival process is a Poisson process with intensity . This type of trac is called random trac or Pure Chance Trac type One, PCTI. We consider two cases: a. n = : Poisson distribution (Sec. 7.2). The peakedness is in this case equal to one: Z = 1. b. n < : Truncated Poisson distribution (Sec. 7.3). 2. Engset-case (B Binomial-case): There is a limited number of sources S. The individual source has a constant call (arrival) intensity when it is idle. When it is busy the call intensity is zero. The

8.2. BINOMIAL DISTRIBUTION

147

arrival process is thus state-dependent. If i sources are busy, then the arrival intensity is equal to (S i) . This type of trac is called Pure Chance Trac type Two, PCTII. We consider the following two cases: a. n S: Binomial distribution (Sec. 8.2). The peakedness is in this case less than one: Z < 1. b. n < S: Truncated Binomial distribution (Sec. 8.3). 3. Palm-Wallstrmcase (P Pascal-case): o There is a limited number of sources S. If at a given instant we have i busy sources, then the arrival intensity equals (S +i) . Again we have two cases: a. n = : Pascal distribution = Negative Binomial distribution (Sec. 8.6). In this case peakedness is greater than one: Z > 1. b. n < : Truncated Pascal distribution (truncated negative Binomial distribution) (Sec. 8.7). As the Poisson process may be obtained by an innite number of sources with a limited total arrival intensity , the Erlang-case may be considered as a special case of the two other cases:
{S , 0}

lim

S = . S = .

For any nite state i we then have a constant arrival intensity: (S i)

The three trac types are referred to as BPP-trac according to the abbreviations given above (Binomial & Poisson & Pascal). As these models include all values of peakedness Z > 0, they can be used for modelling trac with two parameters: mean value A and peakedness Z. For arbitrary values of Z the number of sources S in general becomes non-integral. Performancemeasures: The performance parameters for loss systems are time congestion E, Call congestion B, trac congestion C, and the utilisation of the channels. Among these, trac congestion C is the most important characteristic. These measures are derived for each of the above-mentioned models.

8.2

Binomial Distribution

We consider a system with a limited number of sources (subscribers) S. The individual source switches between the states idle and busy. A source is idle during a time interval which is exponentially distributed with intensity , and the source is busy during an exponentially distributed time interval (service time, holding time) with intensity (Fig. 8.2). This kind

148

CHAPTER 8. LOSS SYSTEMS WITH FULL ACCESSIBILITY

of sources are called sporadic sources or on/o sources. This type of trac is called Pure Chance Trac type Two (PCTII) or pseudo-random trac. The number of channels/trunks n is in this section assumed to be greater than or equal to the number of sources (n S), so that no calls are lost. Both n and S are assumed to be integers, but it is possible to deal with nonintegral values (Iversen & Sanders, 2001 [43]).
. . . . .. . . .. ... ... . . . . . . . . . . . . . . . . . . . . . . . . . . . . . . . . . ........................................................................................................................................................................................... ... .. ............................................................................................................................................................................................. . . . . .. .. . . . . . .. .. .. .. .. .. . . .. .. .. . . ... ... ... . ... ... .... ... .. ... ... .. . .. . .. ..................... . . . ........................................... .......................................... ... . ... . . ... ...................... ................ ..... ... . .. . ..................... . . . . . . . . . . . . . . . .

State

Busy

Idle

Time

arrival

departure

arrival

Figure 8.2: Every individual source is either idle or busy, and behaves independent of all other sources. (S 1)
S 1 S

...... ...... ..... ........... ............... . ..... ......... .... ..... ..... .... ... ........ ...... ... .... .... . ... ...... ......... . ... .. .............. ..... .... .... ....... . . .... .. .. .. .. .. .. .. .. .. . . . . . . . . . . . . . . . . . . . . . . . . . . . . . . . . . . . .. . .. .. . . .. .. ... .. .. . .. ... .. .. . ................ ............... .............. ..... .............. ...... ... . ..... .. .. .... ... ... ... ..... ........ ......... ...... .............. .............. ......

(S 1)

...... ............... ............... .................. ..... ........ ... ....... ..... ... ....... .... .. ... .... ........... ..... .... ........... .. . .. .. . .. ... . ... ... .. .. . ... ... . . .. .. .. .. .. .. .. . . . . . . . . . . . . . . . . . . . . . . . . . . . . . . . . . . . . . .. .. .. .. .. .. .. .. .. . . . . . .. . .... . ......... ....... ........... ... ................... ............... .. . .... .. .. .. .. .... ... ... ..... ... ................. ................ ............... ...........

.... .......... .... ...... ... .. .. .. .. . . . . . . . . . . . . . . . . . .. .. .. .. .. . ... ............. ... ........

Figure 8.3: State transition diagram for the Binomial case (Sec. 8.2). The number of sources S is less than or equal to the number of circuits n (S n).

8.2.1

Equilibrium equations

We are only interested in the steady state probabilities p(i), which are the proportion of time the process spend in state [ i ]. We base our calculations on the state transition diagram in Fig. 8.3. We consider cuts between neighbouring states and nd: S p(0) = p(1) , (S 1) p(1) = 2 p(2) , ... ... (8.1) (S i 1) p(i 1) = i p(i) , (S i) p(i) = (i + 1) p(i + 1) , ... ... 1 p(S 1) = S p(S) .

8.2. BINOMIAL DISTRIBUTION All state probabilities are expressed by p(0): p(1) p(2) ... p(i) S = p(0) = (S 1) p(1) 2 ... S = p(0) 1 = p(0) ... S 2 ...
i 1

149

,
2

S (S i 1) = p(i 1) = p(0) i i (S i) p(i) (i + 1) ... p(S 1) S


1

,
i+1

p(i + 1) = ... p(S) =

= p(0) ... = p(0)

S i+1 ... S S
S

The total sum of all probabilities must be equal to one: 1 = p(0) S 1+ 1


S

S + 2

S ++ S

= p(0) 1 +

(8.2)

where we have used Newtons Binomial expansion. By letting = / we get: p(0) = 1 . (1 + )S (8.3)

The parameter is the oered trac per idle source (number of call attempts per time unit for an idle source the oered trac from a busy source is zero) and we nd: p(i) = S i S i i 1 (1 + )S
i

1+

1 1+

Si

which is the Binomial distribution (Tab. 6.1). Finally, we get by introducing: a = 1/ = = , 1+ + 1/ + 1/ S i ai (1 a)Si , i = 0, 1, . . . , S , 0 S n, (8.4)

p(i) =

150

CHAPTER 8. LOSS SYSTEMS WITH FULL ACCESSIBILITY

In this case, when a call attempt from an idle source never is blocked, the parameter a is equal to the carried trac y per source (a = y), which is equivalent to the probability that a source is busy at a random instant (the proportion of time the source is busy). This is also observed from Fig. 8.2, as all arrival and departure points on the time axes are regeneration points (equilibrium points). A cycle from start of a busy state (arrival) till start of the next busy state is representative for the whole time axes, and time averages are obtained by averaging over one cycle. Notice that for a systems with blocking we have y = a (cf. Sec. 8.3). The Binomial distribution obtained in (8.4) is sometimes called the Bernoulli distribution in teletrac theory, but this should be avoided as we in statistics use this name for a two-point distribution. Formula (8.4) can be derived by elementary considerations. All subscribers can be split into two classes: idle subscribers and busy subscribers. The probability that an arbitrary subscriber belongs to class busy is y = a, which is independent of the state of all other subscribers as the system has no blocking and call attempts always are accepted. There are in total S subscribers (sources) and the probability p(i) that i sources are busy at an arbitrary instant is given by the Binomial distribution (8.4) & Tab. 6.1.

8.2.2

Trac characteristics of Binomial trac

We summarize denitions of parameters given above: = call intensity per idle source, 1/ = mean service (holding) time, = / = oered trac per idle source. (8.5) (8.6) (8.7)

By denition, the oered trac of a source is equal to the carried trac in a system with no congestion, where the source freely switches between states idle and busy. Therefore, we have the following denition of the oered trac: a= = oered trac per source, 1+ (8.8) (8.9) (8.10) (8.11)

A = S a = total oered trac, y = carried trac per source Y = S y = total carried trac.

Oered trac per idle source is a dicult concept to deal with because the proportion of time a source is idle depends on the congestion. The number of calls oered by a source

8.2. BINOMIAL DISTRIBUTION

151

depends on the number of channels (feed-back): a high congestion results in more idle time for a source and thus in more call attempts. Time congestion: E = 0 E = p(n) = an Carried trac:
S

S < n, S = n. (8.12)

= Sy =
i=0

i p(i) (8.13)

= S a = A,

which is the mean value of the Binomial distribution (8.4). In this case with no blocking we of course have a = y and: Trac congestion: C= Number of call attempts per time unit:
S

AY = 0. A

(8.14)

=
i=0

p(i) (S i)
S

= S
i=0

i p(i) = S S a

= S (1 y) . As all call ttemptd are accepted we get: Call congestion: B = 0. Trac carried by channel : Random hunting: a = Sy Y = . n n (8.16) (8.15)

Sequential hunting: complex expression derived by L.A. Joys (1971 [56]). Improvement function: Fn (A) = Yn+1 Yn = 0 . (8.17)

152 Peakedness (Tab. 6.1):

CHAPTER 8. LOSS SYSTEMS WITH FULL ACCESSIBILITY

Z =

S a (1 a) 2 = , Sa 1 < 1. 1+ (8.18)

Z = 1a=

We observe that the peakedness Z is independent of the number of sources and always less than one. Therefore it corresponds to smooth trac. Duration of state i: This is exponentially distributed with rate: (i) = (S i) + i , 0 i S n. (8.19)

Finite source trac is characterized by number of sources S and oered trac per idle source . Alternatively, we in practice often use oered trac A and peakedness Z. We have the following relations between the two representations: A = S Z = , 1+ (8.20)

1 , 1+ 1Z , Z A . 1Z

(8.21)

= S =

(8.22) (8.23)

8.3

Engset distribution

The only dierence in comparison with Sec. 8.2 is that number of sources S now is greater than or equal to number of trunks (channels), S n. Therefore, call attempts may experience congestion.
........... ................ ................ . .................. ..... .. . ... ... ........ .. .. ... ... .... ........... ..... .... ..... .. . . .. . .. ............... .... . .. .... ...... .. .. . .. ... .. .. .. .. . . .. . . . . . . . . . . . . . . . . . . . . . . . . . . . . . . . . . . .. .. . . .. .. . . .. ... .. .. .. ... .. .. ...... .. ............... ..... .......... .... .. ....... ... . ...... ..... ... ..... . . ... ...... ...... ... .... ............... .............. .............. ..............

(S 1)

........... ................ ................ . .................. ..... ... ... ... ........ .. .. ... ... .... ........... ..... .... .. . . .. . . .. .... ... . ... .. . .. .. .. .. . . . . . . . . . . . . . . . . . . . .. .. .. .. ... . ..... . ... . ... ... . ..... . . .... ... ......... ..... ... .. . ... ........ ...... ... ..... ................. ................ ............... ..............

(S i)

........... ................ ................ . .................. ..... ... ... ... ........ .. .. ... ........ ... .... ........... ..... .... ........... .. . . .. . . .... ... ... ... .. .. . . . .. ... ... .. .. .. .. . .. . . . . . . . . . . . . . . . . . . . . . . . . . . . . . . . . . .. .. .. . . .. .. .. . ... ... .. .. ..... ... . .................. ............... ... . ...... . . . . .. . .... ... . .... ... . ..... ... ...... ...... ... ...... ................ ............... .............. ..............

(S n+1)

n1

(n1)

Figure 8.4: State transition diagram for the Engset case with S > n, where S is the number of sources and n is the number of channels.

8.3. ENGSET DISTRIBUTION

153

8.3.1

State probabilities

The cut equations are identical to (8.1), but they only exist for 0 i n (Fig. 8.4). The normalisation equation (8.2) becomes: S 1 = p(0) 1 + 1 S ++ n
n

From this we obtain p(0) and letting = / the state probabilities become: S i
n

i S j .
j

p(i) =

(8.24)

j=0

In the same way as above we may by using (8.8) rewrite this expression to a form, which is analogue to (8.4): S i
n

ai (1 a)Si S j , aj (1 a)Sj 0 i n, (8.25)

p(i) =

j=0

from which we directly observe why it is called a truncated Binomial distribution (cf. truncated Poisson distribution (7.10)). The distribution (8.24) & (8.25) is called the Engset distribution after the Norwegian T. Engset (18651943) who rst published the model with a nite number of sources (1918 [26]).

8.3.2

Trac characteristics of Engset trac

The Engset-distribution results in more complicated calculations than the Erlang loss system. The essential issue is to understand how to nd the performance measures directly from the state probabilities using the denitions. The Engset system is characterised by the parameters = / = oered trac per idle source, S = number of sources, and n = number of channels. Time congestion E: this is by denition equal to the proportion of time the system is blocking new call attempts, i.e. p(n) (8.24): S n n
n

En,S () = p(n) =

j=0

S j

,
j

S n.

(8.26)

154

CHAPTER 8. LOSS SYSTEMS WITH FULL ACCESSIBILITY

Call congestion B: this is by denition equal to the proportion of call attempts which are lost. Only call attempts arriving at the system in state n are blocked. During one unit of time we get the following ratio between the number of blocked call attempts and the total number of call attempts: Bn,S () = p(n) (S n)
n

p(j) (S j)
j=0

S n
n

n (S n) S j . (S j)
j

j=0

Using S i we get: S1 n n
n

S i = S

S1 , i

Bn,S () =

j=0

S1 j j

Bn,S () = En,S1() ,

S n.

(8.27)

This result may be interpreted as follows. The probability that a call attempt from a random source (subscriber) is rejected is equal to the probability that the remaining (S 1) sources occupy all n channels. This is called the arrival theorem, and it can be shown to be valid for both loss and delay systems with a limited number of sources. The result is based on the product form among sources and the convolution of sources. As E increases when S increases, we have Bn,S () = En,S1 () < En,S ().

Theorem 8.1 Arrival-theorem: For all systems with a limited number of sources a random source upon arrival will observe the state of the system as if the source itself does not belong to the system.

The PASTAproperty is included in this case because an innite number of sources minus one is still an innite number.

8.3. ENGSET DISTRIBUTION Carried trac: By applying the cut equation between state [ i 1 ] and state [ i ] we get:
n

155

=
i=1 n

i p(i) (S i + 1) p(i 1) (S i) p(i)


i=0 n

(8.28)

=
i=1 n1

(8.29)

=
i=0

(S i) p(i) (S n) p(n) , (8.30)

= (S Y ) (S n) E ,

as E = En,S () = p(n). This is solved with respect to Y : Y = {S (S n) E} . 1+ (8.31)

Trac congestion C = Cn,S (A). This is the most important congestion measure. The oered trac is given by (8.20) and we get: C = AY A S {S (S n) E} 1+ 1+ , S 1+ Sn E. S (8.32)

C =

We may also nd the carried trac if we know the call congestion B. The number of accepted call attempts from a source which is on the average idle 1/ time unit before it generate one call attempt is 1 (1 B), and each accepted call has an average duration 1/. Thus the carried trac per source, i.e. the proportion of time the source is busy, becomes: y= The total carried trac becomes: Y =Sy =S (1 B) . 1 + (1 B) (8.33) (1 B)/ . 1/ + (1 B)/

156

CHAPTER 8. LOSS SYSTEMS WITH FULL ACCESSIBILITY

Equalizing the two expressions for the carried trac (8.31) & (8.33) we get the following relation between E and B: S B E= . (8.34) S n 1 + (1 B) Number of call attempts per time unit:
n

=
i=0

p(i) (S i) (8.35)

= (S Y ) ,

where Y is the carried trac (8.28). Thus (S Y ) is the average number of idle sources which is evident. Historically, the total oered trac was earlier dened as /. This is, however, misleading because we cannot assign every repeated call attempt a mean holding time 1/. Also it has caused a lot of confusion because the oered trac by this denition depends upon the system (number of channels). With few channels available many call attempts are blocked and the sources are idle a higher proportion of the time and thus generate more call attempts per time unit. Lost trac: A = AC = S S n E 1+ S (8.36)

(S n) E. 1+

Duration of state i: This is exponentially distributed with intensity: (i) = (S i) + i , (n) = n , Improvement function: Fn,S (A) = Yn+1 Yn . (8.38) 0 i < n, i = n. (8.37)

Example 8.3.1: Call average and time average Above we have dened the state probabilities p(i) under the assumption of statistical equilibrium as the proportion of time the system spends in state i, i.e. as a time average. We may also study how the state of the system looks when it is observed by an arriving or departing source (user) (call average). If we consider one time unit, then on the average (S i) p(i) sources will observe the system in state [ i ] just before the arrival epoch, and if they are accepted they will bring the system

8.4. RELATIONS BETWEEN E, B, AND C

157

into state [ i + 1 ]. Sources observing the system in state n are blocked and remain idle. Therefore, arriving sources observe the system in state [ i ] with probability: n,S, (i) = (S i) p(i)
n

i = 0, 1, . . . n .

(8.39)

(S j) p(j)
j=0

In a way analogue to the derivation of (8.27) we may show that in agreement with the arrival theorem (Theorem 8.1) we have as follows: n,S, (i) = pn,S1, (i 1) , i = 0, 1, . . . , n . (8.40)

When a source leaves the system and looks back it observes the system in state [ i 1 ] with probability: i p(i) n,S, (i 1) = n , i = 1, 2, . . . , n . (8.41) j p(j)
j=1

By applying cut equations we immediately get that this is identical with (8.39), if we include the blocked customers. On the average, sources thus depart from the system in the same state as they arrive to the system. The process will be reversible and insensitive to the service time distribution. If we make a lm of the system, then we are unable to determine whether time runs forward or backward.

8.4

Relations between E, B, and C

From (8.34) we get the following relation between E = En,S () and B = Bn,S () = En,S1(): E = S B S n 1 + (1 B) (S n) E (1 + ) S + (S n) E or 1 S n = E S 1 1 = B 1+ (1 + ) 1 B . , (8.42)

B =

or

1 S + S n E

(8.43)

The expressions to the right-hand side are linear in the reciprocal blocking probabilities. In (8.32) we obtained the following simple relation between C and E: C = E = S n E, S S C. S n (8.44) (8.45)

158

CHAPTER 8. LOSS SYSTEMS WITH FULL ACCESSIBILITY

If we in (8.44) express E by B (8.42), then we get C expressed by B: C = B , 1 + (1 B) (1 + ) C . 1+C (8.46)

B =

(8.47)

This relation between B and C is general and may also be derived as follows. The carried trac Y corresponds to (Y ) accepted call attempts per time unit. The average number of idle sources is (S Y ), so the average number of call attempts per time unit is (S Y ) (8.35). The call congestion becomes: B = (S Y ) Y (S Y ) (S Y ) Y . (S Y )

By denition Y = A (1 C) and from (8.20) we have S = A (1 + )/. Inserting this we get: B = A(1 + ) A(1 C) A (1 C) A(1 + ) A(1 C) (1 + ) C 1+C q.e.d.

B =

From the last equation we see that for small values of the call congestion B the trac congestion is Z times bigger than the call congestion: C B = Z B. 1+ (8.48)

From (8.46) and (8.27) we get for Engset trac: Cn,S () < Bn,S () < En,S () . (8.49)

8.5

Evaluation of Engsets formula

If we try to calculate numerical values of Engsets formula directly from (8.26) (time congestion E), then we will experience numerical problems for large values of S and n. In the following we derive various numerically stable recursive formul for E and its reciprocal I = 1/E. When the time congestion E is known, it is easy to obtain the call congestion B and the trac congestion C by using the formul (8.43) and (8.44). Numerically it is also simple to nd any of the four parameters S, , n, E when we know three of them. Mathematically we may assume that n and eventually S are non-integral.

8.5. EVALUATION OF ENGSETS FORMULA

159

8.5.1

Recursion formula on n

From the general formula (7.27) recursive in n we get using x = (S x) and = / :


x1 Ex1,S () x x1 + x Ex1,S ()

Ex,S () =

Ex,S () =

(S x+1) Ex1,S () , x + (S x+1) Ex1,S ()

E0,S () = 1 .

(8.50)

Introducing the reciprocal time congestion In,S () = 1/En,S (), we nd the recursion formula: Ix,S () = 1 + x Ix1,S () , (S x + 1) I0,S () = 1 . (8.51)

The number of iterations is n. Both (8.50) and (8.51) are analytically exact, numerically stable and accurate recursions for increasing values of x. However, for decreasing values of x the numerical errors accumulate and the recursions are not reliable.

8.5.2

Recursion formula on S

Let us denote the normalised state probabilities of a system with n channels and S1 sources by pn,S1(i). We get the state probabilities of a system with S sources and n channels by convolving these state probabilities with the state probabilities of a single source which are given by {p1,1 (0) = 1 a, p1,1 (1) = a}. We then get states from zero to n + 1, truncate the state space at n, and normalise the state probabilities (cf. Example 3.2.1) (assuming p(x) = 0 when x < 0): qn,S (i) = (1a) pn,S1(i) + a pn,S1(i 1) , i = 0, 1, . . . , n . (8.52)

The obtained state probabilities qn,S (i) are not normalised, because we truncate at state [ n ] and exclude the last term for state [ n+1 ]: qn,S (n + 1) = a pn,S1(n). The normalised state probabilities pn,S (i) for a system with S sources and n channels are thus obtained from the normalised state probabilities pn,S1 (i) for a system with S 1 sources by: qn,S (i) , 1 a pn,S1(n)

pn,S (i) =

i = 0, 1, . . . , n .

(8.53)

160

CHAPTER 8. LOSS SYSTEMS WITH FULL ACCESSIBILITY

The time congestion En,S () for a system with S sources can be expressed by the time congestion En,S1 () for a system with S 1 sources by inserting (8.52) in (8.53): En,S () = pn,S (n) = (1a) pn,S1 (n) + a pn,S1 (n1) 1 a pn,S1 (n) (1a) En,S1 () + a = n E () (S n) n,S1 , 1 a En,S1 ()

where we have used the balance equation between state [n 1, S 1] and state [n, S 1]. Replacing a by using (8.8) we get: n En,S1 () + S n En,S1 () En,S () = . 1 + En,S1 () Thus we obtain the following recursive formula: En,S () = En,S1 () S , S n 1 + {1 En,S1 ()} S >n , En,n () = an . (8.55)

The initial value is obtained from (8.12). Using the reciprocal blocking probability I = 1/E we get: S n In,S () = (8.56) {In,S1() a} , S > n , In,n () = an . S (1a) For increasing S the number of iterations is S n. However, numerical errors accumulate due to the multiplication with (S/(S n) which is greater than one, and the applicability is limited. Therefore, it is recommended to use the recursion (8.58) given in the next section for increasing S. For decreasing S the above formula is analytically exact, numerically stable, and accurate. However, the initial value should be known beforehand.

8.5.3

Recursion formula on both n and S

If we insert (8.50) into (8.55), respectively (8.51) into (8.56), we nd: En,S () = S a En1,S1 () , n + (S n)a En1,S1() n S n In1,S1 () + , Sa S E0,Sn () = 1 , (8.57)

In,S () =

I0,Sn () = 1 ,

(8.58)

which are recursive in both the number of servers and the number of sources. Both of these recursions are numerically accurate for increasing indices and the number of iterations is n (Joys, 1967 [54]).

8.5. EVALUATION OF ENGSETS FORMULA

161

From the above we have the following conclusions for recursion formul for the Engset formula. For increasing values of the parameter, recursion formul (8.50) & (8.51) are very accurate, and formul (8.57) & (8.58) are almost as good. Recursion formul (8.55) &(8.56) are numerically unstable for increasing values, but unlike the others stable for decreasing values. In general, we have that a recursion, which is stable in one direction, will be unstable in the opposite direction.

Example 8.5.1: Engsets loss system We consider an Engset loss system having n = 3 channels and S = 4 sources. The call rate per idle source is = 1/3 calls per time unit, and the mean service time (1/) is 1 time unit. We nd the following parameters: = 1 = 3 1 = 1+ 4 erlang (oered trac per idle source),

a =

erlang (oered trac per source), erlang (oered trac),

A = S a=1 Z = 1 A 3 = S 4

(peakedness).

From the state transition diagram we obtain the following table:

i 0 1 2 3 Total

(i) 4/3 3/3 2/3 1/3

(i) 0 1 2 3

q(i) 1.0000 1.3333 0.6667 0.1481 3.1481

p(i) 0.3176 0.4235 0.2118 0.0471 1.0000

i p(i) 0.0000 0.4235 0.4235 0.1412 0.9882

(i) p(i) 0.4235 0.4235 0.1412 0.0157 1.0039

We nd the following blocking probabilities: Time congestion: E3,4 1 3 1 3 1 3 = p(3) = 0.0471 , AY 1 0.9882 = = 0.0118 , A 1
3

Trac congestion:

C3,4

Call congestion:

B3,4

= {(3) p(3)}
i=0

(i) p(i)

0.0157 = 0.0156 . 1.0039

We notice that E > B > C, which is a general result for the Engset case (8.49) & (Fig. 8.6). By

162

CHAPTER 8. LOSS SYSTEMS WITH FULL ACCESSIBILITY

applying the recursion formula (8.51) we, of course, get the same results: E0,4 1 3 1 3 1 3 1 3 = 1, (4 1 + 1) 1 1 4 3 = , 1 7 1 + (4 1 + 1) 3 1 (4 2 + 1) 1 4 3 7 2 + (4 2 + 1) 1 3 (4 3 + 1) 1 2 3 9 3 + (4 3 + 1) 1 3
4 7

E1,4

E2,4

2 , 9 4 = 0.0471 , 85

E3,4

2 9

q.e.d.

Example 8.5.2: Limited number of sources The inuence from the limitation in the number of sources can be estimated by considering either the time congestion, the call congestion, or the trac congestion. The congestion values are shown in Fig. 8.6 for a xed number of channels n, a xed oered trac A, and an increasing value of the peakedness Z corresponding to a number of sources S, which is given by S = A/(1 Z) (8.23). The oered trac is dened as the trac carried in a system without blocking (n = ). Here Z = 1 corresponds to a Poisson arrival process (Erlangs B-formula, E = B = C). For Z < 1 we get the Engset-case, and for this case the time congestion E is larger than the call congestion B, which is larger than the trac congestion C. For Z > 1 we get the Pascal-case (Secs. 8.6 & 8.7 and Example 8.7.1).

8.6

Pascal Distribution (Negative Binomial)

In the Binomial case the arrival intensity decreases linearly with an increasing number of busy sources. Palm & Wallstrm introduced a model where the arrival intensity increases o linearly with the number of busy sources (Wallstrm, 1964 [100]). The arrival intensity in o state i is given by: i = (S + i), 0 i n , (8.59) where and S are positive constants. The holding time are still assumed to be exponentially distributed with intensity . In this section we assume the number of channels is innite. We then set up a state transition diagram (Fig. 8.5 with n innite) and nd the steady state probabilities which only exist for < . We obtain the state probabilities: p(i) = S i
i

0 i < ,

< ,

(8.60)

8.7. TRUNCATED PASCAL DISTRIBUTION where

163

S S+i1 = (1)i . (8.61) i i Formula (8.60) is the Negative Binomial distribution, also called the Pascal distribution (Tab. 6.1). The trac characteristics of this model are obtained by an appropriate substitution of the parameters of the Binomial distribution. This is dealt with in the following section, which deals with a more realistic case.

8.7

Truncated Pascal distribution

We consider the same trac process as in Sec. 8.6, but now we restrict the number of servers to a limited number n. The restriction < is no more necessary as we always will obtain statistical equilibrium with a nite number of states. The state transition diagram is shown in Fig. 8.5, and state probabilities are given by: S i
n

p(i) =

j=0

S j

0 i n.

(8.62)

This is the truncated Negative Binomial (Pascal) distribution. Formally it is obtained from the Bernoulli/Engset case by the the following substitutions: S is replaced by is replaced by S, . (8.63) (8.64)

By these substitutions all formul of the Bernoulli/Engset cases are valid for the truncated Pascal distribution, and the same computer programs can be use for numerical evaluation. It can be shown that the state probabilities (8.62) like state probabilities for Erlang and Engset loss systems are valid for arbitrary holding time distribution (Iversen, 1980 [38]). Assuming exponentially distributed holding times, this model has the same state probabilities as Palms rst normal form, i.e. a system with a Poisson arrival process having a random intensity distributed as a gamma-distribution (inter-arrival times become Pareto distributed, which is a heavy-tailed distribution. The model is used for modelling overow trac which has a peakedness greater than one. Thus we get from (8.21) by the above substitution (8.64): Z= 1 > 1, 1 = <1 (8.65)

Remember that the peakedness of a trac stream is calculated for an innite number of channels. For the Pascal case we get (cf. (8.49)): Cn,S () > Bn,S () > En,S () . (8.66)

164
S

CHAPTER 8. LOSS SYSTEMS WITH FULL ACCESSIBILITY


........... ............... ................ . .................. ..... .. . .... ... ........ ..... ... ... .. .... ........... ..... .... .. . . .. . .. ............ .. ... .. ..... ........ . .. . .. .. ... .. .. .. .. .. . . . . . .. . . . . . . . . . . . . . . . . . . . . . . . . . . . . . . .. .. . . .. .. .. . .. .. ... . . .. ... .. .. . ... .. . ................ ............... .. ................. ... .... ... ......... ...... ... .... ... . . ... ..... ................ ............... .............. .... .......

(S +1)

.... ..... ..... ........... ............. ...... ......... ..... .... ..... .... ... ... ... ... .... ........... ..... .... .... ........... ... .... .. . .. .. .. .. .. . .. .. . .. . . . . . . . . . . . . . . . . . . .. . .. ... .. . .. ..... ... . ... ... . ... .. ... ... .......... ...... ... .... ... ......... ...... ... .. . .. ................. ................ ................ ...............

(S +i)

.... .... ..... ..... .......... ...... ......... ...... ......... ..... .... ..... .... ... ... .... ........... ..... .... ........... .... ........... .... .... ........... . .. .. .. .. .. .. .. .. . .. .. .. .. .. .. .. .. . . . .. . . . . . . . . . . . . . . . . . . . . . . . . . . . . . . .. .. . .. .. . . .. . ... ... ... ... ... .. ... . .. ..... . . .. ... . . .. ... ... ......... .... ... .......... .... ... ......... ..... ... ......... .. . . .. ................. ................ ................ ...............

(S +n1)

n1

(n1)

Figure 8.5: State transition diagram for the Pascal (truncated Negative Binomial) case.
Example 8.7.1: Peakedness: numerical example In Fig. 8.6 we keep the number of channels n and the oered trac A xed, and calculate the blocking probabilities for increasing peakedness Z. For Z > 1 we get the Pascal-case. For this case the time congestion E is less than the call congestion B which is less than the trac congestion C. We observe that both the time congestion and the call congestion have a maximum value. Only the trac congestion gives a reasonable description of the performance of the system.

Example 8.7.2: Pascal loss system We consider a Pascal loss system with n = 4 channels and S = 2 sources. The arrival rate is = 1/3 calls/time unit per idle source, and the mean holding time (1/) is 1 time unit. We nd the following parameters when we for the Engset case let S = 2 (8.63) and = 1/3 (8.64): = 1 = , 3 1 = , 1+ 2 1 2 =1 3 . 2 erlang,

a =

A = S a = 2 1 1 = 1+ 1

Z =

1 3

From a state transition diagram we get the following parameters:

i 0 1 2 3 4 Total

(i) 0.6667 1.0000 1.3333 1.6667 2.0000

(i) 0 1 2 3 4

q(i) 1.0000 0.6667 0.3333 0.1481 0.0617 2.2099

p(i) 0.4525 0.3017 0.1508 0.0670 0.0279 1.0000

i p(i) 0.0000 0.3017 0.3017 0.2011 0.1117 0.9162

(i) p(i) 0.3017 0.3017 0.2011 0.1117 0.0559 0.9721

8.7. TRUNCATED PASCAL DISTRIBUTION Congestion Probability [%] 16


. ... ... ... ... ... ... ... ... ... ... . .. ... ... ... ... ... ... ... ... ... ... . .. ... ... ... ... ... ... ... ... ... ... . . ... ... ... ... ... ... ... ... ... ... . . ... ... ... ... ... ... ... ... ... ... . ... ... ... ... ... ... ... ... ... ... ... ... ... ... ... ... ... ... ... ... .. ... ... ... ... ... ... ... ... ... .. ... ... ... ... ... ... ... ... ... .. .. ... ... ... ... ... ... ... ... .. .. ... ... ... ... ....................................... ........................................... ... .............................. ............................ ... ................... .................. ... ... .............. ............. ... ... ........... ........... . .. ........ ......... .. .. ....... ....... .. ...... ... .......... .. ................................................................... .. ..... .. . .... ................................................................................................................................ ...................................... .. ..... ................................ ......... . . . .. .. ........... ............ .. ....... .. ....... .. .. ... . .. .. ...... .... .. ..... .. ... .... ..... ....... .... . .. .... .. .. ... .... ..... ... . . .... ... ... . .. . ... . .. .. ... .. ... ... .. ... ... .. ... ... .. ... .. ... .. ... ... ... ... ... . . .. .. ... .. ... ... .. ... .. .. .. .. ... .. .. .. .. ... ... .. ... .. .. .. ... .. ... .. . . .. .. ... .. ... .. .. ... .. ... .. .. .. ... ... . .. .. .. .. .. .. ... ... .. ... ..... . ... .. . . ... .. ... .... .. .... .... .. ... .... .. .. ... .. ... .. ............ ............ ...................... ...................... ..

165

12

B E

0.0

0.5

1.0

1.5

2.0

2.5 Peakedness Z

Figure 8.6: Time congestion E, Call congestion B and Trac congestion C as a function of peakedness Z for BPPtrac i a system with n = 20 trunks and an oered trac A = 15 erlang. More comments are given in Example 8.5.2 and Example 8.7.1. For applications the trac congestion C is the most important, as it is almost a linear function of the peakedness.

We nd the following blocking probabilities:

Time congestion:

E4,2 C4,2

1 3 1 3

= p(4) = 0.0279 . AY 1 0.9162 = = 0.0838 . A 1


4

Trac congestion:

Call congestion:

B4,2

1 3

= {(4) p(4)}
i=0

(i) p(i)

0.0559 = 0.0575 . 0.9721

We notice that E < B < C, which is a general result for the Pascal case. By using the same

166

CHAPTER 8. LOSS SYSTEMS WITH FULL ACCESSIBILITY

recursion formula as for the Engset case (8.50), we of course get the same results: E0,2 1 3 1 3 1 3 1 3 1 3 = 1.0000 , 1 2 = , 2 5 1+ 3 1
3 3 2 3

E1,2

E2,2

3 3

2 5

2+
4 3

2 5

1 , 6 2 , 29 5 = 0.0279 179

E3,2

4 3

1 6

3+
5 3

1 6

I4,2

5 3

2 29

4+

2 29

q.e.d.

Updated: 2006.02.13

Chapter 9 Overow theory


In this chapter we consider systems with restricted (limited) accessibility, i.e. systems where a subscriber or a trac ow only has access to k specic channels from a total of n (k n). If all k channels are busy, then a call attempt is blocked even if there are idle channels among the remaining (nk) channels. An example is shown in Fig. 9.1, where we consider a hierarchical network with trac from A to B, and from A to C. From A to B there is a direct (primary) route with n1 channels. If they all are busy, then the call is directed to the alternative (secondary) route via T to B. In a similar way, the trac from A to C has a rst-choice route AC and an alternative route ATC. If we assume the routes TB and TC are without blocking, then we get the accessibility scheme shown to the right in Fig. 9.1. From this we notice that the total number of channels is (n1 + n2 + n12 ) and that the trac AB only has access to (n1 + n12 ) of these. In this case sequential hunting among the routes should be applied so that a call only is routed via the group n12 , when all n1 primary channels are busy. It is typical for a hierarchical network that it possesses a certain service protection. Independent of how high the trac from A to C is, then it will never get access to the n1 channels. On the other hand, we may block calls even if there are idle channels, and therefore the utilisation will always be lower than for systems with full accessibility. The utilisation will, however, be bigger than for separate systems with the same total number of channels. The common channels allows for a certain trac balancing between the two groups. Historically, it was necessary to consider restricted accessibility because the electro-mechanical systems had very limited intelligence and limited selector capacity (accessibility). In digital systems we do not have these restrictions, but still the theory of restricted accessibility is important both in networks and in guaranteeing the grade-of-service.

167

168

CHAPTER 9. OVERFLOW THEORY

B
n1 n1

A A
n12

T A C
n2 n12

n2

C
Figure 9.1: Telecommunication network with alternate routing and the corresponding accessibility scheme, which is called an ODellgrading. We assume the links between the transit exchange T and the exchanges B and C are without blocking. The n12 channels are common for both trac streams.

9.1

Overow theory

The classical trac models assume that the trac oered to a system is pure chance trac type one or two, PCTI or PCTII. In communication networks with alternative trac routing, the trac which is lost from the primary group is oered to an overow group, and it has properties dierent from PCT trac (Sec. 6.4). Therefore, we cannot use the classical models for evaluating blocking probabilities of overow trac.

Example 9.1.1: Group divided into two Let us consider a group with 16 channels which is oered 10 erlang PCTI trac. By using Erlangs Bformula we nd the blocking probability E = 2.23% and the lost trac 0.2230 erlang. We now assume sequential hunting and split the 16 channels into a primary group and an overow group, each of 8 channels. By using Erlangs Bformula we nd the overow trac from the primary group equal to 3.3832 erlang. This trac is oered to the overow group. Using Erlangs Bformula again, we nd the lost trac from the overow group: A = 3.3832 E8 (3.3832) = 0.0493 [erlang] . The total blocking probability in this way becomes 0.493%, which is much less than the correct result 2.23%. We have made an error by applying the Bformula to the overow trac, which is not PCTI trac, but more bursty.

In the following we describe two classes of models for overow trac. We can in principle study the trac process either vertically or horizontally. By vertical studies we calculate the state probabilities (Sec. 9.1.19.4.3). By horizontal studies we analyse the distance between call arrivals, i.e. the inter-arrival time distribution (9.5).

9.1. OVERFLOW THEORY A


.. .................. .................. . . .. .................. .................. . . n Kostens system

169

. . ................... ................... ..

. . ................... .................. . n Brockmeyers system

. . ................... ................... ..

. ................... ................... ..

. ................... .................. .. n Generalised system

. ................... ................... ..

. .................... ................... ..

Figure 9.2: Dierent overow systems described in the literature.

9.1.1

State probability of overow systems

Let us consider a full accessible group with ordered (sequential) hunting. The group is split into a limited primary group with n channels and an overow group with innite capacity. The oered trac A is assumed to be PCTI. This is called Kostens system (Fig. 9.2). The state of the system is described by a two-dimensional vector: p(i, j), 0 i n, 0 j , (9.1)

which is the probability that at a random point of time i channels are occupied in the primary group and j channels in the overow group. The state transition diagram is shown in Fig. 9.3. Kosten (1937 [68]) analysed this model and derived the marginal state probabilities:

p(i, ) =
j=0 n

p(i, j),

0 i n,

(9.2)

p( , j) =
i=0

p(i, j),

0 j < .

(9.3)

Riordan (1956 [87]) derived the moments of the marginal distributions. Mean value and peakedness (variance/mean ratio) of the marginal distributions, i.e. the trac carried by the two groups, become: Primary group: m = A {1 En (A)} , v = Z = 1 A {En1 (A) En (A)} , m Z = 1 Fn1 (A) = 1 an 1 . (9.4) (9.5)

170

CHAPTER 9. OVERFLOW THEORY . . . . . . . . . .. . . . .. . . . . . . . . . . . . . . . . . . . . j +1 j +1 j +1 ..... ........... j +1 . . . . . . ..................................................................................... . ... . . .. . . . . . . 0j 1j 2j . ...................................................... n j ........ . . ... . . . n .......................................... 1 2 3 . . . . . . . . . . . j j j ......... ........... j . . . . .. . . .. . . . . . . . . . . . . . . . . . . . . . . . . . . . . . . .. . . . . . . .. . . . . . . . . . . . . . . . . . . . . . . . . . . . . . . . . . . . . . . . . . . . . . .2 .2 .2 . . . . ...... ........... 2 . . . . . . . . . . .. ... .. . . . . . . .. .. .. . .. .. .. . . . .. .. .. ... ........... . . ..... ..... ..... ...... ... .... ..................................................................................................................................................... . ........................................................ ................. ... .. ........................ . .. . ... ... ... ... ... .. .. .................. ... . . . . . . . . . . . . . . . . . . . . . . . . . . . . . . . . . 0 1 . . 1 1 . . . . 2 1 . . . . . ...................................................... n 1 ........ . . . . . . . . . . .. .. . ....................... ....................... ..................... .. .. . .. . . . ......................... ........................ ...................... ... ... ... ... ... . ... .... ..... ....... ... ... ... ... ... ... ... ......... ......... ........ ......... ... ... . .. . . . n ..................................... . . . 1 2 3 . . . . . . . . . . . . . . . . . . . . . . . . . . . . . . . . . . . . . . . .1 . .1 .1 . . . . ......... ............1 . . . . . . . .. . . .. .. .. . . . .. .. . .. . . .. .. .. . .. .. . .... .. .. ... . .... ..... ............ . ... ... . ................................................................................................................................................. . .............................................................................. .. ... ......................... .. . . ... ... ... ... ... .. . ...................... . . . . . . . . . . . . . . . . . . . . . . . . . . . . . . . . . . 0 0 . . . . . 1 0 . . 2 0 . . . . . . ..................................................... n 0 ......... . . . . . . .. ...................... ..... ................ ..................... .. . .. .. . .. . .. . .. . ........................ ........................ ..................... .. . . . . ... ... ... ... ... . .... .... .. .... ..... .. .. ... ... ... ... .......... ......... ......... ..... .. ........ ..... . n ................... 1 2 3 .
. . . . . . . . . . . . . . . . . . . . . . . . . . . . . . . . . . . . . . . . . . . . . . . . . . . . . . .. .. .. . . . . .. .. .. . . . .. . . . . . . ....... ....... ....... ....... ....... ....... .. .. .. ... ... ... ... ... .. .. .. .. ... ... ... ... ... ... ......................... ...................... ........................ .. . . ........................ ...................... .. ....................... .. . . . .. . . . . . .. . . . . . . . . . . . . . . . . . . . . . . . . . . . . . . . . . . . . . . . . .. . . . . . . . ..................... ...................... ..................... .. . .. .. .. . .. . .. .. .... .. .................. ... ......................... ...................... ... .. .. ... ... ............. ............. ............ ........... ........... ........... . . . . . . . . . . . . . . . . . . . . . . . . . . . . . . . . . . . . . . . . . . . . . . . . . . . ... ... ... . .. . .. .. . . .. . . . . . . .

Figure 9.3: State transition diagram for Kostens system, which has a primary group with n channels and an unlimited overow group. The states are denoted by (i, j), where i is the number of busy channels in the primary group, and j is the number of busy channels in the overow group. The mean holding time is chosen as time unit. where Fn1 (A) is the improvement function of Erlangs B-formula. Secondary group = Overow group: m = A En (A) , A v = Z = 1m+ 1. m n+1A+m (9.6) (9.7)

Experience shows that the peakedness Z is a good measure for the relative blocking probability a trac stream with a given mean value is subject to. In Fig. 9.4 we notice that the peakedness of overow trac has a maximum for a xed trac and an increasing number of channels. Peakedness has the dimension [channels]. Peakedness is applicable for theoretical calculations, but dicult to estimate accurately from observations. For PCTI trac the peakedness is equal to one, and the blocking is calculated by using the Erlang-B formula. If the peakedness is less than one (9.5), the trac is called smooth and it experiences less blocking than PCTI trac. If the peakedness is larger than one, then the trac is called bursty and it experiences larger blocking than PCTI trac. Overow trac is usually bursty (9.7). Brockmeyer (1954 [10]) derived the state probabilities and moments of a system with a limited overow group (Fig. 9.2), which is called Brockmeyers system. Bech (1954 [6])

9.2. EQUIVALENT RANDOM TRAFFIC METHOD

171

did the same by using matrix equations, and obtained more complicated and more general expressions. Brockmeyers system is further generalised by Schehrer who also derived higher order moments for nite overow groups. Wallstrm (1966 [101]) calculated state probabilities and moments for overow trac of a o generalised Kosten system, where the arrival intensity depends either upon the total number of calls in the system or the number of calls in the primary group.

3.00 2.75 2.50 2.25 2.00 1.75 1.50 1.25 1.00

Z
................. ................... ........ ....... ..... ..... ..... .... ..... .... .... .... .... .... ... .... .. .... ... ... . .. ... ... . . ... ... . . ... ... . ... ... ... ... ... .. .. .. ... ... .. ... .. .. .. .. .. ... .. .. . .. .. ... .. .. .. .. .. .. . .. .. .. .. .. .. .. .. .................... .. ..................... .. .... .... .. .... ... .... .... ... .. .... ... ........ .. ...... ... ... ... .. ... ... .. ... ... ... .. ... .. ... ... . ... .. ... ... .. ... ... . . ... ... ... .. ... .. . ... .. ... .. .. .. .. ... ... .. ... .. ... .. .. . . . ... ... .. .. .. .. ... . . ... .. .. ... .. .. ... . . ... ... .. .. .. .. . . ... ... .. .. ... .. .. ... . . . ... ... .. .. .. .. ... ... .. ... .. ... ... .... . . .. .... .. .... .. ............ ............ . ... .... . .... .... .... .... ... .... ... ... .... ... . . .... .... .... .... ... . .... . ... .... .... .... . . .... . .... .. .... ... ... ... .. ... ..... ... .... . .... ... .... ... .. .... .. .. ... .. . .... . .... .. .... ........ ... ... ......... .. ..... . . . . ..... ... ... .. ..... .. ... ... .. .. . ..... . ... ... ..... .. .. ... ... ..... ... .. .. ... . . ..... .... ...... ... .. .. ... ..... ... .. .. . .... ...... .... ...... .. .. ... ... .. .. ...... .... . . .... . ..... .. .. .... ... .... ... .. .. . . . .... .... .. .. ... .. .. ... . . . .... .... .. .. ... ..... .. .. ... .... . . ..... .......... ... ..... ... .............. ... . .. ..... ..... ... .. . .. ....... ... ... .. ........ ..... ..... . ... .... .... ... ...... .... .... ...... .. .. .. . ...... .. .. .... ...... ....... .......... .. .. .. .. ....... ....... . .. ... . . ........ ........ .. .. ... ... ........ .. ... ........ .. ... . . ......... .. ......... ..... .. .. ..... .. ... .. .......... .. .. ... .......... . .. ..... ..... ........... ........... .. ....... ... ... ... .. . .. ............. ..... . ..... ............ ............. . . . ...... . .... ....... ...... ...... .. ..... ...... ...... .............. ...... ............... . . ....... ....... ..... . ................. .. ... ............. ..... .. . ..... ... ... ..... ........ ........ . .. .. .. ..... .......... . .......... .... ......... ..... ..... .... .. . . . ...... ............ ............ ....... .. .. ... .... .... .. .. ..... ........ ............... ............... . ........ .. .......... ................... ................... .... .. .......... .... .... ..... ....... ......................... ............. .......................... .............. . .. . .. .... ... .................................... ..................................... .................. ..................... . .. .. . .. ............................. .. . . .......................... ............................. ................................ .... ... .... . .. .... ..... ................................................. .......................................................... ... ... .... ............................................................... .... .. .... ............................................ ... .. ... .. . . ......... .. . . ..... ........ .. ...

A 20

10

12

16

20

24

n 28

Figure 9.4: Peakedness Z of overow trac as a function of number of channels for a xed value of the oered trac. Notice that Z has a maximum. When n becomes large call attempts are seldom blocked and the blocked attempts will be mutually independent. Therefore, the process of overowing calls converges to a Poisson process (Chap. 6).

9.2

Equivalent Random Trac method

This equivalent method is also called the ERTmethod, Wilkinsons method or WilkinsonBretschneiders method. It was published independently at the same time in USA by Wilkinson (1956 [102]) and in Germany by Bretschneider (1956 [8]). It plays a key role when dimensioning telecommunication networks.

172

CHAPTER 9. OVERFLOW THEORY


.. ......................... ......................... . .

m1,1 , v1
.. .......................... ......................... . .

. . . . . . . . . . . . . . . . . . . . . . . . . . . .

. . . . . . . . . . . . . . . . . . . . . . . . . . . .

. . . . . . . . . . . . . . . . . . . . . . . . . . .

m1,2 , v2 . . .
.. .......................... .......................... .

. . . . .
. . . . . . . .

. . . . .
. . . . . . . .

. . . . . l
. . . . . . . .

m1,g , vg

.... .... .... .... .... .... .... .... .... .... .... .... .... .... .... .... .... .... .... .... .... .... .... .... .... .... .... .... .... .... .... .... .... .... .... .... .... ... ... ... ... ... ... ... ... ... ... ... ... ... ... ... ... ... ... ... ... ... ... ... ... ... ... ... ... ... ... ... ... ... ... ... ... ...

Ax

. ........................... .......................... ..

nx

. .......................... .......................... ..

m, v

Figure 9.5: Application of the ERT-method to a system having g independent trac streams oered to a common group of channels. The aggregated overow process of the g trac streams is said to be equivalent to the trac overowing from a single full accessible group with the same mean and variance of the overow trac. (9.8) & (9.9).

9.2.1

Preliminary analysis

Let us consider a group with channels which is oered g trac streams (Fig. 9.5). The trac streams may for instance be trac which is oered from other exchanges to a transit exchange, and therefore they cannot be described by classical trac models. Thus we do not know the distributions (state probabilities) of the trac streams, but we are satised (as it is often the case in applications of statistics) by characterising the i th trac stream by its mean value m1,i and variance vi . With this simplication we will consider two trac streams to be equivalent, if they have same mean value and variance. The total trac oered to the group with m=
i=1

channels has the mean value:


g

m1,i .

(9.8)

We assume that the trac streams are independent (non-correlated), and thus the variance of the total trac stream becomes:
g

v=
i=1

vi .

(9.9)

The total trac is characterised by m and v. So far we assume that m < v. We now consider this trac to be equivalent to a trac ow, which is lost from a full accessible group and has same mean value m and variance v. In Fig. 9.5 the upper system is replaced by the equivalent system at the lower part of Fig. 9.5, which is a full accessible system with (nx + ) channels oered the trac Ax . For given values of m and v we therefore solve equations (9.6) and (9.7) with respect to n and A. It can be shown there that a unique solution exists, and it will be denoted by (nx , Ax ).

9.2. EQUIVALENT RANDOM TRAFFIC METHOD The lost trac is found from the Erlangs B-formula: A = Ax Enx + (Ax ) . As the oered trac is m, the trac congestion of the system becomes: C= A . m

173

(9.10)

(9.11)

Notice: the blocking probability is not Enx + (Ax ). We should remember the last step (9.11), where we relate the lost trac to the originally oered trac, which in this case is given by m (9.8). We notice that if the overow trac is from a single primary group with PCTI trac, then the method is exact. In the general case with more trac streams the method is approximate, and it does not yield the exact mean blocking probability.
Example 9.2.1: Paradox In Sec. 6.3 we derived Palms theorem, which states that by superposition of many independent arrival processes, we locally get a Poisson process. This is not contradictory with (9.8) and (9.9), because these formul are valid globally.

9.2.2

Numerical aspects

When applying the ERTmethod we need to calculate (m, v) for given values of (A, n) and vice versa. It is easy to obtain (m, v) for given (A, n) by using (9.4) & (9.5). To obtain (A, n) for given (m, v), we have to solve two equations with two unknown. It requires an iterative procedure, since En (A) cannot be solved explicitly with respect to neither n nor A (Sec. 7.5). However, we can solve (9.7) with respect to n: n=A
v m+ m m 1, v m+ m 1

(9.12)

so that we know n for given A. Thus A is the only independent variable. We can use NewtonRaphsons iteration method to solve the remaining equation, introducing the function: f (A) = m A En (A) = 0 . For a proper starting value A0 we improve this iteratively until the resulting values of m and v/m become close enough to the known values. Yngv Rapp (1965 [86]) has proposed a good approximate solution for A, which can be used e as initial value A0 in the iteration: v v Av+3 1 . (9.13) m m

174

CHAPTER 9. OVERFLOW THEORY

From A we get n, using (9.12). Rapps approximation is sucient accurate for practical applications, except when Ax is very small. The peakedness Z = v/m has a maximum, obtained when n is little larger than A (Fig. 9.4). For some combinations of m and v/m the convergence is critical, but when using computers we can always nd the correct solution. Using computers we operate with non-integral number of channels, and only at the end of calculations we choose an integral number of channels greater than or equal to the obtained results (typical a module of a certain number of channels (8 in GSM, 30 in PCM, etc.). When using tables of Erlangs Bformula, we should in every step choose the number of channels in a conservative way so that the blocking probability aimed at becomes worst case. The above-mentioned method presupposes that v/m is larger than one, and so it is only valid for bursty trac. Individual trac stream in Fig. 9.5 are allowed to have vi /mi < 1, provided the total aggregated trac stream is bursty. Bretschneider ([9], 1973) has extended the method to include a negative number of channels during the calculations. In this way it is possible to deal with smooth trac (EERT-method = Extended ERT method).

9.2.3

Parcel blocking probabilities

The individual trac streams in Fig. 9.5 do not have the same mean value and variance, and therefore they do not experience equal blocking probabilities in the common overow group with channels. From the above we calculate the mean blocking (9.11) for all trac streams aggregated. Experiences show that the blocking probability experienced is proportional to the peakedness Z = v/m. We can split the total lost trac into individual lost trac parcels by assuming that the trac lost for stream i is proportional to the mean value mi and to the peakedness Zi = vi /mi of the stream. We obtain:
g

=
i=1

A ,i
g

= cA
i=1

m1,i

vi m1,i

= cA v, from which we nd the constant c = 1/v. The (trac) blocking probability for trac stream i, which is called the parcel blocking probability for stream i, then becomes: Ci = A ,i vi = A . mi v (9.14)

9.2. EQUIVALENT RANDOM TRAFFIC METHOD

175

Furthermore, we can divide the blocking among the individual groups (primary, secondary, etc.). Consider the equivalent group at the bottom of Fig. 9.5 with nx primary channels and secondary (overow) channels, we can calculate the blocking probability due to the nx primary channels, and the blocking probability due to the secondary channels. The probability that the trac is lost by the channels is equal to the probability that the trac is lost by the nx + channels, under the condition that the trac is oered to the channels: H(l) = A Enx +l (A) Enx +l (A) = . A Enx (A) Enx (A) Enx +l (A) . Enx (A) (9.15)

The total loss probability can therefore be related to the two groups: Enx +l (A) = Enx (A) (9.16)

By using this expression, we can nd the blocking for each channel group and then for example obtain information about which group should be increased by adding more channels.
Example 9.2.2: Example 9.1.1 continued In example 9.1.1 the blocking probability of the primary group of 8 channels is E8 (10) = 0.3383. The blocking of the overow group is H(8) = The total blocking of the system is: E16 (10) = E8 (10) H(8) = 0.3383 0.06592 = 0.02231 . E16 (10) 0.02231 = = 0.06592 . E8 (10) 0.3383

Example 9.2.3: Hierarchical cellular system We consider a cellular system HCS covering three areas. The trac oered in the areas are 12, 8 and 4 erlang, respectively. In the rst two cells we introduce micro-cells with 16, respectively 8 channels, and a common macro-cell covering all three areas is allocated 8 channels. We allow overow from micro-cells to macro-cells, but do not rearrange the calls from macro- to micro-cells when a channel becomes idle. Furthermore, we look away from hand-over trac. Using (9.6) & (9.7) we nd the mean value and the variance of the trac oered to the macro-cell:

Cell i 1 2 3 Total

Oered trac Ai 12 8 4 24

Number of channels ni (j) 16 8 0

Overow mean m1,i 0.7250 1.8846 4.0000 6.6095

Overow variance vi 1.7190 3.5596 4.0000 9.2786

Peakedness Zi 2.3711 1.8888 1.0000 1.4038

176

CHAPTER 9. OVERFLOW THEORY

The total trac oered to the macro-cell has mean value 6.61 erlang and variance 9.28. This corresponds to the overow trac from an equivalent system with 10.78 erlang oered to 4.72 channels. Thus we end up with a system of 12.72 channels oered 10.78 erlang. Using the Erlang-B formula, we nd the lost trac 1.3049 erlang. Originally we oered 24 erlang, so the total trac blocking probability becomes B = 5.437%. The three areas have individual blocking probabilities. Using (9.14) we nd the approximate lost trac from the areas to be 0.2434 erlang, 0.5042 erlang, and 0.5664 erlang, respectively. Thus the trac blocking probabilities become 2.03%, 6.30% and 14.16%, respectively. A computer simulation with 100 million calls yields the blocking probabilities 1.77%, 5.72%, and 15.05%, respectively. This corresponds to a total lost trac equal to 1.273 erlang and a blocking probability 5.30%. The accuracy of the method of this chapter is sucient for real applications.

9.3

Fredericks & Haywards method

Fredericks (1980 [29]) has proposed an equivalence method which is simpler to use than Wilkinson-Bretschneiders method. The motivation for the method was rst put forward by W.S. Hayward. Fredericks & Haywards equivalence method also characterises the trac by mean value A and peakedness Z (0 < Z < ) (Z = 0 is a trivial case with constant trac). The peakedness (7.7) is the ratio between the variance v and the mean value m1 of the state probabilities, and it has the dimension [channels]. For random trac (PCTI) we have Z = 1 and we can apply the Erlang-B formula. For peakedness Z = 1 Fredericks & Haywards method proposes that the system has the same blocking probability as a system with n/Z channels, oered trac A/Z, and thus peakedness Z = 1. For the latter system we may apply the ErlangB formula: E(n, A, Z) E n A , , 1 Z Z
n EZ

A Z

(9.17)

When Z = 1 we assume the trac is PCT-I and apply Erlangs Bformula for calculating the congestion. It is the trac congestion we obtain when using the method (cf. Sec. 9.3.1). For xed value of the blocking probability in the Erlang-B formula we know (Fig. 7.4) that the utilisation increases, when the number of channels increases: the larger the system, the higher utilisation for a xed blocking probability. Fredericks & Haywards method thus expresses that if the trac has a peakedness Z larger than PCTI trac, then we get a lower utilisation than the one obtained by using Erlangs Bformula. If peakedness Z < 1, then we get a higher utilisation. We avoid solving the equations (9.6) and (9.7) with respect to (A, n) for given values of (m, v). The method can easily be applied for both peaked and smooth trac. In general we

9.3. FREDERICKS & HAYWARDS METHOD

177

get an non-integral number of channels and thus need to evaluate the Erlang-B formula for a continuous number of channels. Basharin & Kurenkov has extended the method to comprise multi-slot (multi-rate) trac, where a call requires d channels from start to termination. If a call uses d channels in stead of one (change of scale), then the mean value becomes d times bigger and the variance d 2 times bigger. Therefore, the peakedness becomes d times bigger. In stead of reducing number of channels by the factor Z, we may x the number of channels and make the slot-size Z times bigger: n A A (n, A, Z, d) n, , 1, d Z , , 1, d . (9.18) Z Z Z If we have more trac streams oered to the same group, then it may be an advantage to keep the number of channels xed, but then we get the problem that d Z in general will not be integral.
Example 9.3.1: Fredericks & Haywards method If we apply Fredericks & Haywards method to example 9.2.3, then the macro-cell has (8/1.4038) channels and is oered (6.6095/1.4038) erlang. The blocking probability is obtained from Erlangs B-formula and becomes 0.19470. The lost trac is calculated from the original oered trac (6.6095 erlang) and becomes 1.2871 erlang. The blocking probability of the system becomes E = 1.2871/24 = 5.36%. This is very close to the result obtained (5.44%) by the ERTmethod.

Example 9.3.2: Multi-slot trac We shall later consider service-integrated system with multi-rate (multi-slot) trac. In example 10.4.3 we consider a trunk group with 1536 channels, which is oered 24 trac streams with individual slot-size and peakedness. The exact total trac congestion is equal to 5.950%. If we calculate the peakedness of the oered trac by adding all trac streams, then we nd peakedness Z = 9.8125 and a total mean value equal to 1536 erlang. Fredericks & Haywards method results in a total trac congestion equal to 6.114%, which thus is a conservative estimate (worst case).

9.3.1

Trac splitting

In the following we shall give a natural interpretation of Fredericks & Haywards method and at the same time discuss splitting of trac streams. We consider a trac stream with mean value A, variance v, and peakedness Z = v/A. We split this trac stream into g identical sub-streams. A single sub-stream then has the mean value A/g and peakedness Z/g because the mean value is reduced by a factor g and the variance by a factor g 2 (Example 3.3.2). If we choose the number of sub-streams g equal to Z, then we get the peakedness Z = 1 for each sub-stream. Let us assume the original trac stream is oered to n channels. If we also split the n channels into g sub-group (one for each sub-stream), then each subgroup has n/g channels.

178

CHAPTER 9. OVERFLOW THEORY

Each sub-group will then have the same blocking probability as the original total system. By choosing g = Z we get peakedness Z = 1 in the sub-streams, and we may (approximately) use Erlangs Bformula for calculating the blocking probability. This is a natural interpretation of Fredericks & Haywards method. It can easily be extended to comprise multislot trac. If every call requires d channels during the whole connection time, then by splitting the trac into d sub-streams each call will use a single channel in each of the d sub-groups, and we will get d identical systems with single-slot trac. The above splitting of the trac into g identical trac streams shows that the blocking probability obtained by Fredericks-Haywards method is the trac congestion. The equal splitting of the trac at any point of time implies that all g trac streams are identical and thus have the mutual correlation one. In reality, we cannot split circuit switched trac into identical sub-streams. If we have g = 2 streams and three channels are busy at a given point of time, then we will for example use two channels in one sub-stream and one in the other, but anyway we obtain the same optimal utilisation as in the total system, because we always will have access to an idle channel in any sub-group (full accessibility). The correlation between the sub-streams becomes smaller than one. The above is an example of using more intelligent strategies so that we maintain the optimal full accessibility. In Sec. 6.3.2 we studied the splitting of the arrival process when the splitting is done in a random way (Raikovs theorem 6.2). By this splitting we did not reduce the variation of the process when the process is a Poisson process or more regular. The resulting sub-stream point processes converge to Poisson processes. In this section we have considered the splitting of the trac process, which includes both the arrival process and the holding times. The splitting process depends upon the state. In a sub-process, a long holding time of a single call will result in fewer new calls in this sub-process during the following time interval, and the arrival process will no longer be a renewal process. Most attempts of improving Fredericks & Haywards equivalence method are based on reducing the correlation between the sub-streams, because the arrival processes for a single sub-stream is considered as a renewal process, and the holding times are assumed to be exponentially distributed. From the above we see that these approaches are deemed to be unsuccessful, because they will not result in an optimal trac splitting. In the following example we shall see that the optimal splitting can be implemented for packet switched trac with constant packet size. If we split a trac stream into a sub-stream so that a busy channel belongs to the sub-stream with probability p, then it can be shown that the sub-stream has peakedness Zp given by: Zp = 1 + p (Z 1) , where Z is the peakedness of the original stream. From this random splitting of the trac process we see that the peakedness converges to one, when p becomes small. This corresponds to a Poisson process and this result is valid for any trac process.

9.4. OTHER METHODS BASED ON STATE SPACE

179

Example 9.3.3: Inverse multiplexing If we need more capacity in a network than what corresponds to a single channel, then we may combine more channels in parallel. At the originating source we may then distribute the trac (packets or cells in ATM) in a cyclic way over the individual channels, and at the destination we reconstruct the original information. In this way we get access to higher bandwidth without leasing xed broadband channels, which are very expensive. If the trac parcels are of constant size, then the trac process is split into a number of identical trac streams, so that we get the same utilisation as in a single system with the total capacity. This principle was rst exploited in a Danish equipment (Johansen & Johansen & Rasmussen, 1991 [53]) for combining up to 30 individual 64 Kbps ISDN connections for transfer of video trac for maintenance of aircrafts. Today, similar equipment is applied for combining a number of 2 Mbps connections to be used by ATM-connections with larger bandwidth (IMA = Inverse Multiplexing for ATM) (Techguide, 2001 [96]), (PostigoBoix & Garc ia-Haro & Aguilar-Igartua, 2001 [83]).

9.4

Other methods based on state space

From a blocking point of the view, the mean value and variance do not necessarily characterise the trac in the optimal way. Other parameters may better describe the trac. When calculating the blocking with the ERTmethod we have two equations with two unknown variables (9.6 & 9.7). The Erlang loss system is uniquely dened by the number of channels and the oered trac Ax . Therefore, it is not possible to generalise the method to take account of more than two moments (mean & variance).

9.4.1

BPP trac models

The BPPtrac models describe the trac by two parameters, mean value and peakedness, and are thus natural candidates to model trac with two parameters. Historically, however, the concept and denition of trac congestion has due to earlier denitions of oered trac been confused with call congestion. As seen from Fig. 8.6 only the trac congestion makes sense for overow calculations. By proper application of the trac congestion, the BPP model is very applicable.

Example 9.4.1: BPP trac model If we apply the BPPmodel to the overow trac in example 9.2.3 we have A = 6.6095 and Z = 1.4038. This corresponds to a Pascal trac with S = 16.37 sources and = 0.2876. The trac congestion becomes 20.52% corresponding to a lost trac 1.3563 erlang, or a blocking probability for the system equal to E = 1.3563/24 = 5.65%. This result is quite accurate.

180

CHAPTER 9. OVERFLOW THEORY

9.4.2

Sanders method

Sanders & Haemers & Wilcke (1983 [93]) have proposed another simple and interesting equivalence method also based on the state space. We will call it Sanders method. Like Fredericks & Haywards method, it is based on a transformation of state probabilities so that the peakedness becomes equal to one. The method transforms a nonPoisson trac with (mean, variance) = (m, v) into a trac stream with peakedness one by adding a constant (zerovariance) trac stream with mean v m so that the total trac has mean equal to variance v. The constant trac stream occupies v m channels permanently (with no loss) and we increase the number of channels by this amount. In this way we get a system with n + (v m) channels which are oered the trac m + (v m) = v erlang. The peakedness becomes one, and the blocking probability is obtained using Erlangs B-formula, and so we nd the trac lost from the equivalent system. This lost trac is divided by the originally oered trac to obtain the trac congestion C. The blocking probability relates to the originally oered trac m. The method is applicable for both both smooth m > v and bursty trac m < v and requires only the evaluation of the ErlangB formula with a continuous number of channels.

Example 9.4.2: Sanders method If we apply Sanders method to example 9.2.3, we increase both the number of channels and the oered trac by v m = 2.6691 (channels/erlang) and thus have 9.2786 erlang oered to 10.6691 channels. From Erlangs B-formula we nd the lost trac 1.3690 erlang, which is on the safe side, but close to the results obtained above. It corresponds to a blocking probability E = 1.3690/24 = 5.70%.

9.4.3

Berkeleys method

To get an ERTmethod based on only one parameter, we can in principle keep either n or A xed. Experience shows that we obtain the best results by keeping the number of channels xed nx = n. We are now in the position where we only can ensure that the mean value of the overow trac is correct. This method is called Berkeleys equivalence method (1934). Wilkinson-Bretschneiders method requires a certain amount of computations (computers), whereas Berkeleys method is based on Erlangs B-formula only. Berkeleys method is applicable only for systems, where the primary groups all have the same number of channels.

Example 9.4.3: Group divided into primary and overow group If we apply Berkeleys method two example 9.1.1, then we get the exact solution, and from this special case originates the idea of the method.

9.5. METHODS BASED ON ARRIVAL PROCESSES

181

Example 9.4.4: Berkeleys method We consider example 9.2.3 again. To apply Berkeleys method correctly, we should have the same number of channels in all three micro-cells. Let us assume all micro-cells have 8 channels (and not 16, 8, 0, respectively). To obtain the overow trac 6.6095 erlang the equivalent oered trac is 13.72 erlang to the 8 primary channels. The equivalent system then has a trac 13.72 erlang oered to (8 + 8 =) 16 channels. The lost trac obtained from the Erlang-B formula becomes 1.4588 erlang corresponding to a blocking probability 6.08%, which is a value a little larger than the correct value. In general, Berkeleys method will be on the safe side.

9.5

Methods based on arrival processes

The models in Chaps. 7 & 8 are all characterised by a Poisson arrival process with state dependent intensity, whereas the service times are exponentially distributed with equal mean value for all (homogeneous) servers. As these models all are independent of the service time distribution (insensitive, i.e. the state probabilities only depend on the mean value of the service time distribution), then we may only generalise the models by considering more general arrival processes. By using general arrival processes the insensitivity property is lost and the service time distribution becomes important. As we only have one arrival process, but many service processes (one for each of the n servers), then we in general assume exponential service times to avoid complex models.

9.5.1

Interrupted Poisson Process

In Sec. 6.4 we considered Kuczuras Interrupted Poisson Process (IPP) (Kuczura, 1977 [71]), which is characterised by three parameters and has been widely used for modelling overow trac. If we consider a full accessible group with n servers, which are oered calls arriving according to an IPP (cf. Fig. 6.7) with exponentially distributed service times, then we can construct a state transition diagram as shown in Fig. 9.6. The diagram is two-dimensional. State (i, j) denotes that there are i calls being served (i = 0, 1, . . . , n), and that the arrival process is in phase j (j = a : arrival process on, j = b : arrival process o ). By using the node balance equations we nd the equilibrium state probabilities p(i, j). Time congestion E becomes: E = p(na) + p(nb) . Call congestion B becomes: B= p(na)
n

(9.20)

E.

(9.21)

p(ia)
i=0

Trac congestion C is dened as the proportion of the oered trac which is lost. The

182

CHAPTER 9. OVERFLOW THEORY


..... ..... ........... . ......... .............. .............. ..... ......... ..... ......... .... ..... ..... ... .... ... ....... ...... ... .... .... .... ....... ......... ..... .... ........... ..... .......... ... .... ....... ................ ... . . .. ... . .... ....... .. .... .. . .. .. .. .. .. .. .. .. .. .. .. .. .. . .. . . . . . . . . . . . . . . . . . . . . . . . . . . . . . . . . . . . . . . . . . . . . . . . . . .. .. .. . . . .. .. .. . . .. ... .. . . .. .. ... .. . .. .... . ..... ................ . ..... . .. ....... ..... . ..... . ......... ....... ........ .. . . ... .. ........ ...... ... .. ........ ...... ... . ... ...... .. . . ... .. .... .... ........ ... ...... .... ....... ... ...... ... ....... .. .. .............. .. .. .. ............. ............. . . . ............. . ............. ............ . .. .. .. .. . .. . . . . . . . . .. . . . . . . . . . . . . . . . . . . . . . . . . . . . . . . . . . . . . . . . . . . . . . . . . . . . . . . . . . . . . . . . . . . . . . . . . . . . . . . . . . . . . . . . . . . . . . . . . . . . . . . . . . . . . . . . . . . . . . . . . . . . . . . . . . . . . . . . . . . . . . . . . . . . . . . . . . . . . . . . . . . . . . . . . . . . . . . . . . .. .. .. .. . .. .. . . . .. .. .. .. .. .. .. .. . . . . ... ... ... ... ... ... .. .. . .... .. . .... . .. ....... ...... ... ........... ........... ..... ...... ..... ...... .... ...... .. .. .. ... ... ... .. .. .. .. .. . . . . . . . . . . .. . . . . . . . . . . . . . .. . . .... ............... . ................... . . .. ... ............... . . . . . . . . . ..... .............. ... ............... ... ............... . . . . .. . . . .. . . . . . . . .. .. .. . . . .. .. .. . .. . . . . ... ... ... ... ... ... .. .. .. .. .. .. .......... .......... .......... .......... .......... ..........

0a

1a

2a

0b

1b

2b

... .......... .................. ... ..... .... ... .... ... ........ ........ .... . ... ... . .. .. .. .. .. .. . . . . . . . . . . . . . . . . .. .. .. .. ... ... . ............ .... ................ . .. .... .. . ..... ... .. ...... ........ .. .. ........ ....... .. .. .... .. .. . .. . . . . . . . . . . . . . . . . . . . . . . . . . . . . . . . . . . . . . . . . . . . . . . . . . . . . . . . . . . .. .. . .. .. . ... ... .. .......... .. ... . ........... .. .. .. ... .. .. . . .. . . . . . ................... ................... . . . . . . . . . .. . .. . . ... ... .. .. .......... ..........

na

nb

Figure 9.6: State transition diagram for a full accessible loss system with n servers, IPP arrival process (cf. Fig. 6.7) and exponentially distributed service times (). oered trac is equal to: A= The carried trac is: Y =
i=0

p(on) = . p(on) + p(o) +


n

i {p(ia) + p(ib)} .

(9.22)

From this we obtain C = (A Y )/A. In fact, the trac congestion will be equal to the call congestion as the arrival process is a renewal process. But this is dicult to derive from the above. As shown in Sec. 6.4.1 the inter-arrival times are hyper-exponentially H2 distributed.

9.5.2

Cox2 arrival process

In Sec. 6.4 we noticed that a Cox2 arrival process is more general than an IPP (Kuczura, 1977 [71]). If we consider Cox2 arrival processes as shown in Fig. 4.10, then we get the state transition diagram shown in Fig. 9.7. From this we nd under the assumption of statistical equilibrium the state probabilities and the following performance measures. Time congestion E: E = p(na) + p(nb) . Call congestion B: B= p 1 p(na) + 2 p(nb)
n n

(9.23) . (9.24)

p 1
i=0

p(ia) + 2
i=0

p(ib)

Trac congestion C. The oered trac is the average number of call attempts per mean service time. The mean inter-arrival time is: ma = 1 1 2 + (1 p)1 + (1 p) = . 1 2 1 2

9.5. METHODS BASED ON ARRIVAL PROCESSES


..... ..... ..... ........... ........... ........... ..... ......... ..... ......... ..... ......... ..... ..... ..... ... ... ... .... .... ........... ..... .... ........... ..... .... .... .... .... ........... ... . ................ ... . .......... ... .. . .. .... ....... .. ... . .... . .. . .. .. .. .. .. .. .. .. .. .. .. .. . .. . . . . . . . . . . . . . . . . . . . . . . . . . . . . . . . . . . . . . . . . . . . . . . . . . .. .. .. . . . .. .. .. . . .. ... .. . . .. .. ... .. . .. .... . ..... . ................ ........ ....... . ... .. . ...... ....... . ......... ....... . ........ .... . ... ... .. .. ..... ... ... . .. .. ....... ... ... . .. .. ...... ... ... . . ..... . ... ... ..... ... ... ..... . . . ............... .. . . .............. ... . ............... ... .............. ... . ............. .. . ............. .. . . . . . . . . .. .. .. . . . .. .. .. . . . . . . . . . .. .. .. .. .. . .. . . . . . . . . .. .. .. . . . .. .. .. . . . . . . .. .. .. . . . .. .. .. . . . . . . .. .. .. . . . .. .. .. . . . . . . .. .. .. . . . .. .. .. . . . 1. 1. 1. . . . .. .. .. .. .. .. . . . . . . .. . . . 2... 2... 2... .. .. . . . . . . . . . . . . .. .. .. . . . . . . .. .. .. . . . .. .. .. . . . . . . . . . .. .. .. . . . .. .. .. .. .. .. . . . .. .. .. .. .. .. . . . .. .. .. .. .. . .. . . .... .. ..... .. ..... .. ...... ...... ...... .... .. ............. .... ....... .... ....... .... ....... .. .. .. ... ... ... .. .. .. .. .. . . . . . . . . . . .. . . . . . . . . . . . . . .. . .... ............... . ................... . .. .................. . . . . . . . . . . . . . . ... ................ ..... .............. ... ............... . . . . .. . .. . .. . . . . . .. . . .. .. . .. . .. .. .. . . . . .. ... ... ... ... .. .... .. .. .. .. .. ........... .......... .......... .......... .......... ........

183
... .......... .................. ... ..... .... ... .... ... ........ .... ... ... ... ... . ... . .. .. .. .. .. . . . . . . . . . . . . . . . . .. .. .. .. ... ... . ............. .... ................ .. . . .... ..... ... .. . ... . ................. ... . ... .............. ... . ... .. . ... . . . .. . . .. . . . . .. . . .. . . . . .. . . .. . . . . . .. . . .. . . . . . .. . . .. . . . . .. . . .. . . . 1. .. . .. . . . . . . .. 2... . . . . . . . . . .. . . . . .. . .. . .. . . .. .. .. .. . .. .. . .. .. . . .. . . .. ........... .. ............ . .. . .. ... .. .. . . .. . . . . . . ................... ................... . . .. . . . . . . .. . .. . . ... ... .. .. .......... ..........

p1

p1

p1

p1

0a

1a

2a

na

0b

1b

2b

nb

Figure 9.7: State transition diagram for a full accessible loss system with n servers, Cox2 arrival processes (cf. Fig. 4.10) and exponentially distributed service times (). The oered trac then becomes A = (ma )1. The carried trac is given by (9.22) applied to Fig. 9.7 and thus we can nd the trac congestion C. If we generalise the arrival process to a Coxk arrival process, then the state-transition diagram is still two-dimensional. By the application of Coxdistributions we can in principle take any number of parameters into consideration. If we generalise the service time to a Coxk distribution, then the state transition diagram becomes much more complex for n > 1, because we have a service process for each server, but only one arrival process. Therefore, in general we always generalise the arrival process and assume exponentially distributed service times.
2006-03-22

184

CHAPTER 9. OVERFLOW THEORY

Chapter 10 Multi-Dimensional Loss Systems


In this chapter we generalise the classical teletrac theory to deal with service-integrated systems (ISDN and B-ISDN). Every class of service corresponds to a trac stream. Several trac streams are oered to the same trunk group. In Sec. 10.1 we consider the classical multi-dimensional Erlang-B loss formula. This is an example of a reversible Markov process which is considered in more details in Sec. 10.2. In Sec. 10.3 we look at more general loss models and strategies, including service-protection (maximum allocation) and multi-slot BPPtrac. The models all have the so-called productform property, and the numerical evaluation is very simple by using the convolution algorithm for loss systems, implemented in the tool ATMOS (Sec. 10.4). In Sec. 10.5 we review other algorithms for the same problem. The models considered are all based on exible channel/slot allocation. They can be generalised to arbitrary circuit switched networks with direct routing, where we calculate the end-to-end blocking probabilities (Chap. 11). All models considered are insensitive to the service time distribution, and thus they are robust for applications. At the end of the chapter we consider other algorithms.

10.1

Multi-dimensional Erlang-B formula

We consider a group of n trunks (channels, slots), which is oered two independent PCTI trac streams: (1 , 1 ) and (2 , 2 ). The oered trac becomes A1 = 1 /1 , respectively A2 = 2 /2 . Let (i, j) denote the state of the system, i.e. i is the number of calls from stream 1 and j is 185

186

CHAPTER 10. MULTI-DIMENSIONAL LOSS SYSTEMS

the number of calls from stream 2. We have the following restrictions: 0 i n, 0 j n, 0 i+ j n. The state transition diagram is shown in Fig. 10.1. Under the assumption of statistical equilibrium the state probabilities are obtained by solving the global balance equations for each node (node equations), in total (n + 1)(n + 2)/2 equations. As we shall see in the next section, this diagram corresponds to a reversible Markov process, which has local balance, and furthermore the solution has product form. We can easily show that the global balance equations are satised by the following state probabilities which may be written on product form: p(i, j) = p(i) p(j) Ai Aj = Q 1 2 , i! j! (10.2) (10.1)

where p(i) and p(j) are one-dimensional truncated Poisson distributions, Q is a normalisation constant, and (i, j) full the above restrictions (10.1). As we have Poisson arrival processes, which have the PASTA-property (Poisson Arrivals See Time Averages), the time congestion, call congestion, and trac congestion are all equal for both trac streams, and they equals P (i + j = n). By the Binomial expansion or by convolving two Poisson distributions we nd the following aggregated state probabilities, where Q is obtained by normalisation: (A1 + A2 )x p(i + j = x) = Q , x!
n

(10.3)

=
=0

(A1 + A2 ) . !

(10.4)

This is the Truncated Poisson distribution (7.9) with the oered trac: A = A1 + A2 . (10.5)

We may also interpret this model as an Erlang loss system with one Poisson arrival process and hyper-exponentially distributed holding times in the following way. The total arrival process is a superposition of two Poisson processes with the total arrival rate: = 1 + 2 , (10.6)

10.1. MULTI-DIMENSIONAL ERLANG-B FORMULA

187

.................. ................... ... ... .. .. .. .. . . . . . . . . . . . . . . . . . .. . .. ... .. ... .. ................... .................. . .. . .. . . . ... . . ... . ... . . . . . . . . . . . . . . . . . . . . . . . . . . . . . . . . . . . . . . . . . . . . . . . . . . . . . . . . . . . ... . . ... . ... . ... . . . . .. .......... . ... ............... ................. ...... ...... ...... ....... ... .. . .... ... .. ... .. .. .. .................................. .. .. . .. . . ...... . ............................... . . . .... . . . . . . . . . . . . . . . . .. . . . . ............................... . . . .. . ... .. .. . .... . .. ....... ........................... .. .. . ... ... ... .. .. .... ... ... ................. ................. ................. ............... . .. . . . . . .. . . . . . .. .. . ... . ... . . ... . . ... . . . .. . ... ... . . . . . . . . . . . . . . . . . . . . . . . . . . . . . .. . ... . ... . ... ... . . . .. ... . . .. . .. . . . .. . . .. . . .. .. . . . . . . .

0, n

n2

0, n1

1, n1

.. .. .

.. .. .

.. .. .

(n1) 2

(n1) 2 ..... ...

.. .. .

.. .

.. .. . .

.. .. .

.. .. .

.. .. . .

.. .. . .. . . . . . .. . . . .. . .. . . . .. .. . . . . . .. .. . . . . . ... ... . . . .. ... . ... .. . . ... . ... . . . . . . . . . . . . . . . . . . . . . . . . . . . .. .. . . . . . . . . . . . . . . . . . . . . ... . ... . ... .. . . . .. . . . ... ... .. . .. ... .. . . . ... ... .. .. . . . . . .. .. . . .. . . . . .. . . .. .......... ... .......... .......... . .. .. . ... .. .... ...... ... ...... ...... . ...... ...... ....... ...... ....... ..... ...... ...... ....... ... . . . . .. .. ... .. ... ... .. .. .. .. .. .. ................. ... . .. .. . . . . ................................ . ...... ... . ... . .. . ...... .. . ................................ ................................ . ................................ . ................ . . . . . . . . . . . . . . . . . .. . . . .. . . . . . . . . . .. . .. . . .... . . . . ................................ ............................... ............... . ........................ . .......... . . . . .. .. .. . ... .. .. . ... .. ....... . .... ...... .. . .. ........ ....... ........................... ....... .. .. . . .. . ... . ... ... .... .... .. .. .. ..... ... ... .. ................. ................ ................ .. ............... .............. .. ............. . . . . . . . .. . . . . .. . . . .. . . . .. ... ... .. . . . ... ... ... . . . .. .. . . . . . . ... . .. ... . . . . ... ... ... . . . . . . . . . . . . . . . . . . . . .. .. . . . . . . . . . . . . . . . . . . . . . . . . . . . . . . . .. .. . . . . . . . . . . . . . . . . . . . . . . . . . . . . . . . . .. . . . . . . .. . . . . . . . . . . . . . . . . . . . . . . . . . . . . . . . .. . . . .. . . . . . . . . . . . . . . . . . . . . . . . . . . . . . . . . . . . . . . . . . . . . . . . . . . . . .. .. .. . . . . ... . ... . ... ... ... ... . . . . ... .. ... . . . .. .. . . .. . . .. . .. .. . . . .. .. .. . . . . ............... ... ............ ... ............ ... ............ .................. .................. .................. .................... .... .... .... .... .. .. .. .. ... ... ... .. .. .. .. .. .. ................................. .................................. ................. .. .. .. . . ................................ ................................. .............. .. .. ... . ... . .. .... ................. ................ ... . ... . . . . . . . ... . ... . ... ... . ..... .... . .... ..... . . . . . . . . . . . . . . . . . . . . . . . . . . . . . .. .. .. .. .. . . . . . . ............................... ............................... ................ . .. . ...... .. ...... .......... ................................. ................................. ................. . . .... . .... . .... .................. .. ... . . ... .. . .. .. .. . .. . .. .. .. .. . .. ... .. .. .. . .. . .. ... ... . . ... . .... .... .... ..... ................... ................... ................... .................. ... .... .... .... ............ ............ ............ ........... . . . . . . .. .. . . . . . . .. .. .. .. . . . . . . . .. .. .. .. . . . . ... . ... ... ... . . . ... ... ... ... . . . . ... ... ... ... . . . . . . . . . . . . .. . . . . . . . . . . . . . . . . . . . . . . . . . . . . . . . . . . . . . . . . . . . . . . . . . . . . . . . . . . . . . . . . . . . . . . . . . . . . . . . . . . . . . . . . . . . . . . . . . . . . . . . . . . . . . . . . . . . . . . . . . . . . . . . . . . . . . . . . . . . . . . . . . . . . . . . . . . . . . . . . . . . . . . . . . . . . . . . . . . . . . . . . . . . . . . . . . . . . . . . . . . . . . . . . . . . . . . . . . . . . .. . .. . . . .. .. .. .. .. . . . . .. .. . . . . ... ... ... ... . . . ... . .. .. ... .. . . . . . .. .. .. . . . . .. . . . . . .. .. .. . . . . . . . .. ......... . ......... . . . . . . ...... ...... ................. ........... ...... .................. ..... ..... .................. ................... .................. ..... ............. ..... ............. . .. .. .. .. ... .. ... ... .. .. .. .. ... ... ... .. .. .. .. .. .. .. .. .. .. .. . . . .. ................. ................................ ................................. . . ... . ... . ... . .. .. . ... . ................ ... ... .. . ................. ................. ... . . ... .. .. ................................ ............................ ... ................................ . .. . . . ............................ .. . .. . ... . . . . ... . . . .... . . . . . . . . . . . . . . . . . . . . . . . . . . . . . . . . . . . . . .. . ... . ... . .. . . . . . . ...... ..... ..... ...... . . . . . . . . . .. . .... . ... . .. . .. ................ ...... .......................... ...... ......................... ................................ . . .. . ................ ................. .. . . .... .... .. .................................. .................................. ................. .................................. .. .. .. ... .. .. .. . . . .. .. .. ... ... ... .. ... ... .. .. .. .. ... ... .. .. .. .. .. .... .... .... .................. .................. .................. ................. ................. ................. ................. ............... ............... ...............

.. .. .

2..................

0, 2

1, 2

2, 2

21

2 2

2 2

2 2

0, 1

1, 1

2, 1

n1, 1

21

(n1) 1

0, 0

1, 0

2, 0

n1, 0

n, 0

21

(n1) 1

n1

Figure 10.1: Two-dimensional state transition diagram for a loss system with n channels which are oered two PCTI trac streams. This is equivalent to a state transition diagram for the loss system M/H2 /n, where the hyper-exponential distribution H2 is given by (10.7).

188

CHAPTER 10. MULTI-DIMENSIONAL LOSS SYSTEMS

and the holding time distribution is hyper-exponentially distributed: f (t) = 1 2 1 e1 t + 2 e2 t . 1 + 2 1 + 2 (10.7)

We weight the two exponential distributions according to the relative number of calls per time unit. The mean service time is 1 1 2 1 A1 + A2 m1 = + = , 1 + 2 1 1 + 2 2 1 + 2 A , which is in agreement with the oered trac. m1 = (10.8)

Thus we have shown that Erlangs loss model is valid for hyper-exponentially distributed holding times. This is a special case of the general insensitivity property of Erlangs B formula. We may generalise the above model to N trac streams: AiN Ai1 Ai2 1 2 p(i1 , i2 , , iN ) = Q N , i1 ! i2 ! iN !
N

0 ij n ,
j=1

ij n ,

(10.9)

which is the general multi-dimensional Erlang-B formula. By a generalisation of (10.3) we notice that the global state probabilities can be calculated by the following recursion, where q(x) denotes the relative state probabilities, and p(x) denotes the absolute state probabilities: q(x) = 1 x
n N

Aj q(x 1) ,
j=1

q(0) = 1 ,

(10.10)

Q(n) =
i=0

q(i) , q(x) , Q(n)

p(x) =

0 x n.

(10.11)

If we use the recursion with normalisation (Sec. 7.4), then we get the recursion formula for ErlangB. Formula (10.10) is similar to the balance equations for the Poisson case when:
N

A=
j=1

Aj .

The time congestion is E = p(n), and as the PASTA-property is valid, this is also equal to the call congestion and the trac congestion. The numerical evaluation is dealt with in detail in Sec. 10.4. Multi-dimensional systems were rst mentioned by Erlang and more thoroughly dealt with by Jensen in the Erlangbook (Jensen, 1948 [49]).

10.2. REVERSIBLE MARKOV PROCESSES

189

10.2

Reversible Markov processes

In the previous section we considered a two-dimensional state transition diagram. For an increasing number of trac streams the number of states (and thus equations) increases very rapidly. However, we may simplify the problem by exploiting the structure of the state transition diagram. Let us consider the two-dimensional state transition diagram shown in Fig. 10.2. The process is reversible if there is no circulation ow in the diagram. Thus, if we consider four neighbouring states, then the ow in clockwise direction must equal the ow in the opposite direction (Kingman, 1969 [64]), (Sutton, 1980 [95]). From Fig. 10.2 we have: Clockwise: [i, j] [i, j + 1] [i + 1, j + 1] [i + 1, j] Counter clockwise: [i, j] [i + 1, j] [i + 1, j + 1] [i, j + 1] [i + 1, j] : [i + 1, j + 1] : [i, j + 1] : [i, j] : p(i, j) 1 (i, j) p(i+1, j) 2 (i+1, j) p(i+1, j +1) 1 (i+1, j +1) p(i, j +1) 2 (i, j +1) . [i, j + 1] : [i + 1, j + 1] : [i + 1, j] : [i, j] : p(i, j) 2 (i, j) p(i, j +1) 1 (i, j +1) p(i+1, j +1) 2 (i+1, j +1) p(i+1, j) 1 (i+1, j) ,

We can reduce both expressions by the state probabilities and then obtain the condition given by (10.12). It can be shown that a necessary and sucient condition for reversibility is that the following two expressions are equal: Clockwise: 2 (i, j) 1 (i, j +1) 2 (i+1, j +1) 1 (i+1, j) Counter clockwise: 1 (i, j) 2 (i+1, j) 1 (i+1, j +1) 2 (i, j +1) . If these two expressions are equal, then there is local or detailed balance. A necessary condition for reversibility is thus that if there is a ow (an arrow) from state i to state j, then there must also be a ow (an arrow) from j to i. We may locally apply cut equations between any two connected states. Thus from Fig. 10.2 we get: p(i, j) 1 (i, j) = p(i + 1, j) 1 (i + 1, j) . (10.13) (10.12)

We can express any state probability p(i, j) by state probability p(0, 0) by choosing any path between the two states (Kolmogorovs criteria). We may for instance choose the path: (0, 0), (1, 0), . . . , (i, 0), (i, 1), . . . , (i, j) ,

190
. . . . .. .. .. . . . . . . . . . . . . . . . . .

CHAPTER 10. MULTI-DIMENSIONAL LOSS SYSTEMS . . .


. . . . . . . . . . . . . . . . . .. . . .. .. . . .

. . .

 1 (i, j +1)    .. ................................................................................ ............................................................................... . ................... .................. . . .. . ................................................ i, j +1 .... i+1, j +1 ... .. . ............. . . . .............. ............. ............. . ............................................ .. ................................................................................................................................ ... ........... . . . . . .. .. . . . .. .. . 1 (i+1, j +1) . ... .. . . . . . . . . . . . . . . . . . . .
. . . . . . . . . . . . . . . . . . . . . . . . . . . . . . . . . . . . . . . . . . . . . . . . . . . . . . . . . . . . . . . . . . . . . . . . . . . . . . . . . . . . . . . . . . . . . . . . . . . . . . . . . . . . . . . . . . . . . . . . . . . . . . . . . . . . . . . . . . . . . . . . . . . . . . ... .. .. . . . . . . . . . . . . . . . . . . . . . . . . . . . . . . . . . . . . . . . . . . . . . . . . . . . . . . . . . . . . . . . . . . . . . . . . . . . . . . . . . . . . . . . . . . . . . . . . . . . . . . . . . . . . . . . . . . . . . . . . . . . . . . . . . . . . . . . . . . . . . . . . . . . . . . . . . ... .. . . . .

. . . . . .. . .. . . . . . . . . . . . . . . . . .

. . .

. . . . . . . . . . . . . . . . . .. . .. . .. . .

. . .

2 (i, j)

2 (i, j +1)

2 (i+1, j)

2 (i+1, j +1)

................................................ i, j .. . ............................................ ... . .


. . .. . . .. . . . . . . . . . . . . . . . . .

.. . ................................................................................ ............................................................................... .

1 (i, j)

  . . . . ..
.. . ... . . . . . . . . . . . . . . . . .

............................................................................... .............................................................................. . .. .

. . .

. . . . . . . . . . . . . . . . .. . .. .. . . .

i+1, j
. . . . . . . . . . . . . . . . . . .. .. . .. . . .

.. ................... ................... . ................... ................... .. ..

1 (i+1, j)

. . .

. . .

. . .

Figure 10.2: Kolmogorovs criteria: a necessary and sucient condition for reversibility of a two-dimensional Markov process is that the circulation ow among four neighbouring states in a square equals zero: Flow clockwise = ow counter-clockwise (10.12). and we then obtain the following balance equation: p(i, j) = 1 (0, 0) 1 (1, 0) 1 (i1, 0) 2 (i, 0) 2 (i, 1) 2 (i, j 1) p(0, 0) 1 (1, 0) 1 (2, 0) 1 (i, 0) 2 (i, 1) 2 (i, 2) 2 (i, j)

We nd p(0, 0) by normalisation of the total probability mass. The condition for reversibility will be fullled in many cases, for example for: 1 (i, j) = 1 (i) , 2 (i, j) = 2 (j) , 1 (i, j) = i 1 , 2 (i, j) = j 2 . (10.16) (10.17)

If we consider a multi-dimensional loss system with N trac streams, then any trac stream may be a state-dependent Poisson process, in particular BPP (Bernoulli, Poisson, Pascal) trac streams. For Ndimensional systems the conditions for reversibility are analogue to (10.12). Kolmogorovs criteria must still be fullled for all possible paths. In practice, we experience no problems, because the solution obtained under the assumption of reversibility will be the correct solution if and only if the node balance equations are fullled. In the following section we use this as the basis for introducing a very general multi-dimensional trac model.

10.3. MULTI-DIMENSIONAL LOSS SYSTEMS

191

10.3

Multi-Dimensional Loss Systems

In this section we consider generalisations of the classical teletrac theory to cover several trac streams oered to a single channel/trunk group. Each trac stream may have individual parameters and may be state-dependent Poisson arrival processes with multi-slot trac and class limitations. This general class of models is insensitive to the holding time distribution, which may be class dependent with individual parameters for each class. We introduce the generalisations one at a time and present a small case-study to illustrate the basic ideas.

10.3.1

Class limitation

In comparison with the case considered in Sec. 10.1 we now restrict the number of simultaneous calls for each trac stream (class). Thus, we do not have full accessibility, but unlike overow systems where we physically only have access to specic channels, then we now have access to all channels, but at any instant we may only occupy a limited number. This may be used for the purpose of service protection (virtual circuit protection = class limitation = threshold priority policy). We thus introduce restrictions to the number of simultaneous calls in class j as follows: 0 ij nj n , j = 1, 2, . . . , N , (10.18) where
N

nj > n .
j=1

If the latter restriction is not fullled, then we get separate groups corresponding to N ordinary independent one-dimensional loss systems. Due to the restrictions the state transition diagram is truncated. This is shown for two trac streams in Fig. 10.3. We notice that the truncated state transition diagram still is reversible and that the value of p(i, j) relative to the value p(0, 0) is unchanged by the truncation. Only the normalisation constant is modied. In fact, due to the local balance property we can remove any state without changing the above properties. We may consider more general class limitations to sets of trac streams so that any trac stream has a minimum (guaranteed) number of allocated channels.

10.3.2

Generalised trac processes

We are not restricted to consider PCTI trac only as in Sec. 10.1. Every trac stream may be a state-dependent Poisson arrival process with a linear state-dependent death (departure)

192 2................ . . . . . . + n2+ + j+ + 0 + 0 +

CHAPTER 10. MULTI-DIMENSIONAL LOSS SYSTEMS

. .

.. .. . .

.. .. .

.. .. .

.. .. . .

blocking for stream 1 blocking for stream 2


. . .. .. . .. .. .

.. .. . ....

(i, j) + i

.. .. .

.. .. ... .

.. .

.. .. .

.. .. .

.. .. .

.. . .. .

.. ..

.. .. .

+ n1

.. .. . .

i+j = n
.. .. . .. .. . .. .. . . .. .. .

.. .. .

.. .. . + n

.... .. ............ ............ ..

Figure 10.3: Structure of the state transition diagram for two-dimensional trac processes with class limitations (cf. 10.18). When calculating the equilibrium probabilities, state (i, j) can be expressed by state (i, j 1) and recursively by state (i, 0), (i 1, 0), and nally by (0, 0) (cf. (10.15)). rate (cf. (10.16) and (10.17)). The system still fulls the reversibility conditions given by (10.12). Thus, the product form still exists for BPP trac streams and more general statedependent Poisson processes. If all trac streams are Engset (Binomial) processes, then we get the multi-dimensional Engset formula (Jensen, 1948 [49]). As mentioned above, the system is insensitive to the holding time distributions. Every trac stream may have its own individual holding time distribution.

10.3.3

Multi-slot trac

In service-integrated systems the bandwidth requested may depend on the type of service. Thus a voice telephone call requires one channel (slot) only, whereas for example a video service may require d channels simultaneously. We get the additional restrictions: 0 dj ij nj n , and
N

j = 1, 2, . . . , N ,

(10.19)

0
j=1

dj ij n ,

(10.20)

where ij is the actual number of type j calls. The resulting state transition diagram will still be reversible and have product form. The restrictions correspond for example to the physical model shown in Fig. 10.5.

10.3. MULTI-DIMENSIONAL LOSS SYSTEMS


Stream 1: PCTI trac 1 = 2 calls/time unit 1 = 1 (time units1 ) Stream 2: PCTII trac S2 = 4 sources 2 = 1/3 calls/time unit/idle source 2 = 1 (time units1 ) 2 = 2 /2 = 1/3 erlang per idle source Z2 = 1/(1 + 2 ) = 3/4 (peakedness) d2 = 2 channels/call A2 = S2 2 /(1 + 2 ) = 1 erlang n2 = 6 = n

193

Z1 = 1 (peakedness) d1 = 1 channel/call A1 = 1 /1 = 2 erlang n1 = 6 = n

Table 10.1: Two trac streams: a Poisson trac process (Example 7.5.1) and a Binomial trac
process (Example 8.5.1) are oered to the same trunk group.

Oered trac Aj is usually dened as the average number of call attempts per mean holding time. If we measure the carried trac Yj as the average number of busy channels, then the lost trac measured in channels becomes:
N N

A =
j=1

Aj dj
j=1

Yj .

(10.21)

Example 10.3.1: Rnnbloms model o The rst example of a multi-slot trac model was published by Rnnblom (1958 [92]). The o paper considers external (outgoing and incoming) trac and internal trac in a PABX telephone exchange with both-way channels. The external trac occupies only one channel per call. The internal trac occupies both an outgoing channel and an incoming channel and thus requires two channels simultaneously. Rnnblom showed that this model has product form. o

Example 10.3.2: Two trac streams Let us illustrate the above models by a small case-study. We consider a trunk group of 6 channels which is oered two trac streams specied in Tab. 10.1. We notice that the second trac stream is a multi-slot trac stream. We may at most have three type-2 calls in our system. We only need to specify the oered trac, not the absolute values of arrival rate and service rate. The oered trac is as usually dened as the trac carried by an innite trunk group. We get the two-dimensional state transition diagram shown in Fig. 10.4. The total sum of all relative state probabilities equals 20.1704. So by normalisation we nd p(0, 0) = 0.0496 and thus the following state probabilities and marginal state probabilities p(i, ) and p(, j).

194

CHAPTER 10. MULTI-DIMENSIONAL LOSS SYSTEMS

PCT-II
. . . .. .. . .. . . . . . . . . . . . . . . . . . . . . . . . . . . . . .......... ........... ... ... .. .. .. . .. . . . . . . . . . . . . . . . . .. .. . .. . .. ... .. .. ........... .. .. . .. ....... . .. . .. . ... ... . . ... . . . . . . 2. . . . . . . . . . . . . . 3 . ..... . ... . . .. . . . . ........ .......... .. .... .... ......... .... .. ............. .... ...... .... ...... ... .... .... .. .. .. .. .. .. .. ........................ ... .. .. .. . ...................... . .................... . .................... ..... . ... . .. .. . . . . . ... . ... . . . . . . . . . . . . . . . . . . . . . . . .. . .. . . . . ..... ..... . . ... . . . .. .. . . ..................... . ..................... . .. . ...................... ....................... .... ... .. . .. .. ... ... .. . .. .. .. ... ... .... .... ............. ....... .... ............ .. ... .. .. .. .. .. .. . ........ .. . .. . .. .... . . . . .. .. . . .. .. . . . . . . ... . ... . ... ... ... ... . . . ... ... ... . . . . . . . . . . .. . . . . . . . . . . . . . . . . 3. . 3. . 3. . . . . . . . . . . . . . . . . . . . . . . . . . . . . 3 . ..... 3 . ..... 3 . ..... . . . . . . . . . ... ... ... . . . . . . . . . .. . . . . . .. . ......... ......... ......... . . .. . .. .. . . . .... .......... .... .... .. ........... .. .. .. ............. ............. ............. ............. . .... ...... . . ... .. ... .. .. .. .. ... .. ... .. .. .. .. .. .. .. .. ... .... ... .... ...................... ....................... . . . ..................... ................. ... ... . ... .. .. ..................... ... . ... .. . ..................... . ... . .. . .. . . ... . . . . .................... ................... . . . . ... . ... . . . . . . . . . . . . . . . . . . . . . . . . . . . . . . . . . . . .. . .. . . .. . .. . . . .. . . .. . .. . . .. . . . . . .................... .... ............... ..................... .... ................ . . . . . ..................... ...................... ...................... ...................... . . . ... . .... . .. . . ... .. . .. . .. . .. .. .. . . ... . ... . .. .. .. ... .. .. . . .. .. .. .. ... .. ... ... ... ... . ... ... .... ..... .... ..... ..... . .... ..... ...... ... ... ... ... ............ ....... .... ......... . .. . . . . .. .. .. .. .. .. .. . ... .. ......... . . . ....... . ....... . ....... . .. ... . . . . . . .. .. .. ... . ... . . . . . ... ... ... . . . ... ... ... ... ... . . . . . . . . . . ... ... ... ... ... . .. . . .. . . . . . . . . . . . . . . . . . . . . . . . . . . . 4. . 4. . 4. . 4. . 4. . . . . . . . . . . . . . . . . . . . . . . . . . . . . . . . . . . . . . . . . . . . . . . . . . . . . . . . . . . . . . 3 . ..... 3 . ..... 3 . ..... 3 . ..... 3 . ..... . . . . . . . . . .. .. .. .. . .. ... .. .. .. . . . . . . . . .. .. .. .. . . . . . . . . .. . . . . . . .. .. . . . . .. . . . . . . .. ......... ......... ........ .. .......... .......... ..... . . . . .. ........... ........... ........... ........... ........... ........... .. . ........... . . . ............ ... ... .. .... ..... ... .. .. ... ... ... ... ... .. .. .. .. ... ... ... ... ... .. ... .. .. .. ... .. .. ....................... ....................... .... ....... ....... ... .. . .... . .. . .. . . ... .. .. .. . ................. .. ....................... .................... . ...................... .. ...................... ...................... ... . ... . . . ..................... ..... .. ..... .................. . .................. . .................. .. . . . . . . . . . . . . .. . ... . . . ... . ... . ..... ..... ..... ... .. . . . . . . . . . . . . . . . . . . . . . . . . . . . . . . . . . . . . . .. . . . . . . . . . . . . . ..... ... . ..... ..... .... . ... . . . . . . . .................... .................... ..................... .................... ..................... .................... . ................. . ................ . . . .. .. . ... .. . .. .. . .. .. .. . .. .. . .. . . ... .. .. ...... ...... ...... ...... .. .... ...... . ....................... ...... ...................... ...... .. .................... ...... ................ .. .. .. .. . . . .. . . .. .. ... ... ... ... ... ... .. .. .. .. .. .... .. .. .... .... .... .. .. .. .. .. .... ..... .... ..... .... .... ........... ....... ... ....... ... .......... .......... .......... .......... ........ ..... .... .... ........ ........ ..... ....

0, 6

0, 4

2 1 2 1 2 1

1, 4

2, 4

0, 2

1, 2

2, 2

3, 2

4, 2

0, 0

1, 0

2, 0

3, 0

4, 0

5, 0

6, 0

............................ ....................... ... . .

PCTI

4 27

2 3

4 3

4 3

4 3

8 3

8 3

16 9

8 9

4 3

2 3

4 15

4 45

Figure 10.4: Example 10.3.2: Six channels are oered both a Poisson trac stream (PCTI)
(horizontal states) and an Engset trac stream (PCTII) (vertical states). The parameters are specied in Tab. 10.1. If we allocate state (0, 0) the relative value one, then we nd by exploiting local balance the relative state probabilities q(i, j) shown below.

10.3. MULTI-DIMENSIONAL LOSS SYSTEMS


p(i, j) j=6 j=4 j=2 j=0 p(i, ) i=0 0.0073 0.0331 0.0661 0.0496 0.1561 i=1 0.0661 0.1322 0.0992 0.2975 i=2 0.0661 0.1322 0.0992 0.2975 0.0881 0.0661 0.1542 0.0441 0.0331 0.0771 0.0132 0.0132 0.0044 0.0044 i=3 i=4 i=5 i=6 p( , j) 0.0073 0.1653 0.4627 0.3647 1.0000

195

The global state probabilities become:

p(0) = p(0, 0) p(1) = p(1, 0) p(2) = p(0, 2) + p(2, 0) p(3) = p(1, 2) + p(3, 0) p(4) = p(0, 4) + p(2, 2) + p(4, 0) p(5) = p(1, 4) + p(3, 2) + p(5, 0) p(6) = p(0, 6) + p(2, 4) + p(4, 2) + p(6, 0)

= = = = = = =

0.0496 0.0992 0.1653 0.1983 0.1983 0.1675 0.1219

Performance measures for trac stream 1: Due to the PASTA-property the time congestion (E1 ), the call congestion (B1 ), and the trac congestion (C1 ) are identical. We nd the time congestion E1 : E1 = p(6, 0) + p(4, 2) + p(2, 4) + p(0, 6) = p(6) , E1 = B1 = C1 = 0.1219 , Y1 = 1.7562 . Performance measures for stream 2: Time congestion E2 (proportion of time the system is blocked for stream 2): E2 = p(0, 6) + p(1, 4) + p(2, 4) + p(3, 2) + p(4, 2) + p(5, 0) + p(6, 0) = p(5) + p(6) , E2 = 0.2894 . Call congestion B2 (Proportion of call attempts blocked for stream 2):

196

CHAPTER 10. MULTI-DIMENSIONAL LOSS SYSTEMS

The total number of call attempts per time unit is obtained from the marginal distribution: xt = 4 3 2 1 0.3647 + 0.4627 + 0.1653 + 0.0073 3 3 3 3

= 1.0616 . The number of blocked call attempts per time unit becomes: x = 4 3 2 1 {p(5, 0) + p(6, 0)} + {p(3, 2) + p(4, 2)} + {p(1, 4) + p(2, 4)} + p(0, 6) 3 3 3 3

= 0.2462 . Hence: B2 = x = 0.2320 . xt

Trac congestion C2 (Proportion of oered trac blocked): The carried trac, measured in the unit [channel], is obtained from the marginal distribution:
6

Y2 =
j=0

j p(, j) ,

Y2 = 2 0.4627 + 4 0.1653 + 6 0.0073 , Y2 = 1.6306 erlang .

The oered trac, measured in the unit [channel ], is d2 A2 = 2 erlang (Tab. 10.1). Hence we get: C2 = 2 1.6306 = 0.1848 . 2

The above example has only 2 streams and 6 channels and the total number of states equals 16 (Fig. 10.4). When the number of trac streams and channels increase, then the number of states increases very fast and we are unable to evaluate the system by calculating the individual state probabilities. In the following section we introduce the convolution algorithm for loss systems which eliminates this problem by aggregation of states.

10.4

Convolution Algorithm for loss systems

We now consider a trunk group with a total of n homogeneous trunks. Being homogeneous means that they have the same service rate. The trunk group is oered N dierent types

10.4. CONVOLUTION ALGORITHM FOR LOSS SYSTEMS

197

of calls, also called streams, or classes. A call of type i requires di trunks (channels, slots) during the whole service time, i.e. all di channels are occupied and released simultaneously. The arrival processes are general state-dependent Poisson processes. For the ith arrival process the arrival intensity in state xi di , that is, when xi calls of type i are being served, is i (xi ). We may restrict the number xi of simultaneous calls of type i so that: 0 xi di ni n . It will be natural to require that ni is an integral multiple of di . This model describes for example the system shown in Fig. 10.5.

1 , Z 1 , d 1

.. . ..............

2 , Z 2 , d 2

. . .

. . .

N , Z N , d N

Li Local exchanges

... ... ... ... ... ... ... ... ... ... ... ... ... ... ... ... ... ... ... ... ... ... ... ... ... ... ... ... ... ... ... ... ... ..... ... ..... ... ..... ... ..... ... ..... ... ..... ... ..... ... ..... ... ..... ... ..... ... ..... ... ..... ... ..... ... ..... ... ..... ... ..... ... ..... ..... ... ... ..... ..... ... ... ..... ..... ... ... ..... ..... ... ... ..... ..... ... ... ..... ..... ... . ..... ..... ..... ..... ..... ..... ..... ... ..... ... ........ ........ .. ...... ...... . ........................................................................................... ........................................................................................... .. .. ..... ..... ..... ..... ..... . . ..... ..... . . ..... ..... ..... .... .. .. ..... ..... ..... ..... ..... . . ..... ..... .... ..... ..... ..... .... .... ..... ..... ... .... .... .... ..... ..... ..... ..... .... ....

n1 n2

nN

Transit exchange

Destination exchange

Figure 10.5: Generalisation of the classical teletrac model to BPPtrac and multi-slot trac. The parameters i and Zi describe the BPPtrac, whereas di denotes the number of slots required.

The system mentioned above can be evaluated in an ecient way by the convolution algorithm rst introduced in (Iversen, 1987 [40]). We rst describe the algorithm, and then explain it in further detail by an example. The convolution algorithm is closely related to the productform.

10.4.1

The algorithm

The algorithm is described by the following three steps: Step 1: Calculate the state probabilities of each trac stream as if it is alone in the system, i.e. we consider classical loss systems as described in Chaps. 7 & 8. For trac

198 stream i we nd:

CHAPTER 10. MULTI-DIMENSIONAL LOSS SYSTEMS

Pi = {pi (0), pi (1), . . . , pi (ni )} ,

i = 1, 2, . . . , N .

(10.22)

Only the relative values of pi (x) are of importance, so we may choose qi (0) = 1 and calculate the values of qi (x) relative to qi (0). If a term qi (x) becomes greater than K (e.g. 1010 ), then we may divide all values qi (j), 0 j x, by K. To avoid any numerical problems in the following it is advisable to normalise the relative state probabilities so that: qi (j) pi (j) = , Qi
ni

j = 0, 1 . . . , ni ,

Qi =
j=0

qi (j) .

As described in Sec. 7.4 we may normalise at each step to avoid any numerical problems. Step 2: By successive convolutions (convolution operator ) we calculate the aggregated state probabilities for the total system excepting trac stream number i: QN/i = P1 P2 Pi1 Pi+1 PN . (10.23)

We rst convolve P1 and P2 and obtain P12 which is convolved with P3 , etc. Both the commutative and the associative laws are valid for the convolution operator, dened in the usual way (Sec. 3.2):
1 u

Pi P j = where

pi (0) pj (0),
x=0

pi (x) pj (1 x), ,
x=0

pi (x) pj (u x) ,

(10.24)

u = min{ni + nj , n} .

(10.25)

Notice, that we truncate the state space at state n. Even if Pi and Pj are normalised, then the result of a convolution is in general not normalised due to the truncation. It is recommended to normalise after every convolution to avoid any numerical problems both during this step and the following. Step 3: Calculate the time congestion Ei , the call congestion Bi , and the trac congestion Ci of stream i. This is done during the convolution: QN = QN/i Pi . This convolution results in:
j j

QN (j) =
x=0

QN/i (j x) pi (x) =
x=0

pi (j) , x

(10.26)

where for pi (j), i denotes the trac stream, j the total number of busy channels, and x x the number of channels occupied by stream number i. Steps 2 3 are repeated for every trac stream. In the following we derive formul for Ei , Bi , and Ci .

10.4. CONVOLUTION ALGORITHM FOR LOSS SYSTEMS Time congestion Ei for trac stream i becomes: Ei =
jSE i

199

pi (j)/Q . x

(10.27)

where SE i = {(x, j) | x j n (x > ni di ) (j > n di )} , The summation is extended to all states SE i where calls belonging to class i are blocked: the set {x > ni di } corresponds to the states where trac stream i has utilised its quota, and (j > n di ) corresponds to states with less than di idle channels. Q is the normalisation constant:
n

Q=
j=0

QN (j) .

(At this stage we usually have normalised the state probabilities so that Q = 1). Call congestion Bi for trac stream i is the ratio between the number of blocked call attempts and the total number of call attempts, both for trac stream i, and for example per time unit. We nd: i SE i i (x) px (j) Bi = ni . (10.28) j i x=0 i (x) px (j) j=0 Trac congestion Ci : We dene as usual the oered trac as the trac carried by an innite trunk group. The carried trac for trac stream i is:
ni j

Yi =
j=0 x=0

x pi (j) . x

(10.29)

Thus we nd: Ci =

Ai Yi . Ai

The algorithm is implemented in the PC-tool ATMOS (ListovSaabye & Iversen, 1989 [74]). The storage requirement is proportional to n as we may calculate the state probabilities of a trac stream when it is needed. In practice we use a storage proportional with n N, because we save intermediate results of the convolutions for later re-use. It can be shown (Iversen & Stepanov, 1997 [42]) that we need (4 N 6) convolutions when we calculate trac characteristics for all N trac streams. Thus the calculation time is linear in N and quadratic in n.

Example 10.4.1: De-convolution In principle we may obtain QN/i from QN by a de-convolution and then during the re-convolution of Pi calculate the performance measures. In this way we need not repeat all the convolutions (10.23)

200

CHAPTER 10. MULTI-DIMENSIONAL LOSS SYSTEMS Stream 3: Pascal trac (Negative Binomial) S3 = 2 sources 3 = 1/3 calls/time unit 3 = 1 (time unit1 ) 3 = 3 /3 = 1/3 erlang per idle source Z3 = 1/(1 + 3 ) = 3/2 d3 = 1 channels/call A3 = S3 (1 Z3 ) = 1 erlang n3 = 4 (max. # of simultaneous calls)

Table 10.2: A Pascal trac stream (Example 8.7.2) is oered to the same trunk as the two trac streams of Tab. 10.1.
for each trac stream. However, when implementing this approach we get numerical problems. The convolution is from a numerical point of view very stable, and therefore the de-convolution will be unstable. Nevertheless, we may apply de-convolution in some cases, for instance when the trac sources are on/osources.

Example 10.4.2: Three trac streams We rst illustrate the algorithm with a small example, where we go through the calculations in every detail. We consider a system with 6 channels and 3 trac streams. In addition to the two streams in Example 10.3.2 we add a Pascal stream with class limitation as shown in Tab. 10.2 (cf. Example 8.7.2). We want to calculate the performance measures of trac stream 3. Step 1: We calculate the state probabilities pi (j) of each trac stream i (i = 1, 2, 3, j = 1, 2, . . . , ni ) as if it were alone. The results are given in Tab. 10.3. Step 2: We evaluate the convolution of p1 (j) with p2 (k), p1 p2 , truncate the state space at n = 6, and normalise the probabilities so that we obtain p12 shown in the Tab. 10.3. Notice that this is the result obtained in Example 10.3.2. Step 3: We convolve p12 (j) with p3 (k), truncate at n, and obtain q123 (j) as shown in Tab. 10.3. The time congestion E3 is obtained from the detailed state probabilities. Trac stream 3 (single slot trac) experiences time congestion, both when all six channels are busy and when the trac stream occupies 4 channels (maximum allocation). From the detailed state probabilities we get: E3 = = q123 (6) + p3 (4) {p12 (0) + p12 (1)} 0.8678 0.1535 + 0.0279 {0.0496 + 0.0992} , 0.8678

E3 = 0.1817 .

10.4. CONVOLUTION ALGORITHM FOR LOSS SYSTEMS State j 0 1 2 3 4 5 6 Total Probabilities p1 (j) 0.1360 0.2719 0.2719 0.1813 0.0906 0.0363 0.0121 p2 (j) 0.3176 0.0000 0.4235 0.0000 0.2118 0.0000 0.0471 q12 (j) Normal. p1 p2 0.0432 0.0864 0.1440 0.1727 0.1727 0.1459 0.1062 0.8711 p12 (j) 0.0496 0.0992 0.1653 0.1983 0.1983 0.1675 0.1219 1.0000 Prob. p3 (j) 0.4525 0.3017 0.1508 0.0670 0.0279 0.0000 0.0000 1.0000 q123 (j) Normal. p12 p3 0.0224 0.0599 0.1122 0.1579 0.1825 0.1794 0.1535 0.8678 p123 (j) 0.0259 0.0689 0.1293 0.1819 0.2104 0.2067 0.1769 1.0000

201

1.0000 1.0000

Table 10.3: Convolution algorithm applied to Example 10.4.2. The state probabilities for the
individual trac streams have been calculated in the examples 7.5.1, 8.5.1 and 8.7.2. Notice that the state {p3 (4) p12 (2)} is included in state q123 (6). The carried trac for trac stream 3 is obtained during the convolution of p3 (i) and p12 (j) and becomes: 4 6i 1 Y3 = i p3 (i) p12 (j) , 0.8678
i=1 j=0

Y3 =

0.6174 = 0.7115 . 0.8678

As the oered trac is A3 = 1, we get: Trac congestion: C3 = 1 0.7115 , 1

C3 = 0.2885 . The call congestion becomes: x , xt where x is the number of lost calls per time unit, and xt is the total number of call attempts per time unit. Using the normalised probabilities from Tab. 10.3 we get {3 (i) = (S3 i) 3 }: B3 = x = 3 (0) {p3 (0) p12 (6)} + 3 (1) {p3 (1) p12 (5)} + 3 (2) {p3 (2) p12 (4)} + 3 (3) {p3 (3) p12 (3)} + 3 (4) p3 (4) {p12 (2) + p12 (1) + p12 (0)} ,

202
x

CHAPTER 10. MULTI-DIMENSIONAL LOSS SYSTEMS


= 0.2503 .
6

xt = 3 (0) p3 (0)
j=0

p12 (j)
5

+ 3 (1) p3 (1)
j=0 4

p12 (j)

+ 3 (2) p3 (2)
j=0 3

p12 (j)

+ 3 (3) p3 (3)
j=0 2

p12 (j)

+ 3 (4) p3 (4)
j=0

p12 (j) ,

xt = 1.1763 . We thus get: B3 = x = 0.2128 . xt

In a similar way by interchanging the order of convolving trac streams we nd the performance measures of stream 1 and 2. The total number of micro-states in this example is 47. By the convolution method we reduce the number of states so that we never need more than two vectors of each n+1 states, i.e. 14 states. By using the ATMOStool we get the following results shown in Tab. 10.4 and Tab. 10.5. The total congestion can be split up into congestion due to class limitation (ni ), and congestion due to the limited number of channels (n).

Input Oered trac i 1 2 3 A 2.0000 1.0000 1.0000 Peaked ness Z 1.00 0.75 1.50

Total number of channels n = 6 Maximum allocation n 6 6 4 Slot size d 1 2 1 Mean holdding time 1 1.00 1.00 1.00 Sources S 4 -2 beta 0 0.3333 -0.3333

Table 10.4: Input data to ATMOS for Example 10.4.2 with three trac streams.

10.5. STATE SPACE BASED ALGORITHMS


Output i 1 2 3 Total Call congestion B 1.769 200E-01 3.346 853E-01 2.127 890E-01 Trac congestion C 1.769 200E-01 2.739 344E-01 2.884 898E-01 2.380 397E-01 Time congestion E 1.769 200E-01 3.836 316E-01 1.817 079E-01 Carried trac Y 1.646 160 1.452 131 0.711 510 3.809 801

203

Table 10.5: Output data from ATMOS for the input data in Tab. 10.4.

Example 10.4.3: Large-scale example To illustrate the tool ATMOS we consider in Tab. 10.6 and Tab. 10.7 an example with 1536 trunks and 24 trac streams. We notice that the time congestion is independent of peakedness Zi and proportional to the slot-size di , because we often have: p(j) p(j 1) . . . p(j di ) for di j. (10.30)

This is obvious as the time congestion only depends on the global state probabilities. The call congestion is almost equal to the time congestion. It depends weakly upon the slot-size. This is also to be expected, as the call congestion is equal to the time congestion with one source removed (arrival theorem). In the table with output data we have in the rightmost column shown the relative trac congestion divided by (di Zi ), using the single-slot Poisson trac as reference value (di = Zi = 1). We notice that the trac congestion is proportional to di Zi , which is the usual assumption when using the Equivalent Random Trac (ERT) method (Chap. 9). The mean value of the oered trac increases linearly with the slot-size, whereas the variance increases with the square of the slot-size. The peakedness (variance/mean) ratio for multi-slot trac thus increases linearly with the slot-size. We thus notice that the trac congestion is much more relevant than the time congestion and call congestion for characterising the performance of the system. If we calculate the total trac congestion using Fredericks & Haywards method (Sec. 9.2), then we get a total trac congestion equal to 6.114 % (cf. Example 9.3.2 and Tab. 10.7). The exact value is 5.950 %.

10.5

State space based algorithms

The convolution algorithm is based on aggregation of trac streams, where we end up with a trac stream which is the aggregation of all trac streams except the one which we are interested in. Another approach is to aggregate the state space into global state probabilities.

204

CHAPTER 10. MULTI-DIMENSIONAL LOSS SYSTEMS

Input Oered traf. i 1 2 3 4 5 6 7 8 9 10 11 12 13 14 15 16 17 18 19 20 21 22 23 24 Ai 64.000 64.000 64.000 64.000 64.000 64.000 32.000 32.000 32.000 32.000 32.000 32.000 16.000 16.000 16.000 16.000 16.000 16.000 8.000 8.000 8.000 8.000 8.000 8.000 Peakedness Zi 0.200 0.500 1.000 2.000 4.000 8.000 0.200 0.500 1.000 2.000 4.000 8.000 0.200 0.500 1.000 2.000 4.000 8.000 0.200 0.500 1.000 2.000 4.000 8.000

Total # of channels n = 1536 Max. sim. # ni 1536 1536 1536 1536 1536 1536 1536 1536 1536 1536 1536 1536 1536 1536 1536 1536 1536 1536 1536 1536 1536 1536 1536 1536 Channels/call di 1 1 1 1 1 1 2 2 2 2 2 2 4 4 4 4 4 4 8 8 8 8 8 8 mht i 1.000 1.000 1.000 1.000 1.000 1.000 1.000 1.000 1.000 1.000 1.000 1.000 1.000 1.000 1.000 1.000 1.000 1.000 1.000 1.000 1.000 1.000 1.000 1.000 Sources S 80.000 4.000 128.000 1.000 0.000 -64.000 -0.500 -21.333 -0.750 -9.143 -0.875 40.000 4.000 64.000 1.000 0.000 -32.000 -0.500 -10.667 -0.750 -4.571 -0.875 20.000 4.000 32.000 1.000 0.000 -16.000 -0.500 -5.333 -0.750 -2.286 -0.875 10.000 4.000 16.000 1.000 0.000 -8.000 -0.500 -2.667 -0.750 -1.143 -0.875

Table 10.6: Input data for Example 10.4.3 with 24 trac streams and 1536 channels. The maximum
number of simultaneous calls of type i (ni ) is in this example n = 1536 (full accessibility), and mht is an abbreviation for mean holding time.

10.5. STATE SPACE BASED ALGORITHMS

205

Output i 1 2 3 4 5 6 7 8 9 10 11 12 13 14 15 16 17 18 19 20 21 22 23 24 Total

Call congestion Bi 6.187 744E-03 6.202 616E-03 6.227 392E-03 6.276 886E-03 6.375 517E-03 6.570 378E-03 1.230 795E-02 1.236 708E-02 1.246 554E-02 1.266 184E-02 1.305 003E-02 1.379 446E-02 2.434 998E-02 2.458 374E-02 2.497 245E-02 2.574 255E-02 2.722 449E-02 2.980 277E-02 4.766 901E-02 4.858 283E-02 5.009 699E-02 5.303 142E-02 5.818 489E-02 6.525 455E-02

Trac congestion Ci 1.243 705E-03 3.110 956E-03 6.227 392E-03 1.247 546E-02 2.502 346E-02 5.025 181E-02 2.486 068E-03 6.222 014E-03 1.246 554E-02 2.500 705E-02 5.023 347E-02 1.006 379E-01 4.966 747E-03 1.244 484E-02 2.497 245E-02 5.019 301E-02 1.006 755E-01 1.972 682E-01 9.911 790E-03 2.489 618E-02 5.009 699E-02 1.007 214E-01 1.981 513E-01 3.583 491E-01 5.950 135E-02

Time congestion Ei 6.227 392E-03 6.227 392E-03 6.227 392E-03 6.227 392E-03 6.227 392E-03 6.227 392E-03 1.246 554E-02 1.246 554E-02 1.246 554E-02 1.246 554E-02 1.246 554E-02 1.246 554E-02 2.497245E-02 2.497 245E-02 2.497 245E-02 2.497 245E-02 2.497 245E-02 2.497 245E-02 5.009 699E-02 5.009 699E-02 5.009 699E-02 5.009 699E-02 5.009 699E-02 5.009 699E-02

Carried trac Yi 63.920 403 63.800 899 63.601 447 63.201 570 62.398 499 60.783 884 63.840 892 63.601 791 63.202 205 62.399 549 60.785 058 57.559 172 63.682128 63.203 530 62.401 763 60.787 647 57.556 771 51.374 835 63.365 645 62.406 645 60.793 792 57.553 828 51.318 316 41.065 660 1444.605

Rel. value Ci /(diZi ) 0.9986 0.9991 1.0000 1.0017 1.0046 1.0087 0.9980 0.9991 1.0009 1.0039 1.0083 1.0100 0.9970 0.9992 1.0025 1.0075 1.0104 0.9899 0.9948 0.9995 1.0056 1.0109 0.9942 0.8991

Table 10.7: Output for Example 10.4.3 with input data given in Tab. 10.6. As mentioned earlier
in Example 9.3.2, Fredericks-Haywards method results in a total congestion equal to 6.114 %. The total trac congestion 5.950 % is obtained from the total carried trac and the oered trac.

206

CHAPTER 10. MULTI-DIMENSIONAL LOSS SYSTEMS

10.5.1

Fortet & Grandjean (Kaufman & Robert) algorithm

In case of Poisson arrival processes the algorithm becomes very simple by generalizing (10.10). Let pi (x) denote the contribution of stream i to the global state probability p(x):
N

p(x) =
i=1

pi (x) .

(10.31)

Thus the average number of channels occupied by stream i when the system is in global state x is x pi (x). Let trac stream i have the slot-size di . Due to reversibility we will have local balance for every trac type. The local balance equation becomes: x pi (x) i = i p(x di ) , di x = di , di + 1, . . . n . (10.32)

The left-hand side is the ow from state [ x ] to state [ x di ] due to departures of type i calls. The right-hand side is the ow from global state [ x di ] to state [ x ] due to arrivals of type i. It does not matter whether x is a integer multiple of di , as we only consider average values. From (10.32) we get: pi (x) = 1 di Ai p(x di ) . x (10.33)

The total state probability p(x) is obtained by summing over all trac streams (10.31): p(x) = 1 x
N

di Ai p(x di ) ,
i=1

p(x) = 0 for x < 0 .

(10.34)

This is Fortet & Grandjeans algorithm (Fortet & Grandjean, 1964 [28]) The algorithm is usually called Kaufman & Roberts algorithm, as it was re-discovered by these authors in 1981 (Kaufman, 1981 [58]) (Roberts, 1981 [88]).

10.5.2

Generalized algorithm

The above model can easily be generalized to BPP-trac (Iversen, 2005 [44]) x pi (x) x di i = p(x di) Si i pi (x di ) i . di di (10.35)

On the right-hand side the rst term assumes that all type i sources are idle during one time unit. As we know xdi pi (x di ) di

10.5. STATE SPACE BASED ALGORITHMS

207

type i sources on the average are busy in global state xdi we reduce the rst term with the second term to get the right value. Thus we get: 0 x<0 p(0) x=0 p(x) = (10.36) N pi (x) x = 1, 2, . . . , n
i=1

where

pi (x) =

d i Si i x di i p(x di ) pi (x di) x i x i x < di .

(10.37) (10.38)

pi (x) = 0

The state probability p(0) is obtained by the normalization condition:


n n N

p(j) =
j=0 j=0 i=1

pi (j) = 1 .

(10.39)

Above we have used the parameters (Si , i ) to characterize the trac streams. Alternatively we may also use (Ai , Zi) related to (Si , i) by the formul (8.20) (8.23). Then (10.37) becomes: di Ai x di 1 Zi pi (x) = p(x di ) pi (x di ) (10.40) x Zi x Zi For Poisson arrivals we of course get (10.34). In practical evaluation of the formula we will use normalization in each step as described in Sec. 7.4.1. This results in a very accurate and eective algorithm. In this way also the number of operations and the memory requirements become very small, as we only need to store the di previous state probabilities of trac stream i, and the max{di } previous values of the global state probabilities. The number of operations is linear in number of channels.

Performance measures By this algorithm we are able to obtain performance measures for each individual trac stream. Time congestion: Call attempts of stream i require di idle channel and will be blocked with probability:
n

Ei =
x=ndi +1

p(i) .

(10.41)

208

CHAPTER 10. MULTI-DIMENSIONAL LOSS SYSTEMS

Trac congestion: From the state probabilities pi (x) we get the total carried trac of stream i:
n

Yi =
i=1

x pi (x) .

(10.42)

Thus the trac congestion of stream i becomes: Ci = The total carried trac is
N

Ai di Yi . Ai di

(10.43)

Y =
j=1

Yi ,

(10.44)

so the total trac congestion becomes: C= AY , A (10.45)

where A is the total oered trac measured in channels:


N

A=
j=1

di Ai .

Call congestion: This is obtained from the trac congestion by using (8.47): Bi = (1 + i ) Ci . 1 + i Ci (10.46)

The total call congestion cannot be obtained by this formula as we do not have a global value of . But from individual carried trac and call congestion we may nd the total number of oered calls and accepted calls for each stream and from this the total call congestion.

Example 10.5.1: Generalized algorithm We evaluate Example 10.3.2 by the general algorithm. Table 10.8 shows the non-normalized state probabilities when we let state zero equal to one. Table 10.9 shows the normalized state probabilities and the carried trac of each stream in each state. In a computer program we would normalize state probabilities after each iteration (increasing number of lines by one) and calculate the aggregated total trac for each stream. This trac value should of course also be normalized in each step. In this way we only need to store the previous di values and the carried trac of each trac stream. We get the following performance measures, which of course are the same as obtained by convolution

10.5. STATE SPACE BASED ALGORITHMS

209

State x 0 1 2 3 4 5 6 Total
2 6

Poisson q1 (x) 0
2 1 2 2 2 3

Engset q2 (x) 0 0
2 4 10 = 4 2 3 3 2 4 16 20= 9 3 3 2 4 10 3 214 =2 4 3 4 3 3 2 4 3 1 16 4 5 3 9 = 16 5 3 9 2 4 4 1 180 4 6 3 2 = 135 6 3

Total q(x) 1 2
10 3

1=2 2=2
10 3

2 4 2 5

20 9

4 4
152 45 332 135 2723 135

4=2 4=
152 45

8 5 152 135

Table 10.8: Example 10.5.1: relative state probabilities for Example 10.3.2 evaluated by the generalized algorithm.

State x 0 1 2 3 4 5 6 Total

Poisson p1 (x) 0.0000 0.0992 0.0992 0.1102 0.0992 0.0793 0.0558 x p1 (x) 0.0000 0.0992 0.1983 0.3305 0.3966 0.3966 0.3349 1.7562

Engset p2 (x) 0.0000 0.0000 0.0661 0.0881 0.0992 0.0881 0.0661 x p2 (x) 0.0000 0.0000 0.1322 0.2644 0.3966 0.4407 0.3966 1.6306 p(x) 0.0496 0.0992 0.1653 0.1983 0.1983 0.1675 0.1219 1.0000

Total y = x p(x) 0.0000 0.0992 0.3305 0.5949 0.7932 0.8373 0.7315 3.3867

Table 10.9: Example 10.5.1: absolute state probabilities and carried trac yi(x) = x pi (x) for Example 10.3.2 evaluated by the generalized algorithm.

210
algorithm.

CHAPTER 10. MULTI-DIMENSIONAL LOSS SYSTEMS

E1 = p(6) E2 = p(5) + p(6) C1 C2 = =


211.7562 21 121.6306 12 (1+0)0.1219 1+00.1219 (1+1/3)0.1847 1+(1/3)0.1847

= 0.1219 = 0.2894 = 0.1219 = 0.1847 = 0.1219 = 0.2320

B1 = B1 =

10.6

Final remarks

The convolution algorithm for loss systems was rst published in (Iversen, 1987 [40]). A similar approach to a less general model was published in two papers by Ross & Tsang (1990 [90]), (1990 [91]) without reference to this original paper from 1987 even though it was known by the authors. The generalized algorithm in Sec. 10.5.2 is new and includes Delbroucks algorithm (Delbrouck, 1983 [22]) which is more complex to evaluate. Compared with all other algorithms the generalized algorithm requires much less memory and operations to evaluate. By normalizing the state probabilities in each iteration we get a very accurate and simple algorithm. In principle, we may apply the generalized algorithm for BPPtrac to calculate the global state probabilities for (N 1) trac streams and then use the convolution algorithm to calculate the performance measures for the remaining trac stream we want to evaluate. The convolution algorithm is more general than the generalized algorithm as it allows for minimum and maximum allocation of channels to each trac stream. The generalized algorithm does not keep account of the actual number of calls for each stream. The convolution algorithm furthermore allows for arbitrary state-dependent arrival processes.
Updated 2006-02-20

Chapter 11 Dimensioning of telecom networks


Network planning includes designing, optimising, and operating telecommunication networks. In this chapter we will consider trac engineering aspects of network planning. In Sec. 11.1 we introduce trac matrices and the fundamental double factor method (Kruithofs method) for updating trac matrices according to forecasts. The trac matrix contains the basic information for choosing the topology (Sec. 11.2) and trac routing (Sec. 11.3). In Sec. 11.4 we consider approximate calculation of end-to-end blocking probabilities, and describe the Erlang x-point method (reduced load method). Sec. 11.5 generalises the convolution algorithm introduced in Chap. 10 to networks with exact calculation of end-to-end blocking in virtual circuit switched networks with direct routing. The model allows for multislot BPP trac with minimum and maximum allocation. The same model can be applied to hierarchical cellular wireless networks with overlapping cells and to optical WDM networks. In Sec. 11.6 we consider service-protection mechanisms. Finally, in Sec. 11.7 we consider optimising of telecommunication networks by applying Moes principle.

11.1

Trac matrices

To specify the trac demand in an area with K exchanges we should know K 2 trac values Aij (i, j = 1, . . . , K), as given in the trac matrix shown in Tab. 11.1. The trac matrix assumes we know the location areas of exchanges. Knowing the trac matrix we have the following two interdependent tasks: Decide on the topology of the network (which exchanges should be interconnected ?) Decide on the trac routing (how do we exploit a given topology ?) 211

212

CHAPTER 11. DIMENSIONING OF TELECOM NETWORKS

TO
K

FROM 1 . . . i . . . j . . . K
K

1 A11 . . . Ai1 . . . Aj1 . . .

i A1i . . . Aii . . . Aji . . .

j A1j . . . Aij . . . Ajj . . .

K A1K . . . AiK . . . AjK . . .

Ai =
k=1

Aik

A1 . . . Ai . . . Aj . . . AK
K K

AK1 AKi AKj AKK Akj A 1 A i A j A K


i=1

A j =
k=1

Ai =
j=1

A j

The trac matrix has the following elements: Aij = is the trac from i to j. Aii = is the internal trac in exchange i. Ai = is the total outgoing (originating) trac from i. Aj = is the total incoming (terminating) trac to j. Table 11.1: A trac matrix. The total incoming trac is equal to the total outgoing trac.

11.1.1

Kruithofs double factor method

Let us assume we know the actual trac matrix and that we have a forecast for future row sums O(i) and column sums T (i), i.e. the total incoming and outgoing trac for each exchange. This trac prognosis may be obtained from subscriber forecasts for the individual exchanges. By means of Kruithofs double factor method (Kruithof, 1937 [69]) we are able to estimate the future individual values Aij of the trac matrix. The procedure is to adjust the individual values Aij , so that they agree with the new row/column sums: Aij Aij S1 , S0 (11.1)

where S0 is the actual sum and S1 is the new sum of the row/column considered. If we start by adjusting Aij with respect to the new row sum Si , then the row sums will agree, but the column sums will not agree with the wanted values. Therefore, next step is to adjust the obtained values Aij with respect to the column sums so that these agree, but this implies that the row sums no longer agree. By alternatively adjusting row and column sums the values

11.1. TRAFFIC MATRICES

213

obtained will after a few iterations converge towards unique values. The procedure is best illustrated by an example given below.

Example 11.1.1: Application of Kruithof s double factor method We consider a telecommunication network having two exchanges. The present trac matrix is given as:

1 1 2 Total 10 30 40

2 20 40 60

Total 30 70 100

The prognosis for the total originating and terminating trac for each exchange is:

1 1 2 Total 50

Total 45 105

100

150

The task is then to estimate the individual values of the matrix by means of the double factor method. Iteration 1: Adjust the row sums. We multiply the rst row by (45/30) and the second row by (105/70) and get:

1 1 2 Total 15 45 60

2 30 60 90

Total 45 105 150

The row sums are now correct, but the column sums are not. Iteration 2: Adjust the column sums:

1 1 2 Total 12.50 37.50 50.00

2 33.33 66.67 100.00

Total 45.83 104.17 150.00

214

CHAPTER 11. DIMENSIONING OF TELECOM NETWORKS

We now have the correct column sums, whereas the column sums deviate a little. We continue by alternately adjusting the row and column sums: Iteration 3:

1 1 2 Total 12.27 37.80 50.07

2 32.73 67.20 99.93

Total 45.00 105.00 150.00

Iteration 4:

1 1 2 Total 12.25 37.75 50.00

2 32.75 67.25 100.00

Total 45.00 105.00 150.00

After four iterations both the row and the column sums agree with two decimals.

There are other methods for estimating the future individual trac values Aij , but Kruithofs double factor method has some important properties (Bear, 1988 [5]):

Uniqueness. Only one solution exists for a given forecasts. Reversibility. The resulting matrix can be reversed to the initial matrix with the same procedure. Transitivity. The resulting matrix is the same independent of whether it is obtained in one step or via a series of intermediate transformations, (for instance one 5-year forecast, or ve 1-year forecasts). Invariance as regards the numbering of exchanges. We may change the numbering of the exchanges without inuencing the results. Fractioning. The single exchanges can be split into sub-exchanges or be aggregated into larger exchanges without inuencing the result. This property is nor exactly fullled for Kruithofs double factor method, but the deviations are small.

11.2. TOPOLOGIES

215

11.2

Topologies

In Chap. 1 we have described the basic topologies as star net, mesh net, ring net, hierarchical net and non-hierarchical net.

11.3

Routing principles

This is an extensive subject including i.a. alternative trac routing, load balancing, etc. In (Ash, 1998 [3]) there is a detailed description of this subject.

11.4

Approximate end-to-end calculations methods

If we assume the links of a network are independent, then it is easy to calculate the end-to-end blocking probability. By means of the classical formul we calculate the blocking probability of each link. If we denote the blocking probability of link i by Ei , then we nd the end-to-end blocking probability for a call attempt on route j as follows: Ej = 1
iR

(1 Ei ) ,

(11.2)

where R is the set of links included in the route of the call. This value will be worst case, because the trac is smoothed by the blocking on each link, and therefore experience less congestion on the last link of a route. For small blocking probabilities we have: Ej
iR

Ei .

(11.3)

11.4.1

Fix-point method

A call will usually occupy channels on more links, and in general the trac on the individual links of a network will be correlated. The blocking probability experienced by a call attempt on the individual links will therefore also be correlated. Erlangs x-point method is an attempt to take this into account.

216

CHAPTER 11. DIMENSIONING OF TELECOM NETWORKS

11.5

Exact end-to-end calculation methods

Circuit switched telecommunication networks with direct routing have the same complexity as queueing networks with more chains. (Sec. 14.9) and Tab. 14.3). It is necessary to keep account of the number of busy channels on each link. Therefore, the maximum number of states becomes:
K

(ni + 1) .
i=1

(11.4)

Link 1 2 K

Route 1 d11 d12 d1K 2 d21 d22 d2K N dN 1 dN 2 dN K

Number of channels n1 n2 nK

Table 11.2: In a circuit switched telecommunication network with direct routing dij denoted the slot-size (bandwidth demand) of route j upon link i (cf. Tab. 14.3).

11.5.1

Convolution algorithm

The convolution algorithm described in Chap. 10 can directly be applied to networks with direct routing, because there is product form among the routes. The convolution becomes multi-dimensional, the dimension being the number of links in the network. The truncation of the state space becomes more complex, and the number of states increases very much.

11.6

Load control and service protection

In a telecommunication network with many users competing for the same resources (multiple access) it is important to specify service demands of the users and ensure that the GoS is fullled under normal service conditions. In most systems it can be ensured that preferential subscribers (police, medical services, etc.) get higher priority than ordinary subscribers when they make call attempts. During normal trac conditions we want to ensure that all subscribers for all types of calls (local, domestic, international) have approximately the same

11.6. LOAD CONTROL AND SERVICE PROTECTION

217

service level, e.g. 1 % blocking. During overload situations the call attempts of some groups of subscribers should not be completely blocked and other groups of subscribers at the same time experience low blocking. We aim at the collective misery. Historically, this has been fullled because of the decentralised structure and the application of limited accessibility (grading), which from a service protection point of view still are applicable and useful. Digital systems and networks have an increased complexity and without preventive measures the carried trac as a function of the oered trac will typically have a form similar to the Aloha system (Fig. 6.4). To ensure that a system during overload continues to operate at maximum capacity various strategies are introduced. In stored program controlled systems (exchanges) we may introduce call-gapping and allocate priorities to the tasks (Chap. 13). In telecommunication networks two strategies are common: trunk reservation and virtual channels protection.
T C

service protecting route last choice route

single choice route

primary route = high usage route

Figure 11.1: Alternative trac routing (cf. example 11.6.2). Trac from A to B is partly carried on the direct route (primary route = high usage route), partly on the secondary route via the transit exchange T.

11.6.1

Trunk reservation

In hierarchical telecommunication networks with alternative routing we want to protect the primary trac against overow trac. If we consider part of a network (Fig. 11.1), then the direct trac AT will compete with the overow trac from AB for idle channels on the trunk group AT . As the trac AB already has a direct route, we want to give the trac AT priority to the channels on the link AT. This can be done by introducing trunk (channel) reservation. We allow the ABtrac to access the ATchannels only if there are more than r channels idle on AT (r = reservations parameter). In this way, the trac AT will get higher priority to the ATchannels. If all calls have the same mean holding time (1 = 2 = ) and

218

CHAPTER 11. DIMENSIONING OF TELECOM NETWORKS

PCT-I trac with single slot trac, then we can easily set up a state transition diagram and nd the blocking probability. If the individual trac streams have dierent mean holding times, or if we consider Binomial & Pascal trac, then we have to set up an N-dimensional state transition diagram which will be non-reversible. In some states calls of a type having been accepted earlier in lower states may depart but not be accepted, and thus the process is non-reversible. We cannot apply the convolution algorithm developed in Sec. 10.4 for this case, but the generalized algorithm in Sec. 10.5.2 can easily be modied by letting pi (x) = 0 when x nri . An essential disadvantage by trunk reservation is that it is a local strategy, which only consider one trunk group (link), not the total end-to-end connection. Furthermore, it is a one-way mechanism which protect one trac stream against the other, but not vice-versa. Therefore, it cannot be applied to mutual protection of connections and services in broadband networks.

Example 11.6.1: Guard channels In a wireless mobile communication system we may ensure lower blocking probability to hand-over calls than experienced by new call attempts by reserving the last idle channel (called guard channel) to hand-over calls.

11.6.2

Virtual channel protection

In a service-integrated system it is necessary to protect all services mutually against each other and to guarantee a certain grade-of-service. This can be obtained by (a) a certain minimum allocation of bandwidth which ensures a certain minimum service, and (b) a maximum allocation which both allows for the advantages of statistical multiplexing and ensures that a single service do not dominate. This strategy has the fundamental product form, and the state probabilities are insensitive to the service time distribution. Also, the GoS is guaranteed not only on a link basis, but end-to-end.

11.7

Moes principle

Theorem 11.1 Moes principle: the optimal resource allocation is obtained by a simultaneous balancing of marginal incomes and marginal costs over all sectors.

In this section we present the basic principles published by Moe in 1924. We consider a system with some sectors which consume resources (equipment) for producing items (trac). The problem can be split into two parts:

11.7. MOES PRINCIPLE

219

a. Given that a limited amount of resources are available, how should we distribute these among the sectors? b. How many resources should be allocated in total? The principles are applicable in general for all kind of productions. In our case the resources correspond to cables and switching equipment, and the production consists in carried trac. A sector may be a link to an exchange. The problem may be dimensioning of links between a certain exchange and its neighbouring exchanges to which there are direct connections. The problem then is: a. How much trac should be carried on each link, when a total xed amount of trac is carried? b. How much trac should be carried in total? Question a is solved in Sec. 11.7.1 and question b in Sec. 11.7.2. We carry through the derivations for continuous variables because these are easier to work with. Similar derivations can be carried through for discreet variables, corresponding to a number of channels. This is Moes principle (Jensen, 1950 [50]).

11.7.1

Balancing marginal costs

Let us from a given exchange have direct connections to k other exchanges. The cost of a connection to an exchange i is assumed to to be a linear function of the number of channels: Ci = c0i + ci ni , The total cost of cables then becomes:
k

i = 1, 2, . . . , k .

(11.5)

C (n1 , n2 , . . . , nk ) = C0 +
i=1

ci ni ,

(11.6)

where C0 is a constant. The total carried trac is a function of the number of channels: Y = f (n1 , n2 , . . . , nk ) . As we always operate with limited resources we will have: f = Di f > 0 . ni (11.8) (11.7)

220

CHAPTER 11. DIMENSIONING OF TELECOM NETWORKS

In a pure loss system Di f corresponds to the improvement function, which is always positive for a nite number of channels because of the convexity of Erlangs Bformula. We want to minimise C for a given total carried trac Y : min{C} given Y = f (n1 , n2 , . . . , nk ) . (11.9)

By applying the Lagrange multiplier , where we introduce G = C f , this is equivalent to: min {G (n1 , n2 , . . . , nk )} = min {C (n1 , n2 , . . . , nk ) [f (n1 , n2 , . . . , nk ) Y ]} A necessary condition for the minimum solution is: G f = ci = ci Di f = 0, ni ni or 1 D1 f D2 f Dk f = = = = . c1 c2 ck i = 1, 2, . . . , k , (11.11) (11.10)

(11.12)

A necessary condition for the optimal solution is thus that the marginal increase of the carried trac when increasing the number of channels (improvement function) divided by the cost for a channel must be identical for all trunk groups (7.33). It is possible by means of second order derivatives to set up a set of necessary conditions to establish sucient conditions, which is done in Moes Principle (Jensen, 1950 [50]). The improvement functions we deal with will always full these conditions. If we also have dierent incomes gi for the individual trunk groups (directions), then we have to include an additional weight factor, and in the results (11.12) we shall replace ci by ci /gi .

11.7.2

Optimum carried trac

Let us consider the case where the carried trac, which is a function of the number of channels (11.7) is Y . If we denote the revenue with R(Y ) and the costs with C(Y ) (11.6), then the prot becomes: P (Y ) = R(Y ) C(Y ) . (11.13) A necessary condition for optimal prot is: dP (Y ) =0 dY dR dC = , dY dY (11.14)

i.e. the marginal income should be equal to the marginal cost.

11.7. MOES PRINCIPLE Using:


k

221

P (n1 , n2 , . . . , nk ) = R (f (n1 , n2 , . . . , nk )) the optimal solution is obtained for: P dR Di f ci = 0, = ni dY which by using (11.12) gives:

C0 +
i=1

ci ni

(11.15)

i = 1, 2, . . . , k ,

(11.16)

dR = . (11.17) dY The factor given by (11.12) is the ratio between the cost of one channel and the trac which can be carried additionally if the link in extended by one channel. Thus we shall add channels to the link until the marginal income equals the marginal cost (7.35).
Example 11.7.1: Optimal capacity allocation We consider two links (trunk groups) where the oered trac is 3 erlang, respectively 15 erlang. The channels for the two systems have the same cost and there is a total of 25 channels available. How should we distribute the 25 channels among the two links? From (11.12) we notice that the improvement functions should have the same values for the two directions. Therefore we proceed using a table:

A1 = 3 erlang n1 3 4 5 6 7 F1,n (A1 ) 0.4201 0.2882 0.1737 0.0909 0.0412

A2 = 15 erlang n2 17 18 19 20 21 F1,n (A2 ) 0.4048 0.3371 0.2715 0.2108 0.1573

For n1 = 5 and n2 = 20 we use all 25 channels. This results in a congestion of 11.0%, respectively 4.6%, i.e. higher congestion for the smaller trunk group.

Example 11.7.2: Triangle optimisation This is a classical optimisation of a triangle network using alternative trac routing (Fig. 11.1). From A to B we have a trac demand equal to A erlang. The trac is partly carried on the direct route (primary route) from A to B, partly on an alternative route (secondary route) A T B, where T is a transit exchange. There are no other routing possibilities. The cost of a direct connection is cd , and for a secondary connection ct . How much trac should be carried in each of the two directions? The route A T B already carries trac to and from other destinations, and we denote the marginal utilisation for a channel

222

CHAPTER 11. DIMENSIONING OF TELECOM NETWORKS

on this route by a. We assume it is independent of the additional trac, which is blocked from A B. According to (11.12), the minimum conditions become: F1,n (A) a = . cd ct Here, n is the number of channels in the primary route. This means that the costs should be the same when we route an additional call via the direct route and via the alternative route. If one route were cheaper than the other, then we would route more trac in the cheaper direction.

As the trac values applied as basis for dimensioning are obtained by trac measurements they are encumbered with unreliability due to a limited sample, limited measuring period, measuring principle, etc. As shown in Chap. 15 the unreliability is approximately proportional to the measured trac volume. By measuring the same time period for all links we get the highest uncertainty for small links (trunk groups), which is partly compensated by the abovementioned overload sensitivity, which is smallest for small trunk groups. As a representative value we typically choose the measured mean value plus the standard deviation multiplied by a constant, e.g. 1.0. To make sure, it should further be emphasised that we dimension the network for the trac which shall be carried 12 years from now. The value used for dimensioning is thus additionally encumbered by a forecast uncertainty. We has not included the fact that part of the equipment may be out of operation because of technical errors. ITUT recommends that the trac is measured during all busy hours of the year, and that we choose n so that by using the mean value of the 30 largest, respectively the 5 largest observations, we get the following blocking probabilities: En A30 En A5 0.01 , 0.07 . (11.18)

The above service criteria can directly be applied to the individual trunk groups. In practise, we aim at a blocking probability from A-subscriber to B-subscriber which is the same for all types of calls. With stored program controlled exchanges the trend is a continuous supervision of the trac on all expensive and international routes. In conclusion, we may say that the trac value used for dimensioning is encumbered with uncertainty. In large trunk groups the application of a non-representative trac value may result in serious consequences for the grade-of-service level. During later years, there has been an increasing interest for adaptive trac controlled routing (trac network management), which can be introduce in stored program control digital systems. By this technology we may in principle choose the optimal strategy for trac routing during any trac scenario.

Chapter 12 Delay Systems


In this chapter we consider trac to a system with n identical servers, full accessibility, and an innite number of waiting positions. When all n servers are busy, an arriving customer joins a queue and waits until a server becomes idle. No customers can be in queue when a server is idle (full accessibility). We consider the same two trac cases as in Chaps. 7 & 8. 1. Poisson arrival process (an innite number of sources) and exponentially distributed service times (PCT-I ). This is the most important queueing system, called Erlangs delay system. Using the notation later introduced in Sec. 13.1, this system is denoted as M/M/n. In this system the carried trac will be equal to the oered trac as no customers are blocked. The probability of a positive waiting time, mean queue lengths, mean waiting times, carried trac per channel, and improvement functions will be dealt with in Sec. 12.2. In Sec. 12.3 Moes principle is applied for optimising the system. The waiting time distribution is calculated for the basic service discipline, FirstCome FirstServed (FCFS), in Sec. 12.4. 2. A limited number of sources and exponentially distributed service times (PCT-II ). This is Palms machine repair model (the machine interference problem) which is dealt with in Sec. 12.5. This model has been widely applied for dimensioning for example computer systems, terminal systems, and exible manufacturing system (FMS). Palms machine repair model is optimised in Sec. 12.6.

12.1

Erlangs delay system M/M/n

Let us consider a queueing system M/M/n with Poisson arrival process (M), exponential service times (M), n servers and an innite number of waiting positions. The state of the system is dened as the total number of customers in the system (either being served or 223

224
. .... ...... ..... ............. .................. ..... ......... ..... ... .... ... ... .... ........... ..... .... .... .... ........... .... .... ......... .... .. .. . .. .... ........ .. .... .. .. .. .. .. .. .. .. . . . . . . . . . . . . . . . . . . . . . . . . . . . . . . . . . . .. .. . . .. .. . .. . . ... . ... .. .. . .... . ....... ............... ...... ........ ............... .. ... ... . ...... ...... ... ..... ..... .... . ... ..... .............. ............... .............. ..............

CHAPTER 12. DELAY SYSTEMS


........... .... .......... . ............... . .................. ..... .. . ... ....... ...... .... ... ... .... .. ......... . ... .... .... .. .. ... . ... .. . ... . .. .. .. .. .. . . . . . . . . . . . . . . . . . . .. . .. . .. . .. .. . .. . ... . .... . .... . ....... ... ... ......... ...... ... .... ... ....... ...... ... .... . . ................ ............... .............. ..............

. .... ........... .... .......... . ............... . ......... ........ .................. ..... ......... ..... .. . ... ....... ...... .... ... ........ ..... ... .... .. ......... . ... .... ........... ..... .... .. .... .. . .. .... ..... .. ... . ... ... . .. ... . .. .. .. .. .. .. .. .. . . . . . . . . . . . . . . . . . . . . . . . . . . . . . . . . . . . . . .. .. . . .. .. . .. .. . .. . .. .. .. . . .. . .. . .... . ..... . ..... . ..... . ........ ..... . ........ .... ... .. .. .... .... ... . ...... ...... ... . ...... ...... ... . .. . . . . ... ..... ... ..... ............... .............. .............. ............... .............. ..............

n+1

(i+1)

Figure 12.1: State transition diagram of the M/M/n delay system having n servers and an unlimited number of waiting positions. waiting in queue). We are interested in the steady state probabilities of the system. By the procedure described in Sec. 7.4 we set up the state transition diagram shown in Fig. 12.1. Assuming statistical equilibrium, the cut equations become: p(0) = p(1) , p(1) = 2 p(2) , . . . . . . . . .

p(i) = (i+1) p(i+1) , . . . . . . . . . (12.1)

p(n1) = n p(n) , p(n) = n p(n+1) , . . . . . . . . .

p(n + j) = n p(n+j +1) . As A = / is the oered trac, we get: i p(0) A , 0 i n, i! p(i) = in Ai p(n) A = p(0) , i n. n n! n in By normalisation of the state probabilities we obtain p(0) :

(12.2)

1=
i=0

p(i) , A A2 + 2 + ... n n

1 = p(0) 1 +

A A2 An + ++ 1 2! n!

1+

12.2. TRAFFIC CHARACTERISTICS OF DELAY SYSTEMS

225

The innermost brackets have a geometric progression with quotient A/n. The normalisation condition can only be fullled for: A < n. (12.3) Statistical equilibrium is only obtained for A < n. Otherwise, the queue will continue to increase against innity. We obtain: p(0) =
n1

1 Ai An n + i! n! n A

A < n.

(12.4)

i=0

Equations (12.2) and (12.4) yield the steady state probabilities.

12.2

Trac characteristics of delay systems

For evaluation of the capacity and performance of the system, several characteristics have to be considered. They are expressed by the steady state probabilities.

12.2.1

Erlangs C-formula

When the Poisson arrival process is independent of the state of the system, the probability that an arbitrary arriving customer has to wait in the queue is equal to the proportion of time all servers are occupied (PASTAproperty: Poisson Arrivals See Time Average). The waiting time is a random variable denoted by W. For an arbitrary arriving customer we have: E2,n (A) = p{W > 0}

p(i) =
i=n

= p(i)
i=n

p(i)

i=0

= p(n) Erlangs Cformula:

n . nA

(12.5)

An n n! n A E2,n (A) = , 2 An1 An n A A ++ + 1+ + 1 2! (n 1)! n! n A

A < n.

(12.6)

226

CHAPTER 12. DELAY SYSTEMS

This delay probability depends only upon A, the product of and s, not upon the parameters and s individually. The formula has several names: Erlangs Cformula, Erlangs second formula, or Erlangs formula for waiting time systems. It has various notations in literature: E2,n (A) = D = Dn (A) = p{W > 0} . As customers are either served immediately or put into queue, the probability that a customer is served immediately becomes: Sn = 1 E2,n (A) . The carried trac Y equals the oered trac A, as no customers are rejected and the arrival process is a Poisson process:
n

=
i=1 n

i p(i) +
i=n+1

n p(i)

(12.7)

=
i=1

p(i1) + p(i1) i=n+1

= A,

where we have exploited the cut balance equations. The queue length is a random variable L. The probability of having customers in queue at a random point of time is:

p{L > 0} =
i=n+1

p(i) =

A n A 1 n

p(n) ,

p{L > 0} = where we have used (12.5).

A A p(n) = E2,n (A) . nA n

(12.8)

12.2.2

Numerical evaluation

The formula is similar to Erlangs B-formula (7.10) except for the factor n/(n A) in the last term. As we have very accurate recursive formul for numerical evaluation of Erlangs B-formula (7.29) we use the following relationship for obtaining numerical values of the C-

12.2. TRAFFIC CHARACTERISTICS OF DELAY SYSTEMS formula: E2,n (A) = n E1,n (A) n A (1 E1,n (A)) E1,n (A) , 1 A {1 E1,n (A)} /n A < n.

227

= We notice that:

(12.9)

E2,n (A) > E1,n (A) , as the term A {1 E1,n (A)} /n is the average carried trac per channel in the corresponding loss system. For A n, we have E2,n (A) = 1 as it is a probability and all customers are delayed. Erlangs C-formula may in an elegant way be expressed by the B-formula as noticed by B. Sanders: 1 1 1 = , E2,n (A) E1,n (A) E1,n1 (A) I2,n (A) = I1,n (A) I1,n1 (A) , where I is the inverse probability (7.30): I2,n (A) = 1 . E2,n (A) (12.10) (12.11)

Erlangs C-formula has been tabulated in Moes Principle (Jensen, 1950 [50]) and is shown in Fig. 12.2.

12.2.3

Mean queue lengths

We distinguish between the queue length at an arbitrary point of time and the queue length when there are customers waiting in the queue. Mean queue length at an arbitrary point of time: The queue length L at an arbitrary point of time is called the virtual queue length. This is the queue length experienced by an arbitrary customer as the PASTA-property is valid due to the Poisson arrival process (time average = call average). We get the mean queue length

228 E2 (A) Probability of delay 1.0 0.9 0.8 0.7 0.6 0.5 0.4 0.3 0.2 0.1 0.0

CHAPTER 12. DELAY SYSTEMS n

. . . . . . . . . . . . . . . . . . . . . . . . . . . . . . . . . . . . . . . . . . . . . . . . . . . . . . . . . . . . . . . . . . . . . . . . . . . . . . . . . . . . . . . . . . . . . . . . . . . . . . . . . . . . . . . . . . . . . . . . . . . . . . . . . . . . . . . . . . . . . . . . . . . . . . . . . . . . . . . . . . . . . . . . . . . . . . . . . . . . . . . . . . . . . . . . . . . . . . . . . . . . . . . . . . . . . . . . . . . . . . . . . . . . . . . . . . . . . . . . . . . . . . . . . . . . . . . . . . . . . . . . . . . . . . . . . . . . . . . . . . . . . . . . . . . . . . . . . . . . . . . . . . . . . . . . . . . . . . . . . . . . . . . . . . . . . . . . . . . . . . . . . . . . . . . . . . . . . . . . . . . . . . . . . . . . . . . . . . . . . . . . . . . . . . . . . . . . . . . . . . . . . . . . . . . . . . . . . . . . . . . . . . . . . . . . . . . . . . . . . . . . . . . . . . . . . . . . . . . . . . . . . .. . . . . . . . . . . . . . .. . . . . . . . . . . .. . . . . . . . . . . . . . . .. . . . . . . . . . . . .. . . . . . . . . . . . . . . . .. . . . . . . . . . . . . . . . .. . . . . . . . . . . . . . .. . . . . . . . . . . . . . .. . . . . . . . . . . . . . .. . . . . . . . . . . . . . .. . . . . . . . . . . . . . .. . . . . . . . . . .. . . .. . . . . . . . . . . .. .. . . . . . . . . . . . .. . .. . . . . . . . . . . . .. .. . . . . . . . .. . . .. . .. . . . . . . .. . . .. . .. . . . . . . . . .. . . . .. . . . . . . . . .. . . .. . . . . . . . .. . .. . . .. . . . . . . . .. .. . . . .. . .. . . . . . . . .. .. . . .. . . . . . .. . . .. . .. .. . . . . . . . .. .. . .. . . . . . .. . . . .. .. . . .. . .. . . . . . . .. .. . . . .. .. . . . . . .. .. .. . .. .. . . . . . . ... .. ... .. .. . . . . . . . . . ... . . .. ... .. .. . . .. .. . . . .. . .. . .. ... . . ... . . .. . .. .. . . ... ... .. . . . . .. .. ... . .. . .. ... . . . . . . . .. ... ... ... . ... . . .. . .... ... . ... .. ... . . .. . .... . .... ... ... . . .. . .... .. ... .... . . ... . .. .. ... . .. .... .. ... .... . . . .. . . . ... ... .... ..... .... ..... . . . ... .... .... ...... . . ... ...... . .. . ... ... ... ...... ....... . . ........ . . . . . . .. .... .. . . . . .... ..... ........................................................................................................................................ ............ ........ . . . . . ...... . . .... ..... ... . ............. . . . . ............................................................................... . . . . . ................................................

10

15

10

11

12 13 14 15 Oered trac A

Figure 12.2: Erlangs Cformula for the delay system M/M/n. The probability E2,n (A) for a positive waiting time is shown as a function of the oered trac A for dierent values of the number of servers n. Ln = E{L} at an arbitrary point of time:
n

Ln = 0
i=0

p(i) +

(in) p(i)
i=n+1 in

(in) p(n)
i=n+1 i

A n

= p(n)
i=1

A n

A = p(n) n

i=1

(A/n)

A n

As A/n c < 1, the series is uniformly convergent, and the dierentiation operator may be

12.2. TRAFFIC CHARACTERISTICS OF DELAY SYSTEMS put outside the summation: Ln = p(n) A n (A/n) A/n 1 (A/n) = p(n) A/n {1 (A/n)}2

229

= p(n)

A n , nA nA A . nA (12.12)

Ln = E2,n (A)

The average queue length may be interpreted as the trac carried by the queueing positions and therefore it is also called the waiting time trac. Mean queue length, given the queue is greater than zero: The time average is also in this case equal to the call average. The conditional mean queue length becomes:

(in) p(i) Lnq =


i=n+1

p(i)
i=n+1

p(n) =

A/n (1 A/n)2 A p(n) nA (12.13)

n nA

By applying (12.8) and (12.12) this is of course the same as: Lnq = Ln , p{L > 0}

where L is the random variable for queue length.

12.2.4

Mean waiting times

Here also two items are of interest: the mean waiting time W for all customers, and the mean waiting time w for customers experiencing a positive waiting time. The rst one is an indicator for the service level of the whole system, whereas the second one is of importance

230

CHAPTER 12. DELAY SYSTEMS

for the customers, which are delayed. Time averages will be equal to call averages because of the PASTA-property. Mean waiting time W for all customers: Littles theorem tells that the average queue length is equal to the arrival intensity multiplied by the mean waiting time: Ln = Wn . (12.14) where Ln = Ln (A), and Wn = Wn (A). From (12.12) we get by considering the arrival process: 1 Ln A = E2,n (A) . nA As A = s, where s is the mean service time, we get: s Wn = E2,n (A) . nA Wn = Mean waiting time w for delayed customers: The total waiting time is constant and may either be averaged over all customers (Wn ) or only over customers, which experience positive waiting times (wn ) (3.20): Wn = wn E2,n (A) , wn = s . nA (12.16) (12.17)

(12.15)

Example 12.2.1: Single server queueing system M/M/1 This is the system appearing most often in the literature. The state probabilities (12.2) are given by a geometric series: p(i) = (1 A) Ai , i = 0, 1, 2, . . . , (12.18) as p(0) = 1A. The probability of delay become: E2,1 (A) = A . The mean queue length Ln (12.12) and the mean waiting time for all customers Wn (12.15) become: L1 = W1 = A2 , 1A As . 1A (12.19)

(12.20)

From this we observe that an increase in the oered trac results in an increase of Ln by the third power, independent of whether the increase is due to an increased number of customers () or an increased service time (s). The mean waiting time Wn increases by the third power of s, but only by the second power of . The mean waiting time wn for delayed customers increases with the second power of s, and the rst power of . An increased load due to more customers is thus better than an increased load due to longer service times. Therefore, it is important that the service times of a system do not increase during overload.

12.3. MOES PRINCIPLE FOR DELAY SYSTEMS

231

Example 12.2.2: Mean waiting time w when A 0 Notice, that as A 0, we get wn = s/n (12.17). If a customer experiences waiting time (which seldom happens when A 0), then this customer will be the only one in the queue. The customer must wait until a server becomes idle. This happens after an exponentially distributed time interval with mean value s/n. So wn never becomes less than s/n.

12.2.5

Improvement functions for M/M/n

The marginal improvement of the trac carried when we add one server can be expressed in several ways. The decrease in the proportion of total trac (= the proportion of all customers) that experience delay is given by: F2,n (A) = A {E2,n (A) E2,n+1 (A)} . (12.21)

The decrease in mean queue length (= trac carried by the waiting positions) becomes by using Littles law (12.14): FL,n (A) = Ln (A) Ln+1 (A) = {Wn (A) Wn+1 (A)} , (12.22)

where Wn (A) is the mean waiting time for all customers when the oered trac is A and the number of servers is n (12.15). Both (12.21) and (12.22) are tabulated in Moes Principle (Jensen, 1950 [50]) and are simple to evaluate by a calculator or computer.

12.3

Moes principle for delay systems

Moe rst derived his principle for queueing systems. He studied the subscribers waiting times for an operator at the manual exchanges in Copenhagen Telephone Company. Let us consider k independent queueing systems. A customer being served at all k systems has the total average waiting time i Wi , where Wi is the mean waiting time of ith system which has ni servers and is oered the trac Ai . The cost of a channel is ci , eventually plus a constant cost, which is included in the constant C0 below. Thus the total costs for channels becomes:
k

C = C0 +
i=1

ni ci .

(12.23)

If the waiting time also is considered as a cost, then the total costs to be minimised becomes f = f (n1 , n2 , . . . , nk ). This is to be minimised as a function of number of channels ni in the

232

CHAPTER 12. DELAY SYSTEMS

individual systems. If the total average waiting time is W , then the allocation of channels to the individual systems is determined by: min {f (n1 , n2 , . . . , nk )} = min C0 +
i

ni ci +
i

Wi W

(12.24)

where (theta) is Lagranges multiplier. As ni is integral, a necessary condition for minimum, which in this case also can be shown to a be sucient condition, becomes: 0 < f (n1 , n2 , . . . , ni 1, . . . , nk ) f (n1 , n2 , . . . , ni , . . . , nk ) , 0 f (n1 , n2 , . . . , ni , . . . , nk ) f (n1 , n2 , . . . , ni +1, . . . , nk ) , which corresponds to: Wni 1 (Ai ) Wni (Ai ) > Wni (Ai ) Wni +1 (Ai ) where Wni (Ai ) is given by (12.15). Expressed by the improvement function for the waiting time FW,n (A) (12.22) the optimal solution becomes: ci FW,ni 1 (A) > FW,ni (A) , i = 1, 2, . . . k . (12.27) The function FW,n (A) is tabulated in Moes Principle (Jensen, 1950 [50]). Similar optimisations can be carried out for other improvement functions.
Example 12.3.1: Delay system We consider two dierent M/M/n queueing systems. The rst one has a mean service time of 100 s and the oered trac is 20 erlang. The cost-ratio c1 / is equal to 0.01. The second system has a mean service time equal to 10 s and the oered trac is 2 erlang. The cost ration equals c2 / = 0.1. A table of the improvement function FW,n (A) gives: n1 = 32 channels and n2 = The mean waiting times are: W1 = 0.075 s. W2 = 0.199 s. This shows that a customer, who is served at both systems, experience a total mean waiting time equal to 0.274 s, and that the system with less channels contributes more to the mean waiting time. 5 channels.

(12.25)

ci , ci , (12.26)

12.4. WAITING TIME DISTRIBUTION FOR M/M/N, FCFS

233

The cost of waiting is related to the cost ratio. By investing one monetary unit more in the above system, we reduce the costs by the same amount independent of in which queueing system we increase the investment. We should go on investing as long as we make prot. Moes investigations during 1920s showed that the mean waiting time for subscribers at small exchanges with few operators should be larger than the mean waiting time at larger exchanges with many operators.

12.4

Waiting time distribution for M/M/n, FCFS

Queueing systems, where the service discipline only depends upon the arrival times, all have the same mean waiting times. In this case the strategy has only inuence upon the distribution of waiting times for the individual customer. The derivation of the waiting time distribution is simple in the case of ordered queue, FCFS = FirstCome FirstServed. This discipline is also called FIFO, FirstIn FirstOut. Customers arriving rst to the system will be served rst, but if there are multiple servers they may not necessarily leave the server rst. So FIFO refers to the time for leaving the queue and initiating service. Let us consider an arbitrary customer. Upon arrival to the system, the customer is either served immediately or has to wait in the queue (12.6). We now assume that the customer considered has to wait in the queue, i.e. the system may be in state [n + k], (k = 0, 1, 2, . . .), where k is the number of occupied waiting positions just before the arrival of the customer. Our customer has to wait until k + 1 customers have completed their service before an idle server becomes accessible. When all n servers are working, the system completes customers with a constant rate n , i.e. the departure process is a Poisson process with this intensity. We exploit the relationship between the number representation and the interval representation (5.4): The probability p{W t} = F (t) of experiencing a positive waiting time less than or equal to t is equal to the probability that in a Poisson arrival process with intensity (n ) at least (k+1) customers arrive during the interval t (6.1): (nt)i nt e . i! i=k+1

F (t | k waiting) =

(12.28)

The above was based on the assumption that our customer has to wait in the queue. The conditional probability that our customer when arriving observes all n servers busy and k

234 waiting customers (k = 0, 1, 2, ) is:

CHAPTER 12. DELAY SYSTEMS

pw (k) =

p(n + k)

p(n) = p(n)
i=0 k

A n

p(n + i)
i=0

A n

A n

A n

k = 0, 1, . . . .

(12.29)

This is a geometric distribution including the zero class (Tab. 6.1). The unconditional waiting time distribution then becomes:

F (t) =
k=0

pw (k) F (t | k) ,
k

(12.30) (nt)i nt e i! i=k+1 1


k=0

F (t) =
k=0

A n

A n

i1

= ent

i=1

(nt)i i!

A n

A n

as we may interchange the two summations when all terms are positive probabilities. The inner summation is a geometric progression:

i1

k=0

A 1 n

A n

A 1 n 1 A n A n

i1

k=0

A n

1
i

1 (A/n)i 1 (A/n)

= 1

12.4. WAITING TIME DISTRIBUTION FOR M/M/N, FCFS Inserting this we obtain: F (t) = ent

235

i=1

(nt)i i!

A n

= ent

i=0

(nt)i i!

i=0

(nt)i i! ,

A n

= ent ent ent A/n F (t) = 1 e(n A)t , F (t) = 1 e (n ) t ,

n > A,

t > 0.

(12.31)

i.e. an exponential distribution. Apparently we have a paradox: when arriving at a system with all servers busy one may: 1. Count the number k of waiting customers ahead. The total waiting time will then be Erlang(k+1) distributed. 2. Close the eyes. Then the waiting time becomes exponentially distributed. The interpretation of this is that a weighted sum of Erlang distributions with geometrically distributed weight factors is equivalent to an exponential distribution. In Fig. 12.3 the phasediagram for (12.30) is shown, and we notice immediately that it can be reduced to a single exponential distribution (Sec. 4.4.2 & Fig. 4.9). Formula (12.31) conrms that the mean waiting time wn for customers who have to wait in the queue becomes as shown in (12.17). The waiting time distribution for all (an arbitrary customer) becomes (3.19): Fs (t) = 1 E2,n (A) e(nA) t , A < n, t 0, (12.32)

and the mean value of this distribution is Wn in agreement with (12.15). The results may be derived in an easier way by means of generation functions.

12.4.1

Sojourn time for a single server

When there is only one server, the state probabilities (12.2) are given by a geometric series p(i) = (1 A) A i (12.18) for all i 0. Every customer spends an exponentially distributed time interval with intensity in every state. A customer who nds the system in state [ i ] shall stay in the system an Erlang(i+1) distributed time interval. Therefore, the sojourn

236
A n A n

CHAPTER 12. DELAY SYSTEMS


A n A n

.... .. ................... ................... .. .

.... .. ............... ............... .. . .

n .........................................

... .. .. .............................. ............................. . ... .. .............................. ............................. . .. .. . .. . . . . . . . . . . . . . . . . . . . . . . . . . . . . . . . . . . . . . . . . . . . . . . . . . . . . . . . . . . . . . . . . . . . . . . . . . . . . . . . . . . . . . . . . . . . . . . . . . . . . . . . . . . . . . . . . . . . . . . . . . . . . . . . . . . . . . . . . . . .. ... ... .. ... . .. .. . .. .. . .. .. . . . . ....................................................................... ............................................................ ... . . . ............ . .. ... . .. .. ...

1 A n

1 A n

... .... .. ......................... ........................ .... . ............................... .............................. . ... . . . . . . . . . . . . . . . . . . . . . . . . . . . . . . . . . . . . . . . . . . . . . . . . . . . . . . . . . . . . . . . . . . . . . . . . . . . . . . . . . . . . . . . . . . . . . . . . . . . . . . . . . . . . . . . . . . . . . . . . . . . . . . . . . . . . . . . . . . . . .. ... ... .. ... . .. .. .. .. . . .. . . . . .......................................................................... ................................................ .. . ................................ ...... .. . .... . . .. .. ...

1 A n

1 A n

Figure 12.3: The waiting time distribution for M/M/nFCFS becomes exponentially distributed with intensity (n ). The phase-diagram to the left corresponds to a weighted sum of Erlang-k distributions (Sec. 4.4.2) as the rate out of all phases is n (1A/n) = n. time in the system (waiting time + service time), which also is called the response time, is exponentially distributed with intensity ( ) (cf. Fig. 4.9): F (t) = 1 e( )t , > , t 0. (12.33)

This is identical with the waiting time distribution of delayed customers. The mean sojourn time may be obtained directly using W1 from (12.20) and the mean service time s: m = W1 + s = m = As +s, 1A (12.34)

s 1 = , 1A

where = 1/s is the service rate. We notice that mean sojourn time is equal to mean waiting time for delayed customers (12.17).

12.5

Palms machine repair model

This model belongs to the class of cyclic queueing systems and corresponds to a pure delay system with a limited number of customers (cf. Engset case for loss systems). The model was rst considered by Gnedenko in 1933 and published in 1934. It became widely known when C. Palm published a paper in 1947 [80] in connection with a theoretical analysis of manpower allocation for servicing automatic machines. A number of S machines, which usually run automatically, are serviced by n repairmen. The machines may break down and then they have to be serviced by a repairman before running again. The problem is to adjust the number of repairmen to the number of machines so that the total costs are minimised (or the prot optimised). The machines may be textile machines which stop when they run out of thread; the repairmen then have to replace the empty spool of a machine with a full one.

12.5. PALMS MACHINE REPAIR MODEL Density function 1 .. 5 2 101 5 2 102 5 2 103 5 2 104 0 5 10 15 20 25

237

. . . . . . A= 8 . .. . .. . . n = 10 . .. ... .. .. ... ... .. .. .... .. . .. .......... .. ............ .. .............. .. .... ......... .. ..... ........ .. ..... ......... .. ..... ........ .. ..... ........ .. .. .. .. ..... .. ........... .. .. . ....... .. ..... ....... .. ...... ...... . . . .... ...... . . . ... ...... ......... . . . . .............. . . . .............. . . . .......... . . ............. ... .. ... .. .. . .. . .. . . ... ... .. .. .... . .. . . ... .. .. . ......... ...... . .. .. . . ......... .. .. . .. .. .... . . ..... . ... . .. .. .. .. . . . . .. ... . . . ... .. .. . .. . .

FCFS

.. .. .. .. .... .. ... .. .. .... ... .. .. .... ... .. .. .... .. ... .. .. .... ... .. .. .... ... .. .. .... ... .. .. .... ... .. .. .... ... .. .. .... ... .. .. .... ... .. .. .... ... .. .. .... .. ... .. .. .. .. .. .. .. .. .. .

......

......

SIRO

......

. . . .LCFS .......

30

35

40

45

50 55 60 Waiting time

Figure 12.4: Density function for the waiting time distribution for the queueing discipline FCFS, LCFS, and SIRO (RANDOM). For all three cases the mean waiting time for delayed calls is 5 time-units. The form factor is 2 for FCFS, 3.33 for LCFS, and 10 for SIRO. The number of servers is 10 and the oered trac is 8 erlang. The mean service time is s = 10 time-units.

This Machine-Repair model or Machine Interference model was also considered by Feller (1950 [27]). The model corresponds to a simple closed queueing network and has been successfully applied to solve trac engineering problems in computer systems. By using Kendalls notation (Chap. 13) the queueing system is denoted by M/M/n/S/S, where S is the number of customers, and n is the number of servers. The model is widely applicable. In the Web the machines corrrespond to clients whereas the repairmen correspond to servers. In computer terminal systems the machines correspond to the terminals and a repairman corresponds to a computer managing the terminals. In a computer system the machine may correspond to a disc storage and the repairmen correspond to input/output (I/O) channels. In the following we will consider a computer terminal system as the background for the development of the theory.

238

CHAPTER 12. DELAY SYSTEMS

12.5.1

Terminal systems

Time division is an aid in oering optimal service to a large group of customers using for example terminals connected to a mainframe computer. The individual user should feel that he is the only user of the computer (Fig. 12.5).

Figure 12.5: Palms machine-repair model. A computer system with S terminals (an interactive system) corresponds to a waiting time system with a limited number of sources (cf. Engset-case for loss systems). The individual terminal changes all the time between two states (interactive) (Fig. 12.6): the user is thinking (working), or the user is waiting for a response from the computer.

The time interval the user is thinking is a random variable Tt with mean value mt . The time interval, when the user is waiting for the response from the computer, is called the response time R. This includes both the time interval Tw (mean value mw ), where the job is waiting for getting access to the computer, and the service time itself Ts (mean value ms ). Tt + R is called the circulation time (Fig. 12.6). At the end of this time interval the terminal returns to the same state as it left at the beginning of the interval (recurrent event). In the following we are mainly interested in mean values, and the derivations are valid for all work-conserving queueing disciplines (Sec. 13.4.2).

12.5. PALMS MACHINE REPAIR MODEL Terminal state


. . . .. . .. ... . ... ... . . . . . . . ....................... . . ....................... ...................... . . . .. . . ...................... . . . .. . . . . . . . . . . . . . . . . . . . . . . . . . . . . . . ....... ... ...... ... . . . . . . . . . . . . . . . . . . . . . . . . . . . . . . . . .... .... .... ... ... ... . . . . . . . . . . . .... ... . . . .. . .......................... ......................... .......................... .......................... . . . . . . . . . . . . . . . . . . .. . .. . ........................................................................................................................................................................................ .................................................................. .................................................................................................................... . . . . . .. .... . . . . . . . . . . . . . . . . . . . . . . .............. . . . ............. .. ........................... ................. ........ ..................... .................... ........ ........ . . .. . .. .. . . . . . . . . . . . . . . . . . .

239

Circulation time

Service

Waiting

Thinking

Tt

Tw

Ts

Time

Figure 12.6: The individual terminal may be in three dierent states. Either the user is working actively at the terminal (thinking), or he is waiting for response from the computer. The latter time interval (response time) is divided into two phases: a waiting phase and a service phase.

12.5.2

State probabilities single server

We consider now a system with S terminals, which are connected to one computer. The thinking times for every thinking terminal are so far assumed to be exponentially distributed with the intensity = 1/mt , and the service (execution) time at the computer is also assumed to be exponentially distributed with intensity = 1/ms . When there is queue at the computer, the terminals have to wait for service. Terminals being served or waiting in queue have arrival intensity zero. State [ i ] is dened as the state, where there are i terminals in the queueing system (Fig. 12.5), i.e. the computer is either idle (i = 0) or working (i > 0), and (i1) terminals are waiting when (i > 0). The queueing system can be modelled by a pure birth and death process, and the state transition diagram is shown in Fig. 12.7. Statistical equilibrium always exists (ergodic system). The arrival intensity decreases as the queue length increases and becomes zero when all terminals are inside the queueing system. The steady state probabilities are found by applying cut equations to Fig. 12.7 and expressing all states in terms of state S: (S i) p(i) = p(i + 1), i = 0, 1, . . . , S . (12.35)

By the additional normalization constraint that the sum of all probabilities must be equal to

240
.... ........... .............. ............... ............... ................ . ...... ......... ..... ... .... ... ....... ...... .... ... . ... ........ .... .. .... .. ......... . ... .... ........... ..... . ... .. . ... .. . .. .............. ..... ............... .... .. . . . .. . .. ... . .. .. ... .. . .. .. .. . .. .. .. . . .. .. .. . . . . . . . . . . . . . . . . . . . . . . . . . . . . . . . . . . . . . . . . . . . . . . . . . . .. .. .. . .. .. .. .. .. .. .. .. ... .. ... .. ... . .. .. . . ... . ... . ... . ................ ............... .. ................. ... ........... ...... .... . . ... ......... ...... .. ......... ...... .. . ... ... ..... . .. ................. .... ................. ................ ................ ........... ..............

CHAPTER 12. DELAY SYSTEMS S (S 1) (S 2)


.... ........... ............. ................. ...... ......... ..... .... ... ... ... ... .... ........... ..... .... ........... .... ............ . .. .... ........... . .. .. ... .. . .. .. .. .. . . .. .. .. .. . .. .. .. .. . . . . . . . . . . . . . . . . . . . . . . . . . . . . . . . . .. .. . . .. .. .. .. ... ... .. .. . . . . .... .... .. ................. .. ............ .... ... ... ........... ..... ... ........... .... . . .. .. . . .. .. ..... ................ ................ ............... ...........

S 1 S

Figure 12.7: State transition diagram for the queueing system shown in 12.5. State [ i ] denotes the number of terminals being either served or waiting, i.e. S i denotes the number of terminals thinking. one we nd, introducing = /: p(S i) =
i

i!

p(S)
i

i!
S j

i = 0, 1, . . . , S ,

(12.36)

j=0

j! (12.37)

p(0) = E1,S ( ) . This is the truncated Poisson distribution (7.9).

We may interpret the system as follows. A trunk group with S trunks (the terminals) is oered calls from the computer with the exponentially distributed inter-arrival times (intensity ). When all S trunks are busy (thinking), the computer is idle and the arrival intensity is zero, but we might just as well assume it still generates calls with intensity which are lost or overow to another trunk group (the exponential distribution has no memory). The computer thus oers the trac = / to S trunks, and we have the formula (12.37). Erlangs Bformula is valid for arbitrary holding times (Sec. 7.3.3) and therefore we have: Theorem 12.1 The state probabilities of the machine repair model (12.36) & (12.37) with one computer and S terminals is valid for arbitrary thinking times when the service times of the computer are exponentially distributed. The ratio = / between the time a terminal on average is thinking 1/ and the time the computer on average serves a terminal 1/, is called the service ratio. The service ratio corresponds to the oered trac A in Erlangs B-formula. The state probabilities are thus determined by the number of terminals S and the service ratio . The numerical evaluation of (12.36) & (12.37) is of course as for Erlangs B-formula (7.29).
Example 12.5.1: Information system We consider an information system which is organised as follows. All information is kept on 6 discs

12.5. PALMS MACHINE REPAIR MODEL

241

which are connected to the same input/output data terminal, a multiplexer channel. The average seek time (positioning of the seek-arm) is 3 ms and the average latency time to locate the le is 1 ms, corresponding to a rotation time of 2 ms. The reading time to a le is exponentially distributed with a mean value 0.8 ms. The disc storage is based on rotational positioning sensing, so that the channel is busy only during the reading. We want to nd the maximum capacity of the system (number of requests per second). The thinking time is 4 ms and the service time is 0.8 ms. The service ratio thus becomes 5, and Erlangs B-formula gives the value: 1 p(0) = 1 E1,6 (5) = 0.8082 . This corresponds to max = 0.8082/0.0008 = 1010 requests per second.

12.5.3

Terminal states and trac characteristics

The performance measures are easily obtained from the analogy with Erlangs classical loss system (12.37). Replacing p(0) by E1,S ( ) the computer is working with the probability {1 E1,S ( )}. We then have that the average number of terminals being served by the computer is given by: ns = 1 E1,S ( ) . (12.38) The average number of thinking terminals corresponds to the trac carried in Erlangs loss system: nt = {1 E1,S ( )} = {1 E1,S ( )}. (12.39) The average number of waiting terminals becomes: nw = S ns nt = S {1 E1,S ( )} {1 E1,S ( )} = S {1 E1,S ( )}{1 + } . If we consider a random terminal at a random point of time, we get: p{terminal served} = ps = p{terminal thinking} = pt = p{terminal waiting} = pw = 1 E1,S ( ) ns = , S S nt = S (1 E1,S ( )) , S (12.41) (12.42) (12.40)

{1 E1,S ( )}{1 + } nw = 1 . (12.43) S S We are also interested in the response time R which has the mean value mr = mw + ms . By applying Littles theorem L = W to terminals, waiting positions and computer, respectively, we obtain (denoting the circulation rate of jobs by ): 1 mt mw ms mr = = = = , nt nw ns nw + ns (12.44)

242 or

CHAPTER 12. DELAY SYSTEMS

nw + ns S nt ms = ms . ns ns nt mt Making use of (12.38) and (12.44) we get: = ns ms mr = mr = mr = S ms mt ns S ms mt . 1 E1,S ( ) (12.45)

Thus the mean response time is independent of the time distributions as it is based on (12.38) and (12.44) (Littles Law). However, E1,S ( ) will depend on the types of distributions in the samE way as the Erlang-B formula. If the service time of the computer is exponentially distributed (mean value ms = 1/), then E1,S ( ) will be given by (12.37). Fig. 12.8 shows the response time as a function of the number of terminals in this case. If all time intervals are constant, the computer may work all the time serving K terminals without any delay when: K = = mt + ms ms +1. (12.46)

K is a suitable parameter to describe the point of saturation of the system. The average waiting time for an arbitrary terminal is obtained from (12.45): mw = mr ms
Example 12.5.2: Time sharing computer In a terminal system the computer sometimes becomes idle (waiting for terminals) and the terminals sometimes wait for the computer. Few terminals result in a low utilisation of the computer, whereas many terminals connected will waste the time of the users. Fig. 12.9 shows the waiting time trac in erlang, both for the computer and for a single terminal. An appropriate weighting by costs and summation of the waiting times for both the computer and for all terminals gives the total costs of waiting. For the example in Fig. 12.9 we obtain the minimum total delay costs for about 45 terminals when the cost of waiting for the computer is hundred times the cost of one terminal. At 31 terminals both the computer and each terminal spends 11.4 % of the time for waiting. If the cost ratio is 31, then 31 is the optimal number of terminals. However, there are several other factors to be taken into consideration.

Example 12.5.3: Trac congestion We may dene the trac congestion in the usual way (Sec. 2.3). The oered trac is the trac

12.5. PALMS MACHINE REPAIR MODEL Mean response time [1 ]

243

20

15

10

. . . . . . . . . . . . . . . . . . .. . . .. . . . . . . . . . . . . . .. . . . . .. . . . . . . . . . .. . . . . .. . . . . . . . . . . . . . . . . .. . . . .. . . . . . . . . . . . . . . .. . .. . . . . . . . . . . . . .. . . . .. . . . . . . . . . . . . . . . .. . . . . . . . . . . . . . . . . . . . . .. . . . . . ............................................................................................................................................................................................................................................................................. ........................... ..................................................................................................................................................................................................................... .......................... . . . . . . . . . . . . . . . . . . . . . .. . . . .. . . . . . . . .. . . . .. . . . . . . . . . . . . . . . . . . . . . . . . .. . . . . . .. . . . ... . . . . . . . . . . .. . . . . . .. . . . . . . . . . . . . . ... . . . . . ... . . . . . . . . . . .. . . . . ... . . .. . . . . . . . . . ... . . . . . . . . . ... . . . . . . . . ... . . . . . ... . . . . .. . . . . ... . . . . . . . . . . . .. . . . . ... . . . . . . . . . .. . . . . . . .. . . . . . . . . . . . . . ... . . . . ... . . . . . . .. . . . . . .. . . . . . . . . . . ... . . . . . . ... . . . . . . . . . . .. . . . . .. . . . . .. . . .. . . . . . . . . . . . .................................................................................................................................................................................................................................................. . ..................................................................................................................................................... ...................................... . ................................................................................. ........................... . . . ... . . ..... . . . . . . . . . . . . ... . . . . . ... . . . . . . . . . . .. . . . . . ... . . . . . . . .. . . . . .... . . . . . . . . . . . . . ... . . ... . . . . . . . . . . . . . . .. . . . . . . .. .. . . . . . . . . . . . . .. . . . . . . . . . . . . . . . .. . . . . . .. .. . . . . . . . . . . . .. . . . .. . . . . . . . . . . . . . .. .. . . . .. .. . . . . . . . . . . . .. . . . . .. . . . . . . . . . . . . .. . . . . .. .. . . . . . . . . . . . .. . .. . . . . . . . . . . . . . . . . . .. .. . . . . .. .. . . . . . . . . . . . .. . . . . .. . . . . . . . . . . . . . .. . . . . . .. . . . . . . . . . . . . . . .. . . . . . . ... . . . . . . . . ..................................................... ........................................................................................................................................................................................... . .. ................................................................................................................................................................................................................................................. ............................ ........................... . . . . . . . . . . .. . . . .. . . .... . . . . . . . . . .. . . . . .. . . . . . . . . .. . .... . .. . . . .... . . . . . . . . . . . ... . . . . ... . . . . . . . . . . . . . ... . . . . .. .. ... . . . . . . . . . . . . ... . . . ... . . . . . . . . . . . . . . . ... . ... . . . .. . . . . . . . . . . . . . ... . . . ... . . . . . . . . . . . . . . . ... . . . ... . . . . .. . . . . . .. . . . . . . ... . . ... . . . . . .. . . . . . . . . . ... . . . . ... . . . . . . . . . . . . . .. . .... . . . .... . . .. . . . . . . . . . . .... . . . . . . .... . .. . . . . . . . . . . . .. .... . . . . . .... . . . .. . . . . . . . ........ . . . . . ........ . . . . . . . . . . .. . . .... . . .... . .. .. . . . . . . . . . . ...... . . . ...... . . . .. . . . .... . . ............................................................................................................................................................................................................................................................................... ........................... ................................................................................................................................................................................................................................................. . .... ... . .. . . . . . . . . . . . . . . ....... . . . ....... . . .... . . . . . . . . ................. . .. . . . .................. .. . . . . . . . . . . . . . . . . ......... .. ......... . . . ........ . . ........ . . . . . . . . . . . .. . ................................... ................ ...... . . . . . . .... .... ... ..... ... . ......................... .... .... .... .... .... .... .... ..... .... .... .... .... .... .... .... .... .... .... .... .... ..... .... .... .... ..... .... .... .... .... .... .... .... .... .... .... .... .... ................. .... ... ... ... ... ... ... ... . ... ... ... ... ... ... ... ... ... ... ... ... ... ... ... ... ... .... ... ... ... ... ... ... ... ... ... ... ... ... ............ . ............. . . . . . . . . . . . . . . . . . . . . . . . . . . . . . . . . . . . . . . . . . . . . . . . . . . . . . . . . . . . . . . .

= 30

10

20

30

40 50 Number of terminals S

Figure 12.8: The actual average response time experienced by a terminal as a function of the number of terminals. The service-factor is = 30. The average response time converges to a straight line, cutting the x-axes in S = 30 terminals. The average virtual response time for a system with S terminals is equal to the actual average response time for a system with S + 1 terminals (the Arrival theorem, theorem 8.1).
carried when there is no queue. The oered trac per source is (8.8): a= The carried trac per source is: y= The trac congestion becomes: C = ay a mt + ms mw = , mt + mw + ms mt + mw + ms ms = 1+ mt + ms ms . mt + mw + ms

= 1 C = pw

244

CHAPTER 12. DELAY SYSTEMS

In this case with nite number of sources the trac congestion becomes equal to the proportion of time spent waiting. For Erlangs waiting time system the trac congestion is zero because all oered trac is carried.

Waiting time trac [erlang] 1.0


.. .. .. .. .. .. .. .. .. .. .. .. .. .. .. .. .. .. .. .. .. .. .. .. .. .. .. .. .. .. .. .. .. .. .. .. .. .. .. .. .. .. .. .. .. .. .. .. .. .. .. .. .. .. .. .. .. .. .. .. .. .. .. ... .. .. ..... ..... .... . .. ...... .. ..... ..... .. .. ..... ..... .... .. ..... .. ..... .... .. .. .... .... .... .. .... .. . .. .... .. .. .... .... . . .. .... .. .... .... ... .. .. .... .... .. .. .... .... .. .. .... .... .. ... .. .. ... ... .. ... ... .. ... ... .. .. ... ... .. ... .. ... . .. .. .. ... ... .. ... ... .. ... ... .. .. ... ... .. .. ... ... . .. .. ... ... .. .. ... ... .. ... .. ... .. ... ... .. ... .. ... . .. . .. ... ... .. ... .. ... .. ... ... .. .. ... ... .. .. ... ... . .. .. ... ... .. .. ... ... .. ... .. ... .. ... ... .. ... ...... ... ...... . .. .... ... ..... ... ..... .... ... .... ... ..... .... ... ... ..... ... ..... ... ..... ..... ... ... ... ... .... .... ...... ...... .... .... .... .... ........ ........ ..... ..... .... ..... ...... ....... ............ ............ ......... ......... ............ ............... .................... ................... ............................................................................................. ......................................................................................... .................... ....................

= 30

0.8

Computer

0.6

Per terminal

0.4

0.2

0.0

10

20

30

40

50 60 70 Number of terminals S

Figure 12.9: The waiting time trac (the proportion of time spend waiting) measured in erlang for the computer, respectively the terminals in an interactive queueing system (Service factor = 30).

12.5.4

Machinerepair model with n servers

The above model is easily generalised to n computers. The transition diagram is shown in Fig. 12.10.

12.5. PALMS MACHINE REPAIR MODEL


........... .............. ............... ............... ..... ... .... ... ....... ...... .... .. . ... .... .. ......... ..... .... ... .. .. . .............. .... ............... .. ... .. .. ... .. .. .. .. .. . . . .. . . .. . . . . . . . . . . . . . . . . . . . . . . . . . . . . . . .. . .. .. .. .. .. .. .. ... . .... .. . .. . ... . . ... . ............... .. ........ ..... .. ... ... .......... ...... ... ... ......... ...... ... ..... .. ...... ........ ................ ................ ..... ..... ....

245
(S n) (S n1)
.... ..... ..... ...... ......... ..... .... ... ... .... ........... ................ .. .. .. .. . .. .. .. .. . . . . . . . . . . . . . . . . . . .. . .. .. ... ... .. .. .... . .... .. ... . ... .......... ... ........ .... .. ..... ................ ...........

(S 1)

(S n+2) (S n+1)

.... .... ..... ..... ........... ........... .......... ............. ............. ...... ......... ...... ......... ..... .... ..... ..... ..... .... .... ... .... ... ... ... ... ... .... ........... ..... .... ........... ..... .... ........... ..... .... .... ........... ... .... ........... ... .... ........... .... .... . .. .. . . .. .. .. . . . .. .. .. .. .. . .. . . .. .. ... .. .. .. .. .. .. .. .. . .. .. . .. . . . . . . . . . . . . . . . . . . . . . . . . . . . . . . . . . . . . . . . . . . . . . .. .. .. . .. .. .. .. .. .. .. .. ... ... . ... . . . .. .. .. . . ..... . ... ... ... . ... . ...... .... .. .... . ... ... . ......... ...... ... . ......... ...... ... . ......... .... . ... .... ... ........ ...... ... ........ ...... . . ... ........ ...... ... .. . .. . . .. . . ................. ................ ................ ................ ................ ............... ............... ...............

n1

n+1

(n1)

Figure 12.10: State transition diagram for the machine-repair model with S terminals and n computers. The steady state probabilities become: p(i) = S i
i

p(0) , n
in

0 i n,

(S n)! p(i) = (S i)!

p(n) ,

niS.

(12.47)

where we have the normalisation constraint:


S

p(i) = 1 .
i=0

(12.48)

We can show that the state probabilities are insensitive to the thinking time distribution as in the case with one computer. (We get a state-dependent Poisson arrival process). An arbitrary terminal is at a random point of time in one of the three possible states: ps = p {the terminal is served by a computer}, pw = p {the terminal is waiting for service}, pt = p {the terminal is thinking}. We have: ps 1 = S
n S

i p(i) +
i=0 i=n+1

n p(i)

(12.49)

pt = ps

(12.50) (12.51)

pw = 1 ps pt . The mean utilisation of the computers becomes: = ps ns S = . n n

(12.52)

246 The mean waiting time for a terminal becomes: W = pw 1 . ps

CHAPTER 12. DELAY SYSTEMS

(12.53)

Sometimes pw is called the loss coecient of the terminals, and similarly (1 ) is called the loss coecient of the computers (Fig. 12.9).

Example 12.5.4: Numerical example of scale of economy The following numerical examples illustrate that we obtain the highest utilisation for large values of n (and S). Let us consider a system with S/n = 30 and / = 30 for a increasing number of computers (in this case pt = ).

n p p p a W 1

1 0.0289 0.1036 0.8675 0.8675 3.5805

2 0.0300 0.0712 0.8989 0.8989 2.3754

4 0.0307 0.0477 0.9215 0.9215 1.5542

8 0.0313 0.0311 0.9377 0.9377 0.9945

16 0.0316 0.0195 0.9489 0.9489 0.6155

12.6

Optimising the machine-repair model

In this section we optimise the machine/repair model in the same way as Palm did in 1947. We have noticed that the model for a single repair-man is identical with Erlangs loss system, which we optimised in Chap. 7. We will thus see that the same model can be optimised in several ways. We consider a terminal system with one computer and S terminals, and we want to nd an optimal value of S. We assume the following structure of costs: ct = cost per terminal per time unit a terminal is thinking, cw = cost per terminal per time unit a terminal is waiting, cs = cost per terminal per time unit a terminal is served, ca = cost of the computer per time unit.

12.6. OPTIMISING THE MACHINE-REPAIR MODEL 30


. . . . . . . . . . . . . . . . . . . . . . . . . . . . . . . . . . . . . . . . . . . . . . . . . . . . . . . . . . . . . . . . . . . . . . . . . . . . . . . . . . . . . . . . . . . . . .. .. .. .. .. .. .. .. .. .. .. .. .. .. . .. ... .. ........ .. ........ .. ........ ....... ... ... ........ ........ ... ... ........ ........ ... ........ ........ ... ... ........ ........ ... ...... ... ........ .... ........ .... ........ .... ....... ....... .... .... ........ ........ ..... ..... .. ..... ........ ...... ...... ........ . ......... . ........ ......... . .. ........................................... . ... ..... ................................. ...... ... . ... . . .... . . ... ... .. ... . ... . . .... . .. .. ... . ... . . . ... . . . .... . . ... .. ... .. .. ... .. . . ... . .... . . .. ... . ... . .... . .. ... . ... . ... . . ... . . . . .... . .. ... . .... . . ... .... . . . . . . . . . . . . . . .

247

Total costs C0 [100] . .

25

20

15

10

0 0 10 20

30

40 50 60 Number of terminals S

Figure 12.11: The machine/repair model. The total costs given in (12.57) are shown as a function of number of terminals for a service ratio = 25 and a cost ratio r = 1/25 (cf. Fig. 7.6). The cost of the computer is supposed to be independent of the utilisation and is split uniformly among all terminals. The outcome (product) of the process is a certain thinking time at the terminals (production time). The total costs c0 per time unit a terminal is thinking (producing) becomes: p t c0 = p t ct + p s cs + p w cw + We want to minimise c0 . The service ratio ratio r = cw /ca , we get: c0 = ct + 1 ca . S (12.54)

= mt /ms is equal to pt /ps . Introducing the cost


1 S

p w cw + ps cs + pt pt 1 cs + ca

ca

= ct +

r pw + (1/S) , pt

(12.55)

248

CHAPTER 12. DELAY SYSTEMS

which is to be minimised as a function of S. Only the last term depends on the number of terminals and we get: min {c0 } = min
S S

r pw + (1/S) pt r (nw /S) + (1/S) nt /S r nw + 1 nt r [S {1 E1,S ( )} {1 + }] + 1 {1 E1,S ( )} rS+1 {1 E1,S ( )} +1+ 1 , (12.57) (12.56)

= min
S

= min
S

= min
S

= min
S

where E1,S ( ) Is Erlangs B-formula (12.36). We notice that the minimum is independent of ct and cs , and that only the ratio r = cw /ca appears. The numerator corresponds to (7.31), whereas the denominator corresponds to the carried trac in the corresponding loss system. Thus we minimise the cost per carried erlang in the corresponding loss system. In Fig. 12.11 an example is shown. We notice that the result deviates from the result obtained by using Moes Principle for Erlangs loss system (Fig. 7.6), where we optimise the prot.

Chapter 13 Applied Queueing Theory


Till now we have considered classical queueing systems, where all trac processes are birth and death processes. The theory of loss systems has been successfully applied for many years within the eld of telephony, whereas the theory of delay systems has been applied within the eld of data and computer systems. The classical queueing systems play a key role in queueing theory. Usually, we assume that either the inter-arrival time distribution or the service time distribution is exponentially distributed. For theoretical and physical reasons, queueing systems with only one server are often analysed and widely applied. In this chapter we rst concentrate on the single server queue and analyse this system for general service time distributions, various queueing disciplines, and for customers with priorities.

13.1

Classication of queueing models

In this section we shall introduce a compact notations for queueing systems, called Kendalls notation.

13.1.1

Description of trac and structure

D.G. Kendall (1951 [61]) has introduced the following notation for queueing models: A/B/n where A = arrival process, B = service time distribution, 249

250 n = number of servers.

CHAPTER 13. APPLIED QUEUEING THEORY

For trac processes we use the following standard notations (cf. Sec. 4.5): M D Ek Hn Ph GI G Markovian. Exponential time intervals (Poisson arrival process, exponentially distributed service times). Deterministic. Constant time intervals. Erlang-k distributed time intervals (E1 = M). Hyper-exponential of order n distributed time intervals. Phase-type distributed time intervals. General Independent time intervals, renewal arrival process. General. Arbitrary distribution of time intervals (may include correlation).

Cox Cox-distributed time intervals.

Example 13.1.1: Ordinary queueing models M/M/n is a pure delay system with Poisson arrival process, exponentially distributed service times and n servers. It is the classical Erlang delay system (Chap. 12). GI/G/1 is a general delay system with only one server.

The above mentioned notation is widely used in the literature. For a complete specication of a queueing system more information is required: A/B/n/K/S/X where: K S X = = = the total capacity of the system, or only the number of waiting positions, the population size (number of customers), queueing discipline (Sec. 13.1.2).

K = n corresponds to a loss system, which is often denoted as A/B/nLoss. A superscript b on A, respectively B, indicates group arrival (bulk arrival, batch arrival), respectively group service. C (Clocked) may indicate that the system operates in discrete time. Full accessibility is usually assumed.

13.1.2

Queueing strategy: disciplines and organisation

Customers in a queue waiting to be served can be selected for service according to many dierent principles. We rst consider the three classical queueing disciplines:

13.1. CLASSIFICATION OF QUEUEING MODELS

251

FCFS: First Come First Served. It is also called a fair queue or an ordered queue, and this discipline is often preferred in real-life when customers are human beings. It is also denoted as FIFO: First In First Out. Note that FIFO refers to the queue only, not to the total system. If we have more than one server, then a customer with a short service time may overtake a customer with a long waiting time even if we have FIFO queue. LCFS: Last Come First Served. This corresponds to the stack principle. It is for instance used in storages, on shelves of shops etc. This discipline is also denoted as LIFO: Last In First Out. SIRO: Service In Random Order. All customers waiting in the queue have the same probability of being chosen for service. This is also called RANDOM or RS (Random Selection). The rst two disciplines only take arrival times into considerations, while the third does not consider any criteria at all and so does not require any memory (contrary to the rst two). They can be implemented in simple technical systems. Within an electro-mechanical telephone exchange the queueing discipline SIRO was often used as it corresponds (almost) to sequential hunting without homing. For the three above-mentioned disciplines the total waiting time for all customers is the same. The queueing discipline only decides how waiting time is allocated to the individual customers. In a program-controlled queueing system there may be more complicated queueing disciplines. In queueing theory we in general assume that the total oered trac is independent of the queueing discipline. For computer systems we often try to reduce the total waiting time. It can be done by using the service time as criterion: SJF: Shortest Job First (SJN = Shortest Job Next, SPF = Shortest Processing time First). This discipline assumes that we know the service time in advance and it minimises the total waiting time for all customers. The above mentioned disciplines take account of either the arrival times or the service times. A compromise between these disciplines is obtained by the following disciplines: RR: Round Robin. A customer served is given at most a xed service time (time slice or slot). If the service is not completed during this interval, the customer returns to the queue which is FCFS. PS: Processor Sharing. All customers share the service capacity equally.

252

CHAPTER 13. APPLIED QUEUEING THEORY FB: Foreground Background. This discipline tries to implement SJF without knowing the service times in advance. The server will oer service to the customer who so far has received the least amount of service. When all customers have obtained the same amount of service, FB becomes identical with PS.

The last mentioned disciplines are dynamic as the queueing disciplines depend on the amount of time spent in the queue.

13.1.3

Priority of customers

In real life customers are often divided into N priority classes, where a customer belonging to class p has higher priority than a customer belonging to class p+1. We distinguish between two types of priority: Non-preemptive = HOL: A new arriving customer with higher priority than a customer being served waits until a server becomes idle (and all customers with higher priority have been served). This discipline is also called HOL = Head-Of-the-Line. Preemptive: A customer being served having lower priority than a new arriving customer is interrupted. We distinguish between: Preemptive resume = PR: The service is continued from, where it was interrupted, Preemptive without re-sampling: The service restarts from the beginning with the same service time, and Preemptive with re-sampling: The service starts again with a new service time. The two latter disciplines are applied in for example manufacturing systems and reliability. Within a single class, we have the disciplines mentioned in Sec. 13.1.2. In queueing literature we meet many other strategies and symbols. GD denotes an arbitrary queueing discipline (general discipline). The behaviour of customers is also subject to modelling: Balking refers to queueing systems, where customers with a queue dependent probability may give up joining the queue. Reneging refers to systems with impatient customers which depart from the queue without being served.

13.2. GENERAL RESULTS IN THE QUEUEING THEORY

253

Jockeying refers to the systems where the customers may jump from one (e.g. long) queue to another (e.g. shorter) queue. Thus there are many dierent possible models. In this chapter we shall only deal with the most important ones. Usually, we only consider systems with one server.

Example 13.1.2: Stored Program Controlled (SPC) switching system In SPCsystems tasks of the processors are divided into for example ten priority classes. The priority is updated for example every 5th millisecond. Error messages from a processor have the highest priority, whereas routine tasks of control have the lowest priority. Serving accepted calls has higher priority than detection of new call attempts.

13.2

General results in the queueing theory

As mentioned earlier there are many dierent queueing models, but unfortunately there are only few general results in the queueing theory. The literature is very extensive, because many special cases are important in practice. In this section we shall look at the most important general results. Littles theorem presented in Sec. 5.3 is the most general result which is valid for an arbitrary queueing system. The theorem is easy to apply and very useful in many cases. In general only queueing systems with Poisson arrival processes are simple to deal with. Concerning queueing systems in series and queueing networks (e.g. computer networks) it is important to know cases, where the departure process from a queueing system is a Poisson process. These queueing systems are called symmetric queueing systems, because they are symmetric in time, as the arrival process and the departure process are of same type. If we make a lm of the time development, we cannot decide whether the lm is run forward or backward (cf. reversibility) (Kelly, 1979 [60]). The classical queueing models play a key role in the queueing theory, because other systems will often converge to them when the number of servers increases (Palms theorem 6.1 in Sec. 6.4). Systems that deviate most from the classical models are the systems with a single server. However, these systems are also the simplest to deal with. In waiting time systems we also distinguish between call averages and time averages. The virtual waiting time is the waiting time, a customer experiences if the customer arrives at a random point of time (time average). The actual waiting time is the waiting time, the real customers experiences (call average). If the arrival process is a Poisson process, then the two averages are identical.

254

CHAPTER 13. APPLIED QUEUEING THEORY

13.3

Pollaczek-Khintchines formula for M/G/1

We have earlier derived the mean waiting time for M/M/1 (Sec. 12.2.4) and later we consider M/D/1 (Sec. 13.5). In general the mean waiting time for M/G/1 is given by: Theorem 13.1 Pollaczek-Khintchines formula (193032): W = W = as V , 1A As , 2(1 A) (13.1) (13.2)

s = m2 . (13.3) 2 2 W is the mean waiting time for all customers, s is the mean service time, A is the oered trac, and is the form factor of the holding time distribution (3.10). V =A The more regular the service process is, the smaller the mean waiting time will become. The corresponding results for the arrival process is studied in Sec. 13.6. In real telephone trac the form factor will often be 4 6, in data trac 10 100. Formula(13.2) is one of the most important results in queueing theory, and we will study it carefully.

13.3.1

Derivation of Pollaczek-Khintchines formula

We consider the queueing system M/G/1 and we wish to nd the mean waiting time for an arbitrary customer. It is independent of the queueing discipline, and therefore we may in the following assume FCFS. Due to the Poisson arrival process (PASTAproperty) the actual waiting time of a customers is equal to the virtual waiting time. The mean waiting time W for an arbitrary customer can be split up into two parts: 1. The average time it takes for a customer under service to be completed. When the new customer we consider arrives at a random point of time, the residual mean service time given by (3.25): s m1,r = , 2 where s and have the same meaning as in (13.2). When the arrival process is a Poisson process, the probability of nding a customer being served is equal to A because for a single server system we always have p0 = 1 A (oered trac = carried trac).

13.3. POLLACZEK-KHINTCHINES FORMULA FOR M/G/1

255

The contribution to the mean waiting time from a customer under service therefore becomes: s V = (1 A) 0 + A 2 = m2 . 2 (13.4)

2. The waiting time due to waiting customers in the queue (FCFS). On the average the queue length is L. By Littles theorem we have L= W , where L is the average number of customers in the queue at an arbitrary point of time, is the arrival intensity, and W is the mean waiting time which we look for. For every customer in the queue we shall on an average wait s time units. The mean waiting time due to the customers in the queue therefore becomes: Ls = W s =AW . We thus have the total waiting time (13.4) & (13.5): W = V + AW , W = = V 1A As , 2(1 A) (13.5)

which is Pollaczek-Khintchines formula (13.2). W is the mean waiting time for all customers, whereas the mean waiting time for delayed customers w becomes (A = D = the probability of delay) (3.20): W s w= = . (13.6) D 2(1 A) The above-mentioned derivation is correct since the time average is equal to the call average when the arrival process is a Poisson process (PASTAproperty). It is interesting, because it shows how enters into the formula.

13.3.2

Busy period for M/G/1

A busy period of a queueing system is the time interval from the instant all servers become busy until a server becomes idle again. For M/G/1 it is easy to calculate the mean value of a busy period.

256

CHAPTER 13. APPLIED QUEUEING THEORY

At the instant the queueing system becomes empty, it has lost its memory due to the Poisson arrival process. These instants are regeneration points (equilibrium points), and next event occurs according to a Poisson process with intensity . We need only consider a cycle from the instant the server changes state from idle to busy till the next time it changes state from idle to busy. This cycle includes a busy period of duration T1 and an idle period of duration T0 . Fig. 13.1 shows an example with constant service time. The proportion of time the system is busy then becomes: State

Busy
. .......... ......... .. .

h ...................... Time
. . . .. .. . . .. . . . . . . . . . .. .. . . .. . . . . . . . . . .. .. . . .. . . . . . . . . . . . . . . . . .......... ......... . ..

Idle

. . .. .. . . .. . . . . . . . . . . . . . . . .

Arrivals

.................................... .................................... . . .

T1 ............................................................................................................T0 ...............................

Figure 13.1: Example of a sequence of events for the system M/D/1 with busy period T1 and idle period T0 . mT1 mT1 = = A = s. mT0 +T1 mT0 + mT1 From mT0 = 1/, we get: s . 1A During a busy period at least one customer is served. mT1 = (13.7)

13.3.3

Waiting time for M/G/1

If we only consider customers, which are delayed, we are able to nd the moments of the waiting time distribution for the classical queueing disciplines (Abate & Whitt, 1997 [1]). FCFS : Denoting the ith moment of the service time distribution by mi , we can nd the kth moment of the waiting time distribution by the following recursion formula, where the mean service time is chosen as time unit (m1 = s = 1): mk,F A = 1A
k

j=1

k mj+1 mkj,F , j j+1

m0,F = 1 .

(13.8)

13.4. PRIORITY QUEUEING SYSTEMS: M/G/1

257

LCFS : From the above moments mk,F of the FCFSwaiting time distribution we can nd the moments mk,L of the LCFSwaiting time distribution. The three rst moments become: m1,L = m1,F , m2,L = m2,F , 1A m3,L = m3,F + 3 m1,F m2,F . (1A)2 (13.9)

13.3.4

Limited queue length: M/G/1/k

In real systems the queue length, for example the size of a buer, will always be nite. Customers arriving when the buer is full are blocked. For example in the Internet, this strategy is applied in routers and is called the drop tail strategy. There exists a simple relation between the state probabilities p(i) (i = 0, 1, 2, . . .) of the innite system M/G/1 and the state probabilities pk (i), (i = 0, 1, 2, . . . , k) of M/G/1/k, where the total number of positions for customers is k, including the customer being served (Keilson, 1966 [59]): pk (i) = p(i) , (1 A Qk ) (1 A) Qk , (1 A Qk ) i = 0, 1, . . . , k1 , (13.10)

pk (k) =

(13.11)

where A < 1 is the oered trac, and:

Qk =
j=k

p(j) .

(13.12)

There exists algorithms for calculating p(i) for arbitrary holding time distributions (M/G/1) based on imbedded Markov chain analysis (Kendall, 1953 [62]), where the same approach is used for (GI/M/1). We notice that the above is only valid for A < 1, but for a nite buer we also obtain statistical equilibrium for A > 1. In this case we cannot use the approach described in this section. For M/M/1/k we can use the nite state transition diagram, and for M/D/1/k we describe a simple approach in Sec. 13.5.8, which is applicable for general holding time dsitributions.

13.4

Priority queueing systems: M/G/1

The time period a customer is waiting usually means an inconvenience or expense to the customer. By dierent strategies for organising the queue, the waiting times can be distributed among the customers according to our preferences.

258

CHAPTER 13. APPLIED QUEUEING THEORY

13.4.1

Combination of several classes of customers

The customers are divided into N classes (trac streams). Customers of class i are assumed to arrive according to a Poisson process with intensity i [customers per time unit] and the mean service time is si [time units]. The second moment of the service time distribution is denoted m2i , and the oered trac is Ai = i si . In stead of considering the individual arrival processes, we may consider the total arrival process, which also is a Poisson arrival process with intensity:
N

=
i=1

i .

(13.13)

The resulting service time distribution then becomes a weighted sum of service time distributions of the individual classes (Sec. 3.2: combination in parallel). The total mean service time becomes:
N

s=
i=1

i si ,

(13.14)

and the total second moment is:


N

m2 =
i=1

i m2i .

(13.15)

The total oered trac is:


N N

A=
i=1

Ai =
i=1

i si = s .

(13.16)

The remaining mean service time at a random point of time becomes (13.4): V = = 1 m2 2 1 1 A m2 2 s
N

(13.17)

1 A = 2 1 = A 2

i=1 N

i si Ai
1

i=1 N

i m2i

i=1

i=1

i m2i

13.4. PRIORITY QUEUEING SYSTEMS: M/G/1


N

259

=
i=1 N

i m2i 2

(13.18)

=
i=1

Vi .

(13.19)

U(t) Load function

.. . . . .. . ... . . .. . ... . .. . .. . .. . .. . . .. . .. . . .. .. . . .. .. . . .. . .. . .. . .. . . .. .. . . .. . .. . . .. .. . . .. . .. . .. . .. . . .. .. . .. .. . . . .. ... ... . .. . . . . .. . .. . .. . ... .. . ... . . . . .. . .. . .. . .. . ... . ... . .. . . . .. .. .. . . . .. .. . . . .. . . .. . .. .. . . . .. .. . . . . .. .. . . . .. .. . . . . . . .. .. .. .. . . . . . . .. .. . . . .. .. . . . .. . .. . . .. .. . . . . . . .. .. .. . . . .. . . . .. .. . . . . . .. .. . . ... . . . .... . .. .. .. . .. . . . . . . .. . .. .. . . . .... . .. .. . . . . . . . . .. .. .. .. . . .... . .. . . . . .. .. . . . . . .. .. . .. . . . .. .. .. . . . . . .. .. . .. . . . .. . . . .. .. . .. . .. ... . . . .. . . . .. . . . . .. .. . . .. . . . . .. .. . . .. . .. . . . .. . . .. . .. ... . . .. . . . . .. . . . .. . . . ... .. .. . . . . . . .. .. . . . ... .. . . . . . . .. . .. . .. .. . . .... . ... . . . . .. .. . . . .. . .. .. . . . ... . .. .. . . . . .. .. . . . .. . .. . . ... . . .. .. . .. .. . . . . . . . .. . . . .. .. .. .. . .... . . . .. . .. . . . . . .. . .. . .. . . . .. ... .. .. . ... . . . .. .. . . . . . .. . .. .. . . .. . . . . . .. .. . .. . .. ... . . . .. . . .. . .. . . .. . .. .. . . .. . . . .. . . .. . .. . .. . .. . . .. .. . . . . .. . . .. . . .. . . .. . . . . .. .. . . . . .. . . .. . . . . .. .. . . . . .. . . .. . . .. . . .. . . . . ............. ............ . . .

s7

s3

s6

s2

s5

s1

s4

T1

T2

T3

T4 T5

T6

T7

Time

Figure 13.2: The load function U(t) for the queueing system GI/G/1. If we denote the interarrival time Ti+1 Ti by ai , then we have Ui+1 = max{0, Ui + si ai }, where Ui is the value of the load function at time Ti .

13.4.2

Work conserving queueing disciplines

In the following we shall assume that the service time of a customer is independent of the queueing discipline. The capacity of the server is thus constant and independent of for example the length of the queue. The queueing discipline is said to be work conserving. This will not always be the case in practise. If the server is a human being, the service rate will often increase with the length of the queue, and after some time the server may become exhausted and the service rate decreases. We introduce two functions, which are widely applied in the queueing theory.

260

CHAPTER 13. APPLIED QUEUEING THEORY

Load function U(t) denotes the time, it will require to serve the customers, which has arrived to the system at time t (Fig. 13.2). At a time of arrival U(t) increases with a jump equal to the service time of the arriving customer, and between arrivals U(t) decreases linearly with the slope 1 until 0, where it stays until next arrival time. The mean value of the load function is denoted by U = E{U(t)}. In a GI/G/1 queueing system U(t) will be independent of the queueing discipline, if it is work conserving. The virtual waiting time W (t) denotes the waiting time of a customer, if he arrives at time instant t. The virtual waiting time W (t) depends on the queue organisation. The mean value is denoted by W = E{W (t)}. If the queue discipline is FCFS, then U(t) = W (t). When we consider Poisson arrival processes, the virtual waiting time will be equal to the actual waiting time (PASTA property: time average = call average).

We now consider the load function at a random point of time t. It consists of a contribution V from the remaining service time of a customer being served, if any, and a contribution from customers waiting in the queue. The mean value U = E{U(t)} becomes:
N

U =V +
i=1

Li si .

Li is the queue length for customers of type i. By applying Littles law we get:
N

U = V +
i=1 N

i Wi si

= V +
i=1

Ai Wi .

(13.20)

As mentioned above, U is the independent of the queueing discipline (the system is assumed to be work conserving), and V is given by (13.17) for non-preemptive queueing disciplines. U is obtained by assuming FCFS, as we then have Wi = U:
N

U = V +
i=1

Ai U = V + A U ,

U = U V =

V , 1A AV . 1A

(13.21) (13.22)

Under these general assumptions we get by inserting (13.22) into (13.20) Kleinrocks conservation law (1964 [65]):

13.4. PRIORITY QUEUEING SYSTEMS: M/G/1 Theorem 13.2 Kleinrocks conservation law:
N

261

Ai Wi =
i=1

AV = constant. 1A

(13.23)

The average waiting time for all classes weighted by the trac (load) of the mentioned class, is independent of the queue discipline. Notice that the above is only valid for non-preemptive queueing disciplines. We may thus give a small proportion of the trac a very low mean waiting time, without increasing the average waiting time of the remaining customers very much. By various strategies we may allocate waiting times to individual customers according to our preferences.

13.4.3

Non-preemptive queueing discipline

In the following we look at the M/G/1 priority queueing systems, where customers are divided into N priority classes so that a customer with the priority p has higher priority than customers with priority p + 1. In a non-preemptive system a service in progress is not interrupted. The customers in class p are assumed to have the mean service time sp and the arrival intensity p . In Sec. 13.4.1 we derived parameters for the total process. The total average waiting time Wp of a class p customers can be derived directly by considering the following three contributions: a) The residual service time V for the customer under service. b) The waiting time, due to the customers in the queue with priority p or higher, which are already in the queues (Littles theorem):
p

si (i Wi ) .
i=1

c) The waiting time due to customers with higher priority, which overtake the customer we consider while this is waiting:
p1

si i Wp .
i=1

In total we get:
p p1

Wp = V +
i=1

si i Wi +
i=1

si i Wp .

(13.24)

262

CHAPTER 13. APPLIED QUEUEING THEORY

For customers of class 1, which have highest priority we get under the assumption of FCFS: W1 = V + L1 s1 = V + A1 W1 , W1 = V . 1 A1 (13.26) (13.25)

V is the residual service time for the customer under service when the customer we consider arrives (13.18):
N

V =
i=1

i m2i , 2

(13.27)

where m2i is the second moment of the service time distribution of the ith class. For class 2 customers we nd: W2 = V + L1 s1 + L2 s2 + W2 (s1 1 ). Inserting W1 (13.25), we get: W2 = W1 + A2 W2 + A1 W2 , W2 = W2 = W1 , 1 A1 A2 V . {1 A1 } {1 (A1 + A2 )} (13.28)

(13.29)

In general we nd (Cobham, 1954 [14]): Wp = where: Ap =


i=0

V 1 Ap1
p

1 Ap

(13.30)

Ai ,

A0 = 0 .

(13.31)

The structure in formula (13.30) can be directly interpreted. No matter which class all customers wait until the service in progress is completed {V }. Furthermore, the waiting time is due to customers who have already arrived and have at least the same priority Ap , and customers with higher priority arriving during the waiting time Ap1 .

13.4. PRIORITY QUEUEING SYSTEMS: M/G/1

263

Example 13.4.1: SPC-system We consider a computer which serves two types of customers. The rst type has the constant service time of 0.1 second, and the arrival intensity is 1 customer/second. The other type has the exponentially distributed service time with the mean value of 1.6 second and the arrival intensity is 0.5 customer/second. The load from the two types customers is then A1 = 0.1 erlang, respectively A2 = 0.8 erlang. From (13.27) we nd: 1 0.5 V = (0.1)2 + 2 (1.6)2 = 1.2850 s . 2 2 Without any priority the mean waiting time becomes by using Pollaczek-Khintchines formula (13.2): W = By non-preemptive priority we nd: Type 1 highest priority: W1 = W2 = Type 2 highest priority: W2 = 6.43 s , W1 = 64.25 s . This shows that we can upgrade type 1 almost without inuencing type 2. However the inverse is not the case. The constant in the Conservation law (13.23) becomes the same without priority as with nonpreemptive priority: 0.9 12.85 = 0.1 1.43 + 0.8 14.28 = 0.8 6.43 + 0.1 64.25 = 11.57 . 1.285 = 1.43 s , 1 0.1 W1 = 14.28 s . 1 (A1 + A2 ) 1.2850 = 12.85 s . 1 (0.8 + 0.1)

13.4.4

SJF-queueing discipline: M/G/1

By the SJF-queueing discipline the shorter the service time of a customer is, the higher is the priority. By introducing an innite number of priority classes, we obtain from the formula (13.30) that a customer with the service time t has the mean waiting time Wt (Phipps 1956): Wt = V , (1 At )2 (13.32)

264

CHAPTER 13. APPLIED QUEUEING THEORY

where At is load from the customers with service time less than or equal to t. The SJF discipline results in the lowest possible total waiting time. If these dierent priority classes have dierent costs per time unit when they wait, so that class j customers have the mean service time sj and pay cj per time unit when they wait, then the optimal strategy (minimum cost) is to assign priorities 1, 2, . . . according to increasing ratio sj /cj .

Example 13.4.2: M/M/1 with SJF queue discipline We consider the case with exponentially distributed holding times with the mean value 1/ that is chosen as time unit (M/M/1). Even though there are few very long service times, then they contribute signicantly to the total trac (Fig. 3.2). The contribution to the total trac A from the customers with service time t is {(3.22) multiplied by A = }:
t

At =

0 t

x f (x) dx x ex dx

=
0

= A 1 et (t + 1) . Inserting this in (13.32) we nd Wt as illustrated in Fig. 13.3, where the FCFSstrategy (same mean waiting time as LCFS and SIRO) is shown for comparison as function of the actual holding time. The mean waiting time for all customers with SJF is less than with FCFS, but this is not evident from the gure. The mean waiting time for SJF becomes:

WSJF = =

0 0

Wt f (t) dt V f (t) dt (1 At )2 A et dt {1 A (1 et (t + 1))}2

=
0

which it is not elementary to calculate.

13.4. PRIORITY QUEUEING SYSTEMS: M/G/1

265

W [s1 ] Mean Waiting time 100 90 80 70 60 50 40 30 20 10 0

A=0.9

........................................ .................................................. .................................. .......................... ............ ........... ........ ....... ...... ..... ..... ..... . .. .... .... .... .... ... ... ... ... ... ... . .. ... ... .. .. .. .. .. .. .. .. .. .. .. .. .. .. .. .. .. . .. .. .. . .. . .. . .. .. .. . .. . .. . .. . . . . .. . .. . .. . . . . .. . .. . . . . .. . . . . .. . .. . .. . . . . . . . .. . . . . .. . .. . .. . .. . .. . .. .. .. . .. .. .. . .. .. .. .. .. .. .. .. . . .. .. .......................................................................................................................................................................................................................................................... .......................................................................................................................................................................................................................................................... ... . .. ... ... ... ... . . .... .... .... .... ..... ..... ........ ........ .................... ...................

SJF

FCFS

10 11 12 13 14 Actual Service Time t [s1 ]

Figure 13.3: The mean waiting time Wt is a function of the actual service time in a M/M/1 system for SJF and FCFS disciplines, respectively. The oered trac is 0.9 erlang and the mean service time is chosen as time unit. Notice that for SJF the minimum average waiting time is 0.9 time units, because an eventual job being served must rst be nished. The maximum mean waiting time is 90 time units. In comparison with FCFS, by using SJF 93.6 % of the jobs get shorter mean waiting time. This corresponds to jobs with a service time less than 2.747 mean service times (time units). The oered trac may be greater than one erlang, but then only the shorter jobs get a nite waiting time.

266

CHAPTER 13. APPLIED QUEUEING THEORY

13.4.5

M/M/n with non-preemptive priority

We may also generalise Erlangs classical waiting time system M/M/n with nonpreemptive queueing disciplines, when all classes of customers have the same exponentially distributed service time distribution with mean value s = 1 . Denoting the arrival intensity for class i by i , we have the mean waiting time Wp for class p:
p

Wp = V +
i=1

s Li + Wp n
p

p1

i=1

s i , n
p1

Wp

s = E2,n (A) + n

i=1

s i Wi + Wp n

i=1

s i . n

A is the total oered trac for all classes. The probability E2,n (A) for waiting time is given by Erlangs C-formula, and customers are terminated with the mean inter-departure time s/n when all servers are busy. For highest priority class p = 1 we nd: W1 = E2,n (A) s 1 + A1 W1 , n n s . n A1 (13.33)

W1 = E2,n (A) For p = 2 we nd in a similar way: W2 = E2,n (A) = W1 + W2 =

s 1 s 1 + A1 W1 + A2 W2 + W2 1 n n n n

1 1 A2 W2 + A1 W2 , n n (13.34)

n s E2,n (A) . {n A1 } {n (A1 + A2 )} n s E2,n (A) . n Ap1 n Ap

In general we nd (Cobham, 1954 [14]): Wp = (13.35)

13.4.6

Preemptive-resume queueing discipline

We now assume that an ongoing service is interrupted by the arrival of a customer with a higher priority. Later the service continues from where it was interrupted. This situation is typical for computer systems. For a customer with the priority p, the customers with lower priority do no exist. The mean waiting time Wp for a customer in class p consists of two contributions.

13.4. PRIORITY QUEUEING SYSTEMS: M/G/1

267

a) Waiting time due to customers with higher or same priority, who are already in the queueing system. This is the waiting time experienced by a customer in a system without priority where only the rst p classes exists: Vp , 1 Ap
p

where Vp =
i=1

i m2,i , 2

(13.36)

is the expected remaining service time due to customers with a higher or the same priority and Ap is given by (13.31). b) The waiting time due to the customers with higher priority who arrive during the waiting time or service time and overtake the customer considered:
p1

(Wp + sp )
i=1

si i = (Wp + sp ) Ap1 .

We thus get: Wp = This can be rewritten as follows: Wp (1 Ap1 ) = resulting in: Wp = Vp 1 Ap1 1 Ap + Ap1 sp . 1 Ap1 (13.37) Vp + sp Ap1 , 1 Ap Vp + (Wp + sp ) Ap1 . 1 Ap

In the same way as in Sec. 13.4.4 we may write the formula for the average waiting time for the SJFqueueing discipline with preemptive resume. The total response time becomes: Tp = Wp + sp . (13.38)

Example 13.4.3: SPCsystem (cf. example 13.4.1) We now assume the computer system in Example 13.4.1 is working with the discipline preemptiveresume and nd: Type 1 highest priority: W1 = W2 =
1 2 2 (0.1)

1 0.1

+ 0 = 0.0056 s ,

1.2850 0.1 + 1.6 = 14.46 s . (1 0.1)(1 0.9) 1 0.1

268
Type 2 highest priority: W2 = W1 =
1 2

CHAPTER 13. APPLIED QUEUEING THEORY

0.5 2 (1.6)2 + 0 = 6.40 s , 1 0.8

1.2850 0.8 + 0.1 = 64.65 s . (1 0.8)(1 0.9) 1 0.8

This shows that by upgrading type 1 to the highest priority, we can give these customers a very short waiting time, without disturbing type 2 customers, but the inverse is not the case. The conservation law is only valid for preemptive queueing systems if the preempted service times are exponentially distributed. In the general case a job may be preempted several times and therefore the remaining service time will not be given by V .

13.4.7

M/M/n with preemptive-resume priority

For M/M/n the case of preemptive resume is more dicult to deal with. All customers must have the same mean service time. Mean waiting time can be obtained by rst considering class one alone (12.15), then consider class one and two together, which implies the waiting time for class two, etc. The conservation law is valid when all customers have the same exponentially distributed service time.

13.5

Queueing systems with constant holding times

In this section we focus upon the queueing system M/D/n, FCFS. Systems with constant service times have the particular property that the customers leave the servers in the same order in which they are accepted for service.

13.5.1

Historical remarks on M/D/n

Queueing systems with Poisson arrival process and constant service times were the rst systems to be analysed. Intuitively, one would think that it is easier to deal with constant service times than with exponentially distributed service times, but this is denitely not the case. The exponential distribution is easy to deal with due to its lack of memory: the remaining life-time has the same distribution as the total life-time (Sec. 4.1), and therefore we can forget about the epoch (point of time) when the service time starts. Constant holding times require that we remember the exact starting time. Erlang was the rst to analyse M/D/n, FCFS (Brockmeyer & al., 1948 [11]):

13.5. QUEUEING SYSTEMS WITH CONSTANT HOLDING TIMES

269

Erlang: Erlang: Erlang:

1909 n = 1 1917 n = 1, 2, 3

errors for n > 1, without proof,

1920 n arbitrary explicit solutions for n = 1, 2, 3.

Erlang derived the waiting time distribution, but did not consider the state probabilities. Fry (1928 [30]) also dealt with M/D/1 and derived the state probabilities (Frys equations of state) by using Erlangs principle of statistical equilibrium, whereas Erlang himself applied more theoretical methods. Crommelin (1932 [20], 1934 [21]), a British telephone engineer, presented a general solution to M/D/n. He generalised Frys equations of state to an arbitrary n and derived the waiting time distribution, now named Crommelins distribution. Pollaczek (1930-34) presented a very general time-dependent solution for arbitrary service time distributions. Under the assumption of statistical equilibrium he was able to obtain explicit solutions for exponentially distributed and constant service times. Also Khintchine (1932 [63]) dealt with M/D/n and derived the waiting time distribution.

13.5.2

State probabilities of M/D/1

Under the assumption of statistical equilibrium we now derive the state probabilities for M/D/1 in a simple way. The arrival intensity is denoted by and the constant holding time by h. As we consider a pure waiting time system with a single server we have: Oered trac = Carried trac = h < 1 , i.e. A = Y = h = 1 p(0) , as in every state except zero the carried trac is equal to one erlang. We consider two epochs (points of time) t and t + h at a distance of h. Every customer being served at epoch t (at most one) has left the server at epoch t + h. Customers arriving during the interval (t, t + h) are still in the queueing system at epoch t + h (waiting or being served). The arrival process is a Poisson process. Hence we have a Poisson distributed number of arrivals in the time interval (t, t + h): p(j, h) = p{j calls in h} = (h)j h , e j! j = 0, 1, 2 . . . . (13.40) (13.39)

The probability of being in a given state at epoch t + h is obtained from the state at epoch t by taking account of all arrivals and departures during (t, t + h). By looking at these epochs we obtain a Markov Chain embedded in the original trac process (Fig. 13.4).

270
State

CHAPTER 13. APPLIED QUEUEING THEORY


State Arrivals in (t, t+h)

i+2 i+1 i i i1 t+h Time

3 2 1 0

t Arrival Departure Arrival

Figure 13.4: Illustration of Frys equations of state for the queueing system M/D/1. We obtain Frys equations of state for n = 1 (Fry, 1928 [30]):
i+1

pt+h (i) = {pt (0) + pt (1)} p(i, h) +


j=2

pt (j) p(ij +1, h)

(13.41)

Above we found: p(0) = 1 A and under the assumption of statistical equilibrium pt (i) = pt+h (i), we successively nd: p(1) = (1 A) eA 1 , p(2) = (1 A) eA (1 + A) + e2A , and in general:
i

p(i) = (1 A)
j=1

(1)ij ejA

(jA)ij1 (jA)ij + (i j)! (i j 1)!

i = 2, 3, . . .

(13.42)

The last term corresponding to j = i always equals eiA , as (1)! . In principle p(0) can also be obtained by requiring that all state probabilities must add to one.

13.5.3

Mean waiting times and busy period of M/D/1

For a Poison arrival process the probability of delay D is equal to the probability of not being in state zero (PASTA property): D = A = 1 p(0) . (13.43)

13.5. QUEUEING SYSTEMS WITH CONSTANT HOLDING TIMES

271

W denotes the mean waiting time for all customers and w denotes the mean waiting time for customers experiencing a positive waiting time. We have for any queueing system (3.20): w= W . D (13.44)

W and w are easily obtained by using Pollaczek-Khintchines formula (13.2): W = Ah , 2(1 A) h . 2(1 A) (13.45)

w =

(13.46)

The mean value of a busy period was obtained for M/G/1 in (13.7) and illustrated for constant service times in Fig. 13.1: h mT1 = . (13.47) 1A The mean waiting time for delayed customers are thus half the busy period. It looks like customers arrive at random during the busy period, but we know that are no customers arrive during the last service time of a busy period. The distribution of the number of customer arriving during a busy period can be shown to be given by a Borl distribution: e B(i) = (i A)i1 i A e , i! i = 1, 2, . . . (13.48)

13.5.4

Waiting time distribution: M/D/1, FCFS

This can be shown to be:

p{W t} = 1 (1 )
j=1

{(j )}T +j (j ) e , (T + j)!

(13.49)

where h = 1 is chosen as time unit, t = T + , T is an integer, and 0 < 1. The graph of the waiting time distribution has an irregularity every time the waiting time exceeds an integral multiple of the constant holding time. An example is shown in Fig. 13.5. Formula (13.49) is not suitable for numerical evaluation. It can be shown (Iversen, 1982 [39]) that the waiting time can be written in a closed form, as given by Erlang in 1909:
T

p{W t} = (1 )
j=0

{(j t)}j (jt) e , j!

(13.50)

272 1.0 0.5 0.2 0.1 0.05 0.02 0.01 0.005 0.002 0.001

CHAPTER 13. APPLIED QUEUEING THEORY P(W>t) Complementary waiting time distribution A = 0.5
...... ..... ...... ...... ...... .... . .... .. .... .. . ... ..... ... ...... ... ..... ... ..... ... ...... . ... ..... ... ......... ... ..... ..... .. .. ..... ..... .. .. ..... ..... .. ..... .. ..... .. ..... ..... .. ..... ..... .. .. ..... ..... ... ... ..... ..... ... ..... ... ..... ... ..... ..... ... ..... ... ..... ... ..... ..... ... ... ..... ..... ... ... ..... ..... ... ... ..... ..... ..... ... ..... ... ..... ..... .. .. ..... ..... ... .. ..... ..... .. .. ..... ..... ... ... ..... ..... ... ... ..... ..... ... ..... ..... ... ..... ..... ... ... ..... ..... ... ... ..... ..... ... ... ..... ..... ... ..... ..... ... ..... ..... ... ... ..... ..... ... ... ..... ..... ... ... ..... ..... ..... ... ..... ... ..... ..... ... ... ..... ..... ... ... ..... ..... ... ..... ..... ... ..... ..... ... ... ..... ..... ... ... ..... ..... ... ... ... ... ... ... ... ... ... ... ... ... ... ... ... ... ... ... ... ... ... ... ... ... ... ... ... ... ... ... ... ... ... ... ... ... ... ... ... ... ... ... ... ... ... ... ... ... ... ... ... ... ... ... ... ... ... ..

M/M/1

M/D/1

t [s1 ]

Figure 13.5: The complementary waiting time distribution for all customers in the queueing system M/M/1 and M/D/1 for ordered queue (FCFS). Time unit = mean service time. We notice that the mean waiting time for M/D/1 is only half of that for M/M/1. which is t for numerical evaluation for small waiting times. For larger waiting times we are usually only interested in integral values of t. It can be shown (Iversen, 1982 [39]) that for an integral value of t we have: p{W t} = p(0) + p(1) + + p(t) . (13.51)

The state probabilities p(i) are calculated most accurately by using a recursive formula based on Frys equations of state (13.42): 1 p(i + 1) = p(0, h)
i

p(i) {p(0) + p(1)} p(i, h)


j=2

p(j) p(ij +1, h)

(13.52)

For non-integral waiting-times we are able to express the waiting time distribution in terms of integral waiting times. If we let h = 1, then (13.50) may be a binomial expansion be written in powers of , where t = T + , T integer, 0 < 1 .

13.5. QUEUEING SYSTEMS WITH CONSTANT HOLDING TIMES We nd:


T

273

p{W T + } = e

j=0

( )j p{W T j} , j!

(13.53)

where p{W T j} is given by (13.51). The numerical evaluation is very accurate when using (13.51), (13.52) and (13.53).

13.5.5

State probabilities: M/D/n

When setting up Frys equations of state (13.41) we obtain more combinations:


n n+i

pt+h (i) =
j=0

pt (j) p(i, h) +
j=n+1

pt (j) p(n + i j, h) .

(13.54)

On the assumption of statistical equilibrium (A < n) we can leave the absolute points of time out of account:
n n+i

p(i) =
j=0

p(j) p(i, h) +
j=n+1

p(j) p(in + i j, h),

i = 0, 1, . . .

(13.55)

The system of equations (13.55) can only be solved directly by substitution, if we know the rst n state probabilities {p(0), p(1), . . . , p(n1)}. In practice we may obtain numerical values by guessing an approximate set of values for {p(0), p(1), . . . , p(n1)}, then substitute these values in the recursion formula (13.55) and obtain new values. After a few approximations we obtain the exact values. The explicit mathematical solution is obtained by means of generating functions (The Erlang book, [11] pp. 7583).

13.5.6

Waiting time distribution: M/D/n, FCFS

The waiting time distribution is given by Crommelins distribution:


n1 i

p{W t} = 1
i=0 k=0

p(k)
j=1

{A(j )}(T +j+1)n1i , {(T + j + 1)n 1 i}!

(13.56)

where A is the oered trac and t = T h + , 0 < h. (13.57)

274

CHAPTER 13. APPLIED QUEUEING THEORY

Formula (13.56) can be written in a closed form in analogy with (13.50):


n1 i T

p{W t} =
i=0 k=0

p(k)
j=0

{A(j t)}jn+n1i A(jt) . e {j n + n 1 i}!

(13.58)

For integral values of the waiting time t we have:


n(t+1)1

p{W t} =
j=0

p(j) .

(13.59)

For non-integral waiting times t = T + , T integer, 0 < 1, we are able to express the waiting time distribution in terms of integral waiting times as for M/D/1:
k

p{W t} = p{W T + } = e
j=0

( )j j!

kj

p(i)
i=0

(13.60)

where k = n(T + 1)1 and p(i) is the state probability (13.55). The exact mean waiting time of all customers W is dicult to derive. An approximation was given by Molina: n+1 1 A n h n W E2,n (A) (13.61) n . n+1 nA 1 A n For any queueing system with innite queue we have (3.20): w= where for all values of n: D =1
j=0

W , D
n1

p(j) .

13.5.7

Erlang-k arrival process: Ek /D/r

Let us consider a queueing system with n = r k servers (r, k integers), general arrival process GI, constant service time and ordered (FCFS) queueing discipline. Customers arriving during idle periods choose servers in cyclic order 1, 2, . . . , n 1, n, 1, 2, . . . Then a certain server will serve just every n th customers as the customers due to the constant service time depart from the servers in the same order as they arrive at the servers. No customer can overtake another customer.

13.5. QUEUEING SYSTEMS WITH CONSTANT HOLDING TIMES A group of servers made up from the servers x, x + k, x + 2 k, . . . , x + (r 1) k , 0 < x k.

275

(13.62)

will serve just every k th customer. If we consider the servers (13.62), then considered as a single group they are equivalent to the queueing system GI k/D/r, where the arrival process GI k is a convolution of the arrival time distribution by itself k times. The same goes for the k1 other systems. The trac in these k systems is mutually correlated, but if we only consider one system at a time, then this is a GI k/D/n, FCFS queueing system. The assumption about cyclic hunting of the servers is not necessary within the individual systems (13.62). State probabilities, mean waiting times etc. are independent of the queueing discipline, which is of importance for the waiting time distribution only. If we let the arrival process GI be a Poisson process, then GI k becomes an Erlang-k arrival process. We thus nd that the following systems are equivalent with respect to the waiting time distribution: M/D/rk, FCFS Ek /D/r, FCFS .

Ek /D/r may therefore be dealt with by tables for M/D/n.


Example 13.5.1: Regular arrival processes In general we know that for a given trac per server the mean waiting time decreases when the number of servers increases (economy of scale, convexity). For the same reason the mean waiting time decreases when the arrival process becomes more regular. This is seen directly from the above decomposition, where the arrival process for Ek /D/r becomes more regular for increasing k (r constant). For A = 0.9 erlang per server (L = mean queue length) we nd: E4 /E1 /2: E4 /E2 /2: E4 /E3 /2: E4 /D/2: L = 4.5174 , L = 2.6607 , L = 2.0493 , L = 0.8100 .

13.5.8

Finite queue system: M/D/1/k

In real systems we always have a nite queue. In computer systems the size of the storage is nite and in ATM systems we have nite buers. The same goes for waiting positions in FMS (Flexible Manufacturing Systems). As mentioned in Sec. 13.3.4 the state probabilities pk (i) of the nite buer system are obtained from the state probabilities p(i) of the innite buer system by using (13.10) & (13.11).

276

CHAPTER 13. APPLIED QUEUEING THEORY

Integral waiting times are obtained from the state probabilities, and non-integral waiting times from integral waiting times as shown above (Sec. 13.5.4). For the innite buer system the state probabbilities only exist when the oered trac is less than the capacity (A < n). But for a nite buer system the state probabilities also exist for A > n, but we cannot obtain them by the above-mentioned method. For M/D/1/k the nite buer state probabilities pk (i) can be obtained for any oered trac in the following way. In a system with one server and (k1) queueing positions we have (k+1) states (0, 1, , k). The balance equations for state probabilities pk (i), i = 0, 1, . . . , k 2, yielding k 1 linear equations between the states {pk (0), pk (1), . . . , pk (k 1)} can be set up using Frys equations of state. But it is not possible to write down simple time-independent equations for state k1 and k. However, the rst (k 1) equations (13.41) together with the normalisation requirement
k

pk (j) = 1
j=0

(13.63)

and the fact that the oered trac equals the carried trac plus the rejected trac (PASTA property): A = 1 pk (0) + A pk (k) (13.64) results in (k + 1) independent linear equations, which are easy to solve numerically. The two approaches yields of course the same result. The rst method is only valid for A < 1, whereas the second is valid for any oered trac.

Example 13.5.2: Leaky Bucket Leaky Bucket is a mechanism for control of cell (packet) arrival processes from a user (source) in an ATM system. The mechanism corresponds to a queueing system with constant service time (cell size) and a nite buer. If the arrival process is a Poisson process, then we have an M/D/1/k system. The size of the leak corresponds to the long-term average acceptable arrival intensity, whereas the size of the bucket describes the excess (burst) allowed. The mechanism operates as a virtual queueing system, where the cells either are accepted immediately or are rejected according to the value of a counter which is the integral value of the load function (Fig. 13.2). In a contract between the user and the network an agreement is made on the size of the leak and the size of the bucket. On this basis the network is able to guarantee a certain grade-of-service.

13.6

Single server queueing system: GI/G/1

In Sec. 13.3 we showed that the mean waiting time for all customers in queueing system M/G/1 is given by Pollaczek-Khintchines formula: W = As 2(1 A) (13.65)

13.6. SINGLE SERVER QUEUEING SYSTEM: GI/G/1 where is the form factor of the holding time distribution. We have earlier analysed the following cases: M/M/1 (Sec. 12.2.4): = 2: W = M/D/1 (Sec. 13.5.3): = 1: W = As , 2(1 A) Erlang 1909. As , (1 A) Erlang 1917.

277

(13.66)

(13.67)

It shows that the more regular the holding time distribution, the less becomes the waiting time trac. (For loss systems with limited accessibility it is the opposite way: the bigger form factor, the less congestion). In systems with non-Poisson arrivals, moments of higher order will also inuence the mean waiting time.

13.6.1

General results

We have till now assumed that the arrival process is a Poisson process. For other arrival processes it is seldom possible to nd an exact expression for the mean waiting time except in the case where the holding times are exponentially distributed. In general we may require, that either the arrival process or the service process should be Markovian. Till now there is no general accurate formulae for e.g. M/G/n. For GI/G/1 it is possible to give theoretical upper limits for the mean waiting time. Denoting the variance of the inter-arrival times by va and the variance of the holding time distribution by vd , Kingmans inequality (1961) gives an upper limit for the mean waiting time: GI/G/1: W As 2(1 A) va + vd s2 . (13.68)

This formula shows that it is the stochastic variations, that results in waiting times. Formula (13.68) gives the upper theoretical boundary. A realistic estimate of the actual mean waiting time is obtained by Marchals approximation (Marchal, 1976 [77]): As W 2(1 A) va + vd s2 s2 + vd a2 + vd . (13.69)

where a is the mean inter-arrival time (A = s/a). The approximation is a scaling of Kingmans inequality so it agrees with the Pollaczek-Khintchines formula for the case M/G/1.

278

CHAPTER 13. APPLIED QUEUEING THEORY

13.6.2

State probabilities: GI/M/1

As an example of a non-Poisson arrival process we shall analyse the queueing system GI/M/1, where the distribution of the inter-arrival times is a general distribution given by the density function f (t). Service times are exponentially distributed with rate . If the system is considered at an arbitrary point of time, then the state probabilities will not be described by a Markov process, because the probability of an arrival will depend on the time interval since the last arrival. The PASTA property is not valid. However, if the system is considered immediately before (or after) an arrival epoch, then there will be independence in the trac process since the inter-arrival times are stochastic independent the holding times are exponentially distributed. The arrival epochs are equilibrium points (regeneration points, Sec. 5.2.2), and we consider the so-called embedded Markov chain. The probability that we immediately before an arrival epoch observe the system in state j is denoted by (j). In statistical equilibrium it can be shown that we will have the following result (D.G. Kendall, 1953 [62]): (i) = (1 )i ,

i = 0, 1, 2, . . .

(13.70)

where is the positive real root satisfying the equation: =


0

e(1)t f (t) dt .

(13.71)

The steady state probabilities can be obtained by considering two successive arrival epochs t1 and t2 (similar to Frys state equations, Sec. 13.5.5). As the departure process is a Poisson process with the constant intensity when there are customers in the system, then the probability p(j) that j customers complete service between two arrival epochs can be expressed by the number of events in a Poisson process during a stochastic interval (the inter-arrival time). We can set up the following state equations:
j

t2 (0) =
j=0

t1 (j)

1
i=0

p(i)

t2 (1) =
j=0

t1 (j) p(j) , . . .

(13.72)

. . . t2 (i) =

t1 (j) p(j i+1) .


j=0

13.6. SINGLE SERVER QUEUEING SYSTEM: GI/G/1 The normalisation condition is as usual:

279

t1 (i) =
i=0 j=0

t2 (j) = 1 .

(13.73)

It can be shown that the above-mentioned geometric distribution is the only solution to this system of equations (Kendall, 1953 [62]). In principle, the queueing system GI/M/n can be solved in the same way. The state probability p(j) becomes more complicated since the departure rate depends on the number of busy channels. Notice that (i) is not the probability of nding the system in state i at an arbitrary point of time (time average), but the probability of nding the system in state i immediately before an arrival (call average).

13.6.3

Characteristics of GI/M/1

The probability of immediate service becomes: p{immediate} = (0) = 1 . The corresponding probability of being delayed the becomes: D = p{delay} = . (13.75) (13.74)

The average number of busy servers at a random point of time (time average) is equal to the carried trac (= the oered trac A < 1). The average number of waiting customers, immediately before the arrival of a customer, is obtained via the state probabilities:

L1 =
i=1

(1 ) i (i 1) , 2 . 1

L1 =

(13.76)

The average number of customers in the system before an arrival epoch is: L2 =
i=0

(1 ) i i . 1

(13.77)

280

CHAPTER 13. APPLIED QUEUEING THEORY

The average waiting time for all customers then becomes: W = 1 . 1 (13.78)

The average queue length taken over the whole time axis (the virtual queue length) therefore becomes (Littles theorem): . (13.79) L=A 1 The mean waiting time for the customers, who obtain waiting times, becomes w = w = W , D 1 1 . 1 (13.80)

Example 13.6.1: Mean waiting times GI/M/1 For M/M/1 we nd = m = A. For D/M/1 = d is obtained from the equation: d = e (1 d ) /A , where d must be within (0,1). It can be shown that 0 < d < m < 1 . Thus the queueing system D/M/1 will always have less mean waiting time than M/M/1. For A = 0.5 erlang we nd the following mean waiting times for all customers (13.78): M/M/1: D/M/1: = 0.5 , = 0.2032 , W = 1, W = 0.2550 , w = 2. w = 1.3423 .

where the mean holding time is used as the time unit ( = 1). The mean waiting time is thus far from proportional with the form factor of the distribution of the inter-arrival time.

13.6.4

Waiting time distribution: GI/M/1, FCFS

When a customer arrives at the queueing system, the number of customers in the system is geometric distributed, and the customer therefore, under the assumption that he gets a positive waiting time, has to wait a geometrically distributed number of exponential phases. This will result in an exponentially distributed waiting time with a parameter given in (13.80), when the queueing discipline is FCFS (Sec. 12.4 and Fig. 4.9).

13.7. ROUND ROBIN AND PROCESSOR-SHARING

281

13.7

Round Robin and Processor-Sharing

The Round Robin (RR) queueing model (Fig. 13.6) is a model for a time-sharing computer system, where we wish a fast response time for the shortest jobs. This queueing discipline is also called fair queueing because the available resources are equally distributed among the jobs (customers) in the system.
. . . ...................................................................................................................................... . . . ..................................................................................................................................... . . . . . . . . . . . . . . . . . . . . . . . . . . . . . . . . . . . . . . . . . . . . . . . . . . . . . . .. . . . .. . .. . . . . . . . . . . .. . .. . . ... . . . . . . . . . . . . . . . . . . . . . . . . . . . . . . . . . . . . . . ............ . ............. . . . . .. . .. . .. .. . . ........................... .. . . ............................ . . . . . . . . . . . . ........................................................ . .. . . . ....................................................... ............................. ............................ .. . . . . . . . .. . . . .. . .............................................. ............................................. . .. .. . .. ... ... ... ... ......... ........

Non-completed jobs

1p

New jobs

Queue

CPU

Completed jobs

Figure 13.6: Round robin queueing system. A task is allocated a time slice s (at most) every time it is served. If the task is not nished during this time slice, it is returned to a FCFS queue, where it waits on equal terms with new tasks. If we let s decrease to zero we obtain the queueing discipline PS (Processor Sharing). New time time This jobs are placed in a FCFSqueue, where they wait until they obtain service within a slice (slot) s which is the same for all jobs. If a job does is not completed within a slice, the service is interrupted, and the job is placed at the end of the FCFSqueue. continues until the required total service time is fullled.

We assume that the queue is unlimited, and that new jobs arrive according to a Poisson process (). The service time distribution can be general with the mean value s. The time slice can vary. If it becomes innite, all jobs will be completed the rst time, and we have simply an M/G/1 queueing system with FCFS discipline. If we let the time slice decrease to zero, then we get the PS = Processor-Sharing model, which has a number of nice analytical properties. The PSwas introduced by Kleinrock (1967) and is dealt with in detail in (Kleinrock, 1976 [67]). The Processor-Sharing model can be interpreted as a queueing system where all jobs are served continuously by the server (time sharing). If there are i jobs in the system, each of them obtain the fraction 1/i of the capacity of the computer. So there is no queue, and the queueing discipline is meaningless. When the oered trac A = s is less than one, it can be shown that the steady state probabilities are given by: p(i) = (1 A) Ai , i = 0, 1, . . . , (13.81)

282

CHAPTER 13. APPLIED QUEUEING THEORY

i.e. a geometric distribution with the mean value A/(1 A). The mean holding time (average response time) for the jobs with duration t becomes: Rt = t . 1A (13.82)

If this job was alone in the system, then its holding time would be t. Since there is no queue, we can then talk about an average delay for jobs with duration t: Wt = Rt t = A t. 1A (13.83)

The corresponding mean values for a random job naturally becomes: R = W = s , 1A A s. 1A (13.84) (13.85)

This shows that we obtain exactly the same mean values as for M/M/1 (Sec. 12.2.4). But the actual mean waiting time becomes proportional to the duration of the job, which is often a desirable property. We dont assume any knowledge in advance about the duration of the job. The mean waiting time becomes proportional to the mean service time. The proportionality should not be understood in the way that two jobs of the same duration have the same waiting time; it is only valid on the average. In comparison with the results we have earlier obtained for M/G/1 (Pollaczek-Khintchines formula (13.2)) the results may surprise the intuition. A very useful property of the Processor-Sharing model is that the departure process is a Poisson process as the arrival process (Sec. 14.2). It is intuitively explained by the fact that the departure process is obtained from the arrival process by a stochastic shifting of the individual arrival epochs. The time shift is equal to the response time with a mean value given by (13.82) (Sec. 6.3.1, Palms theorem). The Processor-Sharing model is very useful for analysing time-sharing systems and for modelling queueing networks (Chap. 14).

Chapter 14 Networks of queues


Many systems can be modelled in such a way that a customer achieves services from several successive nodes, i.e. once he has obtained service at one node, then he goes on to another node. The total service demand is composed of service demands at several nodes. Hence, the system is a network of queues, a queueing network where each individual queue is called a node. Examples of queueing networks are telecommunication systems, computer systems, packet switching networks, and FMS (Flexible Manufacturing Systems). In queueing networks we dene the queue-length in a node as the total number of customers in the node, including customers being served. The aim of this chapter is to introduce the basic theory of queueing networks, illustrated by applications. Usually, the theory is considered as being rather complicated, which is mainly due to the complex notation. However, in this chapter we will give a simple introduction to general analytical queueing network models based on product forms, the convolution algorithm, the MVAalgorithm, and examples. The theory of queueing networks is analogous to the theory of multidimensional loss systems (Chap. 10 & 11). In Chap. 10 we considered multi-dimensional loss systems whereas in this chapter we are looking at networks of queueing systems.

14.1

Introduction to queueing networks

Queueing networks are classied as closed and open queueing networks. In closed queueing networks the number of customers is xed whereas in open queueing networks the number of customers is varying. In principle, an open network can be transformed into a closed network by adding an extra node. 283

284

CHAPTER 14. NETWORKS OF QUEUES

Erlangs classical waiting system, M/M/n, is an example of an open queueing system, whereas Palms machine/repair model with S terminals is a closed network. If there is more than one type of customers, the network can be a mixed closed and open network. Since the departure process from one node is the arrival process at another node, we shall pay special attention to the departure process, in particular when it can modelled as a Poisson process. This is investigated in the section on symmetric queueing systems (Sec. 14.2). The state of a queueing network is dened as the simultaneous distribution of number of customers in each node. If K denotes the total number of nodes, then the state is described by a vector p(i1 , i2 , . . . , iK ) where ik is the number of customers in node k (k = 1, 2, . . . , K). Frequently, the state space is very large and it is dicult to calculate the state probabilities by solving node balance equations. If every node is a symmetric queueing system, for example a Jackson network (Sec. 14.3), then we will have product form. The state probabilities of networks with product form can be aggregated and obtained by using the convolution algorithm (Sec. 14.4.1) or the MVAalgorithm (Sec. 14.4.2). Jackson networks can be generalised to BCMPnetworks (Sec. 14.5), where there are N types of customers. Customers of one specic type all belongs to a so-called chain. Fig. 14.1 illustrates an example of a queueing network with 4 chains. When the number of chains increases the state space increases correspondingly, and only systems with a small number of chains can be calculated exactly. In case of a multi-chain network, the state of each node becomes multi-dimensional (Sec. 14.6). The product form between nodes is maintained, and the convolution and the MVAalgorithm are applicable (Sec. 14.7). A number of approximate algorithms for large networks can found in the literature.

Figure 14.1: An example of a queueing network with four open chains.

14.2

Symmetric queueing systems

In order to analyse queueing systems, it is important to know when the departure process of a queueing system is a Poisson process. Four queueing models are known to have this

14.2. SYMMETRIC QUEUEING SYSTEMS property:

285

1. M/M/n. This is Burkes theorem (Burke, 1956 [12]), which states, that the departure process of an M/M/nsystem is a Poisson process. The state space probabilities are given by (12.2): i A 0 i n, i! p(0) , p(i) = (14.1) A in p(n) , i n. n where A = /. 2. M/G/. This corresponds to the Poisson case (Sec. 7.2). From Sec. 6.3 we know that a random translation of the events of a Poisson process results in a new Poisson process. This model is sometimes denoted as a system with the queueing discipline IS, Innite number of Servers. The state probabilities are given by the Poisson distribution (7.6): p(i) = Ai A e , i! i = 0, 1, 2, . . . . (14.3)

3. M/G/1PS. This is a single server queueing system with a general service time distribution and processor sharing. The state probabilities are similar to the M/M/1 case (13.81): p(i) = (1 A) Ai , i = 0, 1, 2, . . . . (14.4)

4. M/G/1LCFS-PR (PR = Preemptive Resume). This system also has the same state space probabilities as M/M/1 (14.4).

In the theory of queueing networks usually only these four queueing disciplines are considered. But for example also for Erlangs loss system, the departure process will be a Poisson process, if we include blocked customers. The above-mentioned four queueing systems are called symmetric queueing systems as they are symmetric in time. Both the arrival process and the departure process are Poisson processes and the systems are reversible (Kelly, 1979 [60]). The process is called reversible because it looks the same way when we reverse the time (cf. when a lm is reversible it looks the same whether we play it forward or backward). Apart from M/M/n these symmetric queueing systems have the common feature that a customer is served immediately upon arrival. In the following we mainly consider M/M/n nodes, but the M/M/1 model also includes M/G/1PS and M/G/1LCFSPR.

286

CHAPTER 14. NETWORKS OF QUEUES

14.3

Jacksons theorem

In 1957, J.R. Jackson who was working with production planning and manufacturing systems, published a paper with a theorem, now called Jacksons theorem (1957 [46]). He showed that a queueing network of M/M/n nodes has product form. Knowing the fundamental theorem of Burke (1956 [12]) Jacksons result is obvious. Historically, the rst paper on queueing systems in series was by R.R.P. Jackson (1954 [45]). Theorem 14.1 Jacksons theorem: Consider an open queueing network with K nodes satisfying the following conditions: a) Each node is an M/M/nqueueing system. Node k has nk servers, and the average service time is 1/k . b) Customers arrive from outside the system to node k according to a Poisson process with intensity k . Customers may also arrive from other nodes to node k. c) A customer, who has just nished his service at node j, immediately transfers to node k with probability pjk or leaves the network with probability:
K

1
k=1

pjk .

A customer may visit the same node several times if pkk > 0. The average arrival intensity k at node k is obtained by looking at the ow balance equations:
K

k = k +
j=1

j pjk .

(14.5)

Let p(i1 , i2 , . . . , iK ) denote the state space probabilities under the assumption of statistical equilibrium, i.e. the probability that there is ik customers at node k. Furthermore, we assume that k = Ak < nk . (14.6) k Then the state space probabilities are given on product form:
K

p (i1 , i2 , . . . , iK ) =
k=1

pk (ik ) .

(14.7)

Here for node k, pk (ik ) is the state probabilities of an M/M/n queueing system with arrival intensity k and service rate k (14.1). The oered trac k /k to node k must be less than

14.3. JACKSONS THEOREM

287

the capacity nk of the node to enter statistical equilibrium (14.6). The key point of Jacksons theorem is that each node can be considered independently of all other nodes and that the state probabilities are given by Erlangs Cformula. This simplies the calculation of the state space probabilities signicantly. The proof of the theorem was derived by Jackson in 1957 by showing that the solution satisfy the balance equations for statistical equilibrium. Jacksons rst model thus only deals with open queueing networks. In Jacksons second model (Jackson, 1963 [47]) the arrival intensity from outside:
K

=
j=1

(14.8)

may depend on the current number of customers in the network. Furthermore, k can depend on the number of customers at node k. In this way, we can model queueing networks which are either closed, open, or mixed. In all three cases, the state probabilities have product form. The model by Gordon & Newell (1967 [31]), which is often cited in the literature, can be treated as a special case of Jacksons second model.
............................ ............................ .. . . .. ............. ............. .. .. .. .. .. . . . . ............................ . . . .......................... ................... ................... .. . . .. . . . . . . .. . . . . .. .. .. .. ... ... ... ............ ......... ..... ............. .... ..... .. .. .. .. . . . . . . . ... . . .................... ................... ................... ... . . . . . . . . . . .................. . . . .. .. .. .. . ... ... ... ... ......... ........

Figure 14.2: State transition diagram of an open queueing network consisting of two M/M/1 systems in series.
Example 14.3.1: Two M/M/1 nodes in series Fig. 14.2 shows an open queueing network of two M/M/1 nodes in series. The corresponding state transition diagram is given in Fig. 14.3. Clearly, the state transition diagram is not reversible: (between two neighbour states there is only ow in one direction, (cf. Sec. 10.2) and apparently there is no product form. If we solve the balance equations to obtain the state probabilities we nd that the solution can be written on a product form: p(i, j) = p(i) p(j) , p(i, j) = (1 A1 ) Ai (1 A2 ) Aj 1 2 ,

where A1 = /1 and A2 = /2 . The state probabilities can be expressed in a product form p(i, j) = p(i) p(j), where p(i) is the state probabilities for a M/M/1 system with oered trac A1 and p(j) is the state probabilities for a M/M/1 system with oered trac A2 . The state probabilities of Fig. 14.3 are identical to those of Fig. 14.4, which has local balance and product form. Thus it is possible to nd a system which is reversible and has the same state probabilities as the nonreversible system. There is regional but not local balance in Fig. 14.3. If we consider a square of four states, then to the outside world there will be balance, but internally there will be circulation via the diagonal state shift.

288

CHAPTER 14. NETWORKS OF QUEUES

In queueing networks customers will often be looping, so that a customer may visit the same node several times. If we have a queueing network with looping customers, where the nodes are M/M/nsystems, then the arrival processes to the individual nodes are no more Poisson processes. Anyway, we may calculate the state probabilities as if the individual nodes are independent M/M/n systems. This is explained in the following example.

Example 14.3.2: Networks with feed back Feedback is introduced in Example 14.3.1 by letting a customer, which has just ended its service at node 2, return to node 1 with probability p21 . With probability 1 p21 the customer leaves the system. The ow balance equations (14.5) gives the total arrival intensity to each node and p21 must be chosen such that both 1 /1 and 2 /2 are less than one. Letting 1 0 and p21 1 we realise that the arrival processes are not Poisson processes: only rarely a new customer will arrive, but once he has entered the system he will circulate for a relatively long time. The number of circulations will be geometrically distributed and the inter-arrival time is the sum of the two service times. I.e. when there is one (or more) customers in the system, then the arrival rate to each node will be relatively high, whereas the rate will be very low if there is no customers in the system. The arrival process will be bursty. The situation is similar to the decomposition of an exponential distribution into a weighted sum of Erlang-k distributions, with geometrical weight factors (Sec. 4.4). Instead of considering a single exponential inter-arrival distribution we can decompose this into k phases (Fig. 4.9) and consider each phase as an arrival. Hence, the arrival process has been transformed from a Poisson process to a process with bursty arrivals.

03 2 02 2 01 2 00

1 1 1

13 2 12 2 11 2 10

1 1 1

23 2 22 2 21 2 20

1 1 1

33 2 32 2 31 2 30

1 1 1

43 2 42 2 41 2 40

Figure 14.3: State transition diagram for the open queueing network shown in Fig. 14.2. The diagram is nonreversible.

14.4. SINGLE CHAIN QUEUEING NETWORKS


03

289
33

 1  2  1  2  1  2  1

13

 1  2  1  2  1  2  1

23

 1  2  1  2  1  2  1

 1  2  1  2  1  2  1

43

2
02


42

12

22

32

2
01


41

11

21

31

2
00


40

10

20

30

Figure 14.4: State transition diagram for two independent M/M/1queueing systems with identical arrival intensity, but individual mean service times. The diagram is reversible.

14.3.1

Kleinrocks independence assumption

If we consider a real-life data network, then the packets will have the same constant length, and therefore the same service time on all links and nodes of equal speed. The theory of queueing networks assumes that a packet (a customer) samples a new service time in every node. This is a necessary assumption for the product form. This assumption was rst investigated by Kleinrock (1964 [65]), and it turns out to be a good approximation in praxis.

14.4

Single chain queueing networks

We are interested in the state probabilities dened by p(i1 , i2 , . . . , ik , . . . , iK ), where ik is the number of customers in node k (1 k K). Dealing with open systems is easy. First we solve the ow balance equation (14.5) and obtain the aggregated arrival intensity to each node (k ). Combining the arrival intensities with the service time distribution (k ) we get the oered trac Ak at each node and then by considering Erlangs delay system we get the state probabilities for each node.

290

CHAPTER 14. NETWORKS OF QUEUES

14.4.1

Convolution algorithm for a closed queueing network

Dealing with closed queueing networks is much more complicated. We only know the relative load at each node, not the absolute load, i.e. c j is obtained, but c is unknown. We can obtain the unnormalised relative state probabilities. Finally, by normalising we get the normalised state probabilities. Unfortunately, the normalisation implies that we must sum up all state probabilities, i.e. we must calculate each (unnormalised) state probability. The number of states increases rapidly when the number of nodes and/or customers increases. In general, it is only possible to deal with small systems. The complexity is similar to that of multi dimensional loss systems (Chapter 10). We will now show how the convolution algorithm can be applied to queueing networks. The algorithm corresponds to the convolution algorithm for loss systems (Chapter 10). We consider a queueing network with K nodes and a single chain with S customers. We assume that the queueing systems in each node are symmetric (Sec. 14.2). The algorithm has three steps: Step 1. Let the arrival intensity to an arbitrary chosen reference node i be equal to some value i . By solving the ow balance equation (14.5) for the closed network we obtain the relative arrival rates k (1 k K) to all nodes. Finally, we have the relative oered trac values k = k /k . Often we choose the above arrival intensity of the reference node so that the oered trac to this node becomes one. Step 2. Consider each node as if it is isolated and has the oered trac k (1 k K). Depending on the actual symmetric queueing system at node k, we derive the relative state probabilities qk (i) at node k. The state space will be limited by the total number of customers S, i.e. 0 i S. Step 3. Convolve the state probabilities for each node recursively. For example, for the rst two nodes we have: q12 = q1 q2 , (14.9) where q12 (i) =
x=0 i

q1 (x) q2 (i x),

i = 0, 1, . . . , S .

When all nodes have been convolved we get: q1,2,...,K = q1,2,...,K1 qK . (14.10)

Since the total number of customers is xed (S) only state q1,2,...,K (S) exists in the aggregated system and therefore this macro-state must have the probability one. We can then normalise all micro-state probabilities. When we perform the last convolution we can derive the performance measures for the last node. By changing the order of convolution of the nodes we can obtain the performance measures of all nodes.

14.4. SINGLE CHAIN QUEUEING NETWORKS

291

. .. ....................................................................................................................................................................................................................... . . . .............................................................................................................. ....................................................................................................... . . . . . . . . . . . . . . . . . . . . . .......... ........... . . . . . .. ... .. . . ... . .. . . . . . .. . . . . . . . ............... .. ...... . ...... .................... . ..... ... . ..... . ...... . ...... . . . . . . . . . . . . . . . . .. . . . .. . . . .. . . . .. . . .. . .. . . ... .. .... . .. . .. . . .......... .. . . ........ .... .... .... .... .... .... .... .... .... .... .... .... .... .... .... ... ... ... ... ... ... ... ... ... ... ... ... ... ... ... ... ... . . .. . . .. . .. . . . .. . . . .. .. . . . .. . . . . . . . .. . . . .. . . . . . .. . . .... . .. . . . . ....... ..... . . . .... ...... . . .. . . . .. . .. . . .. . . . .. .. . .. . .. . . . . . . . . . . . . .. . ...... . . . . .. . . .. . . .................. . . . . ............. ............ . . .................... . . .. .. . . .. . ...... . . .. .... . . . .... . . . . . .. . .. . . .... .. .. . . . . .... .. . . . .. . .. . . .... ... . . .. ... . .... . . . ... . . ... ......... .... ........ .. .... . ........................... ........................... . . . . . . .... ... . ............. . . ............. . . . . . . .... .. . . . . . . . . . . . . . . . . . . . . . .... . . . .. .. . . . . . . . . .. .. .... . . . . . . . . . . . . . . ...... . . . . . . . . . . . . ..... . . . . . . . . . . . . . .... . . . . . . . . . . . ..................................... . . . . . . . .................. . ........................................ . . . . . . .................... ............................ . .. . .. . . .. . . . . . . . .......................... . . . . . . . . . . . . . . . . . . . . . . . . . . . .. . . . . . . . . . . .. .. . . . . . . . . .. . . . . . . . . . . .. . . . . . . . . .. . .. . . ... . . . . . ... . . . . . . . . . . .. . . . . . . . .. . .. . ... .......... ......... . . ........................... ........................... . .. .. . . . . . . . .. . . .. . . . . . .. . .. . . . . .. .. . . . . . . . . . .. . . .. . . . .. . . .. . . . . . .. . . . .. . . . . . .. . .. . . . .. . .. . . . . . . . .. . .. . . . . .. .... .... .... .... .... .... .... .... .... .... .... .... .... .... .... .... .. . ... ... ... ... ... ... ... ... ... ... ... ... ... ... ... ... ..... ..... ... . . .. .... ...... .. .... . .. . . .. . .. .. .. . .. .. . . . . . . .. . . . ...... . .................. .. . . . . .... . ........... .......... . . .................... . .. . . . . . . .. .. . .. ... . ... ........... ...........

1 1

Node 1

Node 2

Figure 14.5: The machine/repair model as a closed queueing networks with two nodes. The terminals correspond to one ISnode, because the tasks always nd an idle terminal, whereas the CPU corresponds to an M/M/1node.
Example 14.4.1: Palms machine/repair model We now consider the machine/repair model of Palm introduced in Sec. 12.5 as a closed queueing network (Fig. 14.5). There are S customers and terminals. The mean thinking time is 1 and 1 the mean service time at the CPU is 1 . In queueing network terminology there are two nodes: 2 node one is the terminals, i.e. an M/G/ (actually it is an M/G/S system, but since the number of customers is limited to S it corresponds to an M/G/ system), and node two is the CPU, i.e. an M/M/1 system with service intensity 2 . The ows to the nodes are equal (1 = 2 = ) and the relative load at node 1 and node 2 are 1 = /1 and 2 = /2 , respectively. If we consider each node in isolation we obtain the state probabilities of each node, q1 (i) and q2 (j), and by convolving q1 (i) and q2 (j) we get q12 (x), (0 x S), as shown in Table 14.1. The last term with S customers (an unnormalised probability) q12 (S) is compounded of: q12 (S) = S 1 + S1 1 + S2 2 2 2 A simple rearranging yields: q12 (S) = S 1 + 2 where =
2

2 S 1 + + 1 1 . 2! S!

2!

+ +

S!

1 2 = . 2 1

292 State i 0 1 2 . . . i . . . S Node 1 q1 (i) 1 1


2 1 2! . . . i 1 i! . . . S 1 S!

CHAPTER 14. NETWORKS OF QUEUES Node 2 q2 (i) 1 2


2 2

Queueing network q12 = q1 q2 1 1 + 2


2 2 + 1 2 + 2 1 2!

. . .
i 2

. . . . . . . . . q12 (S)

. . .
S 2

Table 14.1: The convolution algorithm applied to Palms machine/repair model. Node 1 is an IS-system, and node two is an M/M/1-system (Example 14.4.1).
The probability that all terminals are thinking is identied as the last term (normalised by the sum) (S terminals in node 1, zero terminals in node 2):
S 2

S! +

1+ +

2!

3!

+ +

= E1,S ( ) ,

S!

which is Erlangs B-formula. Thus the result is in agreement with the result obtained in Sec. 12.5. We notice that appears with the same power in all terms of q1,2 (S) and thus corresponds to a constant which disappears when we normalise.

Example 14.4.2: Central server system In 1971 J. P. Buzen introduced the central server model illustrated in Fig. 14.6 to model a multiprogrammed computer system with one CPU and a number of input/output channels (peripheral units). The degree of multi-programming S describes the number of jobs processed simultaneously. The number of peripheral units is denoted by K 1 as shown in Fig. 14.6, which also shows the transition probabilities. Typically a job requires service hundreds of times, either by the central unit or by one of the peripherals. We assume that once a job is nished it is immediately replaced by another job, hence S is constant. The service times are all exponentially distributed with intensity i (i = 1, . . . , K).

14.4. SINGLE CHAIN QUEUEING NETWORKS


.................................................................................................................................................................................................................................................. . .. .. . ................................................................................................................................................................................................................................................ . .. . . .. . ... . . . . . . . . . . . . . . . . . . . . . . . . . . . . . . . . . . . . . . . . . . . . . . . . . . . . . . . . . . ......... . . .......... . .. . .. . . .. ... . .. . . .. . . .. . .. . . .. . . . . . . . . . . . . .................... ..... ...... ...... . .................. . ................... ................ . . . . . . . . . . ................ . . ............... . . . . . . . . . . . . . . . . .. . . . . . .. . . . . . . . . . . ... .. ... . . . .. . . . . . ........... .......... . . . . . . . . . . . . . . . . . . . . . . . . . . . . . . . . . . . . . . . . . . . . . . . . . .......................................................................................... . . .. . . . . .......................................................................................... . . . . . . . . . . . . . . .. . . . . . . . .. . . . . . . . . . . . . . . . . . . . . . . . . . . . . . . . . . . . . . . . ....... . . ........ . .. . .. . . . . . . .. . . . . . .. ... . . . . . . ... . .. . . . . .. . . . . . . .. . . . . . . .. . . . . . . . . . . . . . . . . . . . . . . ................... . . . .................... . .. . . . . . . . . ................. . .................... ................... . . . . . .. . . .. . ............ . . . .................... . . ............ . . . . . . . . . . . . . . .. . . . . .. . . .. . . .. . . .. . . . .. . . . . . ... .. . . . . . . . . . .. .. .. . ... . . .. . . . . . ... . . . . . . ..................................... ............................ ........... . ................. . ........... ..... .. . .. . .. . .... . ................. ........ ................... . ................................... . . .. . ................ . ............... . . . . . . . . . . . . . . . . . . . . . . . . .. . . . .. . . .. . . . ... .. . ... . . . ........... .......... . . . . . . . . . . . . . . . . . . . . . . . . . . . . . . . . . . . . . . . . . . . . . . . . . . . . . . . . . . . . . . . . . . .... .... .... . . . . .... ...... . . .... .. . . . .. . . .. .. . . .. . . . . . . . . . . .................... . ................. .. .. . .. . . . . . . . ................... . ................... ................... ................ ... . . . . . . .. .. .. .. ... . ... ............ ....... ...

293

S circulating tasks

p12

new tasks

p11

p13

CPU

p1K

I/O channels Figure 14.6: Central server queueing system consisting of one central server (CPU) and (K1) I/Ochannels. A xed number of tasks S are circulating in the system.
Buzen drew up a scheme to evaluate this system. The scheme is a special case of the convolution algorithm. Let us illustrate it by a case with S = 4 customers and K = 3 nodes and: 1 = 1 , 28 2 = 1 , 40 3 = 1 , 280

p11 = 0.1 , The relative loads become:

p12 = 0.7 ,

p13 = 0.2 .

1 = 1 ,

2 = 1 ,

3 = 2 .

If we apply the convolution algorithm we obtain the results shown in Table 14.2. The term q123 (4) is made up by: q123 (4) = 1 16 + 2 8 + 3 4 + 4 2 + 5 1 = 57 .

Node 3 serves customers in all states except for state q3 (0) q12 (4) = 5. The utilisation of node 3 is therefore a3 = 52/57. Based on the relative loads we now obtain the exact loads: a1 = 26 , 57 a2 = 26 , 57 a3 = 52 . 57

294 State i 0 1 2 3 4 Node 1 q1 (i) 1 1 1 1 1 Node 2 q2 (i) 1 1 1 1 1

CHAPTER 14. NETWORKS OF QUEUES Node 1*2 q12 = q1 q2 1 2 3 4 5 Node 3 q3 1 2 4 8 16 Queueing network q123 = (q1 q2 ) q3 1 4 11 26 57

Table 14.2: The convolution algorithm applied to the central server system.
The average number of customers at node 3 is: L3 = {1 (4 2) + 2 (3 4) + 3 (2 8) + 4 (1 16)} / 57 , L3 = 144 . 57

By changing the order of convolution we get the average queue lengths L1 and L2 and ends up with: L1 = 42 , 57 L2 = 42 , 57 L3 = 144 . 57

The sum of all average queue lengths is of course equal to the number of customers S. Notice, that in queueing networks we dene the queue length as the total number of customers in the node, including customers being served. From the utilisation and mean service time we nd the average number of customers nishing service per time unit at each node: 1 = 26 1 , 57 28 2 = 26 1 , 57 40 3 = 52 1 . 57 280

Applying Littles result we nally obtain the mean sojourn time Wk = Lk /k : W1 = 45.23 , W2 = 64.62 , W3 = 775.38 .

14.4.2

The MVAalgorithm

The Mean Value Algorithm (MVA) is an algorithm to calculate performance measures of queueing networks. It combines in an elegant way two main results in queueing theory: the arrival theorem (8.27) and Littles law (5.20). The algorithm was rst published by Lavenberg & Reiser (1980 [72]).

14.4. SINGLE CHAIN QUEUEING NETWORKS

295

We consider a queueing network with K nodes and S customers (all belonging to a single chain). The relative loads of the nodes are denoted by k (k = 1, 2, . . . , K). The algorithm is recursively in the number of customers, i.e. a network with x + 1 customers is evaluated from a network with x customers. Assume that the average number of customers at node k is Lk (x) where x is the total number of customers in the network. Obviously
K

Lk (x) = x .
k=1

(14.11)

The algorithm goes recursively in two steps: Step 1: Increase the number of customers from x to (x + 1). According to the arrival theorem, the (x + 1)th customer will see the system as a system with x customers in statistically equilibrium. Hence, the average sojourn time (waiting time + service time) at node k is: For M/M/1, M/G/1PS, and M/G/1LCFSPR: Wk (x + 1) = {Lk (x) + 1} sk . For M/G/: Wk (x + 1) = sk . where sk is the average service time in node k which has nk servers. As we only calculate mean waiting times, we may assume FCFS queueing discipline. Step 2: We apply Littles law (L = W ), which is valid for all systems in statistical equilibrium. For node k we have: Lk (x+1) = c k Wk (x+1) , where k is the relative arrival rate to node k. The normalising constant c is obtained from the total number of customers::
K

Lk (x+1) = x + 1 .
k=1

(14.13)

By these two steps we have performed the recursion from x to (x + 1) customers. For x = 1 there will be no waiting time in the system and Wk (1) equals the average service time sk . The MVAalgorithm is below shown for a single server nodes, but it is fairly easy to generalise to nodes with either multiple servers or even innite server discipline.

296

CHAPTER 14. NETWORKS OF QUEUES

Example 14.4.3: Central server model We apply the MVAalgorithm to the central server model (Example 14.4.2). The relative arrival rates are: 1 = 1 , 2 = 0.7 , 3 = 0.2 .
Node 1 S = 1 W1 (1) = L1 (1) = L1 (1) = S = 2 W1 (2) = L1 (2) = L1 (2) = 28 W2 (1) = c 1 28 L2 (1) = 0.25 L2 (1) = 1.25 28 W2 (2) = c 1 1.25 28 L2 (2) = 0.4545 L2 (2) = Node 2 40 W3 (1) = c 0.7 40 L3 (1) = 0.25 L3 (1) = 1.25 40 W3 (2) = c 0.7 1.25 40 L3 (2) = 0.4545 L3 (2) = 1.4545 40 W3 (3) = c 0.7 1.4545 40 L3 (3) = 0.6154 L3 (3) = 1.6154 40 W3 (4) = c 0.7 1.6154 40 L3 (4) = 0.7368 L3 (4) = Node 3 280 c 0.2 280 0.50 1.50 280 c 0.2 1.50 280 1.0909 2.0909 280 c 0.2 2.0909 280 1.7692 2.7692 280 c 0.2 2.7692 280 2.5263

S = 3 W1 (3) = 1.4545 28 W2 (3) = L1 (3) = c 1 1.4545 28 L2 (3) = L1 (3) = 0.6154 L2 (3) = S = 4 W1 (4) = 1.6154 28 W2 (4) = L1 (4) = c 1 1.6154 28 L2 (4) = L1 (4) = 0.7368 L2 (4) =

Naturally, the result is identical to the one obtained with the convolution algorithm. The sojourn time at each node (using the original time unit): W1 (4) = 1.6154 28 = 45.23 , W2 (4) = 1.6154 40 = 64.62 , W3 (4) = 2.7693 280 = 775.38 . Example 14.4.4: MVA-algorithm applied to the machine/repair model We consider the machine/repair model with S sources, terminal thinking time A and CPU service time equal to one time unit. As mentioned in Sec. 12.5.2 this is equivalent to Erlangs loss system with S servers and oered trac A. It is also a closed queueing network with two nodes and S customers in one chain. If we apply the MVAalgorithm to this system, then we get the recursion formula for the ErlangB formula (7.29). The relative visiting rates are identical, as a customer alternatively visits node one and two: 1 = 2 = 1. Node 1 S = 1 W1 (1) = L1 (1) L1 (1) = = A W2 (1) = c 1 A L2 (1)
A 1+A

Node 2 1 c11
1 1+A

= =

L2 (1)

S = 2 W1 (2) = L1 (2) L1 (2) = = A

A W2 (2) = c 1 A L2 (2)
1+A 2 1+A+ A ! 2

1+

1 1+A

= c 1 (1 + = 2A

1 1+A )

L2 (2)

1+A 2 1+A+ A ! 2

14.5. BCMP QUEUEING NETWORKS

297

We know that the queue-length at the terminals (node 1) is equal to the carried trac in the equivalent ErlangB system and that all other customers stay in the CPU (node 2). We thus have in general:

Node 1 S = x W1 (x) = L1 (x) L1 (x) = A W2 (x) = c A L2 (x) =

Node 2 1 + L2 (x 1) c {1 + L2 (x 1)}

= A {1 Ex (A)} L2 (x)

= x A {1 Ex (A)}

From this we have the normalisation constant c = 1 Ex (A) and nd for the (x+1)th customer:

L1 (x + 1) + L2 (x + 1) = c A + c {1 + L2 (x)} = c A + c {1 + x A (1 Ex )} = x + 1, Ex+1 = A Ex , x + 1 + A Ex

because we know c = 1 Ex+1 . This is just the recursion formula for the ErlangB formula.

14.5

BCMP queueing networks

In 1975 the second model of Jackson was further generalised by Baskett, Chandy, Muntz and Palacios (1975 [4]). They showed that queueing networks with more than one type of customers also have product form, provided that: a) Each node is a symmetric queueing system (cf. Sec. 14.2: Poisson arrival process Poisson departure process). b) The customers are classied into N chains. Each chain is characterised by its own mean service time si and transition probabilities pij . Furthermore, a customer may change from one chain to another chain with a certain probability after nishing service at a node. A restriction applies if the queueing discipline at a node is M/M/n (including M/M/1): the average service time must be identical for all chains in a node. BCMPnetworks can be evaluated with the multi-dimensional convolution algorithm and the multidimensional MVA algorithm.

298

CHAPTER 14. NETWORKS OF QUEUES

Mixed queueing networks (open & closed) are calculated by rst calculating the trac load in each node from the open chains. This trac must be carried to enter statistical equilibrium. The capacity of the nodes are reduced by this trac, and the closed queueing network is calculated by the reduced capacity. So the main problem is to calculate closed networks. For this we have more algorithms among which the most important ones are convolution algorithm and the MVA (Mean V alue Algorithm) algorithm.

14.6

Multidimensional queueing networks

In this section we consider queueing networks with more than one type of customers. Customers of same type belong to a specic class or chain. In Chap. 10 we considered loss systems with several types of customers (services) and noticed that the product form was maintained and that the convolution algorithm could be applied.

14.6.1

M/M/1 single server queueing system


1
. ................... ................... .. . .. . ....................... ....................... .. . .. ..... ..... ..... ..... ..... ..... ..... ..... ..... ..... ..... ........... .... ...... ..... ..... ... .. .. ..... ..... .. . . . ...... ...... . . . .............................................. . . .... ... ......................... . . .. ... . . . . ....................... ... . ............................................. ... . . . . . . ..... . ...... .. . .. .. .. ..... ..... .. ... . ... .. ............ ..... ........... ..... . ... ..... ..... ..... ..... ... . ..... .... .... . ....................... ...................... .. . .

A1 = 1 /1

. ................... ................. . . . ...

A2 = 2 /2

Figure 14.7: An M/M/1queueing system with two types (chains) of customers. Fig. 14.7 illustrates a single server queueing system with N = 2 types of customers (chains). Customers arrive to the system according to a Poisson arrival process with intensity j (j = 1, 2). State (i, j) is dened as a state with i type 1 customers and j type 2 customers. The service intensity i,j in state (i, j) can be chosen such that it is state dependent, for example: i j ij = 1 + 2 . i+j i+j The service rates can be interpreted in several ways corresponding to the symmetric single server queueing system. One interpretation corresponds to processor sharing, i.e. all (i + j) customers share the server and the capacity of the server is constant. The state dependency is due to the dierence in service rates between the two types of customers; i.e. the number of customers terminated per time unit depends on the types of customers currently being served. Another interpretation corresponds to an M/M/1 system. If we assume 1 = 2 , then it

14.6. MULTIDIMENSIONAL QUEUEING NETWORKS


. . . . . .. . .. . .. . .. . . . .. .. . . ... ... . . . ... ... . . .. . ... ... . . . . . . . . . . . . . . . . . . . . . . . . . . . . . . . . . . . . . . . . . . . . . . . . . . . . . . . . . . . . . . . . . . . . . . . . . . . . . . . . . . . . . . . . . . . . . . . . . . . . . . . . . ... ... . . .. ... ... .. . . . . .. .. . .... .. . .... .. . . . .. .. ............. ............ . . . . . ... ... ... . ........ ... . ........ ... . ... . .. .. .. .. .. .. ... ... .. .. .. .. . . .. .. ......... ........ ........ ........ ... ................................... ......................................................... ... .. ............................... . .............................. . .. . .. . .... .. ..................................................... . ................................. . ... . . . . . . . . . . . . . . . . . . . . . . . . . . . . . . . . . . . . . ...... ...... . . ............................... . ....................................................... . . .................................. .................................. . . . .. . .. ................................. .......................................................... .... .... .. . .. .. .... .. .... .. .. .. . .. . .. ... .. ... ... ... ... .. .. .... ..... ..... ........ .. ... ........ .. . .... .. . . ..... .. . . .. ..... .... ............ . . . .. . . .. ... ... . . ... ... . . . . ... ... . . . . . . . . . . . . . . . . . . . . . . . . . . . . . . . . . . . . . . . . . . . . . . . . . . . . . . . . . . . . . . . . . . . . . . . . . . . . . . . . . . . . . . . . . . . . . . . . . . . . . . . . . . . . . . . . . . . . . . . . . . . . . . . . . . . . . . . . . . . . . . . . . . . . . . . . . . . . . . . . . . . . . . . . . . . . . . . . . . . . . . . . . . . . . . . . . . . . . . . . . . . . . . . . . .. . . . . ... ... . . . ... ... . . ... . . .. . . .. . ....... ... . ....... .. .. . ............. ............. .. .. .... .... . .. . .. .... .... .... .... ... ... . . .. .. ... ... .. .. .. .. .. .. . . . ................................... ............................................................ .................................. ........................................................... ... . .. ................................... .................................. .. . . .. ... .. . . ... . ... . ... ...... ..... ..... . . . . . . . . . . . . . . . . . . . . . . . . . . . . . ... . ... . . ................................. ........................................................ . . . .. . ..... .. .................................. .................................. ... ... ....... .... .... .. . ................................. ........................................................... .. . .... .. . . .. .. .. ... .. ... ... .. ... ... .... ..... ........ ..... ........ ................ .. . . ..... . . . .. .... . ............ .. .. . . .. .. . . .. .. . . ... ... . . . . ... ... . . . . . .. . .. . . . . . . . . . . . . . . . . . . . . . . . . . . . . . . . . . . . . . . . . . . . . . . . . . . . . . . . . . . . . . . . . . . . . . . . . . . . . . . . . . . . . . . . . .. .. . . .. . . ... ... . . ... ... . . .. ... . . .. .. . .. .. . . . . .. .. . . . .

299

i1, j

i, j

i 1 i+j

j 2 i+j1 1

j 2 i+j

i1, j1

i, j 1

i 1 i+j1

Figure 14.8: State transition diagram for a multidimensional M/M/1system with processor sharing.

can be shown that the customer being served is with probability i/(i+j) type 1, and with probability j/(i+j) type 2. This is independent of the service discipline. Part of the state transition diagram is given by Fig. 14.8. The diagram is reversible, since the ow clockwise equals the ow counter-clockwise. Hence, there is local balance and all state probabilities can be expressed by p(0, 0): Ai Aj 1 2 (i + j)! p(0, 0) . i! j!

p(i, j) =

(14.14)

By normalisation we get p(0, 0):


p(i, j) = 1 .
i=0 j=0

In comparison with the multidimensional ErlangB formula we now have the additional factor (i+j)!. The product form between chains (inside a node) is lost, but the product form between nodes will still be maintained. If there are N dierent types of customers (chains) the state probabilities for a single node

300 becomes:

CHAPTER 14. NETWORKS OF QUEUES

N N

ij ! Ajj
i

p(i) = p(i1 , i2 , . . . , iN ) =
j=1

j=1 N

p(0) . ij !

(14.15)

j=1

This can be expressed by the polynomial distribution (4.37):


N

p(i) =
j=1

Ajj

i1 + i2 + + iN i1 , i2 , . . . , iN

p(0) .

(14.16)

For an unlimited number of queueing positions the state probabilities of the total number of customers are: p(j) = p{i1 + i2 + + iN = j} . If i = , then the system is identical to an M/M/1 System with arrival rate = p(j) = (A1 + A2 + + AN )j p(0) = Aj (1 A) . The Binomial expansion is used to obtain this result. The state transition diagram in Fig. 14.8 can also be interpreted as the state transition diagram of an M/G/1-LCFS-PR (preemptive resume) system. It is obvious that M/G/1-LCFS-PR is reversible because the process follows exactly the same path in the state transition diagram away from state zero as back to state zero. The state transition diagram can be shown to be insensitive to the service time distribution so that it is valid for M/G/1 queueing system. Fig. 14.8 corresponds to a state transition diagram for a single server queueing system with hyperexponentially distributed service times (cf. (10.7)), e.g. M/H2 /1LCFSPR or PS. Notice, that for M/M/1 (FCFS, LCFS, SIRO) it is necessary to assume that all customers have the same mean service time, which must be exponentially distributed. Other ways, the customer being served will not be a random customer among the (i + j) customers in the system. In conclusion, single server queueing systems with more types of customers will only have product form when the node is a symmetric queueing system: M/G/1PS, M/G/1LCFS PR, or M/M/1 with same service time for all customers.
i

i :

14.7. CLOSED QUEUEING NETWORKS WITH MULTIPLE CHAINS

301

14.6.2

M/M/n queueing system

We may also carry through the above for a system with n servers. For (i + j) n we get the same relative state probabilities as for the multidimensional Erlang-B formula. For (i + j) > n we only get a simple interpretation when i = , i.e. when all types (chains) of customers have the same mean holding time. We then nd the state probabilities given in (10.9), and the system has product form. M/M/ may be considered as a special case of M/M/n and has already been dealt with in connection with loss systems (Chap. 12).

14.7

Closed queueing networks with multiple chains

Dealing with queueing networks having multiple chains is analogous to the case with a single chain. The only dierence is that the classical formul and algorithms are replaced by the corresponding multi-dimensional formul.

14.7.1

Convolution algorithm

The algorithm is essentially the same as in the single chain case: Step 1. Consider each chain as if it is alone in the network. Find the relative load at each node by solving the ow balance equation (14.5). At an arbitrary reference node we assume that the load is equal to one. For each chain we may choose a dierent node as reference node. For chain j in node k the relative arrival intensity j is obtained k from (we use the upper index to denote the chain):
K

j k where: K = number of nodes, N = number of chains,

=
i=1

pj j , i ik

j = 1, . . . , N ,

(14.17)

pj = the probability that a customer of chain j jumps from node i to node k. ik We choose an arbitrary node as reference node, e.g. node 1, i.e. j = 1. The relative 1 load at node k due to customers of chain j is then:
j k = j sj k k

where sj = is the mean service time at node k for customers of chain j. Note, j is an k index not a power.

302

CHAPTER 14. NETWORKS OF QUEUES

Step 2. Based on the relative loads found in step 1, we obtain the multi-dimensional state probabilities for each node. Each node is considered in isolation and we truncate the state space according to the number of customers in each chain. For example for node k (1 k K): pk = pk (i1 , i2 , . . . , iN ) , 0 ij Sj , j = 1, 2, . . . N ,

where Sj is the number of customers in chain j. Step 3. In order to nd the state probabilities of the total network, the state probabilities of each node are convolved together similar to the single chain case. The only dierence is that the convolution is multi-dimensional. When we perform the last convolution we may obtain the performance measures of the last node. Again, by changing the order of nodes, we can obtain the performance measures of all nodes. The total number of states increases rapidly. For example, if chain j has Sj customers, then the total number of states in each node becomes:
N

(Sj +1) .
j=1

The number of ways N chains with Sj customers in chain j can be distributed in a queueing network with K nodes is:
N

C=
j=1

C(Sj , kj )

(14.18)

where kj (1 kj < k) is the number of nodes visited by chain j and: C(Sj , kj ) = Sj + k j 1 kj 1 = Sj + k j 1 Sj . (14.19)

The algorithm is best illustrated with an example.


Example 14.7.1: Palms machine-repair model with two types of customers As seen in Example 14.4.1, this system can be modelled as a queueing network with two nodes. Node 1 corresponds to the terminals (machines) while node 2 is the CPU (repair man). Node 2 is a single server system whereas node 1 is modelled as a Innite Server system. The number of customers in the chains are (S1 = 2, S2 = 3) and the mean service time in node k is sj . The relative k load of chain 1 is denoted by 1 in node 1 and by 2 in node 2. Similarly, the load af chain 2 is denoted by 1 , respectively 2 . Applying the convolution algorithm yields: Step 1. Chain 1: Relative load: Chain 2: Relative load: S1 = 2 customers 1 = 1 s1 , 1 S2 = 3 customers 1 = 2 s2 , 1 2 = 1 s1 . 2 2 = 2 s2 . 2

14.7. CLOSED QUEUEING NETWORKS WITH MULTIPLE CHAINS


Step 2. For node 1 (IS) the relative state probabilities are (cf. 10.9):
2 1 2 2 1 1 2 2 2 1 1 4 3 1 6 3 1 1 6 3 2 1 1 12

303

q1 (0, 0) = 1 q1 (1, 0) = 1 q1 (2, 0) = 2 1 2

q1 (0, 2) = q1 (1, 2) = q1 (2, 2) = q1 (0, 3) = q1 (1, 3) = q1 (2, 3) =

q1 (0, 1) = 1 q1 (1, 1) = 1 1 q1 (2, 1) = 2 1 1 2

For node 2 (single server) (cf. 14.15) we get: q2 (0, 0) = 1 q2 (1, 0) = 2 q2 (2, 0) = 2 2 q2 (0, 1) = 2 q2 (1, 1) = 2 2 2 q2 (2, 1) = 3 2 2 2
2 q2 (0, 2) = 2 2 q2 (1, 2) = 3 2 2 2 q2 (2, 2) = 6 2 2 2 3 q2 (0, 3) = 2 3 q2 (1, 3) = 4 2 2 3 q2 (2, 3) = 10 2 2 2

Step 3. Next we convolve the two nodes. We know that the total number of customers are (2, 3), i.e. we are only interested in state (2, 3): q12 (2, 3) = q1 (0, 0) q2 (2, 3) + q1 (1, 0) q2 (1, 3) + q1 (2, 0) q2 (0, 3) + q1 (0, 1) q2 (2, 2) + q1 (1, 1) q2 (1, 2) + q1 (2, 1) q2 (0, 2) + q1 (0, 2) q2 (2, 1) + q1 (1, 2) q2 (1, 1) + q1 (2, 2) q2 (0, 1) + q1 (0, 3) q2 (2, 0) + q1 (1, 3) q2 (1, 0) + q1 (2, 3) q2 (0, 0)

304
Using the actual values yields:

CHAPTER 14. NETWORKS OF QUEUES

3 q12 (2, 3) = + 1 10 2 2 2

3 + 1 4 2 2 2 + 1 6 2 2 2

2 3 1 2 2

2 + 1 1 3 2 2 +

2 1 1 2 2 2
2 1 1 2 2 2 2 3 1 2 2 6 3 2 1 1 1 12

+ + +

2 1 3 2 2 2 2 2 2 1 1 2 4 3 1 1 2 6

+ + +

Note that 1 and 2 together (chain 1) always appears in the second power whereas 1 and 2 (chain 2) appears in the third power corresponding to the number of customers in each chain. Because of this, only the relative loads are relevant, and the absolute probabilities are obtain by normalisation by dividing all the terms by q12 (2, 3). The detailed state probabilities are now easy to 3 obtain. Only in the state with the term (2 1 )/12 is the CPU (repair man) idle. If the two types 1 of customers are identical the model simplies to Palms machine/repair model with 5 terminals. In this case we have: 1 2 3 E1,5 (x) = 12 1 1 . q12 (2, 3) Choosing 1 = 1 = and 2 = 2 = 1, yields:
1 12 3 2 1 1 q12 (2, 3)

5 /12 10 + 4 + 1 2 + 6 + 32 + 1 3 + 3 2 + 3 + 1 4 + 1 3 + 1 4 + 2 2 2 4 6 6 5 5! , 2 3 4 5 1++ + + + 2 3! 4! 5!

1 5 12

i.e. the ErlangB formula as expected.

14.8

Other algorithms for queueing networks

Also the MVAalgorithm is applicable to queueing networks with more chains, but it will not be described here. During the last decade several algorithms have been published. An overview can be found in (Conway & Georganas, 1989 [15]). In general, exact algorithms are not applicable for bigger networks. Therefore, many approximative algorithms have been developed to deal with queueing networks of realistic size.

14.9. COMPLEXITY

305

14.9

Complexity

Queueing networks has the same complexity as circuit switched networks with direct routing (Sec. 11.5 and Tab. 11.2). The state space of the network shown in Tab. 14.3 has the following number of states for every node:
N

(Si + 1) .
i=0

(14.20)

The worst case is when every chain consists of one customer. Then the number of states becomes 2S where S is the number of customers. Node 1 11 12 ... 1N 2 21 22 ... 2N ... K K1 K2 ... KN Population Size S1 S2 ... SN

Chain 1 2 ... N

Table 14.3: The parameters of a queueing network with N chains, K nodes and i Si customers. The parameter jk denotes the load from customers of chain j in node k (cf. Tab. 11.2).

14.10

Optimal capacity allocation

We now consider a data transmission system with K nodes, which are independent single server queueing systems M/M/1 (Erlangs delay system with one server). The arrival process to node k is a Poisson process with intensity k messages (customers) per time unit, and the message size is exponentially distributed with mean value 1/k [bits]. The capacity of node k is k [bits per time unit]. The mean service time becomes: s= 1/k 1 = . k k k

So the mean service rate is k k and the mean sojourn time is given by (12.34): m1,k = 1 . k k k

306

CHAPTER 14. NETWORKS OF QUEUES

We introduce the following linear restriction on the total capacity:


K

F =
k=1

k .

(14.21)

For every allocation of capacity which satises (14.21), we have the following mean sojourn time for all messages (call average):
K

m1 =
k=1

k 1 , k k k
K

(14.22)

where: =

k .
k=1

(14.23)

By applying (13.14) we get the total mean service time: 1 = The total oered trac is then:
K

k=1

k 1 . k

(14.24)

. F Kleinrocks law for optimal capacity allocation (Kleinrock, 1964 [65]) reads: A=

(14.25)

Theorem 14.2 Kleinrocks square root law: The optimal allocation of capacity which minimises m1 (and thus the total number of messages in all nodes) is: k = under the condition that: F >
k=1

k + F (1 A) k
K

k /k
K i=1

i /i

(14.26)

k . k

(14.27)

Proof: This can be shown by introducing Lagrange multiplier and consider:


K

G = m1
k=1

k F

(14.28)

Minimum of G is obtained by choosing k as given in (14.26). With this optimal allocation we nd the mean sojourn time: m1 =
K k=1

k /k

F (1 A)

(14.29)

14.10. OPTIMAL CAPACITY ALLOCATION

307

This optimal allocation corresponds to that all nodes rst are allocated the necessary minimum capacity i /i . The remaining capacity (14.24):
K

F
k=1

i = F (1 A) i

(14.30)

is allocated among the nodes proportional the square root of the average ow k /k . If all messages have the same mean value (k = ), then we may consider dierent costs in the nodes under the restriction that a xed amount is available (Kleinrock, 1964 [65]).

308

CHAPTER 14. NETWORKS OF QUEUES

Chapter 15 Trac measurements


Trac measurements are carried out in order to obtain quantitative information about the load on a system to be able to dimension the system. By trac measurements we understand any kind of collection of data on the trac loading a system. The system considered may be a physical system, for instance a computer, a telephone system, or the central laboratory of a hospital. It may also be a ctitious system. The collection of data in a computer simulation model corresponds to a trac measurements. Billing of telephone calls also corresponds to a trac measurement where the measuring unit used is an amount of money. The extension and type of measurements and the parameters (trac characteristics) measured must in each case be chosen in agreement with the demands, and in such a way that a minimum of technical and administrative eorts result in a maximum of information and benet. According to the nature of trac a measurement during a limited time interval corresponds to a registration of a certain realisation of the trac process. A measurement is thus a sample of one or more random variables. By repeating the measurement we usually obtain a dierent value, and in general we are only able to state that the unknown parameter (the population parameter, for example the mean value of the carried trac) with a certain probability is within a certain interval, the condence interval. The full information is equal to the distribution function of the parameter. For practical purposes it is in general sucient to know the mean value and the variance, i.e. the distribution itself is of minor importance. In this chapter we shall focus upon the statistical foundation for estimating the reliability of a measurement, and only to a limited extent consider the technical background. As mentioned above the theory is also applicable to stochastic computer simulation models.

309

310

CHAPTER 15. TRAFFIC MEASUREMENTS

15.1

Measuring principles and methods

The technical possibilities for measuring are decisive for what is measured and how the measurements are carried out. The rst program controlled measuring equipment was developed at the Technical University of Denmark, and described in (Andersen & Hansen & Iversen, 1971 [2]). Any trac measurement upon a trac process, which is discrete in state and continuous in time can in principle be implemented by combining two fundamental operations: 1. Number of events: this may for example be the number of errors, number of call attempts, number of errors in a program, number of jobs to a computing centre, etc. (cf. number representation, Sec. 5.1.1 ). 2. Time intervals: examples are conversation times, execution times of jobs in a computer, waiting times, etc. (cf. interval representation, Sec. 5.1.2). By combining these two operations we may obtain any characteristic of a trac process. The most important characteristic is the (carried) trac volume, i.e. the summation of all (number) holding times (interval) within a given measuring period. From a functional point of view all trac measuring methods can be divided into the following two classes: 1. Continuous measuring methods. 2. Discrete measuring methods.

15.1.1

Continuous measurements

In this case the measuring point is active and it activates the measuring equipment at the instant of the event. Even if the measuring method is continuous the result may be discrete.
Example 15.1.1: Measuring equipment: continuous time Examples of equipment operating according to the continuous principle are: (a) Electro-mechanical counters which are increased by one at the instant of an event. (b) Recording xy plotters connected to a point which is active during a connection. (c) Amp`re-hour meters, which integrate the power consumption during a measuring period. e When applied for trac volume measurements in old electro-mechanical exchanges every trunk is connected through a resistor of 9,6 k, which during occupation is connected between 48 volts and ground and thus consumes 5 mA. (d) Water meters which measure the water consumption of a household.

15.1. MEASURING PRINCIPLES AND METHODS

311

15.1.2

Discrete measurements

In this case the measuring point is passive, and the measuring equipment must itself test (poll) whether there have been changes at the measuring points (normally binary, on-o). This method is called the scanning method and the scanning is usually done at regular instants (constant = deterministic time intervals). All events which have taken place between two consecutive scanning instants are from a time point of view referred to the latter scanning instant, and are considered as taking place at this instant.
Example 15.1.2: Measuring equipment: discrete time Examples of equipment operating according to the discrete time principle are: (a) Call charging according to the Karlsson principle, where charging pulses are issued at regular time instants (distance depends upon the cost per time unit) to the meter of the subscriber, who has initiated the call. Each unit (step) corresponds to a certain amount of money. If we measure the duration of a call by its cost, then we observe a discrete distribution (0, 1, 2, . . . units). The method is named after S.A. Karlsson from Finland (Karlsson, 1937 [57]). In comparison with most other methods it requires a minimum of administration. (b) The carried trac on a trunk group of an electro-mechanical exchange is in practice measured according to the scanning principle. During one hour we observe the number of busy trunks 100 times (every 36 seconds) and add these numbers on a mechanical counter, which thus indicate the average carried trac with two decimals. By also counting the number of calls we can estimate the average holding time. (c) The scanning principle is particularly appropriate for implementation in digital systems. For example, the processor controlled equipment developed at DTU, Technical University of Denmark, in 1969 was able to test 1024 measuring points (e.g. relays in an electro-mechanical exchange, trunks or channels) within 5 milliseconds. The states of each measuring point (idle/busy or o/on) at the two latest scannings are stored in the computer memory, and by comparing the readings we are able to detect changes of state. A change of state 0 1 corresponds to start of an occupation and 1 0 corresponds to termination of an occupation (lastlook principle). The scannings are controlled by a clock. Therefore we may monitor every channel during time and measure time intervals and thus observe time distributions. Whereas the classical equipment (erlang-meters) mentioned above observes the trac process in the state space (vertical, number representation), then the program controlled equipment observes the trac process in time space (horizontal, interval representation), in discrete time. The amount of information is almost independent of the scanning interval as only state changes are stored (the time of a scanning is measured in an integral number of scanning intervals).

Measuring methods have had decisive inuence upon the way of thinking and the way of formulating and analysing the statistical problems. The classical equipment operating in state space has implied that the statistical analyses have been based upon state probabilities, i.e. basically birth and death processes. From a mathematically point of view these models have been rather complex (vertical measurements).

312

CHAPTER 15. TRAFFIC MEASUREMENTS

The following derivations are in comparison very elementary and even more general, and they are inspired by the operation in time space of the program controlled equipment. (Iversen, 1976 [36]) (horizontal measurements).

15.2

Theory of sampling

Let us assume we have a sample of n IID (Independent and Identically Distributed) observations {X1 , X2 , . . . , Xn } of a random variable with unknown nite mean value m1 and nite variance 2 (population parameters). The mean value and variance of the sample are dened as follows: 1 X = n s2 =
n

Xi
i=1 n

(15.1)

1 n1

Xi2 n X 2
i=1

(15.2)

Both X and s2 are functions of a random variable and therefore also random variables, dened by a distribution we call the sample distribution. X is a central estimator of the unknown population mean value m1 , i.e.: (15.3) E{X} = m1 Furthermore, s2 /n is a central estimator of the unknown variance of the sample mean X, i.e.: 2 {X} = s2 /n. (15.4)

We describe the accuracy of an estimate of a sample parameter by means of a condence interval, which with a given probability species how the estimate is placed relatively to the unknown theoretical value. In our case the condence interval of the mean value becomes: X tn1,1/2 s2 n (15.5)

where tn1,1/2 is the upper (1 /2) percentile of the Students t-distribution with n 1 degrees of freedom. The probability that the condence interval includes the unknown theoretical mean value is equal to (1) and is called the level of condence. Some values of the Students t-distribution are given in Table 15.1. When n becomes large, then the Students t-distribution converges to the Normal distribution, and we may use the percentile of this distribution. The assumption of independence are fullled for measurements taken on dierent days, but for example not for successive measurements by the scanning method within a limited time interval, because the number of busy channels at a given instant will be correlated with the number of busy circuits in the previous and the next scanning. In

15.2. THEORY OF SAMPLING

313

Continuous trac process a b c d 4 3 2 1 0

Time Discrete trac process

a b c d 4 3 2 1 0

Scan

Scanning method a b c d 0 0 0 1 1
. ......... ......... .

1 0 0 1 2

......... ......... . . .. . . ......... ......... .

1 1 1 1 4

. ......... ......... .

1 1 0 1 3

.. . ......... ......... .

.. . ......... ......... .

1 0 0 0 1

. ......... ......... .

0 0 0 0 0

. .......... ..... ... .

. .......... ..... ... .

1 0 1 0 2

. ......... ......... . ......... ......... . ..

0 1 1 0 2

0 1 1 0 2

. . ......... ......... .

0 1 0 0 1

Figure 15.1: Observation of a trac process by a continuous measuring method and by the scanning method with regular scanning intervals. By the scanning method it is sucient to observe the changes of state.

314 n 1 2 5 10 20 40 = 10% 6.314 2.920 2.015 1.812 1.725 1.684 1.645

CHAPTER 15. TRAFFIC MEASUREMENTS = 5% 12.706 4.303 2.571 2.228 2.086 2.021 1.960 = 1% 63.657 9.925 4.032 3.169 2.845 2.704 2.576

Table 15.1: Percentiles of the Students t-distribution with n degrees of freedom. A specic value of corresponds to a probability mass /2 in both tails of the Students t-distribution. When n is large, then we may use the percentiles of the Normal distribution. the following sections we calculate the mean value and the variance of trac measurements during for example one hour. This aggregated value for a given day may then be used as a single observation in the formul above, where the number of observations typically will be the number of days, we measure.

Example 15.2.1: Condence interval for call congestion On a trunk group of 30 trunks (channels) we observe the outcome of 500 call attempts. This measurement is repeated 11 times, and we nd the following call congestion values (in percentage): {9.2, 3.6, 3.6, 2.0, 7.4, 2.2, 5.2, 5.4, 3.4, 2.0, 1.4} The total sum of the observations is 45.4 and the total of the squares of the observations is 247.88 . We nd (15.1) X = 4.1273 % and (15.2) s2 = 6.0502 (%)2 . At 95%level the condence interval becomes by using the t-values in Table 15.1: (2.475.78). It is noticed that the observations are obtained by simulating a PCTI trac of 25 erlang, which is oered to 30 channels. According to the Erlang Bformula the theoretical blocking probability is 5.2603 %. This value is within the condence interval. If we want to reduce the condence interval with a factor 10, then we have to do 100 times as many observations (cf. formula 15.5), i.e. 50,000 per measurements (sub-run). We carry out this simulation and observe a call congestion equal to 5.245 % and a condence interval (5.093 5.398).

15.3

Continuous measurements in an unlimited period

Measuring of time intervals by continuous measuring methods with no truncation of the measuring period are easy to deal with by the theory of sampling described in Sec. 15.2 above.

15.3. CONTINUOUS MEASUREMENTS IN AN UNLIMITED PERIOD

315

. . . . . . . . . . . . . . . . . . . . . . . . . . . . . .

. . . . .

. . . . . . ......... ....... .. . ..

0 a: Unlimited measuring period

Time

. . . . . . . . . . . . . . . . . . . . . . . . . . . . . .

. . . . .

. . . . . .. .......... ....... .. .

0 b: Limited measuring period

Time

Figure 15.2: When analysing trac measurements we distinguish between two cases: (a) Measurements in an unlimited time period. All calls initiated during the measuring period contributes with their total duration. (b) Measurements in a limited measuring period. All calls contribute with the portion of their holding times which are located inside the measuring period. In the gure the sections of the holding times contributing to the measurements are shown with full lines.

For a trac volume or a trac intensity we can apply the formul (3.46) and (3.48) for a stochastic sum. They are quite general, the only restriction being stochastic independence between X and N. In practice this means that the systems must be without congestion. In general we will have a few percentages of congestion and may still as worst case assume independence. By far the most important case is a Poisson arrival process with intensity . We then get a stochastic sum (Sec. 3.3). For the Poisson arrival process we have when we consider a time interval T :

2 m1,n = n = T

316 and therefore we nd:

CHAPTER 15. TRAFFIC MEASUREMENTS

m1,s = T m1,t
2 2 s = T m2 + t 1,t

= T m2,t = T m2 t , 1,t

(15.6)

where m2,t is the second (non-central) moment of the holding time distribution, and t is Palms form factor of the same distribution: =
2 m2,t t = 1+ 2 m2 m1,t 1,t

(15.7)

The distribution of ST will in this case be a compound Poisson distribution (Feller, 1950 [27]). The formul correspond to a trac volume (e.g. erlang-hours). For most applications as dimensioning we are interested in the average number of occupied channels, i.e. the trac intensity (rate) = trac per time unit (m1,t = 1, = A), when we choose the mean holding time as time unit: m1,i = A
2 i =

(15.8) (15.9)

A t T

These formul are thus valid for arbitrary holding time distributions. The formul (15.8) and (15.9) are originally derived by C. Palm (1941 [78]). In (Rabe, 1949 [85]) the formul for the special cases t = 1 (constant holding time) and t = 2 (exponentially distributed holding times) were published. The above formul are valid for all calls arriving inside the interval T when measuring the total duration of all holding times regardless for how long time the stay (Fig. 15.2 a).

Example 15.3.1: Accuracy of a measurement We notice that we always obtain the correct mean value of the trac intensity (15.8). The variance, however, is proportional to the form factor t . For some common cases of holding time distributions we get the following variance of the trac intensity measured: Constant: Exponential distribution: Observed (Fig. 4.3):
2 i = 2 i = 2 i =

A , T A 2, T A 3.83 . T

15.4. SCANNING METHOD IN AN UNLIMITED TIME PERIOD

317

Observing telephone trac, we often nd that t is signicant larger than the value 2 (exponential distribution), which is presumed to be valid in many classical teletrac models (Fig. 4.3). Therefore, the accuracy of a measurement is lower than given in many tables. This, however, is compensated by the assumption that the systems are nonblocking. In a system with blocking the variance becomes smaller due to negative correlation between holding times and number of calls.

Example 15.3.2: Relative accuracy of a measurement The relative accuracy of a measurement is given by the ratio: S= i = m1,i t AT
1/2

= variation coecient.

From this we notice that if t = 4, then we have to measure twice as long a period to obtain the same reliability of a measurement as for the case of exponentially distributed holding times.

For a given time period we notice that the accuracy of the trac intensity when measuring a small trunk group is much larger than when measuring a large trunk group, because the accuracy only depends on the trac intensity A. When dimensioning a small trunk group, an error in the estimation of the trac of 10 % has much less inuence than the same percentage error on a large trunk group (Sec. 7.6.1). Therefore we measure the same time period on all trunk groups. In Fig. 15.5 the relative accuracy for a continuous measurement is given by the straight line h = 0.

15.4

Scanning method in an unlimited time period

In this section we only consider regular (constant) scanning intervals. The scanning principle is for example applied to trac measurements, call charging, numerical simulations, and processor control. By the scanning method we observe a discrete time distribution for the holding time which in real time usually is continuous. In practice we usually choose a constant distance h between scanning instants, and we nd the following relation between the observed time interval and the real time interval (g. 15.3):

Observed time 0h 1h 2h 3h ...

Real time 0 0 1 2 h1 h2 h3 h4 ... h h h h

318 Observed number of scans 5 4 3 2 1 0 0 1 2

CHAPTER 15. TRAFFIC MEASUREMENTS

Interval for the real time (scan) Figure 15.3: By the scanning method a continuous time interval is transformed into a discrete time interval. The transformation is not unique (cf. Sec. 15.4). We notice that there is overlap between the continuous time intervals, so that the discrete distribution cannot be obtained by a simple integration of the continuous time interval over a xed interval of length h. If the real holding times have a distribution function F (t), then it can be shown that we will observe the following discrete distribution (Iversen, 1976 [36]): 1 p(0) = h p(k) = 1 h
h

F (t) dt
0 h 0

(15.10)

{F (t + kh) F (t + (k 1)h)} dt ,

k = 1, 2, . . . .

(15.11)

Interpretation: The arrival time of the call is assumed to be independent of the scanning process. Therefore, the density function of the time interval from the call arrival instant to the rst scanning time is uniformly distributed and equal to (1/h) (Sec. 6.3.3). The probability of observing zero scanning instants during the call holding time is denoted by p(0) and is equal to the probability that the call terminates before the next scanning time. For at xed value of the holding time t this probability is equal to F (t)/h, and to obtain the total probability we integrate over all possible values t (0 t < h) and get (15.10). In a similar way we derive p(k) (15.11). By partial integration it can be shown that for any distribution function F (t) we will always observe the correct mean value:

h
k=0

k p(k) =

t dF (t) .

(15.12)

When using Karlsson charging we will therefore always in the long run charge the correct amount.

15.4. SCANNING METHOD IN AN UNLIMITED TIME PERIOD

319

For exponential distributed holding time intervals, F (t) = 1 e t , we will observe a discrete distribution, Westerbergs distribution (Iversen, 1976 [36]): p(0) = 1 p(k) = 1 1 e h , h
2

(15.13)

1 1 e h h

e(k1) h ,

k = 1, 2, . . .

(15.14)

This distribution can be shown to have the following mean value and form factor: m1 = 1 , h (15.15)

e h + 1 = h h 2. e 1

(15.16)

The form factor is equal to one plus the square of the relative accuracy of the measurement. For a continuous measurement the form factor is 2. The contribution 2 is thus due to the inuence from the measuring principle. The form factor is a measure of accuracy of the measurements. Fig. 15.4 shows how the form factor of the observed holding time for exponentially distributed holding times depends on the length of the scanning interval (15.16). By continuous measurements we get an ordinary sample. By the scanning method we get a sample of a sample so that there is uncertainty both because of the measuring method and because of the limited sample size. Fig. 5.2 shows an example of the Westerberg distribution. It is in particular the zero class which deviates from what we would expect from a continuous exponential distribution. If 2 we insert the form factor in the expression for s (15.9), then we get by choosing the mean holding time as time unit m1,t = 1/ = 1 the following estimates of the trac intensity when using the scanning method: m1,i = A ,
2 i =

A T

eh + 1 eh 1

(15.17)

By the continuous measuring method the variance is 2A/T . This we also get now by letting h 0. Fig. 15.5 shows the relative accuracy of the measured trac volume, both for a continuous measurement (15.8) & (15.9) and for the scanning method (15.17). Formula (15.17) was derived by (Palm, 1941 [78]), but became only known when it was re-discovered by W.S. Hayward Jr. (1952 [33]).
Example 15.4.1: Billing principles Various principles are applied for charging (billing) of calls. In addition, the charging rate if usually

320

CHAPTER 15. TRAFFIC MEASUREMENTS

varied during the 24 hours to inuence the habits of the subscriber. Among the principles we may mention: (a) Fixed amount per call. This principle is often applied in manual systems for local calls (at rate). (b) Karlsson charging. This corresponds to the measuring principle dealt with in this section because the holding time is placed at random relative to the regular charging pulses. This principle has been applied in Denmark in the crossbar exchanges. (c) Modied Karlsson charging. We may for instance add an extra pulse at the start of the call. In digital systems in Denmark there is a xed fee per call in addition to a fee proportional with the duration of the call. (d) The start of the holding time is synchronised with the scanning process. This is for example applied for operator handled calls and in coin box telephones.

15.5

Numerical example

2 For a specic measurement we calculate m1,i and i . The deviation of the observed trac intensity from the theoretical correct value is approximately Normal distributed. Therefore, the unknown theoretical mean value will be within 95% of the calculated condence intervals (cf. Sec. 15.2): m1,i 1, 96 i (15.18) 2 The variance i is thus decisive for the accuracy of a measurement. To study which factors are of major importance, we make numerical calculations of some examples. All formul may easily be calculated on a pocket calculator.

Both examples presume PCTI trac, (i.e. Poisson arrival process and exponentially distributed holding times), trac intensity = 10 erlang, and mean holding time = 180 seconds, which is chosen as time unit. Example a: This corresponds to a classical trac measurement: Measuring period = 3600 sec = 20 time units = T . Scanning interval = 36 sec = 0.2 time units = h = 1/s . (100 observations) Example b: In this case we only scan once per mean holding time: Measuring period = 720 sec = 4 time units = T . Scanning interval = 180 sec = 1 time unit = h = 1/s . (4 observations) From Table 15.5 we can draw some general conclusions:

15.5. NUMERICAL EXAMPLE

321

. . .. ... ... .. .. ... .. ... ... ... .. .. ... ... ... ... ...... ... ... .. ... .. .. ... ... ... ... .. ... ... ... .. ... .. ...... ... ... ... ...... ... ... ... .. . .. ... ... ... . ... ... ... .. .. .. .. .. .. ... ... .. ... .. ... . .. ... .. .. .. ... .. ... ... .. ... ... ... ... ... ... ... ... ... ... ... ... ... ... ... ... ... . . . .. .. .. .. . ... ... ... . ... ... ... ... .. ... .. ... ... .. .. .. ... .. ... ... ... ... ... ... ... .. ... ... ... ... ... .. ... ... ... ... ... ... ... ... ... . .. . . .. .. .. ... ... ... ... . ... ... ... ... .. ... ... ... ... .. ... ... ... ... ... ... ... ... ... .. ... ... ... ... .. ... .. ... ... ... ... ... ... .... ... ... ... .. .. .. ... ... ... ... . ... ... .... ... ... ... ... .. ... .. ... ... ... ... .. ... ... .. ... .... ... ... .. ... ... .. .. ... ... ... ... ... ... ... ... ... ... ... ... . .. ... .. ... ... ... ... ... ... ... .. ... ... .... ... . .. ... ... .. .. ... ... .. ... ... ... . ... .. ... ... ... .. ... ... .. .. .. ... ... ... ... ... ... ... .. ... . . . .. . ... ... ... ... .. ... ... ... ... .. . . . . . ... .. ... ... ... .. ... ... .. ... .. ... ... ... ... ... ... .. ... ... ... ... ... ... ... ... ... .. ... ... ... ... ... ... ... ... ... ... ... ... ... .. ... ... ... ... .. . . . . . . . .. .. ... ... ... ... ... ... ... ... .. ... ... ... ... ... ... .. ... .. ... ... ... ... ... ... ... .. ... .. ... ... ... ... ... ... ... .. ... ... ... .. ... ... ... .. ... ... . . . . . . . .. ... ... ... ... .. ... ... ... .. .. .. ... ... ... ... .. ... ... .. .. .. .. ... ... ... ... ... ... .. ... .. ... ... ... ... ... ... ... .. ... ... ... ... .. ... ... ... ... . . . . . .. ... .. ... ... ... ... ... .. .. ... ... .. .. ... ... .. ... ... .. ... ... .. .. ... .. ... ... ... ... ... ... .. ... ... ... .. ... . .. ... .... .. ... ... ... ... ... ... ... .. ... . . . . .. . .. .. ... .. ... ... .. ... ... .... .. .. ... .. ... ... ... ... .. ... .... .. .. ... ... .... ... .... .. .... ... .. ... ... ... ... ... .. ... ... ... .. ... ... .... .... ... ... .... .. .... . . . .. .. .. .. .. .. .... .... .. ... ... .... .... ... .. .. ... .... .... .. ... .... .... ... ... .. ... ... .... .... .... .... .. ... ... . .. . .. ... .... .... .... .... .. ... .. ... .... .... .. .... .... ... ...... .. ... ... .... .. .. ... .. ... ... .... .... . ... ..... .. .. ... ......... ... ... ......... .. ...... ...... ... .. ... . .. .... ...... ...... .. .. ... ..... .. .... ...... ....... ... ........ ....... . ... .. ............................ . . .. .. ......... . ........................... . .. . .... . .. ... .. .. .. .. .. .. .. . .. .. .. .. .. .. .. .. .. . . .. .. .. .. .. .. .. .. .. . .. ... .. .. .. .. .. .. .. . .. .. .. .. .. .. ... .. .. .. .. .. .. .. .. .. .. .. .. . .. .. .. .. .. .. .. .. .. .. .. .. .. .. .. .. .. .. .. .. .. .. .. .. .. .. .. .. ..

Formfactor

k=1

Scan interval [s1 ]

Figure 15.4: Form factor for exponentially distributed holding times which are observed by Erlang-k distributed scanning intervals in an unlimited measuring period. The case k = corresponds to regular (constant) scan intervals which transform the exponential distribution into Westerbergs distribution. The case k = 1 corresponds to exponentially distributed scan intervals (cf. the roulette simulation method). The case h = 0 corresponds to a continuous measurement. We notice that by regular scan intervals we loose almost no information if the scan interval is smaller than the mean holding time (chosen as time unit).

322 5 Relative accuracy of A

CHAPTER 15. TRAFFIC MEASUREMENTS

2 1 0.5

0.2 0.1 0.05

.... .... .... .... .... .... .... .... .... .... .... .... .... .... .... .... .... .... .... .... .... .... .... .... .... .... .... .... .... .... .... .... .... .... .... .... .... .... .... .... .... .... .... .... .... .... .... .... .... .... .... .... .... .... .... .... .... .... .... .... .... .... .... .... .... .... .... .... .... .... .... .... .... .... .... .... .... .... .... .... .... .... ... ... .... .... .... .... .... .... .... .. ... .. .... .... .... .... .... .... .. .. . ... .... .... .. .. .... .... .... ... .. . ....... .... .... .... .... .... ... ... .... .... .... .... .... .... .... ... .... .... .... .... ........... ... ....... .... .... .... .... .... ..... .... ... ..... .... .... .... .... .... .... ......... .... ... ..... .... .... .... .... . ........ ........ .... .... .... .... .. . ....... ....... .... .... .... .... ... .. .... .... .... ....... .... ...... .... .... ... .... .... ...... ...... ... . .... .... .... .... ... ... ...... .... ...... .... .... .... .. . .... .. .... .... .... ..... ... .. . ... .... .... .... .... ... ..... .... .... .... .... ..... ... .... .... .... .... ..... .... .... .... .... .... .... ..... .... .... .... .... ... .... .... .... .... .... ..... ... .... .... .... .... ... .... .... .... .... .... .... ... .... .... .... .... .... .... .... .... .... .... .... .... .... .... .... .... .... .... .... .... .... .... .... .... ..... .... .... .... .... .... .... .... .... .... .... .... ..... .... .... .... .... .... .... .... .... .... .... .... .... .... .... .... .... .... .... .... .... .... .... .... .... .... .... .... .... .... .... .... .... .... .... .... .... ..... .... .... .... .... .... .... .... .. .... .... . .... .... .... .... .... .... .... .... .... .... .... .... .... .... .... .... .... .... .. . .... .... .... .... .... .... .... .... ... .

A=1

h 6 3 0

0.02 1 2 5 10 20 50 100 200 500 AT Trac volume [s]

Figure 15.5: Using double-logarithmic scale we obtain a linear relationship between the relative accuracy of the trac intensity A and the measured trac volume A T when measuring in an unlimited time period. A scan interval h = 0 corresponds to a continuous measurement and h > 0 corresponds to the scanning method. The inuence of a limited measuring method is shown by the dotted line for the case A = 1 erlang and a continuous measurement taking account of the limited measuring interval. T is measured in mean holding times. By the scanning method we loose very little information as compared to a continuous measurement as long as the scanning interval is less than the mean holding time (cf. Fig. 15.4). A continuous measurement can be considered as an optimal reference for any discrete method. Exploitation of knowledge about a limited measuring period results in more information for a short measurement (T < 5), whereas we obtain little additional information for T > 10. (There is correlation in the trac process, and the rst part of a measuring period yields more information than later parts). By using the roulette method we loose of course more information than by the scanning method (Iversen 1976, [36], 1977 [37]). All the above mentioned factors have far less inuence than the fact that the real holding times often deviate from the exponential distribution. In practice we often observe a form

15.5. NUMERICAL EXAMPLE Example a


2 i

323 Example b
2 i

Continuous Method Unlimited (15.8) Limited

1.0000 1.0000 5.0000 2.2361 0.9500 0.9747 3.7729 1.9424

Scanning Method Unlimited (15.17) 1.0033 1.0016 5.4099 2.3259 Limited 0.9535 0.9765 4.2801 2.0688 Roulette Method Unlimited Limited 1.1000 1.0488 7.5000 2.7386 1.0500 1.0247 6.2729 2.5046

Table 15.2: Numerical comparison of various measuring principles in dierent time intervals. factor about 46. The conclusion to be made from the above examples is that for practical applications it is more relevant to apply the elementary formula (15.8) with a correct form factor than to take account of the measuring method and the measuring period. The above theory is exact when we consider charging of calls and measuring of time intervals. For stochastic computer simulations the trac process in usually stationary, and the theory can be applied for estimation of the reliability of the results. However, the results are approximate as the theoretical assumptions about congestion free systems seldom are of interest. In real life measurements on working systems we have trac variations during the day, technical errors, measuring errors etc. Some of these factors compensate each other and the results we have derived give a good estimate of the reliability, and it is a good basis for comparing dierent measurements and measuring principles.

324

CHAPTER 15. TRAFFIC MEASUREMENTS

BIBLIOGRAPHY

325

Bibliography
[1] Abate, J. & Whitt, W. (1997): Limits and approximations for the M/G/1 LIFO waiting time distribution. Operations Research Letters, Vol. 20 (1997) : 5, 199206. [2] Andersen, B. & Hansen, N.H. & og Iversen, V.B. (1971): Use of minicomputer for telephone trac measurements. Teleteknik (Engl. ed.) Vol. 15 (1971) : 2, 3346. [3] Ash, G.R. (1998): Dynamic routing in telecommunications networks. McGraw-Hill 1998. 746 pp. [4] Baskett, F. & Chandy, K.M. & Muntz, R.R. & Palacios, F.G. (1975): Open, closed and mixed networks of queues with dierent classes of customers. Journal of the ACM, April 1975, pp. 248260. (BCMP queueing networks). [5] Bear, D. (1988): Principles of telecommunication trac engineering. Revised 3rd Edition. Peter Peregrinus Ltd, Stevenage 1988. 250 pp. [6] Bech, N.I. (1954): A method of computing the loss in alternative trunking and grading systems. The Copenhagen Telephone Company, May 1955. 14 pp. Translated from Danish: Metode til beregning af sprring i alternativ trunking- og graderingssystemer. Teleteknik, Vol. 5 (1954) : 4, pp. 435448. [7] Bolotin, V.A. (1994): Telephone circuit holding time distributions. ITC 14, 14th International Teletrac Congress. Antibes Juan-les-Pins, France, June 6-10. 1994. Proccedings pp. 125134. Elsevier 1994. [8] Bretschneider, G. (1956): Bie Berechnung von Leitungsgruppen f r uberieenden u Verkehr. Nachrichtentechnische Zeitschrift, NTZ, Vol. 9 (1956) : 11, 533540. [9] Bretschneider, G. (1973): Extension of the equivalent random method to smooth tracs. ITC7, Seventh International Teletrac Congress, Stockholm, June 1973. Proceedings, paper 411. 9 pp. [10] Brockmeyer, E. (1954): The simple overow problem in the theory of telephone trac. Teleteknik 1954, pp. 361374. In Danish. English translation by Copenhagen Telephone Company, April 1955. 15 pp. [11] Brockmeyer, E. & Halstrm, H.L. & Jensen, Arne (1948): The life and works of A.K. Erlang. Transactions of the Danish Academy of Technical Sciences, 1948, No. 2, 277 pp. Copenhagen 1948. [12] Burke, P.J. (1956): The output of a queueing system. Operations Research, Vol. 4 (1956), 699704. [13] Christensen, P.V. (1914): The number of selectors in automatic telephone systems. The Post Oce Electrical Engineers Journal, Vol. 7 (1914), 271281.

326

BIBLIOGRAPHY

[14] Cobham, A. (1954): Priority assignment in waiting line problems. Operations Research, Vol. 2 (1954), 7076. [15] Conway, A.E. & Georganas, N.D. (1989): Queueing networks exact computational algorithms: A unied theory based on decomposition and aggregation. The MIT Press 1989. 234 pp. [16] Cooper, R.B. (1972): Introduction to queueing theory. New York 1972. 277 pp. [17] Cox, D.R. (1955): A use of complex probabilities in the theory of stochastic processes. Proc. Camb. Phil. Soc., Vol. 51 (1955), pp. 313319. [18] Cox, D.R. & Miller, H.D. (1965): The theory of stochastic processes. Methuen & Co. London 1965. 398 pp. [19] Cox, D.R.& Isham, V. (1980): Point processes. Chapman and Hall. 1980. 188 pp. [20] Crommelin, C.D. (1932): Delay probability formulae when the holding times are constant. Post Oce Electrical Engineers Journal, Vol. 25 (1932), pp. 4150. [21] Crommelin, C.D. (1934): Delay probability formulae. Post Oce Electrical Engineers Journal, Vol. 26 (1934), pp. 266274. [22] Delbrouck, L.E.N. (1983): On the steadystate distribution in a service facility carrying mixtures of trac with dierent peakedness factor and capacity requirements. IEEE Transactions on Communications, Vol. COM31 (1983) : 11, 12091211. [23] Dickmeiss, A. & Larsen, M. (1993): Sprringsberegninger i telenet (Blocking calculations in telecommunication networks, in Danish). Masters thesis. Institut for Telekommunikation, Danmarks Tekniske Hjskole, 1993. 141 pp. [24] Eilon, S. (1969): A simpler proof of L = W . Operations Research, Vol. 17 (1969), pp. 915917. [25] Elldin, A., and G. Lind (1964): Elementary telephone trac theory. Chapter 4. L.M. Ericsson AB, Stockholm 1964. 46 pp. [26] Engset, T.O. (1918): Die Wahrscheinlichkeitsrechnung zur Bestimmung der Whlerzahl a in automatischen Fernsprechmtern. Elektrotechnische Zeitschrift, 1918, Heft 31. Transa lated to English in Telektronikk (Norwegian), June 1991, 4pp. [27] Feller, W. (1950): An introduction to probability theory and its applications. Vol. 1, New York 1950. 461 pp. [28] Fortet, R. & Grandjean, Ch. (1964): Congestion in a loss system when some calls want several devices simultaneously. Electrical Communications, Vol. 39 (1964) : 4, 513526. Paper presented at ITC4, Fourth International Teletrac Congress, London. England, 1521 July 1964.

BIBLIOGRAPHY

327

[29] Fredericks, A.A. (1980): Congestion in blocking systems a simple approximation technique. The Bell System Technical Journal, Vol. 59 (1980) : 6, 805827. [30] Fry, T.C. (1928): Probability and its engineering uses. New York 1928, 470 pp. [31] Gordon, W.J., and & Newell, G.F. (1967): Closed queueing systems with exponential servers. Operations Research, Vol. 15 (1967), pp. 254265. [32] Grillo, D. & Skoog, R.A. & Chia, S. & Leung, K.K. (1998): Teletrac engineering for mobile personal communications in ITUT work: the need to match theory to practice. IEEE Personal Communications, Vol. 5 (1998) : 6, 3858. [33] Hayward, W.S. Jr. (1952): The reliability of telephone trac load measurements by switch counts. The Bell System Technical Journal, Vol. 31 (1952) : 2, 357377. [34] ITU-T (1993): Trac intensity unit. ITUT Recommendation B.18. 1993. 1 p. [35] Iversen, V.B. (1973): Analysis of real teletrac processes based on computerized measurements. Ericsson Technics, No. 1, 1973, pp. 164. Holbk measurements. [36] Iversen, V.B. (1976): On the accuracy in measurements of time intervals and trac intensities with application to teletrac and simulation. Ph.D.thesis. IMSOR, Technical University of Denmark 1976. 202 pp. [37] Iversen, V.B. (1976): On general point processes in teletrac theory with applications to measurements and simulation. ITC-8, Eighth International Teletrac Congress, paper 312/18. Melbourne 1976. Published in Teleteknik (Engl. ed.) 1977 : 2, pp. 5970. [38] Iversen, V.B. (1980): The Aformula. Teleteknik (English ed.), Vol. 23 (1980) : 2, 6479. [39] Iversen, V.B. (1982): Exact calculation of waiting time distributions in queueing systems with constant holding times. NTS-4, Fourth Nordic Teletrac Seminar, Helsinki 1982. 31 pp. [40] Iversen, V.B. (1987): The exact evaluation of multiservice loss system with access control. Teleteknik, English ed., Vol 31 (1987) : 2, 5661. NTS7, Seventh Nordic Teletrac Seminar, Lund, Sweden, August 2527, 1987, 22 pp. [41] Iversen, V.B. & Nielsen, B.F. (1985): Some properties of Coxian distributions with applications. Proceedings of the International Conference on Modelling Techniques and Tools for Performance Analysis, pp. 6166. 57 June, 1985, Valbonne, France. North Holland Publ. Co. 1985. 365 pp. (Editor N. Abu El Ata). [42] Iversen, V.B. & Stepanov, S.N. (1997): The usage of convolution algorithm with truncation for estimation of individual blocking probabilities in circuit-switched telecommunication networks. Proceedings of the 15th International Teletrac Congress, ITC 15, Washington, DC, USA, 2227 June 1997. 13271336.

328

BIBLIOGRAPHY

[43] Iversen, V.B. & Sanders, B. (2001): Engset formul with continuous parameters theory and applications. AEU, International Journal of Electronics and Communications, Vol. 55 (2001) : 1, 3-9. [44] Iversen, V.B. (2005): Algorithm for evaluating multi-rate loss systems. COM Department, Technical University of Denmark. December 2005. 27 pp. Submitted for publication. [45] Jackson, R.R.P. (1954): Queueing systems with phase type service. Operational Research Quarterly, Vol. 5 (1954), 109120. [46] Jackson, J.R. (1957): Networks of waiting lines. Operations Research, Vol. 5 (1957), pp. 518521. [47] Jackson, J.R. (1963): Jobshoplike queueing systems. Management Science, Vol. 10 (1963), No. 1, pp. 131142. [48] Jensen, Arne (1948): An elucidation of A.K. Erlangs statistical works through the theory of stochastic processes. Published in The Erlangbook: E. Brockmeyer, H.L. Halstrm and A. Jensen: The life and works of A.K. Erlang. Kbenhavn 1948, pp. 23100. [49] Jensen, Arne (1948): Truncated multidimensional distributions. Pages 5870 in The Life and Works of A.K. Erlang. Ref. Brockmeyer et al., 1948 [48]. [50] Jensen, Arne (1950): Moes Principle An econometric investigation intended as an aid in dimensioning and managing telephone plant. Theory and Tables. Copenhagen 1950. 165 pp. [51] Jerkins, J.L. & Neidhardt, A.L. & Wang, J.L. & Erramilli A. (1999): Operations measurement for engineering support of high-speed networks with self-similar trac. ITC 16, 16th International Teletrac Congress, Edinburgh, June 711, 1999. Proceedings pp. 895906. Elsevier 1999. [52] Johannsen, Fr. (1908): Busy. Copenhagen 1908. 4 pp. [53] Johansen, K. & Johansen, J. & Rasmussen, C. (1991): The broadband multiplexer, TransMux 1001. Teleteknik, English ed., Vol. 34 (1991) : 1, 5765. [54] Joys, L.A.: Variations of the Erlang, Engset and Jacobus formul. ITC5, Fifth International Teletrac Congress, New York, USA, 1967, pp. 107111. Also published in: Teleteknik, (English edition), Vol. 11 (1967) :1 , 4248. [55] Joys, L.A. (1968): Engsets formler for sannsynlighetstetthet og dens rekursionsformler. (Engsets formul for probability and its recursive formul, in Norwegian). Telektronikk 1968 No 12, pp. 5463. [56] Joys, L.A. (1971): Comments on the Engset and Erlang formulae for telephone trac losses. Thesis. Report TF No. 25/71, Research Establishment, The Norwegian Telecommunications Administration. 1971. 127 pp.

BIBLIOGRAPHY

329

[57] Karlsson, S.A. (1937): Tekniska anordninger fr samtalsdebitering enligt tid (Technio cal arrangement for charging calls according to time, In Swedish). Helsingfors Telefonfrening, Tekniska Meddelanden 1937, No. 2, pp. 3248. o [58] Kaufman, J.S. (1981): Blocking in a shared resource environment. IEEE Transactions on Communications, Vol. COM29 (1981) : 10, 14741481. [59] Keilson, J. (1966): The ergodic queue length distribution for queueing systems with nite capacity. Journal of Royal Statistical Society, Series B, Vol. 28 (1966), 190201. [60] Kelly, F.P. (1979): Reversibility and stochastic networks. John Wiley & Sons, 1979. 230 pp. [61] Kendall, D.G. (1951): Some problems in the theory of queues. Journal of Royal Statistical Society, Series B, Vol. 13 (1951) : 2, 151173. [62] Kendall, D.G. (1953): Stochastic processes occuring in the theory of queues and their analysis by the method of the imbedded Markov chain. Ann. Math. Stat., Vol. 24 (1953), 338354. [63] Khintchine, A.Y. (1955): Mathematical methods in the theory of queueing. London 1960. 124 pp. (Original in Russian, 1955). [64] Kingman, J.F.C. (1969): Markov population processes. J. Appl. Prob., Vol. 6 (1969), 118. [65] Kleinrock, L. (1964): Communication nets: Stochastic message ow and delay. McGraw Hill 1964. Reprinted by Dover Publications 1972. 209 pp. [66] Kleinrock, L. (1975): Queueing systems. Vol. I: Theory. New York 1975. 417 pp. [67] Kleinrock, L. (1976): Queueing systems. Vol. II: Computer applications. New York 1976. 549 pp. [68] Kosten, L. (1937): Uber Sperrungswahrscheinlichkeiten bei Staelschaltungen. Elek. Nachr. Techn., Vol. 14 (1937) 512. [69] Kruithof, J. (1937): Telefoonverkehrsrekening. De Ingenieur, Vol. 52 (1937) : E15E25. [70] Kuczura, A. (1973): The interrupted Poisson process as an overow process. The Bell System Technical Journal, Vol. 52 (1973) : 3, pp. 437448. [71] Kuczura, A. (1977): A method of moments for the analysis of a switched communication networks performance. IEEE Transactions on Communications, Vol. Com25 (1977) : 2, 185193. [72] Lavenberg, S.S. & Reiser, M. (1980): Meanvalue analysis of closed multichain queueing networks. Journal of the Association for Computing Machinery, Vol. 27 (1980) : 2, 313 322.

330

BIBLIOGRAPHY

[73] Lind, G. (1976): Studies on the probability of a called subscriber being busy. ITC8, Eighth International Teletrac Congress, Melbourne, November 1976. Paper 631. 8 pp. [74] ListovSaabye, H. & Iversen V.B. (1989): ATMOS: a PCbased tool for evaluating multiservice telephone systems. IMSOR, Technical University of Denmark 1989, 75 pp. (In Danish). [75] Little, J.D.C. (1961): A proof for the queueing formula L = W . Operations Research, Vol. 9 (1961) : 383387. [76] Maral, G. (1995): VSAT networks. John Wiley & Sons, 1995. 282 pp. [77] Marchal, W.G. (1976): An approximate formula for waiting time in single server queues. AIIE Transactions, December 1976, 473474. [78] Palm, C. (1941): Mttnoggrannhet vid bestmning af trakmngd enligt genomska a a o ningsfrfarandet (Accuracy of measurements in determining trac volumes by the scano ning method). Tekn. Medd. K. Telegr. Styr., 1941, No. 79, pp. 97115. [79] Palm, C. (1943): Intensittsschwankungen im Fernsprechverkehr. Ericsson Technics, No. a 44, 1943, 189 pp. English translation by Chr. Jacobus: Intensity Variations in Telephone Trac. NorthHolland Publ. Co. 1987. [80] Palm, C. (1947): The assignment of workers in servicing automatic machines. Journal of Industrial Engineering, Vol. 9 (1958) : 2842. First published in Swedish in 1947. [81] Palm, C. (1947): Table of the Erlang loss formula. Telefonaktiebolaget L M Ericsson, Stockholm 1947. 23 pp. [82] Palm, C. (1957): Some propositions regarding at and steep distribution functions, pp. 317 in TELE (English edition), No. 1, 1957. [83] PostigoBoix, M. & Garc iaHaro, J. & AguilarIgartua, M. (2001): (Inverse Multiplexing of ATM) IMA technical foundations, application and performance analysis. Computer Networks, Vol. 35 (2001) 165183. [84] Press, W.H. & Teukolsky, S.A. & Vetterling, W.T. & Flannery, B.P. (1995): Numerical recipes in C, the art of scientic computing. 2nd edition. Cambridge University Press, 1995. 994 pp. [85] Rabe, F.W. (1949): Variations of telephone trac. Electrical Communications, Vol. 26 (1949) 243248. [86] Rapp, Y. (1965): Planning of junction network in a multiexchange area. Ericsson Technics 1965, No. 2, pp. 187240. [87] Riordan, J. (1956): Derivation of moments of overow trac. Appendix 1 (pp. 507514) in (Wilkinson, 1956 [102]).

BIBLIOGRAPHY

331

[88] Roberts, J.W. (1981): A service system with heterogeneous user requirements applications to multiservice telecommunication systems. Performance of data communication systems and their applications. G. Pujolle (editor), NorthHolland Publ. Co. 1981, pp. 423431. [89] Roberts, J.W. (2001): Trac theory and the Internet. IEEE Communications Magazine Vol. 39 (2001) : 1, 9499. [90] Ross, K.W. & Tsang, D. (1990): Teletrac engineering for productform circuit switched networks. Adv. Appl. Prob., Vol. 22 (1990) 657675. [91] Ross, K.W. & Tsang, D. (1990): Algorithms to determine exact blocking probabilities for multirate tree networks. IEEE Transactions on Communications. Vol. 38 (1990) : 8, 12661271. [92] Rnnblom, N. (1958): Trac loss of a circuit group consisting of bothway circuits which o is accessible for the internal and external trac of a subscriber group. TELE (English edition), 1959 : 2, 7992. [93] Sanders, B. & Haemers, W.H. & Wilcke, R. (1983): Simple approximate techniques for congestion functions for smooth and peaked trac. ITC10, Tenth International Teletrac Congress, Montreal, June 1983. Paper 4.4b1. 7 pp. [94] Stepanov, S.S. (1989): Optimization of numerical estimation of characteristics of multiow models with repeated calls. Problems of Information Transmission, Vol. 25 (1989) : 2, 6778. [95] Sutton, D.J. (1980): The application of reversible Markov population processes to teletrac. A.T.R. Vol. 13 (1980) : 2, 38. [96] Techguide (2001): Inverse Multiplexing scalable bandwidth solutions for the WAN. Techguide (The Technologu Guide Series), 2001, 46 pp. <www.techguide.com> [97] Vaulot, E. & Chaveau, J. (1949): Extension de la formule dErlang au cas ou le trac est fonction du nombre dabonns occups. Annales de Tlcommunications, Vol. 4 (1949) e e ee 319324. [98] Veir, B. (2002): Proposed Grade of Service chapter for handbook. ITUT Study Group 2, WP 3/2. September 2001. 5 pp. [99] Villn, M. (2002): Overview of ITU Recommendations on trac engineering. ITUT e Study Group 2, COM 2-KS 48/2-E. May 2002. 21 pp. [100] Wallstrm, B. (1964): A distribution model for telephone trac with varying call o intensity, including overow trac. Ericsson Technics, 1964, No. 2, pp. 183202. [101] Wallstrm, B. (1966): Congestion studies in telephone systems with overow facilities. o Ericsson Technics, No. 3, 1966, pp. 187351.

332

BIBLIOGRAPHY

[102] Wilkinson, R.I. (1956): Theories for toll trac engineering in the U.S.A. The Bell System Technical Journal, Vol. 35 (1956) 421514.

Author index
Abate, J., 256, 325 AguilarIgartua, M., 179, 330 Andersen, B., 310, 325 Ash, G.R., 325 Baskett, F., 297, 325 Bear, D., 214, 325 Bech, N.I., 170, 325 Bolotin, V.A., 325 Bretschneider, G., 171, 174, 325 Brockmeyer, E., 170, 268, 325 Burke, P.J., 285, 286, 325 Buzen, J.P., 292 Chandy, K.M., 297, 325 Chaveau, J., 331 Chia, S., 327 Christensen, P.V., 325 Cobham, A., 262, 326 Conway, A.E., 304, 326 Cooper, R.B., 326 Cox, D.R., 87, 326 Crommelin, C.D., 269, 326 Delbrouck, L.E.N., 210, 326 Dickmeiss, A., 326 Eilon, S., 101, 326 Elldin, A., 326 Engset, T.O., 153, 326 Erlang, A.K., 40, 92, 128 Erramilli A., 328 Feller, W., 72, 237, 316, 326 Flannery, B.P., 330 Fortet, R., 206, 326 Fredericks, A.A., 176, 327 Fry, T.C., 106, 269, 270, 327 Garc iaHaro, J., 179, 330 333 Georganas, N.D., 304, 326 Gordon, W.J., 287, 327 Grandjean, Ch., 206, 326 Grillo, D., 327 Haemers, W.H., 180, 331 Halstrm, H.L., 325 Hansen, N.H., 310, 325 Hayward, W.S. Jr., 176, 319, 327 Isham, V., 326 ITU-T, 327 Iversen, V.B., 43, 45, 46, 88, 89, 93, 148, 163, 197, 199, 210, 271, 272, 310, 312, 318, 319, 322, 325, 327, 328, 330 Jackson, J.R., 286, 287, 328 Jackson, R.R.P., 328 Jensen, Arne, 106, 129, 139, 188, 192, 219, 220, 227, 231, 232, 325, 328 Jerkins, J.L., 328 Johannsen, F., 50, 328 Johansen, J., 179, 328 Johansen, K., 179, 328 Joys, L.A., 151, 328 Karlsson, S.A., 311, 329 Kaufman, J.S., 206, 329 Keilson, J., 257, 329 Kelly, F.P., 253, 285, 329 Kendall, D.G., 249, 278, 279, 329 Khintchine, A.Y., 95, 269, 329 Kingman, J.F.C., 189, 329 Kleinrock, L., 260, 281, 289, 306, 307, 329 Kosten, L., 169, 329 Kruithof, J., 329 Kuczura, A., 119, 181, 182, 329

334 Larsen, M., 326 Lavenberg, S.S., 294, 329 Leung, K.K., 327 Lind, G., 326, 330 Listov-Saabye, H., 199, 330 Little, J.D.C., 330 Maral, G., 11, 330 Marchal, W.G., 277, 330 Miller, H.D., 326 Moe, K., 139 Muntz, R.R., 297, 325 Neidhardt, A.L., 328 Newell, G.F., 287, 327 Nielsen, B.F., 88, 89, 327 Palacios, F.G., 297, 325 Palm, C., 62, 82, 92, 114, 136, 236, 316, 319, 330 PostigoBoix, M., 179, 330 Press, W.H., 330 Rnnblom, N., 193, 331 o Rabe, F.W., 316, 330 Raikov, D.A., 117 Rapp, Y., 173, 330 Rasmussen, C., 179, 328 Reiser, M., 294, 329 Riordan, J., 169, 330 Roberts, J.W., 206, 331 Ross, K.W., 210, 331 Samuelson, P.A., 139 Sanders, B., 148, 180, 328, 331 Skoog, R.A., 327 Stepanov, S.N., 135, 199, 327, 331 Sutton, D.J., 189, 331 Techguide, 179, 331 Teukolsky, S.A., 330 Tsang, D., 210, 331 Vaulot, E., 331 Veir, B., 55, 331 Vetterling, W.T., 330 Villn, M., 331 e Wallstrm, B., 162, 171, 331 o Wang, J.L., 328 Whitt, W., 256, 325 Wilcke, R., 180, 331 Wilkinson, R.I., 171, 332

Author index

Index
A-subscriber, 7 accessibility full, 121 delay system, 223 Engset, 145 Erlang-B, 121 restricted, 167 ad-hoc network, 116 Aloha protocol, 111, 126 alternative routing, 167, 168, 217 arrival process generalised, 181 arrival theorem, 154, 294 assignment demand, 11 xed, 11 ATMOS-tool, 199 availability, 121 B-ISDN, 8 B-subscriber, 7 balance detailed, 189 global, 186 local, 189 balance equations, 124 balking, 252 BCMP queueing networks, 297, 325 Berkeleys method, 180 billing, 319 Binomial distribution, 115, 147 trac characteristics, 150 truncated, 153 Binomial expansion, 300 Binomial process, 113, 115 Binomial-case, 146 blocking concept, 45 BPP-trac, 147, 190, 192 335 Brockmeyers system, 169, 170 Burkes theorem, 285 bursty trac, 170 Busy, 50 busy hour, 42, 44 time consistent, 44 Buzens algorithm, 292 call duration, 49 call intensity, 41 capacity allocation, 305 carried trac, 40, 128 carrier frequency system, 10 CCS, 41 central moment, 63 central server system, 292, 293 chain queueing network, 284, 297 channel allocation, 14 charging, 311 circuit-switching, 10 circulation time, 238 class limitation, 191 client-server, 237 code receiver, 7 code transmitter, 7 coecient of variation, 63, 317 complementary distribution function, 62 compound distribution, 72 Poisson distribution, 316 concentration, 45 condence interval, 320 congestion call, 46, 128, 199 time, 47, 128, 199 trac, 47, 128, 199 virtual, 47 connection-less, 11

336 conservation law, 260 control channel, 14 control path, 6 convolution, 70, 110 convolution algorithm loss systems, 196 multiple chains, 301 single chain, 290 cord, 7 Cox distribution, 84 Cox2 arrival process, 182 CSMA, 12 cut equations, 123 cyclic search, 8 D/M/1, 280 data signalling speed, 42 de-convolution, 199 death rate, 65 decomposition, 88 decomposition theorem, 117 DECT, 15 Delbroucks algorithm, 210 density function, 62 dimensioning, 138 xed blocking, 139 improvement principle, 140 direct route, 167 distribution function, 61 drop tail, 257 Ek /D/r, 274 EBHC, 41 EERTmethod, 174 Engset distribution, 152 Engsets formula recursion, 158 Engset-case, 146 equilibrium points, 256 equivalent bandwidth, 33 equivalent system, 172 erlang, 39 Erlang x-point method, 211 Erlangs 1. formula, 127 Erlangs B-formula, 127, 128

INDEX hyper-exponential service, 186, 187 multi-dimensional, 185 recursion, 136 Erlangs C-formula, 225, 226 Erlangs delay system, 223 state transition diagram, 224 Erlang-case, 146 Erlang-k distribution, 80, 115 ERTmethod, 171 exponential distribution, 77, 109, 115 in parallel, 82 decomposition, 88 in series, 80 minimum of k, 79 fair queueing, 281 Feller-Jensens identity, 106 at distribution, 82 at rate, 320 ow-balance equation, 286 forced disconnection, 48 form factor, 63 Fortet & Grandjean algorithm, 206 forward recurrence time, 68 Fredericks & Haywards method, 176 gamma distribution, 91 geometric distribution, 115 GI/G/1, 276 GI/M/1, 278 FCFS, 280 GoS, 138 Grade-of-Service, 138 GSM, 15 hand-over, 15 hazard function, 65 HCS, 175 heavy-tailed distribution, 93, 163 hierarchical cellular system, 175 HOL, 252 hub, 11 human-factors, 52 hyper-exponential distribution, 83 hypoexponential, 80

INDEX IDC, 97 IDI, 98 IID, 98 IMA, 179 improvement function, 129, 231 improvement principle, 140 improvement value, 141, 143 independence assumption, 289 index of dispersion counts, 97 intervals, 98 insensitivity, 130 Integrated Services Digital Network, 8 intensity, 115 inter-active system, 238 interrupted Poisson process, 119, 181 interval representation, 96, 106, 310 inverse multiplexing, 179 IPP, 119, 181 Iridium, 15 ISDN, 8 iterative studies, 3 ITU-T, 222 Jackson net, 286 jockeying, 253 Karlsson charging, 311, 318, 320 Kaufman & Roberts algorithm, 206 Kingmans inequality, 277 Kleinrocks square root law, 306 Kolmogorovs criteria, 189, 190 Kostens system, 169 Kruithofs double factor method, 212 lack of memory, 65 Lagrange multiplier, 220, 232, 306 LAN, 12 last-look principle, 311 LCC, 122 leaky bucket, 276 lifetime, 61 line-switching, 10 Littles theorem, 101 load function, 259, 260 local exchange, 9 log-normal distribution, 93 loss system, 46 lost calls cleared, 122 M/D/1/k, 275 M/D/n, 268, 273 M/G/, 285 M/G/1, 254 M/G/1-LCFS-PR, 285 M/G/1-PS, 285 M/G/1/k, 257 M/M/1, 230, 298 M/M/n, 223, 285, 301 M/M/n, FCFS, 233 M/M/n/S/S, 237 machine repair model, 223 macrocell, 175 man-machine, 2 Marchals approximation, 277 Markovian property, 65 mean value, 63 mean waiting time, 229 measuring methods, 310 continuous, 310, 314 discrete, 310 horizontal, 312 vertical, 311 measuring period unlimited, 314, 317 mesh network, 9, 11 message-switching, 12 microcell, 175 microprocessor, 6 mobile communication, 13 modelling, 2 Moes principle, 139, 218, 231, 328 delay systems, 231 loss systems, 140 multi-dimensional Erlang-B, 185 loss system, 191 multi-rate trac, 177, 192 multi-slot trac, 192 multinomial distribution, 88 multiplexing

337

338 frequency, 10 pulse-code, 10 time, 10 MVA-algorithm single chain, 284, 294 negative Binomial case, 147 negative Binomial distribution, 115 network management, 222 Newton-Raphsons method, 173 node equations, 123 non-central moment, 62 non-preemptive, 252 notation distributions, 92 Kendalls, 249 number representation, 96, 106, 310 ODell grading, 168 oered trac, 40 denition, 122, 146 on/o source, 148 overow theory, 167 packet switching, 11 paging, 15 Palms form factor, 63 Palms identity, 62 Palms machine-repair model, 238 optimising, 246 Palms theorem, 114 Palm-Wallstrm-case, 147 o paradox, 235 parcel blocking, 174 Pareto distribution, 91, 93, 163 Pascal distribution, 115 Pascal-case, 147 PASTA property, 129, 186 PCM-system, 10 PCT-I, 122, 146 PCT-II, 147, 148 peakedness, 126, 130, 170 persistence, 52 point process, 95 independence, 100 simple, 95, 101 stationary, 100 Poisson distribution, 112, 115, 122 calculation, 136 truncated, 127 Poisson process, 105, 115 Poisson-case, 146 polynomial distribution, 88, 300 polynomial trial, 88 potential trac, 42 preemptive, 252 preferential trac, 53 primary route, 167 Processor-Sharing, 281 product form, 186, 286 protocol, 8 PS, 281 pseudo random trac, 148 Pure Chance Trac Type I, 122, 146 Type II, 147 QoS, 138 Quality-of-Service, 138 queueing networks, 283

INDEX

Raikovs theorem, 117 random trac, 146 random variable, 61 in parallel, 71 in series, 70 jth largest, 69 Rapps approximation, 174 reduced load method, 211 regeneration points, 256 regenerative process, 256 register, 6, 7 rejected trac, 41 relative accuracy, 317 reneging, 252 renewal process, 98 residual lifetime, 64 response time, 236 reversible process, 189, 190, 253, 285 ring network, 9 roaming, 15

INDEX roulette simulation, 323 Round Robin, 281 RR, 281 sampling theory, 312 Sanders method, 180 scanning method, 311, 317 secondary route, 167 service protection, 167 service ratio, 247 service time, 49 SJF, 263 slot, 112 SM, 41 smooth trac, 152, 170 sojourn time, 236 space divided system, 6 SPC-system, 7 sporadic source, 148 square root law, 306 standard deviation, 63 star network, 9 state transition diagram general procedure, 134 statistical equilibrium, 124 statistical multiplexing, 45 steep distributions, 80 stochastic process, 4 stochastic sum, 72 store-and-forward, 11 strategy, 3 structure, 3 subscriber-behaviour, 52 superposition theorem, 114 survival distribution function, 62 symmetric queueing systems, 253, 285 table Erlangs B-formula, 136 telecommunication network, 9 telephone system conventional, 5 software controlled, 7 teletrac theory terminology, 4 trac concepts, 39 time distributions, 61 time division, 6 time-out, 48 trac channels, 14 trac concentration, 46 trac intensity, 39, 315 trac matrix, 211 trac measurements, 309 trac splitting, 177 trac unit, 39 trac variations, 42 trac volume, 40, 315 transit exchange, 9 transit network, 9 triangle optimisation, 221 user perceived QoS, 46 utilisation, 42, 140 variance, 63 variate, 61 virtual circuit protection, 191 virtual queue length, 227 virtual waiting time, 253, 260 voice path, 6 VSAT, 11 waiting time distribution, 66 FCFS, 233 Weibull distribution, 66, 91 Westerbergs distribution, 319 Wilkinsons equivalence method, 171 wired logic, 3 work conservation, 259

339

340

INDEX

Technical University of Denmark COM Center, Networks

Teletrac Engineering & Network Planning Course 34 340

Exercise 2.1

(Training)

OFFERED TRAFFIC

1. We consider an Internet-caf. Customers arrive at random. On the average 20 customers e arrive per hour. The average time using a terminal is 15 minutes. Quest. 1.1: Find the oered trac measured in speech minutes during one hour. Quest. 1.2: Find the oered trac measured in erlangs. 2. We consider a cell in a cellular system. There are two arrival processes. Hand-over calls arrive with rate 3 calls per minute, and the mean holding time is 90 seconds. New Calls arrive with 240 calls per hour and the mean holding time is 2 minutes. Quest. 2.1: Find the oered trac for each trac stream and the total oered trac. 3. To a computer system three types of tasks arrive: a) inter-active tasks, b) test tasks, and c) production tasks. All tasks arrive according to a Poissonproces, and the service times are constant. For type a) 15 tasks arrive per minute, and the service time is 1 second. For type b) 3 tasks arrive per minute, and the service time is 5 seconds. For type c) 12 tasks arrive per hour, and the service time is 2 minutes. Quest. 3.1: Find the oered trac for each type and the total oered trac. 4. The arrival process to a systems occurs according to a Poissonproces with rate = 2 calls per time unit. Every call occupies two channels during the whole occupation time, which is exponentially distributed with mean value s = 3 time units. Quest. 4.1: Find the oered trac in calls (connections). Quest. 4.1: Find the oered trac in channels. 5. We consider trac to a digital exchange oering ISDN calls (1 channel per call) and ISDN2 calls (2 channels per call): ISDN calls: Per hour 900 calls arrive and the mean holding time is 2 minutes. ISDN2 calls: Per minute 2 calls arrive and the mean holding time is 150 seconds.

2 Quest. 5.1: Find the oered trac (measured in channels) for each type and the total oered trac. 6. A digital 2.048 Mbps (Mbps = Mega bits per second) link is on the average oered 128 packets per second. A packet contains on the average 1500 bytes (1 byte = 8 bits). Quest. 6.1: Find the utilisation of the link.

1 Technical University of Denmark COM Center, Networks Teletrac Engineering & Network Planning Course 34 340

Solution to Exercise 2.1: OFFERED TRAFFIC Question 1: During one hour the number of speech minutes [SM] is: 20 15 minutes = 300 [SM] This is a trac volume and corresponds to 300/60 = 5 [Eh] (erlanghours). Using minutes as time unit the oered trac in erlang (2.2) becomes: A= 20 15 = 5 [erlang] 60

in agreement with that the trac volume per hour is 5 [Eh]. Question 2: Using the time unit [minutes] we get Hand-over trac: A = 3 New calls trac: Thus the total oered trac becomes: A = 4.5 + 8 = 12.5 [erlang] We get of course the same result using for example hours or seconds as time unit Question 3: Using minutes as time unit we get: Type a: Type b: Type c: Total: Aa = 15 Ab = Ac = At = 3 1 = 0.25 [erlang] 60 5 = 0.25 [erlang] 60 90 = 4.5 [erlang] 60 240 A= 2 = 8 [erlang] 60

12 2 = 0.4 [erlang] 60 0.9 [erlang]

2 Question 4: The oered trac in calls becomes, using the same time unit: Acalls = 2 3 = 6 [erlang (calls)] As every call uses 2 channels the oered trac in channels becomes: Achannels = 6 2 = 12 [erlang (channels)] Question 5: We want to nd the oerd trac in the unit [channels]. We nd using the time unit [minutes]: AISDN = 900 2 60 = 30 [erlang]

AISDN2 = 2 Atotal Question 6:

150 2 = 10 [erlang] 60 = 40 [erlang]

= 30 + 10

Per second the oered trac in bits per second becomes: A = 128 byte bit packet 1500 8 = 1, 536, 000 bits per second. second packet byte =
2006.02.12

Thus the utilization becomes:

1, 536, 000 = 0.75 2, 048, 000

1 Technical University of Denmark COM Center, Networks Teletrac Engineering & Network Planning Course 34 340

Exercise 7.9

(Exam 1996)

ERLANGS LOSS SYSTEM We consider Erlangs loss system with n = 3 servers. The arrival process is a Poisson process with intensity = 0.5 calls per time unit. The service time is exponentially distributed with mean value 4 time units 1. Find the oered trac A. 2. Construct the state transition diagram, and nd the state probabilities when the system is in statistical equilibrium. 3. (Not to be solved 2006) Set up the dierential eguations which describe the system (only for state 2). 4. Find the blocking probability E by applying the recursion formula for Erlangs Bformula (the individual recursions should be visible from the answer). 5. Find under assumption of sequential hunting the trac carried by the individual server. 6. Show that the state probabilities of the system observed by a call (customer) who just left the system (is looking back) are the same as observed by a customer just before arriving at the system when we include blocked customers. (Use the state transition diagram to nd how many customers are arriving/leaving the system in each state).

1 Technical University of Denmark COM Center, Networks Teletrac Engineering & Network Planning Course 34 340

Solution to Exercise 7.9: ERLANGS LOSS SYSTEM Question 1: Oered trac A:

(Exam 1996)

A = / = 0.5/0.25 = 2 [erlang]. Question 2: The state transition diagram becomes as follows:


In statistical equilibrium we get the following cut equations:


1 2 1 2 1 2

p0 = p1 = p2 =

1 4 2 4 3 4

p1 , p2 , p3 ,

p1 = 2 p0 , p2 = p1 = 2 p0 , p3 =
2 3 4 3

p2 =

p0 .

The normalisation condition results in:


3

pi = p0 1 + 2 + 2 +
i=0

4 3

= 1,

p0 = p1 = p2 = p3 =

3 = 0.1579 , 19 6 = 0.3158 , 19 6 = 0.3158 , 19 4 = 0.2105 . 19

2 This result may also be obtained by using formula (7.9) in the textbook: Ai pi = i!
3

i=0

A !

2i /i! . 1 + 2 + 2 + 4/3

Question 3: The dierential equations describing the system are given by (??), (??) and (??): i = 0 : Pt {0, t} = P {0, t} + P {1, t} , 0 < i < n : Pt {i, t} = + P {i 1, t} ( + i ) P {i, t} + (i + 1) P {i + 1, t} , i = n : Pt {n, t} = P {n 1, t} n P {n, t} . These equations are easily obtained in the same way as the node balance equations. 1 1 Pt (0, t) = P (0, t) + P (1, t) , 2 4 Pt (1, t) = Pt (2, t) = 1 1 + 4 2 2 1 + 4 2 P (1, t) + P (2, t) + 2 1 P (0, t) + P (2, t) , 2 4 1 3 P (1, t) + P (3, t) , 2 4

1 3 Pt (3, t) = P (3, t) + P (2, t) . 4 2 In this question only Pt (2, t) was asked for. (We notice that all states are time-dependent. The rate of change of a state probability (the derivative) is increased when we move into the state and decreased when we leave the state. In statistical equilibrium all derivatives become zero, and all state probabilities become independent of time, and thus we get the node equations). Question 4: The recursion formula is given by (7.29): En (A) = A En1 (A) , n + A En1 (A)

E0 (A) = 1 .

3 For A = 2 we get: E1 (2) = 2 21 = , 1+21 3


2 3

2 2 3 E2 (2) = 2+2 E3 (2) = 2 2 5 3+2

2 , 5 4 . 19

2 5

E3 (2) = 4/19, which is in agreement with Question 1. Question 5: From Question 4 we obtain the following carried trac as a function of the number of channels: Ac (i) = A {1 Ei (A)} n=0 : n=1 : Ac (0) = 0.0000 , Ac (1) = 2 1 Ac (2) = 2 1 Ac (3) = 2 1 2 3 2 5 4 19 = 2 = 0.6667 , 3 6 = 1.2000 , 5 30 = 1.5789 . 19

n=2 :

n=3 :

If weaAssume ordered hunting (sequential hunting with homing), then channel i carries the trac ai : ai = Ac (i) Ac (i 1) = A {Ei1 (A) Ei (A)} , a1 = a2 = a3 = 2 = 0.6667 , 3 8 = 0.5333 , 15 36 = 0.3789 . 95

This is in agreement with the table given on the home page (Improvement function for Erlang-B).

4 Question 6: Arriving customers observe the system in state i with probability: i = pi = pi . p0 + p1 + p2 + p3

We look at one time unit and consider mean values (statistical equilibrium). During this time unit customers arrive, and pi customers arrive at the system in state i and bring the system into state i + 1. So a random customer observe the state i just before arriving. This is in agreement with the PASTA-property (Poisson Arrivals See Time Averages). Departing customers which look back just after leaving the system observe the system in state i with probability i : i = (i + 1) pi+1 , p1 + 2 p2 + 3 p3 + l l , p1 + 2 p2 + 3 p3 + l i = 0, 1, 2 ,

3 =

where l is the intensity of blocked customers l = p3 . Again we consider one time unit and mean values. During one time unit i pi customers depart from state i and thus observe state i 1 immediately after they have left. As for blocked customers arriving in state 3 with intensity they observe state 3 when they look back. So if we ask a random customer he observed state i . Applying cut equations we get: p1 = p0 , 2 p2 = p1 , 3 p3 = p2 , and thus: i = pi = pi = i , p0 + p1 + p2 + p3 i = pi = i . The state probabilities immediately before an arrival i are thus equal to the state probabilities immediately after a departure i , and they are also equal to the state probabilities pi at a random point of time.
Updated 2006-02-09

1 Technical University of Denmark COM Center, Networks Teletrac Engineering & Network Planning Course 34 340

Exercise 7.1 ERLANGs BFORMULA At a shopping center there is a gambling hall. People passing by decide at random and independently of each other to enter and play, but if all gambling machines are occupied, then they pass on (alternativ formulation as an Internet cafe). During opening hours there are on the average 40 people per hour which enter to play. People choose the rst idle machine from the entrance and play on the average 6 minutes (exponentially distributed). A gambling machine make on the average an income equal to 100 re per minute it is used. The total expenses for rent of rooms and maintenance is 20 kr. per hour per machine, independent of whether it is used or not. Coins of an equivalent value equal to 100 re are used. (In the following Erlangs B-formula may be calculated using the recursion formula, tables or computers).

1. Calculate the oered trac. 2. What is the net income when the number of machines is 4? 3. Is it protable to have more or fewer machines than 4? What is the optimal number of machines?

In the following we assume the number of machines is 4. 4. How many coins may the owner expect to collect from each machine after 12 hours of opening? 5. How long time does it on the average take before the last customer leaves after closing time, when there is 1, 2, 3 or 4 people playing at the closing time? 6. What is the proportion of time when just one machine (a random one) is idle? 7. What is the proportion of time the machine farthest away from the entrance is idle?

1 Technical University of Denmark COM Center, Networks Teletrac Engineering & Network Planning Course 34 340

Solution to Exercise 7.1 The Erlang Bformula is derived in the textbook in Sec. 7.3. Question 1: The oered trac is equal to the average number of calls per mean holding time (2.2): A = / . Four potential customers arrive per six minutes: A = 4 erlang .

Question 2: Using a table of Erlangs B-formula (see table in the collection of exercises) we nd for the trac carried: Y = A [1 E1,n (A)] = 4 [1 E1,4 (4)] = 4 [1 0.3107] , Y = 2.757 erlang.

The gross income is 60 kr. per erlanghour. Therefore, the net income becomes R = (2.757 60 4 20) kr/hour, R = 85.42 kr/hour . Question 3: We evaluate the net income for dierent number number of machines (see table in collection of exercises, or calculate the ErlangB formula using the recursion formula (7.29)):

Number of machines n 1 2 3 4 5 6

Carried trac Y 0.8000 1.5385 2.1972 2.7573 3.2037 3.5313

Income per hour [kr.] Gross 48.00 92.31 131.83 165.42 192.22 211.88 Net 28.00 52.31 71.83 85.42 92.22 91.88

From the table we see that the optimal number of machines becomes 5. When adding one machine the income should increase by at least 20 kr./hour. This corresponds to that the carried trac should increase by at least 1/3 erlang when adding one machine (Moes principle). The optimal number of machines can thus be obtained directly from a table of the improvement function. Question 4: The carried trac on the individual machines becomes when using sequential hunting (7.16): ai = F1,i1 (A) = A [E1,i1 (A) E1,i (A)] . For a period of 12 opening hours we get: a1 = 0.8000 erlang 576 coins , a2 = 0.7385 erlang 532 coins , a3 = 0.6587 erlang 474 coins , a4 = 0.5601 erlang 403 coins . Question 5: After closing time the arrival rate is is = 0. For the departure process we exploit that the exponential distribution has no memory: a) If only one customer is present at closing hour, then the time until this customer departs (independent of how time the customer has already played) is exponentially distributed with mean value 1/ = 6 minutes: w1 = 6 minutes .

3 b) If two customers are present at closing time, then the rst one departs according to an exponential distribution with mean value (2)1 = 3 minutes. Subsequently, the last one departs as mentioned under a). The total mean value thus becomes: w2 = 9 minutes . c) If three customers are present, the rst one departs according to an exponential distribution with mean value (3)1 = 2 minutes. Afterwards the sequence is as described under b), and the total mean value becomes: w3 = 11 minutes . d) In similar way we derive the average waiting time for the case with four customers present at closing time: w4 = 12.5 minutes . Question 6: The probability of having just one idle machine is equal to the probability of having just three busy machines. From the truncated Poisson distribution (7.9) we get: A3 P (3) = / 3!
4

=0

A !

4 E1,4 (A) = 0.3107 . A

The trac ai carried by machine i at a given point of time is correlated with the trac carried by the other machines at the same time. The values given in Question 4 are mean values and cannot be used for solving this question. Question 7: The last machine is (of course) idle, when it is not working. From Question 4 we therefore get: P {last machine idle} = 1 a4 = 0.4399 .
Updated 2005-02-14

1 Technical University of Denmark COM Center, Networks Teletrac Engineering & Network Planning Course 34 340

Exercise 7.10

(Exam 1999)

ERLANGS LOSS SYSTEM We consider a loss system, which has 4 channels and is oered PCT1 trac. The arrival rate (intensity) is = 1 call per time unit, and the mean service time is 1 = 2 time units. The system is assumed to be in statistical equilibrium.

1. Find the oered trac, and set up the state transition diagram of the system. 2. Find the state probabilities, and nd the time congestion, the call congestion, and the trac congestion. 3. Calculate the time congestion using the recursive formula for Erlangs B-formula. The individual steps of the recursions should appear in the answer. 4. Assume random hunting, and nd the probability that two specic channels are busy (the remaining channels may be busy or idle). 5. How many channels do we need, if the system is dimensioned with an improvement value equal to 0.20? (Apply the results from Question 1). 6. Find the distribution of the number of calls, which are lost during a period, when all 4 channels are busy.

1 Technical University of Denmark COM Center, Networks Teletrac Engineering & Network Planning Course 34 340

Solution to Exercise 7.10

(Exam 1999)

The system considered corresponds to Erlangs loss system, which is dealt with in Chap. 7. Question 1: The oered trac A is: A = / = 1 2 , A = 2 [erlang] . The state transition diagram becomes (cf. Fig. 7.2):
1

         
j
1 1

1 2

2 2

3 2

4 2

Question 2: If we denote the relative state probabilities by q(i) and the absolute state probabilities by p(i), then we get:

q(0) = 1 q(1) = 2 q(2) = 2 q(3) = q(4) =


4 3 2 3

p(0) = p(1) = p(2) = p(3) = p(4) =

3 21 6 21 6 21 4 21 2 21

= 0.1429 = 0.2857 = 0.2857 = 0.1905 = 0.0952

Total = 7

Totalm = 1 = 1.0000

We would of course obtain the same state probabilities by inserting the actual parameters into the truncated Poisson distribution (7.9). The time congestion E becomes: E = p(4) = 2 . 21

2 As the arrival process is a Poisson process, the PASTAproperty is valid, and time congestion, call congestion and trac congestion are all identical (Sec. 7.3.2): 2 E=B=C= . 21 We may of course calculate B and C explicitly, but at exam this would be waste of time. Question 3: By applying the recursion formula for calculating Erlangs B-formula (7.29): Ex (A) = A Ex1 (A) , x + A Ex1 (A) E0 (A) = 1 , x = 1, 2, . . . ,

we nd, letting A = 2 [erlang]: x = 1: x = 2: x = 3: x = 4: x = 5: E1 (2) = E2 (2) = E3 (2) = E4 (2) = E5 (2) = 21 1+21 2 2 3 2+2 2 3 2 2 5 3+2 2 5 4 2 19 4 4 + 2 19 2 2 21 2 5 + 2 21 = = = = = 2 , 3 2 , 5 4 , 19 2 , 21 4 , 109

where we for later use in Question 5 also calculate the blocking probability for 5 channels. The blocking probability for 4 channels corresponds of course to the result obtained in Question 2. As a control we may also verify that the values are in agreement with the table of Erlangs B-formula in the collection of exercises. Question 4: The probability that k specic channels (our channels) are busy is given by Palm-Jacobus formula (Sec. ??). For Erlangs loss system we use (??): H(k) = En (A) , Enk (A) k = 1, 2, . . . n .

For n = 4 and k = 2 we nd using the values from Question 3: H(2) = 2/21 , 2/5 5 . 21

H(2) =

3 We may ourselves by elementary probability theory carry through the derivations which are behind Palm-Jacobus formula. If two arbitrary channels are occupied, then the probability that it is just our two channels will be equal to 1/6 (the number of dierent ways we may choose 2 out of 4 channels). If three channels are busy, then the probability that our two channels are among these will be 1/2. If all four channels are busy, then our two channels will always be busy. Therefore, we get: H(2) = = = 1 1 p(2) + p(3) + p(4) 6 2 1 4 2 1 6 + + 6 21 2 21 21

5 q.e.d. 21 It was a general error at the exam to assume independence between channels and use the probability that a random channels is busy. Question 5: The trac lost A , the trac carried Y , and the trac carried additionally Fn (A), when the number of channels are increased from n to n + 1 (which is equal to the trac an+1 carried by channel n+1 in a system with sequential hunting) becomes: n 0 1 2 3 4 5 As: F1,n1 (A) > FB = 0.20 F1,n (A) , we notice that we need n = 4 channels, because we choose an integral number of channels, which is on the safe side (corresponding to an improvement value less than or equal to the dimensioning criteria). The same result may of course be obtained by using the table of the Improvement function of Erlangs B-formula, given in the collection of exercises. Fig. 7.5 gives the same result for the curve A = 2, but it is more dicult to read accurately. Question 6: The duration of state all channels busy is exponentially distributed with the intensity: 4 = 2 [calls/time unit]. New call attempts arrive according to a Poisson process with rate Al 2.0000 1.3333 0.8000 0.4211 0.1905 0.0734 Y 0.0000 0.6667 1.2000 1.5789 1.8095 1.9266 Fn (A) = an+1 0.6667 0.5333 0.3789 0.2306 0.1171

4 (intensity) = 1 [calls/time unit]. If we are in state 4 channels busy (busy period), then the next event is either: a call attempt which is blocked or termination of the busy period. From Sec. 4.1.1 (minimum of k exponentially distributed stochastic variables) we know the probability of next event: p(call attempt) = 1 = , +4 3 4 2 = . +4 3

p(call terminate) =

Because of the exponentially distributed time intervals the process is a Markov process without memory, and the above probabilities are thus independent of the number of calls already blocked. The probability that i call attempts are blocked during a busy period therefore becomes geometrically distributed: p(i) = 1 3 i 2 , 3 i = 0, 1, 2 .

Comments: This version of the geometric distribution begins with the value zero and thus the mean value and variance becomes (cf. the text of Table. 6.1): m1 = 2 = 1 1 1= , 2/3 2 1 2/3 3 = . 2 (2/3) 4

On the average we stay in state [4] half a time unit. On the average one call arrives per time unit. Therefore, the mean value 1 is correct. 2 Alternative solution 1: Direct calculation The distribution of the number of calls during a busy period with all four channels occupied can also be derived directly. The busy period has the density distribution: f (t) dt = n ent dt = 2 e2t dt . If the busy period has a duration inside the interval (t, t+dt), then the number of calls during this (constant) time interval is Poisson distributed: p(i | t) = = (t)i t e i! ti t e , i! i = 0, 1, .

5 The unconditional distribution for the number of call attempts then becomes:

p(i) =
t=0

p(i | t) f (t) dt ti t e 2 e2t dt i!


t=0 t=0

=
t=0

2 i!

ti e3t dt (3t)i e3t d(3t)

1 2 i+1 i! 3

= =

2 1 i+1 i ! i! 3 2 3 i+1 , i = 0, 1, . . . q.e.d.

where be exploit the denition of the gamma function. Alternative solution 2: Generating functions By applying the theory for the number of events in a Poisson process during a stochastic time interval (Sec. ??), we nd the Z-transform of the distribution of the number of events during a busy period (??): Zn (z) = ((1z)) . For the exponential distribution we have the Laplace-transform (??): 2 4 = , (s) = 4+s 2+s and thus: 2 2 Zn (z) = = , 2 + (1 z) 2 + 1 (1 z) which may be written as: . 1 (1 2 ) z 3 This is the Z-transform of the geometric distribution, derived above. This is seen as follows:

Zn (z) =

2 3

Z =
i=0

(1 )i z i , 1 (1 ) z

which corresponds to the above with = 2 . 3


Revised 2001-03-11

1 Technical University of Denmark COM Center, Networks Teletrac Engineering & Network Planning Course 34 340

Exercise 8.9

(Exam 2000)

ENGSETS LOSS SYSTEM

We consider Engsets loss system with 3 servers, which are oered trac from 4 homogeneous sources. An idle source generates calls with the intensity = 1/2 [calls/time unit], and the service time is exponentially distributed with the mean value 1 = 1 [time unit]. 1. Find the total oered trac from the 4 sources. 2. Set up the state transition diagram, and nd the state probabilities under the assumption of statistical equilibrium. 3. Find the time congestion, call congestion, and trac congestion, using results from Question 1 and 2. 4. Find the distribution (density function) of the number of calls, which are blocked during a period where all three servers are busy. 5. Derive the state probabilities of the system by convolving the state probabilities of 4 single sources and truncate the state probabilities at state 3.

1 Technical University of Denmark COM Center, Networks Teletrac Engineering & Network Planning Course 34 340

Solution to Exercise 8.9 Question 1:

(Exam 2000)

When there is no blocking, a single source has the following time-dependent evolution:

The state probabilities of a single source becomes: p(0) = p(1) = Oered trac per idle source (8.7): = / = 1 . 2 2 , 3 1 . 3

Oered trac per source (= carried trac in a system with no blocking (8.8): = 1 = . 1+ 3

The total carried trac of four sources becomes (8.9): A= 4 [erlang] . 3

Question 2: The resulting state transition diagram is shown in the following gure: The relative state probabilities q(i), respectively the absolute state probabilities p(i), becomes:

2
#
4 2

# j

3 2

# j

2 2

# j

0 1

1 2

2 3

"! "! "! "! Y Y Y

q(0) = 1 q(1) = 2 q(2) = q(3) =


3 2 1 2

p(0) = p(1) = p(2) = p(3) =

2 10 4 10 3 10 1 10

Question 3: The time congestion E is the proportion of time all three channels are busy:

E = p(3) =

1 . 10

The call congestion B is the proportion of call attempts which are blocked. If we denote the arrival intensity (rate) in state i by i, then we have:

B =

3 p(3) 0 p(0) + 1 p(1) + 2 p(2) + 3 p(3)


4 2

2 10

+3 2

1 2 4 10

1 10 +2 2

3 10

+1 2

1 10

B =

1 = 0.0370 . 27

The trac congestion C is the proportion of the oered trac which is blocked:

C=

AY , A

3 where Y is the carried trac:


3

=
i=0

i p(i) 4 3 1 +2 +3 10 10 10

= 1 =

13 . 10
4 3

C =

4 3

13 10

C = C may also be obtained from (8.32): C=

1 = 0.0250 . 40

43 1 1 Sn E = = S 4 10 40

q.e.d.

Question 4: During a busy period we are in state 3. The next call attempt is blocked, if it arrives before an existing call terminates. As in state 3 the arrival intensity is 1 and the departure intensity is 3, we get: 2 p(blocking) = 1 p = 1p =
1 2

3+ 1 . 7

1 2

In a similar way the probability that the next call attempt is accepted becomes: p= 6 . 7

Due to the Markov property (exponentially distributed time intervals) the process has no memory, and after blocking a call attempt the system is still in the same state. Therefore, the number of blocked call attempts during a busy period becomes geometric distributed: p(i) = (1 )i = 1 7
i

6 , 7

i = 0, 1, 2 . . . .

4 The distribution starts in i = 0 and thus has the mean value (cf. the text of Tab. 6.1): 1 7 1 1= 1 = . p 6 6 Question 5: From the state probabilities of a single source, which we obtained in Question 1 we get:

# sources 0 1 2 3 4

1
2 3 1 3

1
2 3 1 3

2
4 9 4 9 1 9

1
2 3 1 3

3
8 27 12 27 6 27 1 27

1
2 3 1 3

4
16 81 32 81 24 81 8 81 1 81

Note that the call congestion for 4 sources in Question 2 is equal to the time congestion with 1 3 sources ( 27 ). We truncate the state probabilities at 3 channels and get:

State Unnormalised Normalised 0 1 2 3


16 81 32 81 24 81 8 81 16 80 32 80 24 80 8 80

= = = =

2 10 4 10 3 10 1 10

This is of course the same as we found in Question 2.


Updated 2005-02-23

1 Technical University of Denmark COM Center, Networks Teletrac Engineering & Network Planning Course 34 340

Exercise 8.11

(exam 2003)

Aloha model with Engset trac We consider an Engset model with S = 4 sources. The mean holding time is chosen as time unit (1 = 1). The arrival rate of an idle source is = 1/3. Both time intervals are exponentially distributed. The number of channels is innite, i.e. n S. The state of the system is dened as the number of busy channels. The above system is a model of a non-slotted Aloha system with S transmitters and exponentially distributed packet lengths.

1. Find the oered trac A. 2. Construct the state transition diagram and nd under the assumption of statistical equilibrium the state probabilities p(i), (i = 0, 1, . . . , 4). 3. Find the state probabilities (i), (i = 0, 1, . . . , 4), as they are observed by an arriving customer just before arrival (call averages). (Use either the state probabilities obtained in question 2 as starting point, or use the arrival theorem). 4. What is the probability that a call arriving in state zero (and thus changing the state of the system into state one) will complete service before next call arrives? This corresponds to a successful call transmission in the Aloha protocol. 5. What is the mean holding time of successfully transmitted calls?

1 Technical University of Denmark COM Center, Networks Teletrac Engineering & Network Planning Course 34 340

Solution to exercise 8.11 Question 1:

(exam 2003)

The Engset case is dealt with in Chap. 8. The oered trac becomes (8.9):

A = S=S where = 1 = , 3
1 3

, 1+

A = 4

1+

1 3

= 1 [erlang] .

Alternative approach: If no calls are blocked, a source will change between being idle for three time units and busy for one time unit (mean values). In this case the carried trac is equal to the oered trac equal to 1/4 (the source is busy 25 % of the time. As we have 4 sources the total oered trac becomes 1 [erlang]. Question 2: We get the following state transition diagram (cf. Fig. 8.4):

If we denote the non-normalised state probabilities by qi and the normalised state probabilities by pi we nd:

q0 = 1 q1 = q2 = q3 = q4 = Sum = Question 3:
4 3 2 3 4 27 1 81 256 81

p0 = p1 = p2 = p3 = p4 =

81 256 108 256 54 256 12 256 1 256

, , , , ,

Sum = 1 .

During one time unit the average number of customers arriving in each state is:

state [0] : State [1]: State [2]: State [3]: state [4] : Total :

4 81 3 256 3 108 3 256 2 54 3 256 1 12 3 256 0 1 3 256

= = = =

27 64 27 64 9 64 1 64

= 0 = 1

In general, the numbers do of course not add to one. It is number of calls and not probabilites. If we considered two time units, they would add to two. After normalising the number of calls arriving in each state by the total number of call arriving per time unit, we obtain the state probabilities observed by arriving calls: (0) = (1) = (2) = (3) = (4) = 27 , 64 27 , 64 9 , 64 1 , 64 0.

3 Alternatively we could use the Arrival Theorem saying that the call averages are equal to the time averages with one source less, i.e. the state probabilities of a system with S = 3 sources and n 3 channels.

The non-normalised state probabilities qi and the normalised state probabilities pi become: p0 = 27 , q0 = 1 64 q1 = 1 q2 = q3 = Sum =
1 3 1 27 64 27

p1 = p2 = p2 =

27 64 9 64 1 64

, , ,

Sum = 1 .

This is of course the same result as obtained above. Question 4: A customer arriving in state zero brings the system into state one. In state one new calls arrive with rate 3 = 1, and the holding time of the considered customer terminates with rate 1. The probability that the rst event is completion of the holding time becomes (Sec. 4.1.1): p{complete service before new arrival} = 1 1 = . 1+1 2

Question 5: In Question 4 the next event took place after an exponential time interval with total rate equal to 2, i.e. successful calls terminates after an exponentially distributed time interval with rate 2 and therefore have the mean holding time: m1,success = 1 2 [time units] .

Thus short calls are in general more successful than longer calls, which is quite obvious. Unsuccessful calls have mean value 1.5 and a random call has the mean value 1.
Updated: 2005-02-17

1 Technical University of Denmark COM Center, Networks Teletrac Engineering & Network Planning Course 34 340

Exercise 8.12

(exam 2005)

Engset model with inhomogeneous sources We consider a full accessible Engset loss system with n = 3 channels. The system is oered trac from S = 4 sources. The arrival rate of an idle source is 1 = 1/2 call attempts per time unit. The mean holding time is chosen as time unit (1 = 1). All time intervals are 1 exponentially distributed. The state of the system is dened as the number of busy channels and every busy source occupies one channel.

1. Find the oered trac. 2. Find the state probabilities of the system by convolving the state probabilities of the 4 individual sources, truncating at 3 channels, and normalising. 3. Find the time congestion E, the call congestion B, and the trac congestion C.

We now add a source dierent from the above sources having both mean idle time and mean holding time equal to one time unit (2 = 2 = 1). The source occupies one channel when it is busy. 4. Find by convolving this source with the above system the time congestion for both types of calls. 5. Find the call congestion for both types of calls by applying the arrival theorem. 6. Find the trac congestion for both types of sources by looking at the individual terms during the convolution (use eventually a two-dimensional state transition diagram as aid).

1 Technical University of Denmark COM Center, Networks Teletrac Engineering & Network Planning Course 34 340

Solution to exercise 8.12 Question 1:

(exam 2005)

If there is no congestion, then on the average a source is on one time unit and o two time units. So the oered trac per source is = 1/3 (8.8). The total oered trac becomes (8.9): 4 A=S= [erlang]. 3 Question 2: The state probabilities of a single source is (Binomial distribution for a single source = Bernoulli distribution): 2 1 p(0) = , p(1) = . 3 3 By using the relative state probabilities 2 : 1 we get by convolving 4 sources:

State S1 0 1 2 3 4 2 1 0 0 0

S2 2 1 0 0 0

S12 4 4 1 0 0

S3 2 1 0 0 0

S123 8 12 6 1 0

S4 2 1 0 0 0

S1234 16 32 24 8 1

Truncated 16 32 24 8

Normalised
2 10 4 10 3 10 1 10

To the right we have the state probabilities. As a control we may of course use the formul in the textbook (8.24) or a state transition diagram:
#
4 2

# j

3 2

# j

2 2

# j

0 1

1 2

2 3

"! "! "! "! Y Y Y

We see that the above state probabilities fulll the node balance equations and add to one. Therefore, this is the unique solution.

2 Question 3: From the state probabilities we get: E = p(3) = 1 = 0.1000 . 10

Using the arrival theorem, the call congestion is the time congestion with one source less, i.e. three sources. From the table for 3 sources (S123 ) we get for this inhomogeneous system: E= The carried trac is:
3

1 1 = = 0.0370 . 8 + 12 + 6 + 1 27

=
i=0

i p(i) 2 4 3 1 +1 +2 +3 10 10 10 10

= 0 =

13 . 10

As the oered trac is A = 4/3 (Question 1) we get: C = AY = A


4 3

4 3

13 10

C =

1 = 0.0250 . 40

We may of course also use the formul of the textbook, e.g. (8.32): C= Question 4: S n 43 1 1 E = = , S 4 10 40 q.e.d.

State S1234 0 1 2 3 2 4 3 1

S5 1 1 0 0 21 21 + 20 + 20 +

Convolution 41 41 + 40 + 31 31 + 11

S12345 =2 =6 =7 =4
2 19 6 19 7 19 4 19

Normalised = 0.1052 = 0.3158 = 0.3684 = 0.2105

3 We still consider a system with 3 channels. The time congestion for both types of sources (both requiring one channel) becomes: E1 = E2 = Question 5: The arrival theorem tells that the call congestion of a source is equal to the time congestion of the same system without this source. Thus we get from Question 3: 1 = 0.1000 . 10 The call congestion of type 1 is the time congestion of a system with 3 type 1 (S123 ) sources and the type 2 source (S5 ): B2 = State 0 1 2 3 S123 8 12 6 1 S5 1 1 0 0 S1235 8 20 18 7
8 53 20 53 18 53 7 53

4 = 0.2105 19

Normalised = = = = 0.1509 0.3774 0.3396 0.1321

Thus the call congestion of type 1 sources becomes: B1 = Question 6: From the convolution scheme in the table in Question 4 we see that all the terms in the rst column corresponds to zero erlang of type 1 sources, the terms in the second column corresponds to one erlang, the terms in the third column corresponds to 2 erlang, and the term in the last column corresponds to 3 erlang. Thus we get the carried trac, remembering that the normalisation factor is 1/19 and looking away from zero terms: Y1 = {1 (4 1 + 4 1) + 2 (3 1 + 3 1) + 3 (1 1)}/19 Y1 = 23 19
4 3

7 = 0.1321 . 53

As the oered trac is A1 =

we get the following trac congestion for sources of type 1: C1 =


4 3

4 3

23 19

C1 =

7 = 0.0921 76

4 In a similar way we identify the terms contribution to trac of type 2 (all by 1 erlang) as terms on the second diagonal in the table of Question 4: Y2 = {2 1 + 4 1 + 3 1}/19 = 9 19
1 2

C2

A2 Y2 = = A2

1 2

9 19

19 18 19

1 = 0.0526 . 19 This question may also be solved by considering the two-dimensional state transition diagram of two streams: one with the four identical sources (horizontally), and one with the additional source (vertically): C2 =
4 3    2 2 01  11  21    1 2 6 6 6

    ? 4 ? 3 ? 2 2 2 2 -

00  10  20  30     1 2 3
We get the following relative state probabilities as the state transition diagram is reversible (Chapter 10): 9 10 19 2 2 4 4 4 8 3 3 6 1 1

The normalisation constant is 19. From the marginal state probabilities we get the carried trac of each type: Y1 = (0 4 + 1 8 + 2 6 + 3 1) /19 = 23 , 19

Y2 = (0 10 + 1 9) /19 9 , 19 which are the values obtained above. =


Updated: 2006-02-27

1 Technical University of Denmark COM Center, Networks Teletrac Engineering & Network Planning Course 34 340

Exercise 9.14

(Exam 2004)

Loss system and overow trac

We consider a full accessible loss system with n = 3 channels. The system is oered Pascal trac, and in state i the arrival rate is (S +i). The number of sources is S = 4. The arrival rate of an idle source is = 1/3. The mean holding time is chosen as time unit (1 = 1). All time intervals are exponentially distributed. The state of the system is dened as the number of busy channels. (Note: An Erlang-B table covering n =1, 10 (step = 1) and A = 0, 10 (step = 0.25) was attached). 1. Show that the oered trac is A = 2 [erlang] and that the peakedness is 1.5 . 2. Construct the state transition diagram and nd under the assumption of statistical equilibrium the state probabilities p(i), i = 0, 1, . . . , 3. 3. Find the time congestion E, the call congestion B, and the trac congestion C. 4. Calculate the trac congestion C by using Fredericks-Haywards method. 5. Calculate the trac congestion C using Sanders method. Assume that the above trac is overow trac from an equivalent system with 4 channels which are oered 5 erlang. 6. Find the trac congestion C using Wilkinson-Bretschneiders ERT-method.

1 Technical University of Denmark COM Center, Networks Teletrac Engineering & Network Planning Course 34 340

Solution to Exercise 9.14

(Exam 2004)

This is the truncated Pascal system dealt with in Sec. 8.7. Question 1: The oered trac is obtained by using the formul for the Engset case by letting S = 4 sources (8.63) and = 1 (8.64). 3 By using (8.8) and (8.9) we get: =

1 = , 3 A = S 1+

1 3 = (4) 1 1 3 A = 2 [erlang] . In a similar way we nd from (??): Z = 1 1+ 1 1 3 2


1 3

Z = We may also use the simple relation (8.21) Z =1 Question 2:

A 2 3 =1 = S 4 2

The state transition diagram becomes (cf. Fig. 8.6):

2
4 3 5 3 6 3

..... ..... ..... ..... ..... ....................... ......... ....... ........... .......... ........... ........... .......... ........... ........... .......... ...... . ........... ......... ...... . ........... ......... . . . ............. ...... ......... ......... ... .................... ... .. ... .. ... ........ ........ ......... .... ... ........ ........ ... .. .. .. .. .. .. .. . ... . .. . .. .. .. .. .. .. .. .. .. .. .. .. .. .. .. . . . .. . . . . . . . . . . . . . . . . . . . . . . . . . . . . . . . . . . . . . . . . . . . . . . . . . . . . . . . . . . . . . . . . . . . . . . . . . . . . . . . . . .. .. . . . . .. .. .. .. .. .. .. .. .. .. .. .. .. .. .. .. .. .. .. ... . .. .. . . ... . .. . ... . .... . ... . .... ....... ...... ...... . . . ....... ........... ....... ........... ........ ...... ....... ... ..... ..... . . . ........... . .. ... ...... ... ... ......... ..... .... . ..... ..... ..... ..... ... .... ...... ...... .... .... .... . ........ ........ ............ .................. .................. ............... ............... ............. ...

By using the cut equations we get the following relative state probabilities:

q(0) = 1 q(1) = q(2) = q(3) = Sum = Question 3:


4 3 10 9 20 27 113 27

p(0) = p(1) = p(2) = p(3) =

27 113 36 113 30 113 20 113

= 0.2389 = 0.3186 = 0.2655 = 0.1770

Sum = 1 = 1.0000

Time congestion The time congestion E is the proportion of time all channels are busy and it is obtained from the state probabilities: 20 E = p(3) = = 0.1770 113 Call congestion The call congestion B is the proportion of call attempts blocked and it is obtained as the ratio between the number of blocked call attempts per time unit and the total number of call attempts per time unit. We nd: B = (S + 3) p(3) (S + 0) p(0) + (S + 1) p(1) + (S + 2) p(2) + (S + 3) p(3)
4 3

27 113

+5 3

7 3 36 113

20 113 +6 3

30 113

+7 3

20 113

B =

35 = 0.2303 152

Alternatively we may use the formul for the Engset case, remembering that S and are

3 negative. For example by using (8.43) we get: B = (S n) E (1 + ) S + (S n) E

20 (4 3) 113 (1 1 ) 3 = 20 4 + (4 3) 113 ( 1 ) 3

35 152

q.e.d.

We may also use the arrival theorem and nd the call congestion as the time congestion in a system with one source less, that is with S = 5 sources. Trac congestion The trac congestion C is the ratio between the blocked trac and the oered trac: C= AY 2Y = A 2

The carried trac Y is obtained from the state probabilities:


3

=
i=0

i p(i) 27 36 30 20 +1 +2 +3 113 113 113 113

= 0 Y =

156 113

2 156 113 C = 2 C = 35 = 0.3097 113

Also in this case we may use formul for the Engset system, for example the simple formula (8.44): C = = = S n E S 4 3 20 4 113 35 152

4 Question 4 Fredericks-Haywards method is presented in Sec. 9.3. A trac with mean value A = 2 erlang and peakedness Z = 1.5 oered to n = 3 channels has the same blocking probability as Erlangs loss system with A/Z = 4/3 erlang oered to n/Z = 2 channels. Thus the call congestion is obtained by using Erlangs B-formula: C = E2 (1.3333). In this case we can easily use the recursion formula (7.29 to nd the numerical value: E0 = 1 E1 = 1 4 = 4 7 1+ 3 1 2+
4 3 4 3

E2 = Thus we nd:

4 3

4 7

4 7

8 29

8 = 0.2759 29 We may also use the table and e.g. use linear interpolation between the values for A = 1.25 and A = 1.50 for n = 2 channels. Then we nd C = 0.2752 which is very close to the above value. C= Question 5 Sanders method is described in Sec. 9.4.2. The variance of the oered trac is: Var = mean peakedness = A Z = 2 1.5 = 3 As the mean value (A = 2) is less than the variance (var = 3) we add one channel with a constant trac 1 erlang. Then the total oered trac is 3 erlang, and the number of channels is 4. The peakedness is now one, as the variance is still three. The lost trac from this system is the same (approximate method) as the trac lost from the original system. Using the table we nd the lost trac: Alost = 3 E4 (3) = 3 0.2061 = 0.6183 As this trac is lost from the original system, the call congestion becomes: C= 0.6183 = 0.3092 2

(The textbook year 2004 is not clear at this point, so also C = 0.2061 will be accepted)

5 Question 6 Using the ERT-method we have a system of 4 + 3 = 7 channels which are oered 5 erlang. The trac lost becomes 5 E7 (5) = 0.6026 This is the trac lost from the (original) system and thus the call congestion of the original system becomes: 0.6021 = 0.3013 C= 2 When we oer 5 erlang to 4 channels, then the overow trac will have the mean value (9.6) m1 = 1.992 and the peakedness (9.7) Z = 1.519, corresponding to a variance Var = 3.038. This is very close to the values in the exercise. The correct values to have an overow trac with mean 2 and peakedness 1.5 are obtained by using a computer program: Ax = 4.876 nx = 3.826 Using these values the ERT-method yields the trac congestion C = 0.3007 In conclusion we obtained the values: BPP Fredericks-Hayward Sanders ERT C = 0.3097 C = 0.2759 C = 0.3093 C = 0.3007

We notice that the values are very similar except for Fredericks-Haywards method, which usually is one of the best. But this is an extreme case with very few channels and very high blocking. We also notice that only the BPP-model works with dierent types of blocking probabilities, and that the trac congestion C is the relevant measure. Historically only the time congestion E and the call congestion B have been considered, and they are quite dierent.
Updated: 2006-02-27

1 Technical University of Denmark COM Center, Networks Teletrac Engineering & Network Planning Course 34 340

Exercise 12.19

(exam 1997)

QUEUEING SYSTEM M/M/3 We consider Erlangs classical queueing system M/M/3 having 3 servers and an unlimited number of queueing positions. Customers arrive according to a Poisson process with intensity = 2 customers per time unit, and the service time is exponentially distributed with intensity = 1 time unit1 . The state of the system is dened as the total number of customers in the system. 1. Find the oered trac. Are the conditions for statistical equilibrium fullled? 2. Construct the state transition diagram, and nd the state probabilities when the system is in statistical equilibrium. 3. Calculate the probability for waiting time (Erlangs Cformula) by using the recursive formula for Erlangs B-formula for calculating the C-formula. The individual steps of the recursion shall appear in the solution. 4. Find (a) the mean queue length at a random point of time, (b) the mean waiting time for all customers, and (c) the mean waiting time for customers experiencing a waiting time > 0. 5. (Advanced question) Assume that the three servers are hunted in sequential order and nd the trac carried by each of the three servers (use the numerical values from Question 3). 6. Draw a phase-transition diagram for the response time (service time + eventual waiting time), and nd the mean value and form factor of this response time.

1 Technical University of Denmark COM Center, Networks Teletrac Engineering & Network Planning Course 34 340

Solution to Exercise 12.19: Question 1:

(Exam 1997)

A=s=

1 = 2 1 = 2 erlang.

The conditions for statistical equilibrium are fullled because A < n (12.3). Question 2: The state transition diagram becomes as follows:
2 0 1 1 2 2 2 3 2 3 3 2 4 3 2 5 3 2

p(0) is obtained by using (12.4) and becomes:

p(0) =

1
2

=0

2 23 3 + ! 3! 3 2

1 9

The other state probabilities are given by (12.2):

1 2i , 9 i! p(i) = 1 2i , 9 3! 3i3

0i3 i3

2 Thus: p(0) = 1 , 9 2 , 9 2 , 9 4 , 27 8 , 81 16 , 243 etc.

p(1) =

p(2) =

p(3) =

p(4) =

p(4) =

Question 3: The probability of experiencing waiting time is given by (12.9): D = p{W > 0} = E2,n (A) = In this case we get: p{W > 0} = E2,3 (2) = 3 E1,3 (2) . 3 2 (1 E1,3 (2)) n E1,n (A) . n A {1 E1,n (A)}

We want to obtain E1,3 (2) by means of the recursion formula (7.29): E1,x (A) = We get: E1,0 (2) = 1 , E1,1 (2) = E1,2 (2) = 2 21 = = 0.6667 , 1+21 3 2 2 2/3 = = 0.4000 , 2 + 2 2/3 5 4 2 4/10 = = 0.2105 . 3 + 2 4/10 19 A E1,x1 (A) , x + A E1,x1 (A) E1,0 (A) = 1 .

E1,3 (2) =

3 The probability of experiencing a positive waiting time thus becomes: p{W > 0} = E2,3 = 4 4 3 = = 0.4444 . 3 + 2 (1 4/19) 19 9

Question 4: The mean queue length is given by (12.12): L3 = E2,3 A 4 2 8 = = = 0.8889 . nA 9 32 9 4 L3 = = 0.4444 9

The mean waiting time for all customers is given by (12.15): W3 =

The mean waiting time for customers which experience a positive waiting time > 0 is obtained from(12.17): W3 4/9 s w= = =1 = E2,3 nA 4/9 Question 5: The carried trac per channel for sequential hunting is given in the advanced part of the textbook: A bi = ai + (1 ai ) E2,n (A) , n where ai is the trac carried by the ith channel in a loss system with sequential hunting ai = Fn (A) = A [E1,n (A) E1,n+1 (A)]: a1 = A {E1,0 (2) E1,1 (2)} = a2 = A {E1,1 (2) E1,2 (2)} = a3 = A {E1,2 (2) E1,3 (2)} = Using the results obtained in Question 3 we get: b1 = b2 = b3 = 62 2 2 1 4 + = = 0.7654 , 3 3 3 9 81 2 7 4 272 8 + = = 0.6715 , 15 3 15 9 405 36 2 59 4 1444 76 + = = = 0.5629 . 95 3 95 9 2565 135 2 , 3 8 , 15 36 . 95

4 Question 6: We get the following phase diagram for the response time (it is convenient to put the service time before the waiting time, but it can easily be done in the right order:
.. ............................. ........... ................ .... ........... ........ ... ... .. . ...

=1

. .......................................................... .. ........... ...... ........... ........... ........ ... ....... ................. ........... ................. . ... ... ................. .............. ... .. ..... ........ . .. .. . . . . . . . . . . . . . . . . . . . . . . . . . . . . . . . . . . . . . . . . . . . . . . . . . . . . . . . . . . . . . . . . . . . . . . . . . . . . . . . . . . . . . . . . . . . . . . . . . . . . . . . . . . . . . . . . . . . . . . . . . . . . . . . . . . . . . . . . . . . ... ... .. . .. ... . .. .. . . . .. .. . . .. . . . . . ............................................................................................................................ . .... ........... ........... ........... ...... ........... ........... ....................... ........... ...... ............ . ........ ........... ........... ................. ........... ........... ... .. ..... ........... ................. ... ... . . . ... .. . .. ...

4 9

1=w = 1

5 9

p0 = 1,

p1 = 4/9, q1 = 1,

p2 = 0 q2 = 4/9, q3 = 0

Referring to the notation in Sec. ?? and in Fig. 4.7 we get the following results. Formula (4.32) yields:
2

m1 =
i=1

qi 13 1 4 = 1.4444 . = + == i 1 9 9

The variance is obtained from (4.34):


2 2 i

= 2
i=1 j=1

1 j

qi i

m1 2
2

= 2

1 1 + 1 1 13 9
2

1 1 + 1 1

4/9 1

13 9

34 9

= 1.6914 ,

from which the form factor is obtained using (3.10): = 1+ 2 , m2 1 1.6914 = 1.8107 . (13/9)2

= 1+

The above procedure is general. It is simpler to consider the response time distribution as a parallel combination of an Erlang-2 distribution and an exponential distribution, all phases having the intensity one.

5 The Erlang-2 distribution has the mean value 2, and the second moment is given by (4.14), which becomes 6. This branch has the weighting factor 4/9. The exponential distribution has the mean value 1 and the second moment 2. This branch has the weighting factor 5/9. The response time then has the mean value, 2. moment, and form factor as follows: m1 = m2 = =
Updated: 2006-03-08

5 13 4 2+ 1= , 9 9 9 5 34 4 6+ 2= , 9 9 9 m2 = 1.8107. m2 1

1 Technical University of Denmark COM Center, Networks Teletrac Engineering & Network Planning Course 34 340

Exercise 12.22

(exam 2001)

WAITING TIME SYSTEM WITH HYSTERESIS We consider a pure delay system where customers arrive according to a Poisson process with intensity = 2 customers per time unit. The service time is exponentially distributed with mean value 0.5 time units. We assume the queueing discipline is FCFS = FIFO. 1. Find the oered trac. The customers are served by one or two servers. There is always at least one server available. The other server opens (start serving customers), when the queue length becomes two. When there are no more customers in the queue, the server which rst becomes idle, is closed. The queue length is dened as the number of waiting customers and does not include customers being served. We dene the state of the system as (i, j), where i is the total number of customers in the system, and j is the number of open servers. Therefore, we obtain the state transition diagram with the states and structure shown in the following gure.

2. Construct the state transition diagram of the system. 3. Find the state probabilities under the assumption of statistical equilibrium. 1 (Hint: p(0, 1) = 2 , p(2, 2) = 14 ). 7 4. Find the average queue length using the state probabilities. 5. Give the waiting time distribution of a customer who arrives in a state (i, 2) , i 3. 6. Give the waiting time distribution in the form of a phase diagram for a customer, which arrives in state (1, 1) and brings the system into state (2, 1), and nd the mean waiting time of the customer. (Hints: Next event takes place after a known time interval and is either a departure or an arrival. In the rst case the waiting time terminates. In the last case the second server starts operating, and as the queue discipline is FCFS = FIFO, we know the remaining waiting time).

1 Technical University of Denmark COM Center, Networks Teletrac Engineering & Network Planning Course 34 340

Solution to exercise 12.22 Question 1:

(exam 2001)

If we denote the mean service time by s the Oered trac becomes: A = s = 2 0.5 = 1 [erlang] . Question 2: The state transition diagram becomes as follows: The arrival rate is always . All states with

one server have departure rate , and all states with two servers have departure rate 2. Question 3: The node balance equations yield ( = = 2 events/time unit):

Node (0,1):

2 p(0, 1) = 2 p(1, 1) , p(1, 1) =


2 7

Node (2,1):

4 p(2, 1) = 2 p(1, 1) , p(2, 1) =


1 7

Node (2,2):

6 p(2, 2) = 4 p(3, 2) , p(3, 2) = p(4, 2) = p(5, 2) =


3 , 28 3 , 56 3 , 112

or in general: p(i, 2) =

3 , 7 2i1

i 3.

where the last equations are obtained by simple cut equations. The tail probabilities are a quotient series with factor 1/2. As a control we have:

p(0, 1) + p(1, 1) + p(2, 1) + p(2, 2) +


2 7

i=3

p(i, 2) = = 1.

2 7

1 7

1 14

1 1 2

3 28

Question 4: The average queue length is obtained from the state probabilities:

L = 1 p(2, 1) +
i=3

(i 2) p(i, 2)

1 3 3 1 1 1 + + 2 +3 +4 7 28 28 2 4 8 x

1 3 3 = + + 7 28 28

xi
i=2
1 x= 2

3 3 1 + + 7 28 28 x

x2 1x

x= 1 2

3 3 2x x2 1 + + 7 28 28 (1 x)2 3 3 1 + + 3 7 28 28 4 . 7

x= 1 2

= =

(The most important is the rst equation based on the denition). Question 5: As we have FCFS queueing discipline, a customer arriving in state (i, 2) , i 3 , bringing the system into state (i+ 1, 2) experiences an Erlang-(i1) distributed waiting time. (This answer is sucient).

3 Additional: If we consider an arbitrary one of the customers arriving in states (i, 2), i 3 , then the waiting time becomes Coxdistributed with constant branching probabilities after the second phase as shown in the following gure (we have to exclude state (2, 2) because the probabilility of this state is not twice the probability of state (3, 2)): Thus customers arriving in state (3, 2)

leave (end waiting) by the rst branch, customers arriving in state 4 leave by the second branch, etc. This Coxdistribution is equivalent to a single exponential distribution with intensity 2 in series with an exponential distribution with intensity as the latter is a weighted sum of Erlang-k distributions with geometric weight factors.

Question 6: We call the customer bringing the system into state (2,1) for the tagged customer. The waiting time of the tagged customer terminates when the next event occurs. The next event is either a departure or an arrival. If a customer departs, then the tagged customer starts service. If a new customer arrives, then the second server opens and the tagged customer is served (FCFS). So in both cases the tagged customer initiates service. So the waiting time is exponentially distributed with intensity + = 4: F (t) = 1 e4t , The phase diagram becomes:

t 0.

Updated: 2005-03-03

1 Technical University of Denmark COM Center, Networks Teletrac Engineering & Network Planning Course 34 340

Exercise 12.24

(Exam 2004)

Palms machine-repair model and generalized processor sharing We consider Palms machine-repair model with four terminals and two servers in parallel. The thinking times are exponentially distributed with mean value 1 = 2 time units, and the service times are exponentially distributed with mean value 1 = 1 time unit. The state of the system is dened in the usual way as the number of terminals being served or waiting. 1. Find the trac oered to the two servers. 2. Construct the state transition diagram, and nd assuming statistical equilibrium the state probabilities p(i), i = 0, 1, . . . , 4 .

3. Find the average number of terminals which are: a. thinking, b. waiting, c. being served. 4. Find the trac congestion C. 5. Find by applying Littles theorem to the queue and to both servers the response time, that is the average waiting + service time.

2 We now assume that the service times are hyper-exponentially distributed as follows: F (t) = 1 9 1 et/7 + 1 e3 t , 10 10 t 0.

The state of the system is now dened as (i, j), where i (i = 0, 1, . . . , 4) is the number of jobs being served in phase one, and j (j = 0, 1, . . . , 4) is the number of jobs being served in phase two, 0 i + j 4 . We also assume that the two servers operate in processor sharing mode when more than two terminals are in the queueing system. Thus the service rate in state (i, j) when i + j > 2 is: i j 21 + 22 . i+j i+j When two or fewer terminals are being served, each terminal has its own server. 6. Construct the two-dimensional state transition diagram. 7. Consider the state transition diagram: a. show it is reversible, b. has it product form? 8. Show that the aggregated state probabilities p(i + j = x), x = 0, 1, . . . , 4 , are the same as the state probabilities obtained in question 2.

1 Technical University of Denmark COM Center, Networks Teletrac Engineering & Network Planning Course 34 340

Solution to Exercise 12.24

(exam 2004)

This is Palms machine-repair model with multiple repairmen which is dealt with in Sec. 12.5.4. Question 1: The oered trac is dened as the trac carried when the capacity is unlimited. This will be the case when we have at least 4 servers. Thus we get the same oered trac as in the Engset case (8.9):

A = S=S = S = 4 A = 4 3

1+

/ 1 + / 0.5 1 + 0.5

Question 2:

..... ..... ....... ..... ..... ..... ....... ....... ....... ....... ....................... ......... ....... ............ .......... ........... ............ .......... ........... ............ .......... ........... ............ .......... ....... .... ........... ......... ..... ........... ......... .... ........... ............. ...... ......... . .................... ..... ..... ..... ... ........ ........ ........ ....... ........ ........ ........ ........ ..... ..... ... ... .. .. .. .. .. .. .. ... . . .. .. .. .. .. .. .. .. .. .. .. .. .. .. .. .. .. .. .. .. . .. . . . . . . . . . . . . . . . . . . . . . . . . . . . . . . . . . . . . . . . . . . . . . . . . . . . . . . . . . . . . . . . . . . . . . . . . . . . . . . . . . . . . . . . . . . . . . . . . . . . . . . . . . . . . .. . . . . .. . .. .. .. .. .. .. .. .. . . . . . .. .. .. .. .. .. .. .. .. .. ... . ... . ... .. ... . .. . .. . .. . .. . ... . ... . ... . ... .... ...... ...... ...... .... .. ... . . .. ...... ....... ...... ...... ....... ....... ....... ........ ... ............ ... ... ... .......... ...... ... ... .......... ...... .......... .... ......... . .... ......... .... .......... . ..... .... .......... ...... .... .......... ......... ...... .... ...... ................... ................... ................... .................. .................. .................. .................. ...............

4 2

3 2

2 2

1 2

The relative state probabilities are obtained by using cut equations:

2
16 87 32 87 24 87 12 87 3 87

q(0) = q(1) = q(2) = q(3) = q(4) = Sum =

1 2
3 2 3 4 3 16 87 16

p(0) = p(1) = p(2) = p(3) = p(4) = Sum =

= 0.1839 = 0.3678 = 0.2759 = 0.1379 = 0.0345

1 = 1.0000

We could also apply the theory of queueing networks to this model, but the above approach based on the state transition diagram is indicated in the question. Question 3: a. The average number of terminals thinking becomes:
4

ns =
i=0

(4 i) p(i)

= 4 p(0) + 3 p(1) + 2 p(2) + 1 p(3) + 0 p(4) ns = 220 = 2.5287 . 87

b. The average number of terminals waiting becomes:


4

nw =
i=2

(i 2) p(i)

= 0 p(2) + 1 p(3) + 2 p(4) nw = 18 = 0.2069 . 87

c. The average number of terminals being served is:


2 4

ns =
i=0

i p(i) + 2
i=3

p(i)

= 0 p(0) + 1 p(1) + 2 p(2) + 2 p(3) + 2 p(4) , ns = 110 = 1.2644 . 87

3 As a control the three numbers add to S = 4. Question 4: The oered trac obtained in question one is A= 4/3, and the carried trac is ns = 110/87. The trac congestion then becomes:
4 3

C =

AY = A

4 3

110 87

C =
#

6 = 0.0517 . 116

04
"! T
9 20 4 4

6
1 20

# c

# E

' "! 1 2 T 4 7
18 20 3 3

03

13
"! T
9 20 3 4

6
2 20

6
1 20

# c

# c E

# E

' "! 1 2 T 3 7
27 20

02

' "! 2 2 T 4 7
18 20 2 3

12

22
"! T
9 20 2 4

6
3 20

6
2 20

6
1 20

# c

# c E

# c E

# E

' "! 1 T 7
36 20

01

' "! 2 2 T 3 7
27 20

11

' "! 3 2 T 4 7
18 20 1 3

21

31
"! T
9 20 1 4

3
4 20

3
3 20

6
2 20

6
1 20

# c

# c E

# c E

# c E

# E

' "! 1
7

00

' "! 2
7

10

' "! 3 2
3

20

' "! 4 2
4

30

40
"!

Question 5:

4 We nd the following relation (the mean service time is one): 1 1 W = = nw ns = W +1 R 1 = = , nw + ns nw + ns ns nw , ns

R = 1+ 64 . 55

R = We thus have:

thinking time: waiting time: service time: circulation time:

= 2 =
9 55

= 1 =
174 55

The number of terminals in each stage (thinking, waiting, service) is proportional to the time spent in each stage. This is Littles theorem when we have the same ow in each stage. This is in agreement with question 3. Question 6: We let state (i, j) correspond to i terminals being served in the phase with mean value 7 and j terminals being served in the phase with mean value 1/3. The state transition diagram is shown in the above gure. For example, in state (0, 0) the total arrival rate is 4/2. With probability 1/10 this is a terminal with mean holding time 7, and with probability 9/10 it is a terminal with mean holding time 1/3. Question 7: We show that the ow clockwise is equal to the ow counter-clockwise in each square. This also conrms that the state transition program is correct. The state transition diagram does not have the product-form property. Question 8: As the state transition diagram is reversible we may express all state probabilities by state p(0, 0). We nd the following relative state probabilities when we let q(0, 0) = 160 000 (for convenience to get integer values which are easy to work with):

243 3 240 21 600 2 268 22 680 7 938 52 920 12 348

96 000 100 800

160 000 224 000 117 600 41 160 7 203 We nd: q(0) = q(0, 0) = 160 000 q(1) = q(1, 0) + q(0, 1) = 320 000 q(2) = q(2, 0) + q(1, 1) + q(0, 2) = 240 000 q(3) = q(3, 0) + q(2, 1) + q(1, 2) + q(0, 3) = 120 000 q(4) = q(4, 0) + q(3, 1) + q(2, 2) + q(1, 3) + q(0, 4) = 30 000 The sum of the the relative state probabilities is 870.000 After normalisation we get: p(0) = p(1) = p(2) = p(3) = p(4) = 16 87 32 87 24 87 12 87 3 87

We see the aggregated state probabilities for hyper-exponential service times relatively are the same as for exponential service times in question 2. This indicates that the machinerepair model with processor sharing is insensitive to the service time distribution. This is analogous to that the two systems M/M/1 and M/G/1PS also have the same state probabilities (Sec. 13.7).
Updated: 2006-03-12

1 Danmarks Tekniske Universitet COM Center Data & Teletrakteori kursus 34 340

Exercise 13.21

(exam 2003)

Priority queueing system We consider a single-server queueing system M/G/1, where the customers arrive according to Poisson processes. There are three types of customers: Type 1: Arrival rate 1 = 0.1 calls per time unit. Service time: constant with mean value 1 time unit. Type 2: Arrival rate 2 = 0.04 calls per time unit. Service time: exponentially distributed with mean value 10 time unit. Type 3: Arrival rate 3 = 0.01 calls per time unit. Service time: at distribution with mean value 40 time units and form factor = 10.

1. Find the oered trac for each type, and the total oered trac. 2. Find the mean waiting time for all calls for each type when only this type is oered to the system.

In the following all three types are oered to the system. 3. Find the mean waiting time for a random customer when there is no priority. Type 1 and type 2 are given preemptive resume priority over type 3, so that the rst two types dont notice type 3. Type 1 has non-preemptive priority over class two. 4. Find the mean waiting time for type 1 and 2. 5. Find the mean waiting time for type 3 (type 1 and type 2 may be merged into one type as regards type 3).

1 Technical University of Denmark COM Center, Networks Teletrac Engineering & Network Planning Course 34 340

Solution to exercise 13.21 Question 1: The oered trac becomes:

(exam 2003)

A1 = 0.1 [erlang] , A2 = 0.4 [erlang] , A3 = 0.4 [erlang] , At = 0.9 [erlang] .

Question 2: By using the PollaczekKhintchine formula (13.4) we nd the mean waiting time for all customers as follows: Wi = Ai si i , 2(1 Ai ) 1 0.1 1 1= [time units] , 2(1 0.1) 18 20 0.4 10 2= [time units] , 2(1 0.4) 3 400 0.4 40 10 = [time units] . 2(1 0.4) 3

W1 = W2 = W3 =

Question 3: It is now simpler to use the following version of the Pollaczek-Khintchine formula (13.2): W = where (13.18) & (13.27): V =
i

V , 1A

i m2,i , 2

2 and the second moment of type i is m2,i = i si . V = 0.1 2 0.04 0.01 1 + 2 102 + 10 402 2 2 2

= 0.05 + 4.00 + 80.00 = 84.05 . W = V , 1 At

W = 840.5 [time units] . Question 4 In this question we can look away from type 3. From the above we get V12 = 4.05 and thus for a non-preemptive system (13.26) & (13.28) & (13.29): W1 = W2 = = Question 5 Using the formula (13.37) we get: W3 = = V123 A1 + A2 + s3 (1 A1 A2 )(1 A1 A2 A3 ) 1 (A1 + A2 ) 0.5 84.05 + 40 0.5 0.1 1 0.5 4.05 = 4.5 [time units] , 1 0.1 V {1 A1 } {1 (A1 + A2 )} W1 = 9 [time units] . 1 0.5

= 1721 [time units] .

Updated: 2005-03-16

1 Technical University of Denmark COM Center, Networks Teletrac Engineering & Network Planning Course 34 340

Exercise 13.23

(exam 2005)

Multi-server priority queueing system We consider an Erlang-C waiting time system with n = 5 servers oered A = 3 erlang (Poisson arrival process and exponential service times). The mean service time is 60 seconds.

1. Find the arrival rate. 2. Find the probability of delay using the recursive formula for calculating Erlang-B and the relationship between Erlang-B and Erlang-C formul. (The details of calculation should be visible from the answer, and the result is E2,5 (3) = 81/343 = 0.2362). 3. Find the mean waiting time for all customers.

We now divide the above oered trac into two priority classes: Class 1: A = 1 erlang for high priority customers, Class 2: A = 2 erlang for low priority customers. Both classes still have the same mean service time 60 seconds. We rst consider the non-preemptive resume priority strategy. 4. Find the mean waiting time for each of the two classes. 5. Show that the conservation law is valid for this system. We then consider the preemptive resume priority strategy. Due to the properties of the exponential distribution the conservation law is also valid for this model. 6. Find the mean waiting time for each of the two classes. (given: E2,5 (1) = 1/261 = 0.0038).

1 Technical University of Denmark COM Center, Networks Teletrac Engineering & Network Planning Course 34 340

Solution to exercise 13.23 Question 1:

(exam 2005)

From the formula A = s we get: = A 3 = s 60 [sec] 1 [sec1 ] = 3 [min1 ] 20

Thus the arrival rate is 3 arrivals (call attempts) per minute or 1/20 arrivals per second. Question 2: Using the recurrence formula for Erlang-B (7.29): Ex (A) = we get: E0 = 1 , E1 = 3 31 = = 0.7500 , 1+31 4 = 9 = 0.5294 , 17 A Ex1 (A) , x + A Ex1 (A) E0 (A) = 1 .

E2

3 3 4 = 2+3

3 4

E3 =

9 3 17 9 = 0.3462 , 9 = 26 3 + 3 17 9 3 26 27 = 0.2061 , 9 = 131 4 + 3 26

E4 =

E5

27 3 131 81 = 0.1101 . = 27 = 736 5 + 3 131

2 Thus we have for Erlangs B-formula E1,5 (3) = 81 = 0.1101 . 736

From the relationship (12.9) we obtain Erlang-C from Erlang-B: E2,n (A) = E1,n (A) 1 A {1 E1,n (A)} /n
81 736 3(181/736) 5

405 3680 1965

E2,n (A) = We may also use 12.11:

81 = 0.2362 . 343

1 1 1 = E2,n (A) E1,n (A) E1,n1 (A) = 736 131 81 27 343 81 81 343 q.e.d.

E2,n (A) =

Question 3: The mean waiting time for all customers is (12.15): Wn = E2,n (A) Wn = = 81 60 s = nA 343 5 3

2430 = 7.0845[ sec] 343 81 = 0.1181 [min] . 686

3 Question 4: The non-preemptive model is dealt with in section 13.4.5. We nd the following mean waiting times using (13.33) = (12.15), respectively (13.34): W1 = E2,n (A) = W1 = = W2 = W2 = = 81 343 s n A1

60 51

1215 = 3.5423 [sec] 343 81 = 0.05904 [min] , 1372 n s E2,n (A) 5 n W1 = W1 , = {n A1 } {n (A1 + A2 )} n A1 A2 2 6075 [sec] = 8.8557 [sec] 686 405 = 0.1476 [min] . 2744

Question 5: Using the conservation law (13.23) we nd the constant for the system without priority: C =AW = 3 = For the non-preemptive strategy we get: Cnp = A1 W1 + A2 W2 = 1 6075 1215 +2 343 686 q.e.d. 7290 2430 = [sec] 343 343

243 = 0.3542 min] . 686

7290 = 21.2536 [sec] 343

We thus conrm that the conservation law is valid.

4 In the textbook the conservation law is only used for single-server queues, but it is also valid for multi-server systems, but with dierent formul. In this qyestion we can of course not use the single-server formul. Question 6: For the preemptive-resume discipline the high priority customers can be considered as being alone. so using Erlang-C with n = 5 channels and A = 1 erlang we nd in the same way as above:

E0 = 1 , E1 = 1 11 = = 0.5000 , 1+11 2 = 1 = 0.2000 , 5 1 = 0.0625 , 16

E2

1 1 2 = 2+1 1 1 5 = 3+1

1 2

E3

1 5

E4 =

1 1 16 1 = 0.0154 , 1 = 65 4 + 1 16 1 1 65 1 = 0.0031 , 1 = 326 5 + 1 65

E5 =

E1,5 (1) =

1 = 0.003067 326 1/326 1(1 1/326) 1 5 1 = 0.003831 261

E2,5 (1) =

5 Thus we get (12.15): W1 = E2,n (A) 1 60 s = nA 261 5 1

= =

15 = 0.05747 [sec] 261 1 = 0.0009579 [min] . 1044

Using the conservation law we have: Cpr = 7290 15 = A1 W1 + A2 W2 = 1 + 2 W2 , 343 261 1897545 = 10.5981 [sec] = 0.1766 [min] . 179046

W2 =

Comments: This model may be used for modelling voice/data in GSM/GPRS, where voice has the highest priority. When the blocking/delay probability of voice is low we can consider the oered trac being equal to the carried trac. Remaining capacity is used by data calls waiting in a buer. Furthermore, the mean holding time of data calls can be chosen independently of the mean holding time of voice calls.
Updated: 2005-07-05

1 Technical University of Denmark COM Center, Networks Teletrac Engineering & Network Planning Course 34 340

Exercise 13.19

(Exam 1997)

LEAKY BUCKET: M/D/1/2 QUEUEING SYSTEM Background: Leaky Bucket is a mechanism for controlling the cell (packet) arrival process of a connection in an ATM-system. The mechanism corresponds to a queueing system with constant service time (cell length = 53 bytes)) and a limited buer. If the arrival process is a Poisson process, then we have an M/D/1/k system. The size of the leak corresponds to the average arrival rate accepted in the long run, whereas the size of the bucket (buer) denotes the excess allowed during a short time interval. When implemented in an ATM system the mechanism operates as a virtual queueing system, where a cell is either accepted immediately or rejected. A counter indicates the value of the load function. A contract between the operator (network) and the user (connection) agrees upon the size of the leak and the bucket, and based on this information the network is able to guarantee a certain quality-of-service. Exercise: We rst consider the queueing system M/D/1, which has Poisson arrival process with intensity = 0.6931 calls per time unit, constant service time which we choose as time unit, and one server. The number of queueing positions is unlimited, and we assume that the system is in statistical equilibrium. 1. Find the rst state probabilities p(0), p(1) and p(2) (notice that e0.6931 = 2). We now assume that there is only one queueing position (M/D/1/2). 2. Find from the state probabilities in Question 1 by applying Keilsons formul in Sec. 13.3.4 the state probabilities q(0), q(1) og q(2) in the nite system. 3. What is the probability that a call: (a) is served immediately? (b) is delayed before service? (c) is rejected? 4. Find by using Littles theorem the mean waiting time for customers which experience a positive waiting time. 5. What is the probability that a busy period (a period where the server is busy) has the duration one time unit? 6. Find the probability that a busy period has the duration i time units.

1 Technical University of Denmark COM Center, Networks Teletrac Engineering & Network Planning Course 34 340

Solution to Exercise 13.19

(Exam 1997)

The constant service the is chosen as time unit (s = h = 1) and = 0.6931. We then nd A = s = 0.6931. Question 1: For a M/D/1 system we obtain the state probabilities p(0), p(1) and p(2) from the formul in Sec. 13.5.2: p(0) = 1 A = 0.3069 , p(1) = (1 A)(eA 1) = 0.3069 , p(2) = (1 A)(e2A eA (1 + A)) = 0.1884 .

Question 2: We now consider an M/D/1/2 system, and we apply he formul in Sec. 13.3.4. First we nd Q2 :

Q2 =
j=2

p(j) = 1 p(0) p(1) ,

Q2 = 0.3862 . Then the state probabilities of the truncated system p0 , p1 and p2 are obtained by normalisation: p0 = p2 (0) = p1 = p2 (1) = p2 = p2 (2) = p(0) 1 A Q2 p(1) 1 A Q2 = 0.3069 = 0.4191 , 1 0.6931 0.3862 0.3069 = 0.4191 , 1 0.6931 0.3862

(1 A) Q2 0.3069 0.3862 = 0.1618 . = 1 A Q2 1 0.6931 0.3862

These state probabilities of course add to one.

2 We may also apply the solution given in Sec. 13.5.8. Using Frys equation of state for state zero we get: p0 = (p0 + p1 ) p(0, h) = (p0 + p1 ) e0.6931 , p0 = (p0 + p1 )/2 , p1 = p0 From (13.64) we get: A = 1 p 0 + A p2 0.6931 = 1 p0 + 0.6931 p2 p2 = 1 p0 1 + 0.6931 0.6921

Using the normalisation restriction (13.63) we get p0 : p0 + p0 + 1 p0 1 + =1 0.6931 0.6921

p0 = 0.4191 and thus the same results as above. Question 3: The probabilities questioned become: 1. p{immediate service} = p0 = 0.4191 , 2. p{waiting before service} = p1 = 0.4191 , 3. p{blocking} = p2 = 0.1618 . Question 4: Customers arriving when the system is in state 0 are served immediately, and customers arriving when the system is in state 2 are rejected. Thus only customers arriving in state 1 experience a positive waiting time > 0. The arrival intensity of these customers is x = p1 . The average queue length is given by: L = 0 p0 + 0 p 1 + 1 p 2 = p 2 . We thus nd: w= L 0.1618 = 0.5570 . = x 0.6931 0.4191

3 Question 5: The probability that a busy period has the duration one time unit is: p{busy period = one time unit} = p{no arrivals during one time unit} = 1 F (1) = 1 (1 e1 ) = e0.6931 = 0.5 Question 6: We notice that there is only one queueing position. Therefore, at least one customer must arrive during each service time (only the rst is accepted) to main a busy period. Immediately after start of a new service time, the queueing position is idle. We dene pa and pb as: pa = p{during the rst i1 periods at least one customer arrives per period} = (1 p{busy period = 1 time unit})i1 = 0.5 i1 , pb = p{no arrivals during ith period} = p{busy period = 1 time unit} = 0.5 . The probability that a busy period has the duration i time units Then becomes: p{busy period = i time units} = pa pb , = 0.5 i1 0.5 , = 0.5 i .

Updated: 2006-03-20

1 Technical University of Denmark COM Center, Networks Teletrac Engineering & Network Planning Course 34 340

Exercise 10.10

(exam 2005)

CDMA cellular system with two service classes We consider a lost calls cleared system with n = 5 channel. Two types of calls are oered to the system. They both have exponentially distributed service times with mean value 1 = 1 [time unit]. Both types arrive according to a Poisson process:

Type one: arrival rate is 1 = 1 call attempt per time unit. Each call requires d1 = 1 channel. Type two: arrival rate is 2 = 0.5 call attempts per time unit. Each call requires d2 = 2 channels. If a call attempt does not obtain both channels, then it is lost.

1. Find the oered trac for each type expressed in number of channels.

We dene the state of the system as the total number of busy channels. A type one call attempt is always accepted in state 0 and 1. In state i it is blocked with probability: ri =
i 2 5 2

2 i 5.

i As we usually dene 2 = 0 when i < 2, this expression is valid for all states 0 i 5. Thus the probability of accepting a type one call in state i becomes:

a1 (i) = (1 ri ) ,

0 i 5.

A type two call attempt behaves like two single-channel calls and is only accepted if both channels are obtained. Thus the probability of accepting a type two calls in state i becomes: a2 (i) = (1 ri )(1 ri+1 ) , 0 i 5.

2. Show that the arrival rates of accepted type one calls, respectively type two calls, as a function of state becomes:

2 1 (0) = 1 (1) = 1 (2) = 1 (3) = 1 (4) = 1 (5) = 1 1


9 10 7 10 4 10

2 (0) = 2 (1) = 2 (2) = 2 (3) = 2 (4) = 2 (5) =

1 2 1 2 1 2 1 2

1
90 100 63 100 28 100

0 0

We now dene the two-dimensional state of the system as {(i, j) , i + j 5}, where i (0 i 5) is the number of channels occupied by type one calls, and j (j = 0, 2, 4) is the number of channels occupied by type 2 calls. The above arrival rates which are functions of the total number of busy channels are still valid. 3. Construct the state transition diagram, using the following structure of states:
                       

04

14

02

12

22

32

00

10

20

30

40

50

4. Show that the state transition diagram is reversible. 5. Find the state probabilities (given: p(0, 0) = 20000/78342). 6. Find the carried trac and the trac congestion for each trac type. 7. Show that the call congestion for a type of call is equal to the trac congestion for the same type of call.

1 Technical University of Denmark COM Center, Networks Teletrac Engineering & Network Planning Course 34 340

Solution to exercise 10.10

(exam 2005)

Background: In third generation (3G) cellular communication systems based on CDMA (Code Division Multiple Access) the capacity of a cell depends on the number of active users in both own cell and neighbouring cells (or more correct: the power used for transmitting the signals). Assume that the nominal capacity of a cell is n channels. Then call attempts may with a positive probability be blocked even if less than n channels are occupied. This is a multi-dimensional system (Chapter 10) with limited accessibility (Chapter 9). Question 1: As we have PCT-I trac, we get: A1 = A2 = Question 2: We have chosen the same blocking probabilities as for Erlangs Interconnection formula = Erlangs ideal grading, where we choose k = 2 channels at random out of n = 5. If i channels are busy, then ri is the probability that both channels we choose are among the i busy channels. We directly nd: 1 1 d1 = 1 = 1 [erlang channel] 1 1 2 1 d2 = 2 = 1 [erlang channel] 2 2

r0 r1 r2 r3 r4 r5 From:

= = = = = =

0 0
1 10 3 10 6 10

a1 (0) = a1 (1) = a1 (2) = a1 (3) = a1 (4) = a1 (5) =

1 1
9 10 7 10 4 10

a2 (i) = (1 ri )(1 ri+1 we nd the following state-dependent acceptance a2 (i) probabilities and blocking probabilities b2 (i) for calls requiring two channels:

2 a2 (0) a2 (1) a2 (2) a2 (3) a2 (4) a2 (5) From: 1 (i) = (1 ri ) 1 , 2 (i) = (1 ri )(1 ri+1 ) 2 , we get the result shown in the exercise. = = = = = = 1
90 100 63 100 28 100

b2 (0) = b2 (1) = b2 (2) = b2 (3) = b2 (4) = b2 (5) =

0
10 100 37 100 72 100

0 0

1 1

Question 3: We nd the following state transition diagram:


4   10 04  14   1 6 6 9 7 4     ? 10 ? 10 10 -

63 200

7 50

1 2

02  12  22  32     1 2 3 6 6 6 6
9 7 4       ? 1 ? 1 ? 10 ? 10 10 00  10  20  30  40  50      

9 20

63 200

7 50

Question 4: From the state transition diagram it is seen that the ow clockwise is equal to the ow counter-clockwise, so the process is reversible. In general we have:

3
. . . . .. . ... . . . . . . . . . . . . . . . . .

. . .

  (1ri+j+2 ) 1  ............................................................................ ............................................................................ . ................. ................ . . .. . .......................................... i, j +2 ... i+1, j +2 ... .................... .................... ........................................................................... ........................................................................... .. .............. .. .............. . .  .   .  . . . . . . . . . . . . . .. .. . . . .. .. .. . . . . . . . (i + 1) 1 . . . . . . . . . . . . . . . . . . . .

. . . . . . . . . . . . . . . . . . .. . .. . .. . . .

. . .

. . . . .. . . .. . . . . . . . . . . . . . . . . .

. . .

. . . . . . . . . . . . . . . . . . ... . .. .. . . .

. . .

(1ri+j )(1ri+j+1) 2

........................................... i, j . . ........................................ ... . .


. . .. ... . . . . . . . . . . . . . . . . .

. . . . . . . . . . . . . . . . . . . . . . . . . . . . . . . . . . . . . . . . . . . . . . . . . . . . . . . . . . . . . . . . . . . . . . . . . .

. . . . . . . . . . . . . . . . . . . . . . . . . . . . . . . . . . . . . . . . . . . . . . . . . . . . . . . . . . . . . . . . . . . . . . . ... .. .. . . .

(1ri+j+1)(1ri+j+2) 2 j +2 2 2
............................................................................ ........................................................................... . . .. ............................................................................ ........................................................................... . .

(1ri+j ) 1 (i + 1) 1

  . . . . .
.. . .. . . . . . . . . . . . . . . . . .

. . . . . . . . . . . . . . . . . . . . . . . . . . . . . . . . . . . . . . . . . . . . . . . . . . . . . . . . . . . . . . . . . . . . . . . . . .

. . . . . . . . . . . . . . . . . . . . . . . . . . . . . . . . . . . . . . . . . . . . . . . . . . . . . . . . . . . . . . . . . . . . . . . ... .. .. . .

j +2 2 2
................. ................ . .. ................. ................ . .

. . .

. . . . . . . . . . . . . . . ... . .. . . . .

i+1, j
. . . . . . . . . . . . . . . . . . ... . .. .. . .

. . .

. . .

. . .

Question 5: We notice that the system does not have product form. By choosing q(0, 0) = 20 000 we nd the following relative state probabilities using local balance equations (reversibility):

q,4 q,2 q,0

2205 1575 630 22570 10000 9000 3150 53567 20000 20000 10000 78342 31575 29630 13150 Total q0, q1, q2,

420 3000 3420 q3,

525 525 q4,

42 42 q5,

As the total sum adds to 78342 we obtain the state probabilities by diving all terms by this constant. Question 6:

4 The carried trac is obtained from the marginal state probabilities:


5

Y1 =
i=0

i p(i, ) 1 29630 + 2 13150 + 3 3420 + 4 525 + 5 42 78342 68500 78342

= =

= 0.8744 As the oered trac is A1 = 1 [erlang] we nd the trac congestion: C1 = C1 = In a similar way we nd:
5

A1 Y1 A1 9842 = 0.1256 . 78342

Y2 =
j=0

j p(, j) 2 22570 + 4 2205 78342 53960 78342

= =

= 0.6888 C2 = C2 = Question 7 To nd the call congestion we have to sum over all states. The number of call attempts blocked per time unit becomes, using the blocking probabilities from Question 2 (taking the A2 Y2 A2 24382 = 0.3112 78342

5 states row by row):

n1 3 6 1 p(2, 0) + p(3, 0) + p(4, 0) + 1 p(5, 0) = 0 p(0, 0) + 0 p(1, 0) + 1 10 10 10 + 1 3 6 p(0, 2) + p(1, 2) + p(2, 2) + 1 p(3, 2) 10 10 10 6 p(0, 4) + 1 p(1, 4) 10

Inserting the state probabilities we nd:

n1 1 3 6 78342 = 10000 + 3000 + 525 + 42 1 10 10 10 + 3 6 1 10000 + 9000 + 3150 + 420 10 10 10 6 1575 + 630 10

n1 9842 . = 1 78342

As the number of call attempts oered per time unit is 1 = 1 we get:

B1 =

9842 = C1 , 78342

q.e.d.

Thus we get the same value as for the trac congestion C1 obtained in Question 6. In a similar way we nd for trac stream 2 the average number of call attempts blocked per

6 time unit (taking the states row by row): n2 37 72 1 p(1, 0) + p(2, 0) + p(3, 0) + 1 p(4, 0) + 1 p(5, 0) = 0 p(0, 0) + 2 10 100 100 + 37 72 p(0, 2) + p(1, 2) + 1 p(2, 2) + 1 p(3, 2) 100 100

+ 1 p(0, 4) + 1 p(1, 4) n2 37 72 1 20000 + 10000 + 3000 + 525 + 42 78342 = 2 10 100 100 + 37 72 10000 + 9000 + 3150 + 420 100 100

+ 1575 + 630 n2 24382 = 2 78342 Thus we get the same value as for the trac congestion obtained in question 6: B2 = 24382 = 0.3112 = C2 , 78342 q.e.d.

We may also nd the call congestion values from the global state probabilities because the arrival process is a Poisson process. In fact, due to the PASTA-property the trac, call, and time congestions are equal (proper denition of time congestion).
Updated: 2005-07-05

7 Additional discussion on global state probabilities From the above we get the following global state probabilities: 20000 p(0) = p(0, 0) = 78342 p(1) = p(1, 0) = 20000 78342 20000 78342 12000 78342 5250 78342 1092 78342

p(2) = p(2, 0) + p(0, 2) =

p(3) = p(3, 0) + p(1, 2) =

p(4) = p(4, 0) + p(2, 2) + p(0, 4) =

p(5) = p(5, 0) + p(3, 2) + p(1, 4) =

We may obtain the global state probabilities in an alternative way by: 1. Calculating global state probabilities assuming full accessibility. 2. Down-scaling the state probabilities by the blocking probabilities for single-slot trac, as a two-slot call behaves like 2 single-slot calls. This is valid because blocking probabilities only depend on the global state probabilities, and not upon the number of calls of a given type. Without state-dependent blocking we have the state transition diagram shown in the following gure: By choosing q(0, 0) = 120 we nd the relative state probabilities using local balance equations (reversibility): q,4 q,2 q,0 30 160 15 60 15 60 30 60 90 q2, 10 20 30 q3, 5 5 q4, 1 1 q5,

326 120 120 426 195 195 Total q0, q1,

8
  104  14   1 6 6

1 2

1 2

    ? 1 ? 1 102  12  22  32     1 2 3 6 6 6 6

1 2

      ? 1 ? 1 ? 1 ? 1 100  10  20  30  40  50      

1 2

1 2

1 2

The global relative state probabilities become: p(0) = p(0, 0) = 120 p(1) = p(1, 0) = 120 p(2) = p(2, 0) + p(0, 2) = 120 p(3) = p(3, 0) + p(1, 2) = 80 p(4) = p(4, 0) + p(2, 2) + p(0, 4) = 50 p(5) = p(5, 0) + p(3, 2) + p(1, 4) = 26

The scaling probabilities are shown below together with the resulting state probabilities:

State 0 1 2 3 4 5

Relative probability 120 120 120 80 50 26

Down-scaling factor 1 1 1
9 10 9 10 9 10

Relative Absolute probability probability 120 120 120 72


20000 78342 20000 78342 20000 78342 12000 78342 5250 78342 1092 78342

7 10 7 10

63 2

4 10

819 125

9 We observe that we get the same result as above. For Poisson arrival process we can nd the call congestion from the global state probabilities: B1 = 0 p(0) + 0 p(1) + = 1 3 6 p(2) + p(3) + p(4) + p(5) 10 10 10

0 + 0 + 2000 + 3600 + 3150 + 1092 , 78342 9842 78342 q.e.d.

B1 = For type 2 we get:

B2 = 0 p(0) + =

1 37 72 p(1) + p(2) + p(3) + p(4) + p(5) 10 100 100

0 + 2000 + 7400 + 8640 + 5250 + 1092 , 78342 24382 78342 q.e.d.

B2 =

Due to the Pasta-property we of course get identical values for time, call, and trac congestion. For non-Poisson arrival processes (e.g. Engset and Pascal) we cannot calculate time call and trac congestion from the global state probabilities, only the time congestion.
Updated: 2006-05-11

1 Danmarks Tekniske Universitet COM Center Data & Teletrakteori kursus 34 340

Exercise 10.9

(exam 2003)

VoIP = Voice over Internet Protocol Background orientation: Vocoders (voice coders) implement a voice activity detector algorithm which uses less bandwidth during silence periods than during active speech periods (talk-spurts). The constant bit-rate during speech activity periods is D. For some common coders (as G.723 and G.729) the constant bit-rate d during silence periods to describe the background noise level and other parameters is a little higher than D/2. The voice activity factor is the proportion of time the call (subscriber) is in the active speech state. This value is usually about 40 %. For simplicity, we choose in the following D = 2, d = 1, and voice activity factor = 0.5. The voice quality deteriorates if more than a few percentage of the talk-spurts are lost. Task: We consider a loss system with n = 5 channels. Call attempts arrive according to a Poisson process with intensity = 2 calls per time unit. We allow at most 4 simultaneous calls in the system. An accepted call is in one out of two states: either silence state where it uses one channel, or active speech state (talk-spurt) where is uses two channels (if possible). We assume that an accepted call starts in silence state (only need one channel at arrival) and only can terminate in the silence state, i.e. the service rate is 0 in active speech state and = 1 (time unit)1 in silence state. (All time intervals are exponentially distributed). Each call changes from silence to speech activity (i.e. attempts to transmit a talk-spurt) with the same rate as it changes from speech activity to silence (terminate a talk-spurt). If there is no idle channel, the talk-spurt is lost and the call stays in silence state (Lost Talk-spurts Cleared model, cf. Engset model). In this task the rate can be chosen arbitrarily, so choose = 1. We dene the state of the system as (i, j), where i (0 i 4) is the number of calls in the system, and j (0 j i) is the number of calls in state active speech. Furthermore, we have i + j 5 due to the total number of channels n = 5. Thus we get the states shown on the following page. 1. Construct from the given states the complete state transition diagram. 2. Show that the state transition diagram is reversible and nd the state probabilities under the assumption of statistical equilibrium. As a control we have p(0, 0) = 3/77. 3. Write down the node balance equation for state (2,1). Find (expressed by state probabilities) the time and call congestion for new call attempts. 4. Find (expressed by state probabilities) the time and call congestion for talk-spurts.

 

 

2,2

3,2

 

 

 

 

1,1

2,1

3,1

4,1

 

 

 

 

 

0,0

1,0

2,0

3,0

4,0

1 Technical University of Denmark COM Center, Networks Teletrac Engineering & Network Planning Course 34 340

Solution to exercise 10.9 Question 1:

(exam 2003)

We obtain the following state transition diagram:

Question 2: For all 4 squares we get that clockwise ow equals counter-clockwise ow (Sec. 10.2): Square (1,0) (1,1) (2,1) (2,0) : Square (2,0) (2,1) (3,1) (3,0) : Square (3,0) (3,1) (4,1) (4,0) : Square (2,1) (2,2) (3,2) (3,1) : 1212 = 2211 2213 = 2321 3214 = 2431 1222 = 2212

Thus we can express all states by state (0,0) by using local balance equations. The relative

2 state probabilities become: 2 2 1 2 4 2 4 4


4 3 8 3 2 3

The relative state probabilities add to 77/3, and by normalising we nd the absolute state probabilities as follows: p(2, 2) = p(1, 1) = p(0, 0) = Question 3: For state [2,1] we have: Flow out : Flow in : (1 + 1 + 2 + 1) p(2, 1) 2 p(1, 1) + 2 p(2, 2) + 2 p(3, 1) + 2 p(2, 0) ,
3 77 6 77 6 77 6 77 12 77 6 77

p(3, 2) = p(3, 1) = p(3, 0) =

12 77 12 77 4 77

p(2, 1) = p(2, 0) =

p(4, 1) = p(4, 0) =

8 77 2 77

p(1, 0) =

The node balance equation is obtained by letting Flow out = Flow in. New call attempts arrive according to a Poisson process and therefore the time congestion E and call congestion B are equal (PASTAproperty). From the state transition diagram we get: E = B = p(3, 2) + p(4, 1) + p(4, 0) = 22 . 77

Extra: trac congestion We do not ask for the trac congestion as this is more complex. Calls only terminate in the silence state, and if there is no blocking, then a call will spend half the time in active speech state. If there is no blocking of talk-spurts the average holding time will become two times 1/, and thus the oered trac becomes A = 2 = 4 [erlang]. The carried trac is obtained from the state probabilities: Y =
i,j

i p(i, j) =

184 . 77

Thus the trac congestion becomes: C= AY 124 31 = = . A 308 77

3 Question 4: For talk-spurts we have a nite number of sources (cf. Engset) and we nd: Time congestion Ets : 20 . Ets = p(3, 2) + p(4, 1) = 77 In states [0,0], [1,1] and [2,2] we have idle channels (and no calls in active speech state), so there is no time congestion in these states. Call congestion Bts : During one time unit we have the following average number of talk spurts: mts = 0 {p(0, 0) + p(1, 1) + p(2, 2)} +1 {p(1, 0) + p(2, 1) + p(3, 2)} +2 {p(2, 0) + p(3, 1)} +3 {p(3, 0) + p(4, 1)} +4 {p(4, 0)} mts = = 0+ 8 30 36 36 + + + 77 77 77 77

110 77 From these mts talk-spurts so many are blocked: mbl = = 1 p(3, 2) + 3 p(4, 1) = 12 24 + 77 77

36 . 77 Thus the call congestion for talk-spurts becomes: Bts = mbl 18 . = mts 55

Extra: trac congestion We do not ask for the trac congestion of talk spurts. If the number of channels were innite, then the average trac would be 4 [erlang] and on average two of these would be in active speech state. So the oered trac of talk spurts is Ats = 2 [erlang]. The average carried trac of talk spurts is Yts =
i,j

j p(i, j) =

74 . 77

4 Thus the trac congestion becomes: Cts = Ats Yts 40 . = Ats 77

Part of the trac congestion is due to the limitation in number of calls.


Updated: 2005-04-11

1 Technical University of Denmark COM Center, Networks Teletrac Engineering & Network Planning Course 34 340

Exercise 10.8

(exam 1998)

MOBILE COMMUNICATION SYSTEM WITH TWO TYPES OF TRAFFIC A mobile communication system with S = 4 subscribers has access to n = 3 channels. All calls accepted occupy one channel during an exponentially distributed time interval with mean value 1 = 1 time unit. The system is operated as a loss system. There are two arrival processes: a. Outgoing calls, generated by the S = 4 subscribers (PCTII trac). An idle source generates = 1 call attempts per time unit. 4 b. Incoming calls, arriving according to a Poisson process with arrival rate = 0.8 call attempts per time unit (PCTI trac). An incoming call, which is accepted, occupy both an idle channel and one of the idle sources, which thus becomes busy without making a call attempt itself. The number of busy sources thus always equals the number of busy channels. 1. Find the incoming oered trac Ai , the outgoing oered trac Ao , and the total oered trac At (assume the trac stream considered is alone). 2. Construct the one-dimensional state transition diagram for the system, when the state of the system is dened as number of busy channels. Find the state probabilities under the assumption of statistical equilibrium. 3. Find the time congestion E, the call congestion B, and the trac congestion C for both trac streams. (The trac congestion for outgoing calls is obtained from the total trac congestion and the known trac congestion for incoming calls). 4. Show that the state transition diagram can be interpreted as a state transition diagram for a single PCT-II trac stream, and nd the equivalent number of sources (nonintegral) and the arrival intensity per idle source. 5. We now distinguish between the two types of trac. Construct the twodimensional state transition diagram, where the state of the system (i, j) denotes that there are i incoming calls and j outgoing calls. Is the state transition diagram reversible? The following question was not included at exam. 6. Find time congestion E, call congestion B, and trac congestion C for both trac streams, when we know the state probabilities of the two-dimensional state transition diagram.

1 Technical University of Denmark COM Center, Networks Teletrac Engineering & Network Planning Course 34 340

Solution to Exercise 10.8 Question 1: It should be pointed out that the oered trac is dened as the trac carried, when the capacity of the system is innite and other trac streams do not exist. 0.8 = = 0.8 [erlang] . 1

Ain = From the formul (8.5 8.9): =

1 , 4

1 = , 4

1 = , 1+ 5 4 = 0.8 [erlang] . 5

Aout = S =

Total oered trac becomes: At = Ain + Aout = 0.8 + 0.8 = 1.6 [erlang] . Question 2: The arrival rate (intensity) in state i is: (4 i) + = (4 i) The departure rate (intensity) in state i is: i = i. Therefore, we get the following state transition diagram: 1 4 + . 4 5

       
36 20

31 20

26 20

The relative state probabilities become: 1 which add to a total of 4.7995. The absolute state probabilities then become: p(0) = 0.2084 = p(1) = 0.3750 = p(2) = 0.2907 = p(3) = 0.1260 = 2000 , 9599 3600 , 9599 2790 , 9599 1209 . 9599 1.8 1.395 0.6045

Question 3: Due to the PASTAproperty time, call and trac congestion of the incoming trac are equal: Ein = Bin = Cin = p(3) = 0.1260 . For the outgoing trac we nd:

Time congestion: Eout = p(3) = 0.1260 . Call congestion: Bout = Trac congestion: 0.25 p(3) = 0.0473 . 1 p(0) + 0.75 p(1) + 0.50 p(2) + 0.25 p(3)

3 We cannot nd this directly, but indirectly, e.g. from the total carried trac: Yt = 0 p(0) + 1 p(1) + 2 p(2) + 3 p(3) = 1.3342 . Yout = Yt Ain (1 Cin ) = 1.3342 0.8 (1 0.1260) = 0.6350 . Then we can nd the trac congestion: C= Aout Yout = 0.206 . Aout

Question 4: For a system with PCT-II trac with a non-integral number of sources, the departure intensity in state i is given by i and the arrival intensity by (S i) . A comparison with the gure in question 2 shows us that: = 1 , = 1 , 4 S = 1.8 = 7.2 . 0.25

Question 5:

03
0.5 3 0.8

02
1 0.75 2 0.5 0.8

12
2 0.8

01
1 1 1 0.75 0.8

11
2 1 0.5 0.8

21
1 0.8

00
1

10
2

20
3

30

The system is not reversible. For example, the circulation ow of the following states are:

4 {00 10 11 01 00}: Clockwise: Anticlockwise: Cf. Fig. 10.2 in the textbook.


Updated: 2005-04-12

1 1 0.8 1 = 0.8 0.8 0.75 1 1 = 0.6

1 Technical University of Denmark COM Center, Networks Teletrac Engineering & Network Planning Course 34 340

Exercise 10.7

(exam 1994)

MULTIDIMENSIONAL LOSS SYSTEM We consider a fully accessible loss system with n = 5 channels, which are oered two trac streams. All arrival processes and service times are of type M (Poisson exponential), and all parameters are given in the same time unit. Trac stream 1: Arrival intensity 1 = 2, Mean service time 1 = 1, 1 Every call requires d1 = 1 server. Trac stream 2: Arrival intensity 2 = 1, Mean service time 1 = 1 , 2 2 Every call requires d2 = 2 servers. If a call attempts is not allocated the required capacity at the time of arrival, then it is lost (Lost Calls Cleared). 1. Find the oered trac for both trac streams measured in number of channnels. 2. Construct a twodimensional state transition diagram of the system, where state (i, j) denotes that i channels are occupied by type 1 calls (i = 0, 1, 2, 3, 4, 5), and j channels are occupied by type 2 calls (j = 0, 2, 4), where we have i+j 5. 3. Show that the state transition diagram is reversible, and nd all state probabilities p(i, j) expressed by p(0, 0). Find time congestion, call congestion and trac congestion for both trac streams. Let p(j) denote the summation of alle states, where the total number of busy servers is j, i.e.
j

p(j) =
x=0

p(x, j x) .

4. Show that the following recursion formula is valid for the system considered:
2

j p(j) =
i=1

i di p(j di ) . i

1 Technical University of Denmark COM Center, Networks Teletrac Engineering & Network Planning Course 34 340

Solution to Exercise 10.7: Question 1: The oered trac is dened as the trac carried when there is no blocking. The trac carried during a time interval is the average number of busy channels. For Poisson arrival processes (in general renewal arrival proceses) we thus have: Ai = i si di , where si is the average holding time and di is the number of channels occupied by a call (slotsize). We nd: A1 = 1 s1 d1 = 2 1 1 A1 = 2 erlang A2 = 2 s2 d2 = 1 A2 = 1 erlang Question 2:
  2E 04 ' 14   1 T T

1 2 2

    c 2 c 2 2E E E 02 ' 12 ' 22 ' 32     1 2 3 T T T T

      c 2 c 2 c 2 c 2 2E E E E E 00 ' 10 ' 20 ' 30 ' 40 ' 50      

Question 3: Flow clockwise must be equal to ow counterclockwise in all four squares {(i, j) = (1, 2), (1, 4), (2, 2), (3, 2)} :

2
  T  

1 i

i-1, j '

 E  T

 

i, j

j 1 i

c   

i-1, j-2 ' 

c   E 

i, j-2

Clockwise: Counterclockwise:

2 1 j i 1 2 i j q.e.d.

Therefore, we can express all state probabilities by p(0, 0). The relative values become, using the same positioning as in the state transition diagram: 1 8 1 2 1 or 15 60 120 30 120 240 120 240 80 160 80 32 2 8 1 2 1 2 2 3 4 3 2 3 4 15

The total sum of all terms in the latter array is 1297, and the absolute state probabilities are obtained by dividing all terms by 1297. Time congestion equals call and trac congestion as all call intensitites are independent of the states (PASTAproperty): E1 = p(1, 4) + p(3, 2) + p(5, 0) E1 = 142 1297

B1 = C1 = E1 = 0, 1095

3 E2 = p(0, 4) + p(1, 4) + p(2, 2) + p(3, 2) + p(4, 0) + p(5, 0) E2 = 357 1297

B2 = C2 = E2 = 0, 2753 Question 4: For the global state probabilities we nd: p(0) = p(0, 0) = p(1) = p(1, 0) = 120 1297 240 1297 300 1297 280 1297 215 1297 142 1297

p(2) = p(2, 0) + p(0, 2) = p(3) = p(3, 0) + p(1, 2) =

p(4) = p(4, 0) + p(2, 2) + p(0, 4) = p(5) = p(5, 0) + p(3, 2) + p(1, 4) = We shall show that the following recursion formula is valid:
2

j p(j) =
i=1

i di p(j di) i

1 2 d1 p(j 1) + d2 p(j 2) 1 2

j p(j) = 2 p(j 1) + p(j 2) We do this by showing it is valid for all possible values of j (notice p(x) = 0 for x < 0 ). j=1: p(1) = 2 p(0) = 240 1297 q.e.d. 600 1297 q.e.d.

j = 2 : 2 p(2) = 2 p(1) + p(0) = p(2) = 300 1297

4 j = 3 : 3 p(3) = 2 p(2) + p(1) = p(3) = 280 1297 q.e.d. 860 1297 840 1297

j = 4 : 4 p(4) = 2 p(3) + p(2) = p(4) = 215 1297 q.e.d.

j = 5 : 5 p(5) = 2 p(4) + p(3) = p(5) =

710 1297

142 q.e.d. 1297 We have thus shown that the recursion formula is valid for the system considered. The recursion formula is usually called Kaufman & Roberts algorithm (Sec. 10.5). Additional remark The global state probabilities above could of course be obtained by the convolution algorithm:

State Stream 1 0 1 2 3 4 5 1 2 2
4 3 2 3 4 15

Stream 2 Convolved Normalised 1 0


1 2

1 2
5 2 7 3 43 24 71 60

120 1297 240 1297 300 1297 280 1297 215 1297 142 1297

0
1 8

Updated: 2005-04-14

1 Technical University of Denmark COM Center, Networks Teletrac Engineering & Network Planning Course 34 340

Exercise 13.7

(Exam 1984)

M/H2 /1 QUEUEING SYSTEM WITH PROCESSOR SHARING Jobs arrive to a computer system according to a Poisson process with intensity . The service time is distributed as a hyper-exponential distribution with two phases denoted by a, respectively b: F (t) = p (1 ea t ) + (1 p) (1 eb t ) .
E

p
E

a
E

1p = q

1. Find the oered trac A. In the following we assume that A < 1. The computer system operates as a single-server system with processor sharing queueing discipline, i.e. if there are x jobs in the system, then a job in phase a is served with rate a /x and a job in phase b is served with rate b /x. The state of the system is dened as (i, j), where i is the number of jobs in phase a, and j is the number of jobs in phase b. The state transition diagram becomes twodimensional with the structure shown in the following gure. 2. Find the missing intensities in connection with the states: (1,1), (1,2), (2,2) and (2,1). 3. Show by considering the above-mentioned four states that the state transition diagram is reversible. An M/M/1 queueing system with the oered trac A (A < 1) has the equilibrium state probabilities: p(i) = p(0) Ai , i = 0, 1, 2, . . .

2 4. Show by expressing the state probabilities by state p(0, 0) that the above M/H2 /1 processor sharing system has the same state probabilities as M/M/1 when we let:
i

p(i) =
x=0

p(x, i x) , i = 0, 1, 2, . . . ,

and we only consider i = 1 and 2.

 c p

 c E

03 '  1 a T 4 q b

13 '  T

 c p

 c E

 c E

02 '  1 a T 3 q b

12 '  T

22 '  T

 c p

 c E

 c E

 c E

01 '  1 a T 2 q b

11 '  T
1 2

21 '  T q
1 3

31 '  T q
1 4

 c p

 c p E

 c p E

 c E

00 ' 

10 ' 

20 ' 

30 ' 

1 Technical University of Denmark COM Center, Networks Teletrac Engineering & Network Planning Course 34 340

Solution to Exercise 13.7 Question 1: The average service time s = m1 is (4.25): s= The oered trac the becomes: A= Question 2: (1 p) p + a b . p (1 p) + . a b

 c p

 c E

03 '  1 a T 4 q b

13 '  T
3 4

b
 c E

 c p

 c p E

02 '  1 a T 3 q b

12 '  2 a T 4
2 3

22 '  T
2 4

b
 c E

 c p

 c p E

 c p E

01 '  1 a T 2 q b

11 '  2 a T 3
1 2

21 '  3 a T 4
1 3

31 '  T
1 4

 c p

 c p E

 c p E

 c E

00 ' 

10 ' 

20 ' 

30 ' 

2 Note: The capacity of the server is constant independent of the state. But the service rate (the intensity at which the customers leave the server) depends on the mix of customers. If most customers in the system has a short service time, then service rate will be high, whereas if most customers have a long service time, then the service rate will be low. Question 3: For the four states considered we have:

Flow clockwise: Flow counter clockwise:

2 2 p b a , 4 3 2 2 p q a b . 4 3

We notice that they are equal. Thus the process is reversible. The same is valid for the other squares, and we may express all states by state p(0, 0) (10.15). Question 4: We nd (q = 1 p): p(0, 1) = (1 p) p(0, 0) , b p p(0, 0) , a

p(1, 0) =

and thus:

p(1) = p(0, 1) + p(1, 0) = p(0, 0) p(1) = p(0) A p 1p + a b q.e.d. ,

where p(0) = p(0, 0) and A is obtained in Question 1.

3 In a similar way we nd: p(0, 2) = (1 p) (1 p) p(0, 0) , b b (1 p) p p(0, 0) , a 1 b 2 p p p(0, 0) . a a

p(1, 1) =

p(2, 0) =

p(2) = p(0, 2) + p(1, 1) + p(2, 0) = p(0, 0)


2

1p b

2p (1 p) + + a b
2

p a

p(2) = p(0, 0) p(2) = p(0) A2

p 1p + a b q.e.d.

1 Technical University of Denmark COM Center, Networks Teletrac Engineering & Network Planning Course 34 340

Exercise 13.8

(Exam 1985)

CYCLIC QUEUEING SYSTEM We Consider the following cyclic queueing system:


'

S = 6 jobs =1 
E 0    t =1 t  t  E E 

=2 

I/O Channels

CPU

Six jobs circulate in a system and are served alternatively at a CPU (exponentially distributed service time with mean value 1 = 1/2 time unit) and at one out of two I/O-channels (full accessibility, exponentially distributed service time with mean value 1 = 1 time unit). If both I/O-channels are idle a job chooses a channel at random. The state of the system is dened as the number of jobs, which are being served or are waiting in queue at the CPU, i.e. the number of jobs in the box on the gure. The system is assumed to be in statistical equilibrium. 1. Construct the state transition diagram of the system and nd the state probabilities. 2. Find the utilisation of the CPU and of each of the two I/Ochannels. 3. Calculate the average queue length at each of the two queueing systems from the state probabilities. Apply Littles law to obtain the total cirxulation time for a random job. (Circulation time = average time of a total cyclus). The state probabilities obtained in Question 1 are time averages, i.e. the state probabilities at a random point of time. 4. Find the state probabilities as they are experienced by by a job which just has nished service at an I/O-channel and arrives at the CPU-queueing system (call average).

2 Then calculate the probability that a job experiences at positive waiting time at the CPU, respectively at the I/Ochannels, and nd the mean waiting times for delayed jobs at the CPU, respectively at the I/Ochannels. 5. (This question has been added after exam) Calculate the state probabilities by using the convolution algorithm for queueing networks.

1 Technical University of Denmark COM Center, Networks Teletrac Engineering & Network Planning Course 34 340

Solution to exercise 13.8: In the following an index i, respectively c, refers to the I/O-channel system, respectively to the CPUsystem. Question 1:
       j j j j j j       

You should immediately use the numerical values for og . By means of the cut equations we easily nd: p(i) = p(6) = Question 2: The utilisation of a server is equal to the trac carried By the server. The utilisation of the CPU is: c = 1 p(0) , c = 11 13 2 , 13 1 . 13 i = 0, 1, . . . , 5 ,

as the CPU carries one erlang in all states except state 0. The trac carried by the two I/O-channels becomes:
4

= 2
i=0

p(i) + 1 p(5)

22 13

2 The two I/O-channels have the same utilisation:

i = i =

1 22 2 13 11 13

The two I/O-channels have the same utilisation as the CPU, which is evident because the mean service time at the I/O-channels is twice the mean service time at the CPU. Question 3: The average queue length at the CPU becomes:

Lc =
i=1

(i 1) p(i) 2 2 2 2 1 +2 +3 +4 +5 , 13 13 13 13 13

= 1 Lc =

25 . 13

At the I/Ochannels the average queue length becomes:

Li =
i=0

(6 2 i) p(i)

= 4 p(0) + 3 p(1) + 2 p(2) + 1 p(3) = 4 Li = 2 2 2 2 +3 +2 +1 , 13 13 13 13

20 . 13

As a check we know that the total number of jobs in the system is 6 and that they stay either

3 at a server or in a queue at the I/O-channels, respectively at the CPU: I/Ok : I/Obetjening : CPU-k : CPUbetjening : Total : Li = 2i = Lc = c = S = 20 13 22 13 25 13 11 13

78 =6 q.e.d. 13 Because the same jobs circulates in the system we get the same in Littles law, independent of where we put a cut. The average number of jobs circulating per time unit is for example obtained at the CPU, which terminates 2 jobs per time unit except in state 0, where it is idle: = 2 {1 p(0)} , 22 . 13 By means of Littles law L = W we nd the mean sojourn time W in each of the four cuts: = I/Oqueue : I/Oservers : CPU-queue : CPUserver : W1 = W2 = W3 = W4 = 10 1 13 20 Li = = 22 13 11 1 13 22 2i = =1 22 13 25 1 13 25 Lc = = 22 13 22 (known) (known)

1 1 13 11 c = = 22 13 2 The total circulation time is equal to the sum: Circulation time = 39 11 (time units)

Applying Littles law to the 4 systems, we nd: L = W , 6 = W = 22 W , 13 78 39 = 22 11 q.e.d.

4 Question 4: Per time unit the following number of customers arrive to the CPU-system: c = 2 p(0) + 2 p(1) + 2 p(2) + 2 p(3) + 2 p(4) + 1 p(5) + 0 p(6) , c = (which we already obtained in Question 3). From this we get the following number of customers per time unit: 2 p(0) 2 p(1) 2 p(2) 2 p(3) 2 p(4) 1 p(5) 0 p(6) observes the system in state 0 , observes the system in state 1 , observes the system in state 2 , observes the system in state 3 , observes the system in state 4 , observes the system in state 5 , observes the system in state 6 . 22 , 13

The call average probabilities thus becomes: (i) = 2 , 11 1 . 11 i = 0, 1, 2, 3, 4 .

(5) =

At the CPU we have postive waiting time in the states 1, 2, 3, 4 and 5: p{positive waiting time at CPU} = 9 . 11

There is waiting time at the I/O-channels in the states 0, 1, 2 og 3: p{positive waiting time at I/O} = 8 . 11

In general, we have (3.20): W =wD

5 where D is the probability of experiencing a positive waiting time, W is the mean sojourn for all jobs (obtained in question 3), and w is the mean sojourn time for delayed jobs. CPU: wc = 1 11 25 W3 = D 9 22 wc = 25 18

As the mean service time is 1/2 the waiting time becomes 8/9. I/Ochannels: wi = 1 11 10 W1 = D 8 11 wi = 5 4

In this case the mean service time is 1, so the mean waiting time becomes 1/4. The call averages may also be obtained from the time averages in a system with one job less (the arrival theorem). Question 5: The state probabilities are obtained as for M/M/2, Respectively M/M/1. Only the relative values are of importance. When concolving the state probabilities we x the number of jobs to 6. To be continued
Updated: 2004-05-25

1 Technical University of Denmark COM Center, Networks Teletrac Engineering & Network Planning Course 34 340

Exercise 14.3

(exam 1996)

QUEUEING NETWORK WITH LOSS SYSTEMS We consider an open queueing network consisting of two loss systems (nodes) in series. Each loss system has two servers. Customers arrive from outside to the rst loss system according to a Poisson process with intensity = 1 calls per time unit. The service times are exponentially distributed with mean value one in both nodes. Customers which are blocked at node one immediately pass on to node two. Customers which are blocked in node two leave the system.
-

Node 1

Node 2

1. Find the oered trac to node one, and nd the state probabilities for this node. 2. Construct the state transition diagram of the system, letting state (i, j) denote i busy servers in node one and j busy servers in node two.

0,2

0,1

0,0

     

1,2

1,1

1,0

     

2,2

2,1

2,0

     

3. Show that the state transition diagram is non-reversible 4. The system can be shown to have product form: p(i, j) = p(i) p(j),

2 where p(i) is obtained in Question 1, and p(j) is identical with p(i), as the two systems have the same oered trac. Show that these state probabilities full the node balance equation for state (1,1). 5. Find from the state transition diagram the probability that a call which is blocked at node one also is blocked at node two. 6. Find the probability that a random call: 1. Is served in both nodes. 2. Is served at node one, but blocked at node two. 3. Is blocked at node one, but served at node two. 4. Is blocked at both nodes.

1 Technical University of Denmark COM Center, Networks Teletrac Engineering & Network Planning Course 34 340

Solution to exercise 14.3: Queueing network of loss systems Question 1:

(exam 1996)

Node one corresponds to a classical Erlang loss system with 2 channels and oered trac 1 erlang: A = = 1 erlang
0 1 1 1 1 2 2

Ai p(i) = 2 i! A ! =0 p0 = 0.4 , p1 = 0.4 , p2 = 0.2 Question 2: The state transition diagram becomes as follows:
1 1

0,2
1 2 1

1,2
2 2 2 1

2,2

0,1

1,1
1 2

2,1

0,0

1,0

2,0

2 Question 3: The diagram cannot be reversible as there only is ow in one direction between many states. Question 4: If we assume product form, then we get the following state probabilities:

2 0.2 1 0.4 0 0.4 i j

0.08 0.08 0.04 0.16 0.16 0.08 0.16 0.16 0.08 0.4 0 0.4 1 0.2 2

For state (1, 1) we obtain the following node balance equations: (1 + 2 + ) p11 = p01 + 21 p20 + 22 p12 Letting 1 = 2 = = 1 we get the following: 3 p11 = p01 + p20 + 2 p12 3 0.16 = 0.16 + 2 0.08 + 2 0.08 0.48 = 0.48 q.e.d.

The node balance equation is thus fullled for state (1, 1). In a similar way we can show this is valid for all other states. As the solution fulls the node balance equation we have shown that the system has product form. Question 5: Customers which are blocked in node one, arrive at node two in one of the states (2,0), (2,1) or (2,2). Among these, customers arriving to node two in state (2,2) are also blocked: P {blocked in node 2 | blocked in node 1} = = p22 p20 + p21 + p22 0.04 1 = 0.08 + 0.08 + 0.04 5

This is the same as for a random customer. It shows again that the two nodes behaves as two independent systems. State p(2, 2) is the probability that a customer is blocked in both nodes.

3 Question 6: Due to independence (product form) we get: p1 = p2 = p3 = p4 = 4 4 16 = = 0.64 5 5 25 4 4 1 = = 0.16 5 5 25 4 1 4 = = 0.16 5 5 25 1 1 1 = = 0.04 5 5 25

The probabilities can also be obtained from the state transition diagram. Per time unit = 1 customers arrive to the system. 1. Per time unit the number of customers which are served at node one and also accepted at node two is equal to: 1 p10 + 2 p20 + 1 p11 + 2 p21 = 0.16 + 0.16 + 0.16 + 0.16 = 0.64 q.e.d.

2. Per time unit the number of customers blocked at node two after being served at node one becomes: 2 p22 + 1 p12 = 2 0.04 + 1 0.08 = 0.16 q.e.d 3. Per time unit the following number of customers are accepted at node 2 after being blocked at node 1: 1 p20 + 1 p21 = 0.16 q.e.d.

4. Per time unit the following number of customers are blocked at both nodes: 1 p22 = 0.04
Updated 2006-04-26

q.e.d.

1 Technical University of Denmark COM Center, Networks Teletrac Engineering & Network Planning Course 34 340

Exercise 14.6

(exam 2001)

MACHINEREPAIR MODEL AS A CYCLIC QUEUEING NETWORK We consider a machinerepair model with 4 customers (sources, terminals). The customers have exponentially distributed thinking times with intensity 1 = 0.5 customers per time unit (node 1). The customers are served by to successive single servers. A customer is rst served at node 2 which is a single server with exponentially distributed service times with mean value 1 = 1 time units. Then a customer continues to node 3 which is a single server 2 with exponentially distributed service times with mean value 1 = 1/2 time units. After 3 nishing service at node 3 the customers return to node 1 and starts a new thinking time. This system is a single chain cyclic queueing network with 3 nodes and 4 identical sources. Node 1 (the terminals) corresponds to an M/M/ queueing system whereas both node 2 and node 3 are M/M/1 singleserver systems. The customers circulate between the nodes in the cyclic sequence 1, 2, 3, 1, 2, . . .. 1. Let the relative load of node 3 be equal to one and nd the relative loads of node 1 and node 2. 2. Find the relative state probabilities of each node considered in isolation. 3. Apply the convolution algorithm to nd the absolute state probabilities of each node. 4. Find the average number of customers in each node. 5. Find the average sojourn (waiting + service) time for a customer in each node, and the average total cycle time. 6. Find the mean queue length observed by a customer at the three nodes if we increase the number of customers from 4 to 5.

1 Technical University of Denmark COM Center, Networks Teletrac Engineering & Network Planning Course 34 340

Solution to exercise 14.6 Question 1:

(exam 2001)

As all three nodes serve the same number of customers ( per time unit), the relative loads are proportional with the mean service times. The relative load is the relative oered trac, which in queueing systems is equal to the carried trac. The oered trac is A = s where s is the mean holding time, but we do not know . We get: 1 = 4 Question 2: The rst node is of type M/M/ and has the relative state probabilities: p(i) = p(0)
i 1 , i!

2 = 2

3 = 1 .

i = 0, 1, . . . .

The second and the third node are both of type M/M/1 and have the relative state probabilities: p(i) = p(0) i , i = 0, 1, . . . . Thus we get the following relative state probabilities:

i 0 1 2 3 4

q1 (i) 1 4 8
32 3 32 3

q2 (i) 1 2 4 8 16

q3 (i) 1 1 1 1 1

In the following we multiply the relative state probabilities q1 (i) of node 1 by 3 to work with integers. Question 3: We now apply the convolution algorithm to nd the absolute state probabilities of each node. The node we want to consider should be the last one convolved. We nd:

2 Node 1: i 0 1 2 3 4 q2 (i) 1 2 4 8 16 q3 (i) 1 1 1 1 1 q23 (i) = q2 q3 1 3 7 15 31 q1 (i) 3 12 24 32 32 q123 (i) = q23 q1 3 21 81 233 569

The term of interest is 569, which is made up of the following contributions:

q1 (0) q23 (4) + q1 (1) q23 (3)+q1 (2) q23 (2)+q1 (3) q23 (1)+q1 (4) q23 (0) = 31 3 = 569 . From this we get the state probabilities of node 1: p1 (0) = p1 (1) = p1 (2) = p1 (3) = p1 (4) = 93 = 0.1634 , 569 180 = 0.3163 , 569 168 = 0.2953 , 569 96 = 0.1687 , 569 32 = 0.0562 . 569 + 15 12 + 7 24 + 3 32 + 1 32

In a similar way we nd the state probabilities of node 2 and node 3.

3 Node 2: i 0 1 2 3 4 q1 (i) 3 12 24 32 32 q3 (i) 1 1 1 1 1 q13 (i) = q1 q3 3 15 39 71 103 q2 (i) 1 2 4 8 16 q123 (i) = q13 q2 3 21 81 233 569

The term of interest is 569, which is made up of the following elements:

q2 (0) q13 (4) + q2 (1) q13 (3) + q2 (2) q13 (2) + q2 (3) q13 (1) + q2 (4) q13 (0) = 1 103 = 569 . From this we get the state probabilities of node 2: p2 (0) = p2 (1) = p2 (2) = p2 (3) = p2 (4) = 103 = 0.1810 , 569 142 = 0.2496 , 569 156 = 0.2742 , 569 120 = 0.2109 , 569 48 = 0.0844 . 569 + 2 71 + 4 39 + 8 15 + 16 3

4 Node 3: q12 (i) = q1 q2 3 18 60 152 336 q123 (i) = q12 q3 3 21 81 233 569

i 0 1 2 3 4

q1 (i) 3 12 24 32 32

q2 (i) 1 2 4 8 16

q3 (i) 1 1 1 1 1

The term of interest is 569, which is made up of the following elements:

q3 (0) q12 (4) + q3 (1) q12 (3) + q3 (2) q12 (2) + q3 (3) q12 (1) + q3 (4) q12 (0) = 1 336 = 569 . + 1 152 + 1 60 + 1 18 + 13

From this we get the state probabilities of node 1: p3 (0) = p3 (1) = p3 (2) = p3 (3) = p3 (4) = Question 4: The average number of customers in each node is obtained from the state probabilities:
4

336 = 0.5905 , 569 152 = 0.2671 , 569 60 = 0.1054 , 569 18 = 0.0316 , 569 3 = 0.0053 . 569

Lj =
i=0

i pj (i) ,

j = 1, 2, 3 .

5 We nd: L1 = L2 = L3 = 932 = 1.6380 , 569 1006 = 1.7680 , 569 338 = 0.5940 . 569

As control we have the total number of customers: L1 + L2 + L3 = 4 . Question 5: The utilisation of the nodes is obtained from the state probabilities. The rst node is an innite-server and the two others are single-server systems, which has the carried trac 1 p(0). For a single-server system the carried trac equals the utilisation. The carried trac becomes:

Node 1: Node 2: Node 3:

Y1 = Y2 Y3

932 = 1.6380 [erlang] , 569 103 = 0.8190 [erlang] , = 1 569 336 = 0.4095 [erlang] . = 1 569

The circulation rate is therefore: = 1 1.6380 = 1 0.8190 = 2 0.4095 = 0.8190 [customers/time unit] . 2

where we as a control have that all three values are equal. The sojourn (waiting + service) time in each node becomes by Littles law: W1 = L1 = 2 [time units], 1006 L2 = = 2.1587 [time units], 569 0.8190 L3 338 = = 0.7253 [time units]. 569 0.8190

W2 =

W3 =

6 The total circulation time becomes: R = 2 + 2.1587 + 0.7253 = 4.8840 [time units]. As a control we have: R= Question 6: We use the arrival theorem which says that the fth customer observes the system as if he does not belong to the system himself. So the fth customer will see the mean values calculated in question 4 for a system with 4 customers.
Updated: 2004-05-04

4 = 4.8840 [time units]. 0.8190

1 Technical University of Denmark COM Center, Networks Teletrac Engineering & Network Planning Course 34 340

Exercise 10.4

(exam 1990)

LOSS SYSTEMS WITH MUTUAL OVERFLOW We consider a loss system with 2 servers. Call attempts arrive according to a Poisson process with intensity 20 calls per hour. The holding times are exponentially distributed with mean value 180 seconds. In the following it will be sucient to give numerical answers.

1. Find the oered trac. 2. Calculate by using the recursion formula for Erlangs B-formula the congestion of the system (time congestion equals call congestion and trac congestion). (Show the individual steps of the recursion).

The above system is in the following called a subsystem. We now consider a system made up of two subsystems of the above type (total arrival rate equals 40 calls per hour, in total 4 fully accessible servers).

3. Construct the one-dimensional state transition diagram for the total system, and calculate under the assumption of statistical equilibrium the state probabilities p(i) (i = 0, 1, 2, 3, 4).

We now keep record of the system a call (a server) belongs. A call attempts rst looks for an idle server in its own subsystem. If both servers in this system are busy, it looks for an idle server in the other subsystem. If both servers also are busy in this system the call is blocked. The state of the system is denoted with (i, j) 0 i, j 2 ,

where i, respectively j, denotes the number of busy servers in subsystem 1, respectively 2.

4. Construct the two-dimensional state transition diagram for this system, using the following states.

2
     

02

12

22

 

 

 

01

11

21

 

 

 

00

10

20

5. Calculate the state probabilities of the two-dimensional state transition diagram by exploiting the symmetry and using the aggregated state probabilities calculated in Question 3. (All states in Question 5 with a certain number of busy servers are in Question 3 aggregated into a single state).

1 Technical University of Denmark COM Center, Networks Teletrac Engineering & Network Planning Course 34 340

Solution to exercise 10.4 Question 1: The oered trac A is equal to the average number of call attempts per mean holding time: A= , 1 } 20 [hour/hour] ,

A = {20

A = 1 erlang . There is one call attempt per mean holding time. Question 2: We calculate Erlangs Bformula for n = 2 circuits and A = 1 erlang. We use the recursion formula in Chap. 7 (7.29): En (A) = where: E0 (A) = 1 . For A = 1 we get: E1 (1) = 11 1 = , 1+11 2
1 2

A En1 (A) , n + A En1 (A)

1 1 2 E2 (1) = 2+1

1 = 0.2 . 5

We may control this by using Erlangs Bformula directly: E2 (A) =


A2 2

1+A+ 1 5

A2 2

E2 (A) =

q.e.d.

2 Question 3: We consider a full accessible loss system with n = 4 channels and A = 2 erlang. If we choose 1 = 3 minutes as time unit, we get = 2 and the following state transition diagram:
     j j j j     

If we put the relative value of state zero equal to one, we nd the relative probabilities: qr (0) = 1 , qr (1) = 2 , qr (2) = 2 , qr (3) = qr (4) = 4 , 3 2 , 3

Sum = 7 . As the total sum must add to one, we get the following absolute probabilities: 3 p(0) = , 21 p(1) = p(2) = p(3) = p(4) = 6 , 21 6 , 21 4 , 21

2 . 21 This may also be obtained directly from the truncated Poisson distribution (A = 2 erlang): Ai p(i) = 4 i! , A ! =0 i = 0, 1, , 4 .

3 Question 4: It is understood that a call keeps the allocated circuit in the other subsystem, even though a circuit becomes idle in its own system. 2 2

 i  

 j i  

 j  

02

12

22

1 1 1

2 2 1

2
  i  

  j i  

  j  

01

11

21

1 1 1

2 2 1

1
  i 

  j i 

  j 

00

10

20

The departure (death) intensities are obvious. The total arrival intensity is in every state equal to 2 (in state (2, 2) all call attempts are blocked, and therefore a call attempt does not result in a state transition, and the arrow/intensity is not shown on the gure). If it is possible, the total arrival intensity is divided into 1 in each of the two directions. If there only is one direction, this will get the total intensity (rate) 2. Extra: It is easy to generalise the model. For example, we can introduce restrictions so that a call from one group (subsystem) only is allowed to occupy a circuit in the other group if both circuits of this are idle. The intensities (2,1) (2,2) and (1,2) (2,2) then becomes equal to one in stead of two. Question 5: Notice, that the state transition diagram is not reversible. (This is e.g. done by looking at the ow clockwise and the ow anti-clockwise in 4 neighbouring states). As we both in Question 3 and Question 4 have an oered trac 2 erlangs to 4 circuits with full accessibility, then it is in both cases the same system we consider. Therefore, we have from Question 3: p(0) = 3 = p(0, 0) , 21

4 p(1) = p(2) = p(3) = p(4) = 6 = p(0, 1) + p(1, 0) , 21 6 = p(0, 2) + p(1, 1) + p(2, 0) , 21 4 = p(2, 1) + p(1, 2) , 21

2 = p(2, 2) . 21 Furthermore, because of symmetry we must have: p(0, 1) = p(1, 0) = p(2, 1) = p(1, 2) = p(0, 2) = p(2, 0) . The only unknown are thus states, where in total two channels are busy: p(0, 2) + p(1, 1) + p(0, 2) = 2 p(0, 2) + p(1, 1) = By looking at the node balance equation for state (0, 2) we nd: 4 p(0, 2) = p(0, 1) + p(1, 2) = = = In summary, we thus have: p(0, 2) = p(0, 1) = p(0, 0) = 5 , 84 12 , 84 p(1, 2) = p(1, 1) = 8 , 84 14 , 84 p(2, 2) = p(2, 1) = 8 , 84 8 , 84 p(0, 2) = p(2, 0) = p(1, 1) = 7 1 = . 42 6 5 , 84 5 , 21 6 . 21 3 , 21 2 , 21

12 12 5 , p(1, 0) = , p(2, 0) = . 84 84 84 We notice that the total sum of probabilities is one. Another control also shows that all node balance equations are fullled.
Updated: 2006-05-08

1 Technical University of Denmark COM Center, Networks Teletrac Engineering & Network Planning Course 34 340

Exercise 14.2

(exam 1988)

QUEUEING NETWORK We consider the following open queueing network: Node 1 M/M/1 a
T E  b  E

Node 2 M/M/1
 b (1 p) E 

1
'

2
c

b p

The queueing network consists of two nodes, both of type M/M/1. Customers arrive from outside according to a Poisson process with intensity a = 0.36 calls per time unit, and the service times are exponentially distributed with mean value 1 = 2 time units, respectively 1 1 = 1 time unit. The probability p that a customer after being served in node two returns 2 to node one is p = 0.1, and b is the total arrival intensity to each of the two nodes. 1. Find under the assumption of statistical equilibrium (using ow-balance equations) the oered trac to each of the two nodes. 2. Which interval is p restricted to, if the system is in statistical equilibrium? In the following we assume that the oered trac to node one, respectively node two, is 0.8 erlang, respectively 0.4 erlang. 3. Assume statistical equilibrium and write down the state probabilities p(i, j) which denotes the probability that there are i customers at node one and j customers at node two. (Exploit the product form solution p(i, j) = p(i) p(j)). 4. We now close the queueing network (e.g. by letting p = 1 , = 0), and we consider the case S = 2 customers. Calculate the utilization of the two nodes (carried trac) by considering it as a central server queueing system (use convolution algorithm).

2 5. A xed number of customers S = 2 corresponds to that we are on the diagonal i + j = 2 in question 3. Show by normalisation of the state probabilities that under the condition that we are on the diagonal, then we nd the same utilisation as in question 4. 6. We consider again the same open queueing network as in questions 13. Construct the two-dimensional state transition diagram of the queueing network, where we have the following state transitions: (a) customer arrives from outside, (b) customer changes from node 1 to node 2, (c) customer changes from node 2 to node 1, (d) customer departs from the system. Is the state transition diagram reversible?
Updated: 2002-04-29

1 Technical University of Denmark COM Center, Networks Teletrac Engineering & Network Planning Course 34 340

Solution to Exercise 14.2 Question 1:

(exam 1988)

If we consider the total queueing network, then ow in must equal ow out for the total system. Therefore, we have (cf. the gure): a = b (1 p) , b = a . 1p 0.36 , 1 0.1

b =

b = 0.40 = 1 = 2 , where 1 and 2 are the total arrival rates to node 1, respectively node 2. The oered trac then becomes: A1 = b = 0.4 2 , 1

A = 0.8 erlang . A2 = b = 0.4 1 , 2

A2 = 0.4 erlang . The same ow balance equation is obtained by considering node one (ow in = ow out): a + p b = b .

Question 2: Because node one always carries twice as much trac as node two, node one will be the bottleneck of the system. For single-server queueing systems we have the following condition for statistical equilibrium: A1 < 1 .

2 The total arrival rate is equal to the departure rate obtained above, and the mean service time in node one is 1/1. Thus we get: a 1 < 1, 1 p 1 0.36 2 < 1, 1p p < 0.28 . Question 3: The system fulls the conditions for product form (Jacksons theorem), and for M/M/1 queueing systems in both nodes we get: p(i, j) = p(i) p(j) , j p(i, j) = (1 A1 ) Ai (1 A2 ) A2 , 1 p(i, j) = 0.2 0.8i 0.6 0.4j ,

p(i, j) = 0.12 0.8i 0.4j , Question 4:

i, j 0 .

If we consider node one as the centralserver, then we have the following relative loads: x1 = 1 , 1 , 2 and thus by the convolution algorithm (Sec. 14.4) (or example 14.4.2): x2 =

x 0 1 2

Node 1 Node 2 1 1 1 1
1 2 1 4

Node 1*2 1
3 2 7 4

The last term is obtained from the product sum: p12 (2) = 1 1 1 7 +1 +11= . 4 2 4

3 The three terms correspond to 0, 1 and 2 customers in node one (respectively 2, 1 and zero customers in node two). The utilization of the two nodes thus becomes: A1 = A1 = A2 = A2 = 1 1 +11 2 , 7/4 6 . 7 1 A1 , 2 3 . 7

Question 5: In Question 3 we found: p(2, 0) = (1 A1 ) A2 (1 A2 ) = 1 48 = 0.0768 , 625 24 = 0.0384 , 625

p(1, 1) = (1 A1 ) A1 (1 A2 ) A2 = p(0, 2) = (1 A1 ) (1 A2 ) A2 = 2

12 = 0.0192 . 625

These are the only three states (upon the diagonal) with exactly two customers in the system. By normalising the state probabilities we get: p(2, 0) = p(1, 1) = p(0, 2) = and thus: 4 , 7 2 , 7 1 , 7 6 , 7 3 . 7

A1 = 1 p(0, 2) = A2 = 1 p(2, 0) =

4 Question 6:

 c

    d d d s d d s d d s d d s d d d p2 p2 p2 d d d d d d 1 dd 1 dd 1 dd  d d c  d d c  d d c  d d d c d d a E a d a E E 22 02 12 32     d d d s d d s d d s d d s d d d p2 p2 p2 dd dd dd 1 1 1 dd dd dd  d d c  d d c  d d c  d d d c d d a E a d a E E 21 01 11 31     d d d s d d s d d s d d s d d d p2 p2 p2 dd dd dd 1 1 1 dd dd dd  d d c  d d c  d d c  d d d c a d a d a d E 10 E 20 E 30 00    

03

a E 13

c  w

a E 23

c  w

a E 33

c  w

where = (1 p) 2 . From the structure of the diagram, it is obvious that the process in not reversible.
Updated: 2006-05-10

1 Technical University of Denmark COM Center, Networks Teletrac Engineering & Network Planning Course 34 340

Exercise 13.22

(Exam 2004)

Airport priority queueing system Let us consider an airport where the server is a single runway. The trac during one morning rush hour consists of arriving aircrafts and departing aircrafts both using the runway as follows: a. Landing: 10 aircrafts arrive per hour. The mean service time is m1,1 = 2 [minutes] and second moment of the service time is m2,1 = 6 [minutes2 ]. b. Starting: 20 aircrafts departs per hour. The mean service time is m1,2 = 1.5 [minutes] and second moment of the service time is m2,2 = 3 [minutes2 ]. 1. Find the oered trac for each type, and the total oered trac. 2. Find the total arrival rate and the mean service time for all aircrafts and use this to control the total oered trac. 3. Find the mean waiting time for an arbitrary aircraft when there is no priority. 4. Find the mean waiting time for both types when landing aircraft have non-preemptive priority over starting aircrafts. Show that the conservation law is fullled for this system.

1 Technical University of Denmark COM Center, Networks Teletrac Engineering & Network Planning Course 34 340

Solution to Exercise 13.22

(exam 2004)

Priority queueing systems are dealt with in Sec. 13.4. Question 1: We have to choose a common time unit and in the following we choose minutes. Landing: The arrival rate is 1 = 1/6 aircrafts per minute, and the mean service time is s1 = 2 minutes. So the oered trac becomes: A1 = 1 s1 = 1 [erlang] 3

Starting: The arrival rate is 2 = 1/3 aircrafts per minute, and the mean service time is s2 = 1.5 minutes. So the oered trac becomes: A2 = 2 s2 = The total oered trac becomes: A = A1 + A2 = 5 6 [erlang] 1 [erlang] 2

We notice that the total oered trac is less than one erlang, so the system is in statistical equilibrium. Question 2: The total arrival rate is obtained by adding the two arrival rates (13.13): = 1 + 2 = = 1 1 + 6 3 1 2

The total mean service time is obtained by weighting the mean values with the relative

2 number of calls of each type (13.14). Considering one minute we nd: s = = s = 1 2 s1 + s2 2 1 2 + 1.5 3 3 5 3 [minutes]

From the total process we nd the total oered trac: A = s= A = 1 5 2 3 q.e.d

5 [erlang] 6

Question 3: To nd the mean waiting time for all customers in a single server system we use PollaczekKhintchines formula (13.4) or (13.2). We rst have to nd the second moment m2 or the form-factor of the total trac process. The second moment is obtained by weighting as for the mean value (13.15) (random variables in parallel). We nd: m2 = = 1 2 m2,1 + m2,2 2 1 6+ 3 3 3

m2 = 4 minutes2 The parameter V becomes: V = m2 2


1 2

= V

= 1

3 We could of course also obtain V by summation over all trac classes (13.18):
2

=
i=1

Vi 1/6 1/3 6+ 3 2 2

= 1 Pollaczek-Khintchines formula (13.4) yields: W = V 1 = 1A 1


5 6

W = 6 [minutes] This is the mean waiting time for all aircraft. If we only consider delayed aircrafts, then the mean waiting time is: W w= = 7.2 A We get of course the same if we use (13.2). The form-factor becomes: = m2 4 36 = = s2 (5/3)2 25

Question 4: We now consider a non-preemptive queueing system where landing aircrafts have priority over starting aircrafts. The formul for this are given in Sec. 13.4.3. For highest priority class the mean waiting time becomes (13.26): W1 = = V 1 A1 1 1 3 2
1 3

W1 = For lowest priority we nd (13.29): W2 =

[minutes]

W1 V = (1 A1 )(1 (A1 + A2 )) 1 (A1 + A2 )

W2 = 9 [minutes]

4 The conservation law (13.23) for this system relates the mean waiting time W without priority to the mean waiting times W1 and W2 with non-preemptive priority discipline: A W = A1 W1 + A2 W2 5 1 3 1 6 = + 9 6 3 2 2 5 = Thus the conservation law is fullled
Updated: 2005-01-04

1 9 + 2 2

q.e.d.

Mini-tasks 2005
1. Engsets model with non-integral number of servers and sources. 2. Loss systems with preemptive priority 3. Overow moments of a multi-service link (Schehrer 2005) 4. Estimating blocking probabilities of multi-ber WDM optical network (paper January 2004) 5. Blocking probabilities in WCDMA system for multi-rate trac (Chapter 10) 6. Bandwidth reservation in the generalised model of the limited-availability group (Glabowski) 7. Service bundling 8. Labourdettes algorithm for link with multi-rate trac (Chapter 10) 9. Iversen & Stepanov algorithm for link with multi-rate trac (Chapter 10) 10. Multi-rate trac with bursty arrivals (recent paper Stochastic Knapsack ....) 11. Calculation of derivatives of blocking probabilities in multi-service systems (Stepanov & Iversen) 12. Reduced load approximations for end-to-end blocking (application: hierarchical multirate loss networks) 13. Multi-Service allocation og multiaccesss wireless systems (UMTS) (Furuskar & Zander 2005) 14. Systems with several trac streams and individual time-out constant (Chapter 12) 15. Waiting time distribution in single-server queueing system with constant service time and nite buer (Chapter 13) 16. Approximate Generalised Processor Sharing (GPS) model (Chapter 14) 17. Lundkvists general theory of trac (from 1956) 18. Call blocking probabilities in a TDCDMA system (December 2004) 19. Mean Value Algorithm for nodes with more servers (Queueing network algorithm, Chapter 14) 20. An analytical model for the performance of Wimax networks 21. Some application oriented mini-tasks

1 Technical University of Denmark COM Center, Networks Teletrac Engineering & Network Planning Course 34 340

Exercise 7.7: ALOHAMODELS

(Exam 1993)

We consider a system M/G/, where messages arrive according to a Poisson process with intensity (rate) , and the holding time distribution is given by F (t) with mean value s = 1. There is an innite number of servers, and the state probabilities of the system depend only upon the holding time distribution via the mean value (insensitivity).

1. Find the oered trac and the state probabilities under the assumption of statistical equilibrium.

A message is only served correctly, if it is alone in the system during the whole holding time. Thus the system must be empty at the arrival time, and no new calls may arrive during the holding time. In the following two questions we assume that the holding time is constant with mean value s = 1. 2. Find the probability that a message is transmitted correctly. 3. Find the trac which is carried correctly, and show that it has a maximum equal to 1 = 0.1839 for = 0.5. 2e Assume in the remaining part of the exercise that the holding time is exponentially distributed with mean value s = 1. 4. Find the probability that a message is transmitted correctly. 5. Show that the total number of correctly transmitted messages per time unit has a maximum (0.2059) for = ( 5 1)/2 = 0.6180. 6. Find the distribution function and the mean value of the holding time of the messages which are transmitted correctly. Then nd the carried trac of messages which are transmitted correctly,

1 Technical University of Denmark COM Center, Networks Teletrac Engineering & Network Planning Course 34 340

Exercise 7.8:

(exam 1994)

BATCH ARRIVALS TO A LOSS SYSTEM We consider a pure loss system with 4 channels. Events occur in the arrival process according to a Poisson process with intensitety = 1 [events/time unit]. Every event corresponds to two call attempts, which are served independently. If there only is one channel idle at the arrival instant (time), then a random one of the two call attempts is rejected. The service time is exponentially distributed with mean value 1 = 1 [time unit].

1. Find the oered trac (measured in channels). 2. Set up the state transition diagram of the system, when we dene state x as the total number of busy channel (x = 0, 1, 2, 3, 4). 3. Show that the state probabilities under the assumption of statistical equilibrium are given by:

12 , 49

12 , 49

12 , 49

8 , 49

5 49

and nd the trac congestion (the proportion of the oered trac which is blocked). 4. Find the time congestion and the call congestion of the system.

1 Technical University of Denmark COM Center, Networks Teletrac Engineering & Network Planning Course 34 340

Exercise 8.10

(exam 2001)

LOSS SYSTEM WITH STATEDEPENDENT ARRIVAL INTENSITY We consider a loss system with n = 2 channels. The state of the system i is dened as the number of busy channels. Customers arrive according to a statedependent Poisson process with intensity 3i (i) = [customers per time unit] , 0 i 3 . 4i For all other states (i) = 0. We choose = 1 customer per time unit, and the service time is exponentially distributed with intensity = 1 customer per time unit. 1. Construct the state transition diagram of the system. 2. Find the state probabilities of the system under the assumption of statistical equilibrium, and give the time congestion E. 3. Find the state probabilities (i) as they are observed by an arbitrary arriving customer, and nd the call congestion B. 4. Find the oered trac, which is dened as the trac carried in a system without blocking, and nd the trac congestion C. 5. Assume that both channels are busy. What is the probability that the next event is a call attempt (which of course will be blocked)? Find the distribution of the number of calls, which are blocked during a busy period. 6. Give the state probabilities as they are seen by a customer, which just has departed from the system. We include customers, which are blocked.

1 Technical University of Denmark COM Center, Networks Teletrac Engineering & Network Planning Course 34 340

Exercise 12.17:

(exam 1995)

QUEUEING SYSTEM WITH LIMITED QUEUE We consider a classical queueing system M/M/2, 6 having 2 servers and 4 queueing positions, so that there at most may be 6 customers in the system. Customers arrive according to a Poisson process with intensity = 1 customer per time unit, and the holding times are exponentially distributed with mean value 1 = 1 time unit. 1. Find the oered trac. 2. Construct a state transition diagram for the system, where we dene state x as the total number of customers in the system (either being served or waiting in queue) (x = 0, 1, . . . , 6). Find the probability that a customer is (a) served immediately, (b) served after delay, or (c) blocked.

We now split the system into two identical subsystems, so that each subsystem has one server and two queueing positions. At the time of arrival a customer chooses one of the two systems with a probability, which is proportional to the number of idle positions (server + queueing positions). The state of the system is dened as (i, j), 0 i, j 3, where i denotes the number of customers in subsystem one and j the number of customers in subsystem two. Subsystem one is thus chosen with probability p(1) = 3i , 6 (i + j)

whereas subsystem two is chosen with probability p(2) = 3j . 6 (i + j)

When a customer has chosen a subsystem, he stays in this subsystem. 3. Construct the two-dimensional state transition diagram of the system. 4. Show that the state transition diagram is reversible. 5. Find the state probabilities. 6. Find the probability that a customer is blocked, and explain why this is greater than the probability obtained in question 2.

1 Technical University of Denmark COM Center, Networks Teletrac Engineering & Network Planning Course 34 340

Exercise 13.7

(Exam 1984)

M/H2 /1 QUEUEING SYSTEM WITH PROCESSOR SHARING Jobs arrive to a computer system according to a Poisson process with intensity . The service time is distributed as a hyper-exponential distribution with two phases denoted by a, respectively b: F (t) = p (1 ea t ) + (1 p) (1 eb t ) .
E

p
E

a
E

1p = q

1. Find the oered trac A. In the following we assume that A < 1. The computer system operates as a single-server system with processor sharing queueing discipline, i.e. if there are x jobs in the system, then a job in phase a is served with rate a /x and a job in phase b is served with rate b /x. The state of the system is dened as (i, j), where i is the number of jobs in phase a, and j is the number of jobs in phase b. The state transition diagram becomes twodimensional with the structure shown in the following gure. 2. Find the missing intensities in connection with the states: (1,1), (1,2), (2,2) and (2,1). 3. Show by considering the above-mentioned four states that the state transition diagram is reversible. An M/M/1 queueing system with the oered trac A (A < 1) has the equilibrium state probabilities: p(i) = p(0) Ai , i = 0, 1, 2, . . .

2 4. Show by expressing the state probabilities by state p(0, 0) that the above M/H2 /1 processor sharing system has the same state probabilities as M/M/1 when we let:
i

p(i) =
x=0

p(x, i x) , i = 0, 1, 2, . . . ,

and we only consider i = 1 and 2.

 c p

 c E

03 '  1 a T 4 q b

13 '  T

 c p

 c E

 c E

02 '  1 a T 3 q b

12 '  T

22 '  T

 c p

 c E

 c E

 c E

01 '  1 a T 2 q b

11 '  T
1 2

21 '  T q
1 3

31 '  T q
1 4

 c p

 c p E

 c p E

 c E

00 ' 

10 ' 

20 ' 

30 ' 

1 Technical University of Denmark COM Center, Networks Teletrac Engineering & Network Planning Course 34 340

Exercise 13.4

(exam 1979)

M/E2 /1 QUEUEING SYSTEM We consider the queueing system M/E2 /1, where calls arrive with intensitety , and the service time is Erlang2 distributed with intensitety 2 in each of the two phases. 1. Find the oered trac. 2. Construct a state transition diagram for the system, where the states both denote number of customers in the system and the phase of the server. 3. Find the mean waiting time for an arbitrary call and the mean waiting time of a call, which is delayed. 4. Find the mean value of the time interval which passes from a random point of time until a customer with very high priority would be served when the queue discipline is nonpreemptive. 5. First part of this question is excluded spring 2004. Find the generating function (or the distribution function) and the mean value and variance of the number of customers which arrives during a service time. 6. Let all customers have the above service time distribution and choose the mean service time as time unit. We consider a queueing system with two classes and an oered trac equal to 0.4 erlang for each class. One of the classes has higher priority. Find the mean waiting time for customers in each of the two classes, when the queueing discipline is (a) nonpreemptive, (b) preemptive resume.

1 Danmarks Tekniske Universitet COM Center Data & Teletrakteori kursus 34 340

Exercise 13.21

(exam 2003)

Priority queueing system We consider a single-server queueing system M/G/1, where the customers arrive according to Poisson processes. There are three types of customers: Type 1: Arrival rate 1 = 0.1 calls per time unit. Service time: constant with mean value 1 time unit. Type 2: Arrival rate 2 = 0.04 calls per time unit. Service time: exponentially distributed with mean value 10 time unit. Type 3: Arrival rate 3 = 0.01 calls per time unit. Service time: at distribution with mean value 40 time units and form factor = 10.

1. Find the oered trac for each type, and the total oered trac. 2. Find the mean waiting time for all calls for each type when only this type is oered to the system.

In the following all three types are oered to the system. 3. Find the mean waiting time for a random customer when there is no priority. Type 1 and type 2 are given preemptive resume priority over type 3, so that the rst two types dont notice type 3. Type 1 has non-preemptive priority over class two. 4. Find the mean waiting time for type 1 and 2. 5. Find the mean waiting time for type 3 (type 1 and type 2 may be merged into one type as regards type 3).

1 Technical University of Denmark COM Center, Networks Teletrac Engineering & Network Planning Course 34 340

Exercise 13.22

(Exam 2004)

Airport priority queueing system Let us consider an airport where the server is a single runway. The trac during one morning rush hour consists of arriving aircrafts and departing aircrafts both using the runway as follows: a. Landing: 10 aircrafts arrive per hour. The mean service time is m1,1 = 2 [minutes] and second moment of the service time is m2,1 = 6 [minutes2 ]. b. Starting: 20 aircrafts departs per hour. The mean service time is m1,2 = 1.5 [minutes] and second moment of the service time is m2,2 = 3 [minutes2 ]. 1. Find the oered trac for each type, and the total oered trac. 2. Find the total arrival rate and the mean service time for all aircrafts and use this to control the total oered trac. 3. Find the mean waiting time for an arbitrary aircraft when there is no priority. 4. Find the mean waiting time for both types when landing aircraft have non-preemptive priority over starting aircrafts. Show that the conservation law is fullled for this system.

1 Technical University of Denmark COM Center, Networks Teletrac Engineering & Network Planning Course 34 340

Exercise 9.14

(Exam 2004)

Loss system and overow trac

We consider a full accessible loss system with n = 3 channels. The system is oered Pascal trac, and in state i the arrival rate is (S +i). The number of sources is S = 4. The arrival rate of an idle source is = 1/3. The mean holding time is chosen as time unit (1 = 1). All time intervals are exponentially distributed. The state of the system is dened as the number of busy channels. (Note: An Erlang-B table covering n =1, 10 (step = 1) and A = 0, 10 (step = 0.25) was attached). 1. Show that the oered trac is A = 2 [erlang] and that the peakedness is 1.5 . 2. Construct the state transition diagram and nd under the assumption of statistical equilibrium the state probabilities p(i), i = 0, 1, . . . , 3. 3. Find the time congestion E, the call congestion B, and the trac congestion C. 4. Calculate the trac congestion C by using Fredericks-Haywards method. 5. Calculate the trac congestion C using Sanders method. Assume that the above trac is overow trac from an equivalent system with 4 channels which are oered 5 erlang. 6. Find the trac congestion C using Wilkinson-Bretschneiders ERT-method.

1 Technical University of Denmark COM Center, Networks Teletrac Engineering & Network Planning Course 34 340

Exercise 9.8

(exam 1985)

OVERFLOW SYSTEM MODELLED BY AN INTERRUPTED POISSON PROCESS We consider an overow system, consisting of a primary group with one channel, and an overow group also with one channel (a Brockmeyer-system). The system is oered random trac type 1 (PCTI). The arrival process is a Poisson process with intensity a = 1 calls per time unit, and the service time is exponentially distributed with intensity = 1 calls per time unit. The arrival process for calls to the overow group may be described by an interrupted Poisson process. 1. Find the parameters for this overow process: (a) call intensity () (b) intensity for ON periods (), (c) intensity for OFF periods (). 2. Construct a state transition diagram for the overow group. The state of the overow system is described by (i, j), where i is the number of busy channels (i = 0, 1), and j is the state of the interrupted Poisson process (j =ON, OFF). 3. Show that the state probabilities are given by: p(0, ON) = p(1, ON) = 3 10 2 10 , , p(0, OFF) = p(1, OFF) = 4 , 10 1 . 10

4. Calculate the time congestion and the call congestion for: (a) the overow system only, (b) the whole system. Compare the last result with the congestion obtained by Erlangs B-formula. 5. Find the trac congestion for the overow group alone (not included at exam).

1 Technical University of Denmark COM Center, Networks Teletrac Engineering & Network Planning Course 34 340

Exercise 10.7

(exam 1994)

MULTIDIMENSIONAL LOSS SYSTEM We consider a fully accessible loss system with n = 5 channels, which are oered two trac streams. All arrival processes and service times are of type M (Poisson exponential), and all parameters are given in the same time unit. Trac stream 1: Arrival intensity 1 = 2, Mean service time 1 = 1, 1 Every call requires d1 = 1 server. Trac stream 2: Arrival intensity 2 = 1, Mean service time 1 = 1 , 2 2 Every call requires d2 = 2 servers. If a call attempts is not allocated the required capacity at the time of arrival, then it is lost (Lost Calls Cleared). 1. Find the oered trac for both trac streams measured in number of channnels. 2. Construct a twodimensional state transition diagram of the system, where state (i, j) denotes that i channels are occupied by type 1 calls (i = 0, 1, 2, 3, 4, 5), and j channels are occupied by type 2 calls (j = 0, 2, 4), where we have i+j 5. 3. Show that the state transition diagram is reversible, and nd all state probabilities p(i, j) expressed by p(0, 0). Find time congestion, call congestion and trac congestion for both trac streams. Let p(j) denote the summation of alle states, where the total number of busy servers is j, i.e.
j

p(j) =
x=0

p(x, j x) .

4. Show that the following recursion formula is valid for the system considered:
2

j p(j) =
i=1

i di p(j di ) . i

1 Danmarks Tekniske Universitet COM Center Data & Teletrakteori kursus 34 340

Exercise 12.16

(exam 1993)

MACHINE REPAIR MODEL We consider a machine/repair model with 4 terminals having exponentially distributed thinking times with intensity = 3 events per time unit. The terminals are served by two dierent singleservers. With probability 1/3 a terminal needs service from server 1 and with probability 2/3 a terminal needs service from server 2. Server 1 has exponentially distributed service time with mean value 1 = 1 time units, and server 2 has exponentially distributed 1 service time with mean value 1 = 1/2 time units. After being served at a single server the 2 terminal starts a new thinking time. The above is a machine/repair model, where there are two types of errors which each need a specialised server.

1. Find for one thinking terminal the arrival rate for jobs to server 1 and server 2, respectively. 2. Construct the state transition diagram of the system, when we dene the state of the systems (i, j), 0 i, j, i + j 4, where i, respectively j, is the number of terminals at server 1, respectively server 2. 3. Show that the state transition diagram is reversible. 4. Find the state probabilites of the system when the system is in statistical equilibrium 5. Find the utilisation (carried trac) of the two single servers. 6. Find the mean waiting time for a terminal, which wants to be served at server 1.

1 Technical University of Denmark COM Center, Networks Teletrac Engineering & Network Planning Course 34 340

Exercise 14.7

(Exam 2002)

ENGSETs MODEL AS A QUEUEING NETWORK We consider Engsets loss system with S = 6 sources and n = 3 channels. The arrival rate of an idle source is = 2 calls per time unit, and the mean service time is 1 = 1 (chosen as time unit). 1. Find the oered trac. 2. Calculate the time congestion E using a formula recursive in n. Each step in the recursion should be visible. We consider a closed queueing network with K = 2 nodes and S = 6 customers. Node one is an innite server (IS) and the service rate is 1 per server. Node two is an M/M/3loss system with service rate 2 per server, which corresponds to an innite server (IS) truncated at state 3. The routing is cyclic so that a customer served in node one goes to node two, and a customer served in node two goes to node one. A customer blocked in node two returns to (i.e. stays in) node one. Assume the circulation rate is c , and let 1 = c /1 and 2 = c /2 . This is a queueing network with blocking, and it has product-form. 3. Find the relative state probabilities of each node as independent systems. 4. Convolve the two nodes into one under the assumption that the total number of customers is 6, and show that the state probabilities p(i), (i = 0, 1, 2, 3) of node two correspond to an Engset loss system with S = 6 sources, n = 3 channels, and = 1 /2 .

1 Technical University of Denmark COM Center, Networks Teletrac Engineering & Network Planning Course 34 340

Exercise 14.5

(exam 1999)

CLOSED QUEUEING NETWORK We Consider a closed queueing network with 2 single-server nodes of type M/M/1. After nishing service in a node the customers passes on to the other node. There is a total of S customers in the queueing network. The service rates (intensities) in node 1 and 2 are 1 and 2 , respectively. We assume the system is in statistical equilibrium. The state of the system i is dened af the number of customers in node 1 (then there will be S i customers in the second node). 1. Construct the one-dimensional state transition diagram of the system. 2. Find the state probabilities of the system and the average number of customers in the two nodes. 3. Find from the state probabilities the intensity with which the customers circulate in the system, and exploit this to nd the cycle time of a customer (assume FCFS-queueing discipline). 4. What is the relationship between mean sojourn times (sojourn time = waiting time + service time) in the two nodes? (apply Littles law). Assume in the following that S = 3 customers, 1 = 1 time units, 1 = 2 time units. 1 2 5. Apply the convolution algorithm to nd the state probabilities of the system, and calculate the average number of customers in the two nodes. 6. Apply the MVAalgorithm to nd the average number of customers in the two nodes.

1 Technical University of Denmark COM Center, Networks Teletrac Engineering & Network Planning Course 34 340

Exercise 8.3

(exam 1980)

M/E2 /2 - LOSS SYSTEM We consider a loss system with two channels (servers). Call attempts arrive according to a Poisson process with intensity calls per time unit. The service time is Erlang-2 distributed with intensity 2 in each of the two phases.

1. Find the oered trac. 2. Construct a state transition diagram of the system, where a state both denotes the number of calls in the system and the phase of the calls. Apply the following states, where a and b denotes the two phases.
           

aa

ab

bb

3. Find under the assumption of statistical equilibrium the state probabilities of the system by exploiting the fact that the truncated Poisson distribution for a given mean holding time is valid for any service time distribution (insensitivity property). 4. The blocking state both channels occupied is initiated from either state a or b. Denote for both of these cases the (Cox)distribution of the duration of the blocking state by applying a graphical (phase diagram) representation of the Coxdistribution. 5. (Advanced, excluded) Write down the Laplace transform of the distribution of the duration of the periods when both channels are busy. 6. (Advanced) Find the mean value and variance of the number of call attempts which are blocked during a period when both channels are busy.

1 Technical University of Denmark COM Center, Networks Teletrac Engineering & Network Planning Course 34 340

Exercise 10.1

(exam 1983)

LOSS SYSTEM WITH MULTIPLE ACCESSIBILITY We consider a loss system with 3 identical servers, serving two dierent types of customers, which arrive according to Poisson processes with intensities: type 1: 1 [customers/time unit], type 2: 2 [customers/time unit]. Both types of customers have the same exponentially distributed service time distribution with mean value m = 1/ [time units]. Customers of type 1 has full accessibility to all three servers. Customers of type 2 are blocked if more than one server is busy at the arrival time. The state of the system is dened as the total number of customers being served. 1. Construct a one-dimensional state transition diagram for this system. 2. Find under the assumption of statistical equilibrium the state probabilities of the system. 3. Find, expressed by the state probabilities, the call congestion for customers of type 1 and type 2. We now dene the state of the system as (i, j), where i is the number of type 1 customers being served, and j is the number of type 2 customers being served. 4. Construct a two-dimensional state transition diagram for this system. 5. Assume, that the state probabilities are known, and nd the trac carried by customers of type 2, when a total of (i + j =) 0, 1, 2, or 3 customers are being served.

1 Technical University of Denmark COM Center, Networks Teletrac Engineering & Network Planning Course 34 340

Exercise 12.23

(Exam 2002)

M/M/1 QUEUEING SYSTEM WITH RESTRICTED ACCESS We consider an M/M/1 queueing system with one server and innite queue. Customers arrive according to a Poisson process with rate customers per time unit, and the service time is exponentially distributed with mean value 1 . The server breaks down with rate (also when it is idle), and is repaired with intensity . All time intervals are exponentially distributed. When the server is down no customers are served. A customer being served when the server breaks down, resumes service when the server has been repaired. Customers arriving during a repair period are lost. The state of the system is dened as (i, j), where i (i = 0, 1, 2, ) is the total number of customers in the system and j (j = 0, 1) is the state of the server (0 = in order, 1 = under repair). 1. Construct the state transition diagram of the system. 2. Find the proportion of time the server is broken down, and working. 3. Find the state probabilities of the system under the assumption of statistical equilibrium. 4. What is the probability (expresses by state probabilities) that a call is served immediately without any delay (due to queue or repair)? is delayed before service? is lost? 5. Which restrictions should be fullled between , , and when the system is able to attain statistical equilibrium? 6. Explain the following Coxdistribution for the time interval from start to termination of service of a customer. ... ..... ... ... .... .. + ..............................................+....................................... ... ..... ... ... .... .. + ..............................................+........................................

.. .............. ............. .. .. .

.. .... .. .. .............. ............. .. .. .............. .............. .. . ........................... .......................... ... . . .. ... . . . . . . . . . . . . . . . . . . . . . . . . . . . . . . . . . . . . . . . . . . . . . . . . . . . . . . . . . . . . . . . . . . . . . . . . . . . . . . . . . . . . . . . . . . . . . . . . . . . . . . . . . . . . . . . . . . . . . . . . . . . . . . . . . . . . . . . . . . . . . . . . . . . . . . . . . . . . . . . . . . . . . . . . . . . . . . . . . . . . . . . . . . . . . . . . . . . . . . . . . . . . . . . . . . . . . . . . . . . . . . . .. .. ... ... ... . .. ... ... . . .. .. .. . . . .. .. .. . . . . .. . . . . . ........................................................................................................... ......... . .................................................................................................................................................................................................................................... . . . .......................................................................................................... . ... .. . . ......

1 Technical University of Denmark COM Center, Networks Teletrac Engineering & Network Planning Course 34 340

Exercise 8.12

(exam 2005)

Engset model with inhomogeneous sources We consider a full accessible Engset loss system with n = 3 channels. The system is oered trac from S = 4 sources. The arrival rate of an idle source is 1 = 1/2 call attempts per time unit. The mean holding time is chosen as time unit (1 = 1). All time intervals are 1 exponentially distributed. The state of the system is dened as the number of busy channels and every busy source occupies one channel.

1. Find the oered trac. 2. Find the state probabilities of the system by convolving the state probabilities of the 4 individual sources, truncating at 3 channels, and normalising. 3. Find the time congestion E, the call congestion B, and the trac congestion C.

We now add a source dierent from the above sources having both mean idle time and mean holding time equal to one time unit (2 = 2 = 1). The source occupies one channel when it is busy. 4. Find by convolving this source with the above system the time congestion for both types of calls. 5. Find the call congestion for both types of calls by applying the arrival theorem. 6. Find the trac congestion for both types of sources by looking at the individual terms during the convolution (use eventually a two-dimensional state transition diagram as aid).

1 Technical University of Denmark COM Center, Networks Teletrac Engineering & Network Planning Course 34 340

Exercise 13.23

(exam 2005)

Multi-server priority queueing system We consider an Erlang-C waiting time system with n = 5 servers oered A = 3 erlang (Poisson arrival process and exponential service times). The mean service time is 60 seconds.

1. Find the arrival rate. 2. Find the probability of delay using the recursive formula for calculating Erlang-B and the relationship between Erlang-B and Erlang-C formul. (The details of calculation should be visible from the answer, and the result is E2,5 (3) = 81/343 = 0.2362). 3. Find the mean waiting time for all customers.

We now divide the above oered trac into two priority classes: Class 1: A = 1 erlang for high priority customers, Class 2: A = 2 erlang for low priority customers. Both classes still have the same mean service time 60 seconds. We rst consider the non-preemptive resume priority strategy. 4. Find the mean waiting time for each of the two classes. 5. Show that the conservation law is valid for this system. We then consider the preemptive resume priority strategy. Due to the properties of the exponential distribution the conservation law is also valid for this model. 6. Find the mean waiting time for each of the two classes. (given: E2,5 (1) = 1/261 = 0.0038).

1 Technical University of Denmark COM Center, Networks Teletrac Engineering & Network Planning Course 34 340

Solution to Exercise 7.7 Question 1: This corresponds to the Poisson case in the textbook (Sec. 7.2). The oered trac equals the carried trac as the capacity is innite: A = s = . Under the assumption of statistical equilibrium the state probabilities are given by the Poisson distribution: Ai A p(i) = e , i = 0, 1, 2, . i! Question 2: A message is transmitted correctly, if it is alone in the system. The probability that the system is in state zero at the arrival time (Poisson arrival process PASTA-property) is given above. The probability that next message arrive later than one time unit is given by the tail of the exponential distribution 1 F (t) = exp( 1): pcorrect = p(0) p{no messages arrive within an interval af duration s=1} = eA e 1 , pcorrect = e2A . Question 3: The trac carried correctly becomes: Ycorrect = A e2 A , Ycorrect = e2 . (This is correct because all messages have the same constant holding time. Thus the trac carried is proportional with the number of calls carried). This function has a maximum, which is obtained by letting the derivative with respect to equal to zero: Ycorrect = e2 2 e2 = e2 {1 2 } ,

2 which is zero for = 1/2. It is easy to see that this corresponds to a maximum. The maximum carried trac becomes: Max {Ycorrect } = 1 . 2e

As a message has a constant duration, the maximum of the correctly carried trac corresponds to maximum of throughput for messages transmitted correctly. Question 4: This corresponds to Question 2, but the holding time is now exponentially distributed with density function: 1 fs (t) = et/s . s The time until next message arrives is exponentially distributed with the distribution function: F (t) = 1 et . The probability that a message is transmitted correctly is as above equal to the probability the the system is in state zero and no new messages arrive during the transmission of a message present: pcorrect = p(0) pcorrect = p(0) pcorrect = e
t=0 t=0

{fs (t)dt {1 F (t)}} , 1 t/s e dt e t s

1 . +1

The probability that one exponential distributed stochastic variable is less than another exponentially distributed variable is given in (4.7). Question 5: Per time unit y = pcorrect = e messages are transmitted correctly. 1+ This function has a maximum when the derivative with respect to is zero. y = 1+ e + e +1 (1 + )2

e 2 + 1 . 2 (1 + )

3 The derivative y takes the value zero for 2 + 1 = 0. We thus have: 51 = = 0.6180 , 2 as the other solution is negative (we require that > 0). We nd: y = 0.2059 . I.e., by exponentially distributed message times we transmit more messages correctly than by constant message times. (But, see the following question). Question 6: We know that the minimum of two exponentially distributed stochastic variables is itself an exponentially distributed stochastic variable with an intensity equal to the sum of the two intensities. The holding time (message length) of messages transmitted correctly has the density function: fk (t) = (1 + ) e(1+) t , and the distribution function: Fk (t) = 1 e(1+) t with the mean value: E{T } = The correctly carried trac thus becomes: Ycorrect = which for = 0.6180 becomes: Ycorrect = 0, 1272 . This means that we transmit a larger number of messages, but the amount (volume) of information transmitted correctly becomes smaller, as it is preferably short messages, which are transmitted correctly. See Example 7.2.1, which deals with the simple Aloha protocol corresponding to the rst three questions, and Example 6.2.2, which deals with a slotted Aloha system.
Updated 2005-05-23

1 . 1+

1 1 e , 1+ 1+

1 Technical University of Denmark COM Center, Networks Teletrac Engineering & Network Planning Course 34 340

Solution to Exercise 7.8 Question 1: The trac oered is dened as the number of call attempts per mean service (holding) time. (The denition is independent of whether the arrivval process is a Poisson process or another process). For each event in the arrival process, which has the intensity , we have two call attempts. We thus get: A = (2) s = (2 1) 1 A = 2 erlang [channels] . If there is no blocking, then on the average two channels will be busy. If we dene the oered trac in terms of events, then the oered trac becomes A = 1 erlang [events] so that if there is no blocking, then on the average there will be one event in the system (each occupying two channels). Question 2: The states of the system are given as {0, 1, 2, 3, 4}. By using the standard procedure for constructing state transition diagrams (Sec. 7.4), and for each state consider possible state transitions due to the arrival process and the service process, we nd get the following state transition diagram: 1 0 1 1 2 1 3 4
  s  s  s  1E     

The state transition diagram may also be set up as follows:


   1 E 2 E 4 0   U  o o  T  2  1  3 1       4  G  1 G  E





2 Notice, that for an arrival event in state 3 we only accept a random one of the two call attempts, whereas the other one is lost. In state 4 we block both call attempts. Blocked call attempts have no inuence upon the state of the system (lost calls cleared). Question 3: The rst part of the question may be answered by showing that (1) the given state probabilities full the node equations and (2) that they add to unity. We may of course also nd the state probabilities ourselves. In the actual case, we may also use cut equations. If we let the relative value of the state probability zero be q(0) = 1, then we nd the following relative state probabilities q(i): Node equation for state 0: 1 q(1) = 1 q(0) , q(1) = 1 q(0) . Node equation for state 1: (1 + 1) q(1) = 2 q(2) , q(2) = 1 q(0) . Node equation for state 2: (2 + 1) q(2) = 1 q(0) + 3 q(3) , q(3) = Node equation for state 3: (3 + 1) q(3) = 1 q(1) + 4 q(4) , q(4) = The normalisation restriction yields:
4

2 q(0) . 3

5 q(0) . 12 5 2 + 3 12 49 q(0) , 12

q(i) = 1 = q(0) 1 + 1 + 1 +
i=0

q(0) =

12 . 49 12 , 49 8 , 49 5 , 49 q.e.d.

p(0) = p(1) = p(2) = p(3) = p(4) =

3 From Question 1 we know the oered trac is A = 2 [erlang]. From the state probabilities we nd the following carried trac Y :
4

=
i=0

i p(i) 80 . 49

= The lost trac becomes: 2 Thus the trac congestion C becomes:

80 18 = [erlang] . 49 49

80 2 AY 49 , C= = A 2 C= 9 = 0.1837 . 49

The trac congestion is per denition equal to the proportion of the oered trac which is lost. For Poisson arrival process, the trac, time, and call congestion are equal (PASTA property). In the case considered the arrival process is a (Poisson) process with twin arrivals, i.e. it is not a Poisson process, because we have two events at the same time. The argument that half the oered trac is lost in state 3, and all oered trac is lost in state 4: 1 8 5 9 + = 2 49 49 49 is not correct, because this argument refers to the call congestion (but the result is correct). See the following question. Question 4: Call congestion B: In state 3 we block half of the call attempts, and in state 4 we block all call attempts. The call congestion is dened as the proportion of call attempts, which are blocked. It becomes: 2 p(3) + 2 p(4) B = 2
4

2 p(i)
i=0

1 p(3) + p(4) , 2 9 B = . 49 =

4 Time congestion E: The time congestion is dened as the proportion of time the system is blocked, or the proportion of time there is no idle capacity. A feasible answer is therefore: E = p(4) = 5 . 49

Other answers as E = B was also accepted at exam if reasonable arguments were given.
Updated: 2005-02-23

1 Technical University of Denmark COM Center, Networks Teletrac Engineering & Network Planning Course 34 340

Solution to exercise 8.10 Question 1:

(exam 2001)

With the given arrival rates we get the following state transition diagram (the closed loop in state 2 is usually excluded): We notice that (3) is also given, but it has no inuence upon the state transition diagram

because it is zero: (3) = 0. Question 2: Denoting the non-normalized state probabilities by qi and the normalized state probabilities by pi we nd:

q0 = 1 q1 = q2 =
3 4 1 4

p0 = p1 = p2 =

4 8 3 8 1 8

, , ,

Sum = 2

Sum = 1 .

The time congestion is the proportion of time all channels are busy: E = p2 = Question 3: The number of customers arriving when the system is in a given state i is both proportional with the state probability pi and the arrival rate i. Per time unit so many customers arrive in the dierent states: 1 . 8

State 0: State 1: State 2: Total:

3 4 2 3 1 2

p0 = p1 = p2 = =

6 16 4 16 1 16 11 16

, , . .

The above is number of calls. Thus we get the following call-average arrival state probabilities: 0 = 6 , 11 1 = 4 , 11 1 . 11 2 = 1 . 11

The call congestion is the proportion of all call attempts blocked: B = 2 = Question 4: The oered trac is dened as the trac carried in a system without blocking. If we have three channels, then no calls are blocked ((3) = 0). We get the following state transition diagram:

Extending the results in question 2 with one state more, we nd:

q0 = 1 q1 = q2 = q3 =
3 4 1 4 1 24

p0 = p1 = p2 = p3 =

24 49 18 49 6 49 1 49

, , , ,

Sum = 49 The oered trac then becomes: 24


3

Sum = 1 . i p(i) =
i=0

A=

33 = 0.6735 . 49

3 The carried trac is obtained from the state probabilities in question 2:


2

Y =
i=0

i p(i) =

5 = 0.6250 . 8

Thus the trac congestion becomes: C=


33 49

33 49

5 8

19 = 0.0720 . 264

Question 5: In the state where both channels are occupied, the arrival rate is 2 = 1 and the departure 2 rate is 2 = 2. Thus the probability that the next event is an arrival becomes (4.7): 2 p(arrival) = 2 + 2 =
1 2 1 2

+2

1 . 5 The call attempt is blocked and the state does not change. As the process is without memory, the number of blocked calls during a busy period becomes geometric distributed: = p(i) = 1 5
i

1 5

i = 0, 1, . . . .
1 5

This is a geometric distribution starting in i = 0. The mean value is m1 = Question 6:

(Tab. 6.1).

We proceed in the same way as in question 3. Per time unit the number of customers departing from the dierent states observe the following state when they look back: State 0: State 1: State 2: 1 p1 = 2 p2 =
1 2 6 11 4 11 1 11

, , .

p2 =

Notice that we have two types of calls departing from state 2: served calls and blocked calls. The customers who see state 2 are the blocked customers. Thus we get the following call-average departure state probabilities: 6 4 1 0 = , 1 = , 2 = . 11 11 11

4 This is the same as in question 3, because the process is reversible. The customers on the average see the same state, when they arrive, as they see when they depart.
Updated: 2005-02-24

1 Technical University of Denmark COM Center, Networks Teletrac Engineering & Network Planning Course 34 340

Solution to exercise 12.17 Question 1: The oered trac is:

(Exam 1995)

A= Question 2:

= 1 erlang.

The state transition diagram becomes as follows:


1 0 1 1 2 1 2 2 1 3 2 1 4 2 1 5 2 1 6

If we denote the relative (unnormalised) state probabilities by q(i) and the normalised by p(i), then we have: 32 q(0) = 1 p(0) = 95 q(1) = q(2) = q(3) = q(4) = q(5) = q(6) = 1 1 2 1 4 1 8 1 16 p(1) = p(2) = p(3) = p(4) = p(5) = 32 95 16 95 8 95 4 95 2 95

1 1 p(6) = 32 95 Due to the PASTA property call averages are equal to time averages. The probability that a customer is served immediately is equal to the probability that the system is in state 0 or 1 at the time of arrival: P {served immediately} = p(0) + p(1) = 64 . 95

The probability that a customer is served after delay is equal to the probability that the

2 system is in state 2, 3, 4, or 5 at the time of arrival: P {served immediately} = 30 . 95

The probability that a customer experiences time congestion is then (due to the PASTA property, time congestion equals call congestion and trac congestion): P {blocking} = p(6) = 1 . 95

Question 3: The state transition diagram is:


1 1 1

03
1 1/4 1 3/4 1/3

13
1 1 2/3 1/2

23
1 1 1/2 1

33
1

02
1 2/5 1 3/5 1/2

12
1 1 1/2 2/3

22
1 1 1/3 1

32
1

01
1 1/2 1 1/2 3/5

11
1 1 2/5 3/4

21
1 1 1/4 1

31
1

00
1

10
1

20
1

30

Question 4: For all squares we have that the ow clockwise equals the ow anticlockwise. For example, for the middle square we get: 1 2 1 2 11= 11 2 3 2 3 Question 5: Because the state transition diagram is reversible we can express all state probabilities relatively to state (0, 0) (10.15), and we nd:

3 3 2 1 0 1/20 1/20 1/20 1/20 1/5 1/2 1 0 3/20 1/10 1/20 3/10 3/20 1/20 1/2 1 1/5 2 1/20 3

Multiplying all values by 20 yields: 3 2 1 0 1 4 10 20 0 sum: 35 1 3 6 10 1 1 2 3 4 2 1 1 1 1 3

20 10 4 = 69

Dividing all terms by 69 gives us the absolute state probabilities: 3 2 1 0 1/69 4/69 10/69 1/69 3/69 6/69 1/69 1/69 2/69 1/69 3/69 1/69

20/69 10/69 4/69 1/69 0 1 2 3

Question 6: Customers are only lost in state (3,3). The probability that a customer is lost thus becomes: P {blocking} = which is larger than 1/95 obtained in question 1. 1 , 69

4 In this system it may happen that customers are waiting in one subsystem at a time when the other subsystem is empty. Therefore, customers are not served as fast as in the full accessible system. A customer spending longer time in the system, may result in blocking of new customers, and thus the total blocking becomes larger.
Revised 2005-03-06

1 Technical University of Denmark COM Center, Networks Teletrac Engineering & Network Planning Course 34 340

Solution to Exercise 13.7 Question 1: The average service time s = m1 is (4.25): s= The oered trac the becomes: A= Question 2: (1 p) p + a b . p (1 p) + . a b

 c p

 c E

03 '  1 a T 4 q b

13 '  T
3 4

b
 c E

 c p

 c p E

02 '  1 a T 3 q b

12 '  2 a T 4
2 3

22 '  T
2 4

b
 c E

 c p

 c p E

 c p E

01 '  1 a T 2 q b

11 '  2 a T 3
1 2

21 '  3 a T 4
1 3

31 '  T
1 4

 c p

 c p E

 c p E

 c E

00 ' 

10 ' 

20 ' 

30 ' 

2 Note: The capacity of the server is constant independent of the state. But the service rate (the intensity at which the customers leave the server) depends on the mix of customers. If most customers in the system has a short service time, then service rate will be high, whereas if most customers have a long service time, then the service rate will be low. Question 3: For the four states considered we have:

Flow clockwise: Flow counter clockwise:

2 2 p b a , 4 3 2 2 p q a b . 4 3

We notice that they are equal. Thus the process is reversible. The same is valid for the other squares, and we may express all states by state p(0, 0) (10.15). Question 4: We nd (q = 1 p): p(0, 1) = (1 p) p(0, 0) , b p p(0, 0) , a

p(1, 0) =

and thus:

p(1) = p(0, 1) + p(1, 0) = p(0, 0) p(1) = p(0) A p 1p + a b q.e.d. ,

where p(0) = p(0, 0) and A is obtained in Question 1.

3 In a similar way we nd: p(0, 2) = (1 p) (1 p) p(0, 0) , b b (1 p) p p(0, 0) , a 1 b 2 p p p(0, 0) . a a

p(1, 1) =

p(2, 0) =

p(2) = p(0, 2) + p(1, 1) + p(2, 0) = p(0, 0)


2

1p b

2p (1 p) + + a b
2

p a

p(2) = p(0, 0) p(2) = p(0) A2

p 1p + a b q.e.d.

1 Technical University of Denmark COM Center, Networks Teletrac Engineering & Network Planning Course 34 340

Solution to exercise 13.4: The service time is Erlang-2 distributed with intensity 2 in each phase:
E

The mean service time becomes (4.11): s= Question 1: The oered trac is equal to the average number of call attempts per mean service time: A = / . In the following we must of course require that < , because we consider a pure delay system in statistical equilibrium. Question 2: In state 0 the state of the system is uniquely dened. In all other states the server may either be in the rst phase a or in the second phase b. The state transition diagram becomes as follows:


1 1 1 + = . 2 2

     t t u e u e u e t 2 e 2 2 e 2 2 2 t e 2 e e e  e  e  e      E 2b E E ib E(i+1)b E 1b    

E 1a



E 2a



E ia



E(i+1)a



Question 3: The form factor of the Erlang-2 distribution is (4.13): 1 3 =1+ = . k 2

2 The mean waiting time W for all calls is obtained by using Pollaczek-Khintchines formula (13.2): As = 2 (1 A) 1 2 1 3 , 2

W =

W =

3 , 4 ( )

(notice < ) .

For calls experiencing waiting time > 0, the mean waiting time becomes (13.44): w= w= W W = , D A 3 1 . 4

The probability D of delay in M/G/1 is equal to A. Question 4: The system is either idle (a) or it is busy (b). a) The probability that the system is idle is (1 A), and the mean value of the time interval considered is then 0. b) The probability that the system is busy is A, and as we consider an arbitrary time instant, the mean value of the remaining service time is given by (3.25): s m1,v = 2 = 3 1 . 4

This can be seen directly because there is equal probability of being in any of the two phases, which each are exponentially distributed with intensity 2: 1 2 In summary, we thus have: V V = (1 A) 0 + A m1,v , = 3 . 4 2 1 1 + 2 2 + 1 1 3 1 = . 2 2 4

3 In the general case we apply formula (13.27), where the second moment m2 is given by (4.14): 3! V = m2 = 2 2 1! Question 5: First part of this question is excluded in year 2005. This question corresponds to Sec. 6.4.3 and it is most easy to answer the question by using the theory for a stochastic sum. The Erlang-2 distribution considered has the Laplacetransform (??): (s) = 2 s+2
2

1 2

(1)

The generating function we looks for thus becomes: Z = ((1 s)) = 2 (1 s) + 2


2

The mean value and the variance becomes ((??) & (??)): a =
2 a =

1 = A,

(which was expected).

2 + 2 x , 1 1 + 2 2 , 2 1 A2 . 2

2 a =

2 a = A +

In an interval of xed length we have a Poisson distributed number of calls with equal mean value and variance. When the observation interval also is a stochastic variable, then we get the same mean value, but a much larger variance. Direct derivation of the distribution function The number of calls arriving according to a Poisson process () during an exponentially distributed time interval (2) is geometrically distributed with the parameter: = 2 +2

4 and starting in class zero (See text to Tab. 6.1): p(i) = (1 )i , Mean value and variance are: m1 = 2 = 1 1= , 2 1 ( + 2 ) = 2 (2 )2 1 2 1 A2 + A 2 . i = 0, 1, 2, . . . .

The geometric distribution has the generating function:

g(s) =
i=0

(1 )i si

1 s (1 ) 2 . (1 s) + 2

The distribution is composed of a sum of two geometric distributions, and we have the same mean value, variance, and generating function as derived earlier The distribution function is a Pascal2 distribution (Tab. 6.1) starting in class i = 0: p(i | k) = i+k1 k1 k (1 )i , i = 0, 1, 2, . . . ,

where k = 2, and is derived above: p(i | 2) = (i + 1) (2 )k i , ( + 2 )k+i i = 0, 1, 2, . . . .

Using terminology from statistics we have a series of Binomial trials. The Pascaldistribution is a Negative Binomialdistribution, and in this case: NB 2, 2 + 2 ,

5 where the mean value is reduced by 2, as we are counting arrivals. Question 6: We choose the time unit 1/ = 1. As both classes have the same service time distribution we have: A1 = 1 = 0.4 erlang . A2 = 2 = 0.4 erlang . Without priority the mean waiting time is obtained from Pollaczek-Khintchines formula: W = 0.8 1 3 = 3. 2 (1 0.8) 2

a) Mean waiting time for each class in a nonpreemptive queueing system is given by formula (13.30) (with only two classes directly in (13.26) and (13.29)): Wp = V (1 Ap1 )(1 Ap )

V has already been obtained in Quest. 4: V = 3 3 0.8 = 0.6 . = 2 4 4

It can also be obtained from (13.27):


2

=
i=1

i m2i 2 3 3 + 0.2 = 0.6 , 2 2 q.e.d.

= 0.2 we thus nd: W1 =

0.6 V = , 1 (1 A1 ) (1 0.4)

W1 = 1 (time units) . W2 = W1 V = , (1 A1 )(1 A1 A2 ) 0.2

W2 = 5 (time units) . As a control, we have that the conservation law gives the same constant as for the case with no priority: Ai Wi = 0.8 3 = 0.4 1 + 0.4 5 = 2.4 .

6 b) Mean waiting time for each class in a preemptive resume queueing system is given in formula (13.37): p i m2i 2 Ap1 i=1 Wp = sp + 1 Ap1 1 Ap1 1 Ap where sp , mean service time for calls belonging to class p (we have sp = 1). 0.4 3 2 W1 = , (1 0.4) W1 = W2 = W2 = 1 , 2 (time units) .
1 2

0.4 0.6 + 1, (1 0.4)(1 0.8) (1 0.4) 17 , 3 (time units) .

W1 can be controlled by using PollaczekKhintchines formula, when only class 1 exists. The conservation law is not valid for for pre-emptive resume queueing disciplines, if the service time in not exponentially distributed.
Updated: 2004-04-22

1 Technical University of Denmark COM Center, Networks Teletrac Engineering & Network Planning Course 34 340

Solution to exercise 13.21 Question 1: The oered trac becomes:

(exam 2003)

A1 = 0.1 [erlang] , A2 = 0.4 [erlang] , A3 = 0.4 [erlang] , At = 0.9 [erlang] .

Question 2: By using the PollaczekKhintchine formula (13.4) we nd the mean waiting time for all customers as follows: Wi = Ai si i , 2(1 Ai ) 1 0.1 1 1= [time units] , 2(1 0.1) 18 20 0.4 10 2= [time units] , 2(1 0.4) 3 400 0.4 40 10 = [time units] . 2(1 0.4) 3

W1 = W2 = W3 =

Question 3: It is now simpler to use the following version of the Pollaczek-Khintchine formula (13.2): W = where (13.18) & (13.27): V =
i

V , 1A

i m2,i , 2

2 and the second moment of type i is m2,i = i si . V = 0.1 2 0.04 0.01 1 + 2 102 + 10 402 2 2 2

= 0.05 + 4.00 + 80.00 = 84.05 . W = V , 1 At

W = 840.5 [time units] . Question 4 In this question we can look away from type 3. From the above we get V12 = 4.05 and thus for a non-preemptive system (13.26) & (13.28) & (13.29): W1 = W2 = = Question 5 Using the formula (13.37) we get: W3 = = V123 A1 + A2 + s3 (1 A1 A2 )(1 A1 A2 A3 ) 1 (A1 + A2 ) 0.5 84.05 + 40 0.5 0.1 1 0.5 4.05 = 4.5 [time units] , 1 0.1 V {1 A1 } {1 (A1 + A2 )} W1 = 9 [time units] . 1 0.5

= 1721 [time units] .

Updated: 2005-03-16

1 Technical University of Denmark COM Center, Networks Teletrac Engineering & Network Planning Course 34 340

Solution to Exercise 13.22

(exam 2004)

Priority queueing systems are dealt with in Sec. 13.4. Question 1: We have to choose a common time unit and in the following we choose minutes. Landing: The arrival rate is 1 = 1/6 aircrafts per minute, and the mean service time is s1 = 2 minutes. So the oered trac becomes: A1 = 1 s1 = 1 [erlang] 3

Starting: The arrival rate is 2 = 1/3 aircrafts per minute, and the mean service time is s2 = 1.5 minutes. So the oered trac becomes: A2 = 2 s2 = The total oered trac becomes: A = A1 + A2 = 5 6 [erlang] 1 [erlang] 2

We notice that the total oered trac is less than one erlang, so the system is in statistical equilibrium. Question 2: The total arrival rate is obtained by adding the two arrival rates (13.13): = 1 + 2 = = 1 1 + 6 3 1 2

The total mean service time is obtained by weighting the mean values with the relative

2 number of calls of each type (13.14). Considering one minute we nd: s = = s = 1 2 s1 + s2 2 1 2 + 1.5 3 3 5 3 [minutes]

From the total process we nd the total oered trac: A = s= A = 1 5 2 3 q.e.d

5 [erlang] 6

Question 3: To nd the mean waiting time for all customers in a single server system we use PollaczekKhintchines formula (13.4) or (13.2). We rst have to nd the second moment m2 or the form-factor of the total trac process. The second moment is obtained by weighting as for the mean value (13.15) (random variables in parallel). We nd: m2 = = 1 2 m2,1 + m2,2 2 1 6+ 3 3 3

m2 = 4 minutes2 The parameter V becomes: V = m2 2


1 2

= V

= 1

3 We could of course also obtain V by summation over all trac classes (13.18):
2

=
i=1

Vi 1/6 1/3 6+ 3 2 2

= 1 Pollaczek-Khintchines formula (13.4) yields: W = V 1 = 1A 1


5 6

W = 6 [minutes] This is the mean waiting time for all aircraft. If we only consider delayed aircrafts, then the mean waiting time is: W w= = 7.2 A We get of course the same if we use (13.2). The form-factor becomes: = m2 4 36 = = s2 (5/3)2 25

Question 4: We now consider a non-preemptive queueing system where landing aircrafts have priority over starting aircrafts. The formul for this are given in Sec. 13.4.3. For highest priority class the mean waiting time becomes (13.26): W1 = = V 1 A1 1 1 3 2
1 3

W1 = For lowest priority we nd (13.29): W2 =

[minutes]

W1 V = (1 A1 )(1 (A1 + A2 )) 1 (A1 + A2 )

W2 = 9 [minutes]

4 The conservation law (13.23) for this system relates the mean waiting time W without priority to the mean waiting times W1 and W2 with non-preemptive priority discipline: A W = A1 W1 + A2 W2 5 1 3 1 6 = + 9 6 3 2 2 5 = Thus the conservation law is fullled
Updated: 2005-01-04

1 9 + 2 2

q.e.d.

1 Technical University of Denmark COM Center, Networks Teletrac Engineering & Network Planning Course 34 340

Solution to Exercise 9.14

(Exam 2004)

This is the truncated Pascal system dealt with in Sec. 8.7. Question 1: The oered trac is obtained by using the formul for the Engset case by letting S = 4 sources (8.63) and = 1 (8.64). 3 By using (8.8) and (8.9) we get: =

1 = , 3 A = S 1+

1 3 = (4) 1 1 3 A = 2 [erlang] . In a similar way we nd from (??): Z = 1 1+ 1 1 3 2


1 3

Z = We may also use the simple relation (8.21) Z =1 Question 2:

A 2 3 =1 = S 4 2

The state transition diagram becomes (cf. Fig. 8.6):

2
4 3 5 3 6 3

..... ..... ..... ..... ..... ....................... ......... ....... ........... .......... ........... ........... .......... ........... ........... .......... ...... . ........... ......... ...... . ........... ......... . . . ............. ...... ......... ......... ... .................... ... .. ... .. ... ........ ........ ......... .... ... ........ ........ ... .. .. .. .. .. .. .. . ... . .. . .. .. .. .. .. .. .. .. .. .. .. .. .. .. .. . . . .. . . . . . . . . . . . . . . . . . . . . . . . . . . . . . . . . . . . . . . . . . . . . . . . . . . . . . . . . . . . . . . . . . . . . . . . . . . . . . . . . . .. .. . . . . .. .. .. .. .. .. .. .. .. .. .. .. .. .. .. .. .. .. .. ... . .. .. . . ... . .. . ... . .... . ... . .... ....... ...... ...... . . . ....... ........... ....... ........... ........ ...... ....... ... ..... ..... . . . ........... . .. ... ...... ... ... ......... ..... .... . ..... ..... ..... ..... ... .... ...... ...... .... .... .... . ........ ........ ............ .................. .................. ............... ............... ............. ...

By using the cut equations we get the following relative state probabilities:

q(0) = 1 q(1) = q(2) = q(3) = Sum = Question 3:


4 3 10 9 20 27 113 27

p(0) = p(1) = p(2) = p(3) =

27 113 36 113 30 113 20 113

= 0.2389 = 0.3186 = 0.2655 = 0.1770

Sum = 1 = 1.0000

Time congestion The time congestion E is the proportion of time all channels are busy and it is obtained from the state probabilities: 20 E = p(3) = = 0.1770 113 Call congestion The call congestion B is the proportion of call attempts blocked and it is obtained as the ratio between the number of blocked call attempts per time unit and the total number of call attempts per time unit. We nd: B = (S + 3) p(3) (S + 0) p(0) + (S + 1) p(1) + (S + 2) p(2) + (S + 3) p(3)
4 3

27 113

+5 3

7 3 36 113

20 113 +6 3

30 113

+7 3

20 113

B =

35 = 0.2303 152

Alternatively we may use the formul for the Engset case, remembering that S and are

3 negative. For example by using (8.43) we get: B = (S n) E (1 + ) S + (S n) E

20 (4 3) 113 (1 1 ) 3 = 20 4 + (4 3) 113 ( 1 ) 3

35 152

q.e.d.

We may also use the arrival theorem and nd the call congestion as the time congestion in a system with one source less, that is with S = 5 sources. Trac congestion The trac congestion C is the ratio between the blocked trac and the oered trac: C= AY 2Y = A 2

The carried trac Y is obtained from the state probabilities:


3

=
i=0

i p(i) 27 36 30 20 +1 +2 +3 113 113 113 113

= 0 Y =

156 113

2 156 113 C = 2 C = 35 = 0.3097 113

Also in this case we may use formul for the Engset system, for example the simple formula (8.44): C = = = S n E S 4 3 20 4 113 35 152

4 Question 4 Fredericks-Haywards method is presented in Sec. 9.3. A trac with mean value A = 2 erlang and peakedness Z = 1.5 oered to n = 3 channels has the same blocking probability as Erlangs loss system with A/Z = 4/3 erlang oered to n/Z = 2 channels. Thus the call congestion is obtained by using Erlangs B-formula: C = E2 (1.3333). In this case we can easily use the recursion formula (7.29 to nd the numerical value: E0 = 1 E1 = 1 4 = 4 7 1+ 3 1 2+
4 3 4 3

E2 = Thus we nd:

4 3

4 7

4 7

8 29

8 = 0.2759 29 We may also use the table and e.g. use linear interpolation between the values for A = 1.25 and A = 1.50 for n = 2 channels. Then we nd C = 0.2752 which is very close to the above value. C= Question 5 Sanders method is described in Sec. 9.4.2. The variance of the oered trac is: Var = mean peakedness = A Z = 2 1.5 = 3 As the mean value (A = 2) is less than the variance (var = 3) we add one channel with a constant trac 1 erlang. Then the total oered trac is 3 erlang, and the number of channels is 4. The peakedness is now one, as the variance is still three. The lost trac from this system is the same (approximate method) as the trac lost from the original system. Using the table we nd the lost trac: Alost = 3 E4 (3) = 3 0.2061 = 0.6183 As this trac is lost from the original system, the call congestion becomes: C= 0.6183 = 0.3092 2

(The textbook year 2004 is not clear at this point, so also C = 0.2061 will be accepted)

5 Question 6 Using the ERT-method we have a system of 4 + 3 = 7 channels which are oered 5 erlang. The trac lost becomes 5 E7 (5) = 0.6026 This is the trac lost from the (original) system and thus the call congestion of the original system becomes: 0.6021 = 0.3013 C= 2 When we oer 5 erlang to 4 channels, then the overow trac will have the mean value (9.6) m1 = 1.992 and the peakedness (9.7) Z = 1.519, corresponding to a variance Var = 3.038. This is very close to the values in the exercise. The correct values to have an overow trac with mean 2 and peakedness 1.5 are obtained by using a computer program: Ax = 4.876 nx = 3.826 Using these values the ERT-method yields the trac congestion C = 0.3007 In conclusion we obtained the values: BPP Fredericks-Hayward Sanders ERT C = 0.3097 C = 0.2759 C = 0.3093 C = 0.3007

We notice that the values are very similar except for Fredericks-Haywards method, which usually is one of the best. But this is an extreme case with very few channels and very high blocking. We also notice that only the BPP-model works with dierent types of blocking probabilities, and that the trac congestion C is the relevant measure. Historically only the time congestion E and the call congestion B have been considered, and they are quite dierent.
Updated: 2006-02-27

1 Technical University of Denmark COM Center, Networks Teletrac Engineering & Network Planning Course 34 340

Solution to exercise 9.8: We consider the following overow system: =1


j

=1
j

a = 1

Primary group

Overow group

The primary group alternates between two states: idle and busy.
T

State

Busy
E Time

Idle

The arrival process to the overow group is a Poisson process which is ON or OFF. The process is OFF, when the primary group is idle (exponentially distributed time interval with intensity a ), and it is ON, when the primary group is busy. (exponentially distributed with intensity ).

  

ON

Arrival process

= a

= a
OFF

=
 

2 We thus see (cf. Fig. 6.6 & 6.7) that the arrival process to the overow group is an IPP (Interrupted Poisson process) with the following parameters: = a = 1 , = a = 1 , = Question 2: We nd the following state transition diagram:


= 1.

a = 1

0,ON '   =1 T =1 =1

 E

 

 T

1,ON

=1

=1

 c 0,OFF '   =1

 c 

1,OFF

We notice it corresponds to IPP/M/n (Fig. 9.6) for n = 1. We get the same state transition diagram if we either a. constructed a two-dimensional state transition diagram for primary and secondary group (Fig. 9.3), or b. keep record of which trunks are occupied. Question 3: By using the node balance equations it is easy to show that the given solution is correct. We nd e.g. for state (0,ON ): Flow out Flow in I.e.: Flow out = = = (1 + 1) p(0, ON) = 6 , 10 6 , 10

1 p(1, ON) + 1 p(0, OFF) = Flow in q.e.d.

3 (The above answer is sucient). We see that: p(ON) = p(0, ON) + p(1, ON) = and: 1 , 2

1 , 2 as the primary group carries 0.5 erlang, i.e. is busy half the time. p(OFF) = p(0, OFF) + p(1, OFF) = Question 4: The time congestion E is per denition equal to the proportion of time the system is busy (all channels occupied). The call congestion B is per denition equal to the proportion of call attempts blocked. (a) The overow system alone: Es = p(1, ON) + p(1, OFF) , 3 . 10 On the average the following number of call attempts arrives to the overow group per time unit: a p(0, ON) + a p(1, ON) Es = On the average the number of call attempts lost per time unit is: a p(1, ON) . Therefore, the call congestion becomes: Bs = a p(1, ON) , a p(0, ON) + a p(1, ON)

2 . 5 We notice that for the overow group the call congestion is greater than the time congestion, because the calls arrive in bursts. Bs = (b) The total system: Both channels are busy in state (1,ON ). Therefore, the time congestion is: Et = p(1, ON) , Et = 2 . 10

4 Call attempts are only blocked in state (1,ON ). Therefore, the call congestion becomes: Bt = a p(1, ON) , a {p(0, OFF) + p(0, ON) + p(1, OFF) + p(1, ON)} Bt = 2 = Et . 10

For a system with n = 2 channels and an oered trac A = 1 erlang, we get from Erlangs Bformula: En (A) = E2 (1) =
A2 2! A2 2!

1+A+
1 2

1+1+ 2 , 10

1 2

E2 (1) = which is equal to the result for the whole system. Question 5: (not exam question)

The trac congestion for the total system is given by Erlangs B-formula. The trac congestion for the overow system alone becomes: The oered trac to the overow system is 0.5 erlang, as the IPP-process is ON half the time (Quest. 3). The carried trac is: Y = p(1, ON ) + p(1, OFF ) = The trac congestion thus becomes: C=
1 2

3 . 10

1 2

3 10

2 , 5

which is equal to the call congestion, because the IPP-process is a renewal arrival process.
Updated: 2004-03-07

1 Technical University of Denmark COM Center, Networks Teletrac Engineering & Network Planning Course 34 340

Solution to Exercise 10.7: Question 1: The oered trac is dened as the trac carried when there is no blocking. The trac carried during a time interval is the average number of busy channels. For Poisson arrival processes (in general renewal arrival proceses) we thus have: Ai = i si di , where si is the average holding time and di is the number of channels occupied by a call (slotsize). We nd: A1 = 1 s1 d1 = 2 1 1 A1 = 2 erlang A2 = 2 s2 d2 = 1 A2 = 1 erlang Question 2:
  2E 04 ' 14   1 T T

1 2 2

    c 2 c 2 2E E E 02 ' 12 ' 22 ' 32     1 2 3 T T T T

      c 2 c 2 c 2 c 2 2E E E E E 00 ' 10 ' 20 ' 30 ' 40 ' 50      

Question 3: Flow clockwise must be equal to ow counterclockwise in all four squares {(i, j) = (1, 2), (1, 4), (2, 2), (3, 2)} :

2
  T  

1 i

i-1, j '

 E  T

 

i, j

j 1 i

c   

i-1, j-2 ' 

c   E 

i, j-2

Clockwise: Counterclockwise:

2 1 j i 1 2 i j q.e.d.

Therefore, we can express all state probabilities by p(0, 0). The relative values become, using the same positioning as in the state transition diagram: 1 8 1 2 1 or 15 60 120 30 120 240 120 240 80 160 80 32 2 8 1 2 1 2 2 3 4 3 2 3 4 15

The total sum of all terms in the latter array is 1297, and the absolute state probabilities are obtained by dividing all terms by 1297. Time congestion equals call and trac congestion as all call intensitites are independent of the states (PASTAproperty): E1 = p(1, 4) + p(3, 2) + p(5, 0) E1 = 142 1297

B1 = C1 = E1 = 0, 1095

3 E2 = p(0, 4) + p(1, 4) + p(2, 2) + p(3, 2) + p(4, 0) + p(5, 0) E2 = 357 1297

B2 = C2 = E2 = 0, 2753 Question 4: For the global state probabilities we nd: p(0) = p(0, 0) = p(1) = p(1, 0) = 120 1297 240 1297 300 1297 280 1297 215 1297 142 1297

p(2) = p(2, 0) + p(0, 2) = p(3) = p(3, 0) + p(1, 2) =

p(4) = p(4, 0) + p(2, 2) + p(0, 4) = p(5) = p(5, 0) + p(3, 2) + p(1, 4) = We shall show that the following recursion formula is valid:
2

j p(j) =
i=1

i di p(j di) i

1 2 d1 p(j 1) + d2 p(j 2) 1 2

j p(j) = 2 p(j 1) + p(j 2) We do this by showing it is valid for all possible values of j (notice p(x) = 0 for x < 0 ). j=1: p(1) = 2 p(0) = 240 1297 q.e.d. 600 1297 q.e.d.

j = 2 : 2 p(2) = 2 p(1) + p(0) = p(2) = 300 1297

4 j = 3 : 3 p(3) = 2 p(2) + p(1) = p(3) = 280 1297 q.e.d. 860 1297 840 1297

j = 4 : 4 p(4) = 2 p(3) + p(2) = p(4) = 215 1297 q.e.d.

j = 5 : 5 p(5) = 2 p(4) + p(3) = p(5) =

710 1297

142 q.e.d. 1297 We have thus shown that the recursion formula is valid for the system considered. The recursion formula is usually called Kaufman & Roberts algorithm (Sec. 10.5). Additional remark The global state probabilities above could of course be obtained by the convolution algorithm:

State Stream 1 0 1 2 3 4 5 1 2 2
4 3 2 3 4 15

Stream 2 Convolved Normalised 1 0


1 2

1 2
5 2 7 3 43 24 71 60

120 1297 240 1297 300 1297 280 1297 215 1297 142 1297

0
1 8

Updated: 2005-04-14

1 Technical University of Denmark COM Center, Networks Teletrac Engineering & Network Planning Course 34 340

Solution to exercise 12.16 (Exam 1993) Question 1: We split a Poisson process into two subprocesses and nd: 1 = p1 = 3 1 = 1 2 = p2 = 3 2 = 2 Question 2:
  T

1 3

2 3

04

 E c 1 

03 ' 13   1 T T 4 2 2 2
 E c 1 E c 2  

02 ' 12 ' 22    1 1 T T T 6 2 4 2 2 2
 E c 2 E c 1 E c 3   

01 ' 11 ' 21 ' 31     1 1 1 T T T T 8 2 6 2 4 2 2 2


 E c 3 E c 2 E c 1 E c 4    

00 ' 10 ' 20 ' 30 ' 40      1 1 1 1

Question 3: We nd that the ow clockwise is equal to the ow counter clockwise in all squares.

2 It is sucient to show this for one square. In general we have:

c # E

(N ij 1) p1
' !

c # E

i, j + 1
" T

1+ i, j +1 1
" T

' !

(N ij) p2

(N ij 1) p2 (N ij) p1
' !

c # E

c # E

i, j
" T

i + 1, j 1
" T

' !

Clockwise: Counter clockwise :

(S i j) p2 (S i j 1) p1 2 1 (S i j) p1 (S i j 1) p2 1 2

We notice that the two expressions are equal so that there is no circulation in the system, i.e. there is local balance. Question 4: As we have local balance all states can be expressed by state (0,0). When choosing q(0, 0) = 1 the relative values become:

3 24 48 60 64 65 261 24 24 12 4 1 65 24 24 12 4 64 24 24 12 60 24 24 48 24 24

The state probabilities become:


24 261 48 261 60 261 64 261 65 261 24 261 24 261 12 261 4 261 1 261 65 261 24 261 24 261 12 261 4 261 64 261 24 261 24 261 12 261 60 261 24 261 24 261 48 261 24 261 24 261

Question 5: From the marginal state probabilities we nd: A1 = A2 = 1 196 65 = 261 261

A1 = A2 = 0.7510 It is noticed that the utilizations of the two servers are equal because of symmetry in the state transition diagram. Question 6: We can nd the state probabilities observed by an arriving user at server one by using the arrival theorem. User observed state probabilities (denoted by (i, j)) are equal to time average state probabilities (denoted by p(i, j)) with one user less.

4
  T  E c 1    T 1 T  E c 1 E c 2  

03

6 12 12 15 16 49

6 6 3 1 16 6 6 3 15 6 6 12 6 6

02 ' 2

01 ' 11 ' 21    1 1 T T T
 E c 2 E c 1 E c 3   

00 ' 10 ' 20 ' 30     1 1 1

Mean queue length + average number of users being served at server one becomes: 0 16 + 1 15 + 2 12 + 3 6 57 Luser = = 49 49 Because each customer has an average service time one, this is equal to the mean waiting time: 57 . Wuser = Luser 1 = 49 Alternative Solution I The above solution is the most obvious one, but there are other ways to solve the question.
       
2 3 1 3

x
c

x
c  

 
1 2

If we apply Littles theorem upon the server, respectively the queue in the upper branch, then we get the following as we have the same rate x in both places.

5 Server: The average number of users at the server is the trac carried by the server, and the average time at the server is the mean service time equal to one. x = = L A = 1 W 1 196 261

Queue: Let Luser be the average queue length at server one and Wuser the unknown mean waiting time. Queue: x = Wuser = Luser Wuser Luser x 261 196 261 196

= Luser =

1 60 + 2 48 + 3 24 261 228 261 261 196 57 , 49 q.e.d.

= Wuser =

Alternative Solution II Step 1: We consider all users (terminals) arriving to server one during one time unit and calculate the total number of users Luser observed at this server by all arriving users (mean values). This will be equal to the total waiting time of all arriving users during one time unit as the service time is one. Step 2: We also calculate the number of arriving customers during one time unit (mean value). From these two steps we nd the mean waiting time Wuser = Luser .

Step 1: By summing over all states and at the same time weighting with the queue length and the

6 number of customers arriving in this state, we nd the following average number of customers, including the customer being served (queue length state probabilities call intensity): Luser = + 0 p(0, 0) 4 + 0 p(0, 1) 3 + 0 p(0, 2) 2 + 0 p(0, 3) 1 + 0 p(0, 4) 0 + 1 p(1, 0) 3 + 1 p(1, 1) 2 + 1 p(1, 2) 1 + 1 p(1, 3) 0 + 2 p(2, 0) 2 + 2 p(2, 1) 1 + 2 p(2, 0) 0 + 3 p(3, 0) 1 + 3 p(3, 1) 0 + 4 p(4, 0) 0 Inserting the numerical values of state probabilities we get: Luser = [ + 0 {1 4 + 4 3 + 12 2 + 24 1 + 24 0} + 1 {4 3 + 12 2 + 24 1 + 24 0} + 2 {12 2 + 24 1 + 24 0} + 3 {24 1 + 24 0} + 4 {24 0} ] /261 Luser = [0 {64} + 1 {60} + 2 {48} + 3 {24} + 4 {0}] /261 = 228 261

Step 2: The mean number of arriving customer to server one during one time unit is obtained from the latest expression for Luser : = {64 + 60 + 48 + 24} /261 = 196/261 Thus we nd: Wuser = 228 228/261 = = 1.1633 , 196/261 196

q.e.d.

The mean waiting time is the same for the service-disciplines FCFS, LCFS anf SIRO. This exercise could also be solved by using the theory of queueing networks in chapter 14.
Updated: 2005-04-18

1 Danmarks Tekniske Universitet COM Center Data & Teletrakteori kursus 34 340

Solution to Exercise 14.7: Question 1:

(Exam 2002)

As given in the textbook (8.8) & (8.9) we have: = = 2, = 2 = , 1+ 3 , 1+ 2 = 4 [erlang] . 3

A = S A = 6 Question 2:

Using the recursion formula for E (8.50) we get: Ex,S () = (S x + 1) Ex1,S () , x + (S x + 1) Ex1,S () E0,S () = 1 .

E0,6 (2) = 1 , 12 1 12 = , 1 + 12 1 13

E1,6 (2) =

10 12 60 13 E2,6 (2) = , 12 = 73 2 + 10 13 8 60 160 73 = 0.6867 . 60 = 233 3 + 8 73

E3,6 (2) =

We may also use the corresponding recursive formula for the inverse blocking probability 1 (8.51) given in the textbook: Ix,S () = Ec,S () x Ix,S () = 1 + Ix1,S () , I0,S () = 1 . (S x + 1)

2 Question 3: According to the description in the exercise we get the following state probabilities:

State 0 1 2 3 4 5 6

Node 1 1 1
2 1 2! 3 1 3! 4 1 4! 5 1 5! 6 1 6!

Node 2 1 2
2 2 2! 3 2 3!

0 0 0

Question 4: By convolution we get for a network with 6 customers the following contributions: C =
6 1 4 2 3 3 5 1 + 1 2 + 1 2 + 1 2 6! 5! 4! 2! 3! 3!

6 1 = 6!

65 6 2 + 1+ 1 1 12

2 1

654 + 123

2 1

6 1 {q(0) + q(1) + q(2) + q(3)} 6! 6 1 Q, 6!

= where

c 2 = 1 2

c 1 = 1 2

and Q = q(0) + q(1) + q(2) + q(3) .

Term 1 corresponds to 6 customers in the rst node and 0 customers in the second node. Term 2 corresponds to 5 customers in the rst node and 1 customers in the second node.

3 Term 3 corresponds to 4 customers in the rst node and 2 customers in the second node. Term 4 corresponds to 3 customers in the rst node and 3 customers in the second node. We can at most have 3 customers in the second node. We notice that the normalised state probabilities correspond to an Engset system (8.25) with S = 6 customers and n = 3 channels: p(i) = q(i) , Q 6 p(i) = i
3

1 2 6 j 1 2

j=0

The oered trac per idle source, usually called , is: = 1 . 2

Set up the state transition diagram for this system!


Updated: 2004-05-04

1 Technical University of Denmark COM Center, Networks Teletrac Engineering & Network Planning Course 34 340

Solution to exercise 14.5


(exam 1999)

We consider a closed queueing network with two nodes as shown in the gure. Question 1: Both nodes are singleserver queueing systems, where the server works, whenever there are customers in the node considered. We dene the state i of the system as the number of customers in node one, and we get the following state transition diagram:
.... ............. . .............. ................ ..... ....... .. . ... .. .. .. . ..... ... .... ........ .... .... ... .. . .. ...... .. . .... .. .. ............ ............. .. . . .. .. . .. . . .. . .. . .. . . . . . . . . . . . . . . . . . . . . . . . . . . . . . . . . . . . . . . . . . . . . . . . . . . . . . .. . .. .. .. . . . . . . .. .. .... ...... ...... . ...... ... ... . .... .... ... .. .. ... . .... .. .. ... .. ....... .... .. .. . . .. ...... ..... .. . .. . . ..... ....... .............. ............. ............. ...

2 1

2 1



.... .... .... .......... .......... ................ ..... ........ ..... ........ ..... ........ .... ... .... ... ... .. ... ........ .... .... ........ .... .... ... ... ......... .... ... ... ......... .... ... .... ... . .. . . ... .. .. . .. .. .. .. .. .. .. .. .. .. . .. . .. .. .. .. . . . . . . . . . . . . . . . . . . . . . . . . . . . . . . . . . . . . . . . . . . . . . . .. .. .. .. . .. . . . . .. .. ... ... ... . ... . ... ... . .. . .. .... . .. ... .. .... . .. ...... .. . .. ... .. ..... .. ..... .. ..... .. ..... .. . .. ....... ..... .. ...... ..... .. ... . ..... ....... .............. .............. .............. .............. .............. ...

2 1

2 1

2 1

i,1



.... ................ ..... ....... . ... .... ........ ... ... ...... ... .... .. .. .. .. .. .. .. . . . .. . . . . . . . . . . . . . . . . . . . . . . . . . . . . . .. . . . .. ... ..... .... ... ............ .. . . . .... .. .. ... . .... ... ... ..... ..... ............ ...

2 1

Question 2: If we denote the relative state probabilities by q(i) and the absolute state probabilities by p(i), then we nd by introducing c = 2 /1 the results shown in the following table. The average number of customers in the two nodes becomes:
S

L1 =
i=0 S

i p(i) ,

L2 =
i=0

(S i) p(i) .

Of course we have the sum L1 + L2 = S.

2 State Probabilities Relative q(0) = 1 q(1) = c q(2) = c2 q(i) = c q(S) = cS 1 cS+1 Total = 1c Question 3: Node one transmits customers to node two with rate (intensity) 1 , except when the system is in state 0. Correspondingly, node 2 transmits customers to node one with rate 2 , except when it is in state S. Therefore, we nd the circulation rate: = 1 {1 p(0)} = 2 {1 p(S)} . This gives us a relation between p(0) and p(S), which of course is in agreement with the above relation. The cycle time tc is obtained by applying Littles law L = W , which here has the form: S = tc , tc = Question 4: Using Littles law we nd: = L1 L2 = , W1 W2 W1 L1 = , W2 L2 S .
i

Absolute 1c 1 cS+1 (1 c) c p(1) = 1 cS+1 (1 c) c2 p(2) = 1 cS+1 p(0) = (1 c) ci p(i) = 1 cS+1 p(3) = (1 c) cS 1 cS+1

Total = 1

3 where L1 and L2 are obtained in question 2. Question 5: With the given parameters, the above state transition diagram becomes as follows:

       
1 2

1 2

1 2

The state probabilities become, as c =

2 1 = : 1 2 8 , 15 4 , 15 2 , 15 1 , 15

p(0) = p(1) = p(2) = p(3) =

and thus: 11 , 15 34 . 15

L1 = L2 =

This result can be applied for control in the following. Convolution algorithm for a closed queueing network: (Sec. 14.4.1): If we choose the relative load of node one as: 1 = 1, then the relative load of node two is equal to 2 = 2.

4 State i 0 1 2 3 Node 1 q1 (i) 1 1 1 1 Node 2 q2 (i) 1 2 4 8 Convolution q1 q2 11 12+11 14+12+11 18+14+12+11 = = = = 1 3 7 15

We know there are three customers in the system, and therefore only the last term in the convolution is of interest: {8 , 4 , 2 , 1} , which by normalization becomes: 8 4 2 1 , , , 15 15 15 15 which is in agreement with the previous result. The average number of customers in the two nodes also become equal to the results derived above: Question 6: MVA-algorithm for a closed queueing network: Sec. 14.4.2): Applying the algorithm we get: Customer 1: As the customer is alone in the system, the sojourn (waiting + service) time in the two nodes become: {W1 (1) = 1 , W2 (1) = 2} . Using Littles law we get the average number of customers in the two nodes: {L1 (1) = 1 , L2 (1) = 2} . As the total number of customers is one, we nd = L1 (1) =
1 3

and: .

1 2 , L2 (1) = 3 3

Customer 2: The second customer has to stay in a node during his own service time and may furthermore

5 have to wait, if the rst customer occupy the server. The total sojourn time in the two nodes thus becomes: W1 (2) = 1+ 1 3 1 = 4 , W2 (2) = 3 1+ 2 3 2= 10 3 .

Using Littles law we nd the average number of customers in the two nodes: L1 = 4 10 , L2 = 3 3
3 7

As the total number of customers is two, we nd = L1 (2) =

and: .

4 10 , L2 (2) = 7 7

Customer 3: The third customer has to stay in a node during his own service time and may have to wait, when the two rst customers occupy the server. The sojourn time in the two nodes therefore becomes: 4 10 11 34 W1 (3) = 1 + 1= , W2 (3) = 1 + 2= . 7 7 7 7 Using Littles law, we nd the average number of customers in the two nodes: L1 (3) = 11 34 , L2 (3) = 7 7
7 15

As the total number of customers is three, we get = L1 (3) =

and ,

11 34 , L2 (3) = 15 15

which is in agreement with the results found above.


updated: 2002-04-22

1 Technical University of Denmark COM Center, Networks Teletrac Engineering & Network Planning Course 34 340

Solution to Exercise 8.3 The service time is Erlang-2 distributed with intensity (rate) 2 in each phase. Therefore, the mean service time becomes: 1 1 1 + = s= 2 2 Question 1: The oered trac is equal to the average number of call attempts per mean service time: A = / Question 2: The given states dene the state transition diagram in a unique way:
 B   B i 2 c   B    c r r rr 2 2 b c   r rr 4 r bb 

aa

ab

Question 3: As the truncated Poisson distribution is valid, we have the following state probabilities: 1 p(0) = 1 + A + A2 /2 p(1) = p(a) + p(b) = A p(0) p(2) = p(aa) + p(ab) + p(bb) = A2 p(0) 2

2 For node 0 we have the following node balance equation: 2 p(b) = p(0) p(b) = and thus from the equation for p(1): p(a) = A p(0) 2 A p(0) 2

The node balance equation for state aa is a follows: 4 p(aa) = p(a) p(aa) = In a similar way we get for state ab: 4 p(aa) + p(b) = (2 + 2) p(ab) p(ab) = and for state bb: 4 p(bb) = 2 p(ab) p(bb) = A2 p(0) 8 A2 p(0) 4 A2 p(0) 8

We notice that the two phases are symmetric. This is not obvious from the beginning. The state ab is in fact composed of the two micro-states ab and ba. Therefore, p(ab) is twice as big as p(aa) and p(bb). Summarising, we have:: p(0) = 1 1+A+ A/2 1+A+
A2 2

p(a) = p(b) =

A2 2

p(aa) = p(bb) =

A2 /8 1 p(ab) = 2 1+A+

A2 2

3 There will be (n + 1) dierent (macro-)states with all n channels busy. The macro-state (i, n i) = i channels in phase a and (n i) servers in phase b is made up from n i dierent micro-states The probability of nding the system in state (i, n i) thus becomes: P (i, n i) = n i
n

j=0

n j

p(n)

n i = p(n) 2n where p(n) is the truncated Poisson distribution(7.9). In a similar way we can derive the state probabilities for all Question 4: Starting in state aa: aa
E
n+2 2

states.

ab
E

1/2 E
c

bb 4
c

1/2
E

Starting in state ab. This is a subset of the rst case: ab


E

1/2 E
c

bb 4
c

1/2
E

4 Question 5: The Laplace-transform of the two distributions obtained in Question 4 can immediately be written down from the phase diagrams: Laa (s) = 1 2 1 2 4 s + 4 4 s + 4
3

+
2

1 2 1 2

4 s + 4 4 s + 4

Lab (s) =

The number of observations per time unit when starting in state aa (see the state transition diagram in Question 2): p(a). The number of observations per time unit when starting in state ab: p(b) . As we have p(a) = p(b), the average number of observations of the two distributions is the same, and the Laplace transform of the distribution of the duration of the periods when both channels are busy becomes: L(s) = L(s) = 1 {Laa (s) + Lab (s)} 2 1 4 4 s + 4 + 1 2 4 s + 4
2

1 4

4 s + 4

In fact this corresponds to the following Coxdistribution, which is a weighted sum of the two Coxdistribution in Question 4. But we can no longer identify the individual states as they are mixed together. 3/4E 1/4
c E

1/3 E
c

4
c

2/3
E

From the Laplace-transform it is easy to write down the distribution functions for the cases we consider here.

5 Question 6: The number of blocked call attempts during a busy period has the following mean value and variance (3.46) & (3.48): m1,n = m1,x
2 2 n = 2 x + m1,x

2 where m1,x and x are the mean value and variance of the distribution in Question 5. They can be derived by dierentiating L(s). As we consider Erlang-k distributions in parallel they can also be obtained directly:

m1,x = =

1 4 1 2

1 4

1 2

2 4

1 4

3 4

Second moment of an Erlang-k distribution (which is not normalised) is: 1 k2 + k 2 From this we get: 1 4 1 4
2

m2 =

1 1 1 2 + 6 + 12 4 2 4 13 2

2 x = m2 m2 1,x

5 = 2

1 4

This is of course in agreement with (4.32) and (4.33), as q1 = 1, q2 = 3/4, q3 = 1/4, a0 = 1, a1 = 3/4, a2 = 1/3, a3 = 0 and j = 4 for all j.

6 Therefore we have: m1,n = mn = A 2 5 2 1 4


2

1 2

2 n = 2

1 2

2 n =

5 A A2 + 32 2

2 If the busy periods were constant time intervals, then n would only contribute with A/2, due to the Poisson process. Because the busy periods are stochastic time intervals we also get a contribution 5 A2 /32.

If we look for the variance of the number of call attempts blocked during e.g. a busy hour, then we get an additional contribution to the variance because the number of busy periods also is a random variable itself.
Updated: 2004-02-29

1 Technical University of Denmark COM Center, Networks Teletrac Engineering & Network Planning Course 34 340

Solution to exercise 10.1 Background: The model considered can be applied if we want to give two trac streams dierent service levels on the same channel group, i.e. one trac stream higher priority than another trac stream. This principle, which is called trunk reservation, is dierent from the principle class limitation, where it is the total number of calls of a given type, which decides whether a new call attempt of this type can be accepted. One example is incoming (type 1) and outgoing (type 2) trac between a PABX (Private Automatic Branch eXchange) of a company and the public telephone exchange. Another application is for cellular systems, where we want to give priority to hand-over calls over new call attempts (guard channels). Question 1:
 i 

1 + 2

 q i 

1 + 2

 q i 

 q 

Question 2: The conditions for statistical equilibrium will always be fullled (loss system with a nite number of states), and we get by applying the cut equations and expressing all state probabilities by state p(0): 1 + 2 p(1) = p(0) , p(2) = 1 + 2 p(1) 2 (1 + 2 )2 p(0) , 22 1 p(2) 3

p(3) =

(1 + 2 )2 1 = p(0) , 3! 3

2 where p(0) is obtained from the normalisation condition: p(0) + p(1) + p(2) + p(3) = 1 .

Question 3: As the arrival process is a Poisson process, then we have call congestion B = time congestion E = trac congestion C (PASTAproperty): B1 = E1 = C1 = p(3) , B2 = E2 = C2 = p(2) + p(3) .

Question 4:

 1 '  T

0,2

 E 

1,2

2 0,1

2
c  1 E '  T 2

 c 1 '  T

1,1

 E 

2,1

2 0,0

2 1,0

2,0

c  1 E '   E 

 c 1 ' 

 c 1 E ' 

3,0

We notice that this state transition diagram is not reversible (cf. Sec. 10.2). We have to solve 9 linear equations with 9 unknown to nd the state probabilities. Question 5: In state (i, j) the carried trac if type 1 is i erlang and the carried trac type 2 is j erlang.

3 Therefore, we have: p(i + j = 0) = p(0, 0) , Type 1: = Y01 = 0 , Type 2: = Y02 = 0 . p(i + j = 1) = p(1, 0) + p(0, 1) , Type 1: = Y11 = p(1, 0) , Type 2: = Y12 = p(0, 1) .

p(i + j = 2) = p(2, 0) + p(1, 1) + p(0, 2) , Type 1: = Y21 = 2 p(2, 0) + 1 p(1, 1) , Type 2: = Y22 = 1 p(1, 1) + 2 p(0, 2) . p(i + j = 3) = p(3, 0) + p(2, 1) + p(1, 2) , Type 1: = Y31 = 3 p(3, 0) + 2 p(2, 1) + 1 p(1, 2) , Type 2: = Y32 = 1 p(2, 1) + 2 p(1, 2) . The total carried trac thus becomes: Type 1: Y1 = Y01 + Y11 + Y21 + Y31 , Type 2: Y2 = Y02 + Y12 + Y22 + Y32 . As a control, we of course have (cf. Question 3): Y1 = A1 (1 B1 ) , Y2 = A2 (1 B2 ) . p(i + j = x) corresponds to p(x) in the one-dimensional model (cf. Question 1).
Updated: 2004-03-10

1 Technical University of Denmark COM Center, Networks Teletrac Engineering & Network Planning Course 34 340

Solution to exercise 12.23: Question 1:

(Exam 2002)

We state transition diagram becomes as follows. Note the similarity with an Interrupted Poisson Process (IPP), but also the service process is interrupted, so that the state transition diagram gets a very simple structure:
.......... . ......... . ......... . ......... ........... ............ ............ ............ .. ... ... ... ... .. .. .. .. ... ... ... ... .. .. .. .. .. .. .. .. . . . . .. .. .. .. . . . . . . . . . . . . . . . . . . . . . . . . . . . . . . . . . . . . . . . . . . . . . . . . . . . . . . . . . . . . .. . .. .. . . . . .. .. .. .. . . . . .. . .. .. .. ... .. .. .. .. .. ... ... .. .. ... .. .. . . . . ..... ...... . . . ........... ........... . .. . . . ........... ............ ............ ...... ..... ............ . . . . . . . . . . . . . ... ... ... ... ... ... ... ... . . . . . . . . . . . .. . .. .. .. . . . . . . . . . . . . . . . . . . . . . . . . . . . . . . . . . . . . . . . . . . . . . . . . . . . . . . . . . . . . . . . . . . . . . . . . . . . . . . . . . . . . . . . . . . . . . . . . . . . . . . . . . . . . . . . . . . . . . . . . . . . . . . . . . . . . . . . . . . . . . . . . . . . . . . . . . . . . . . . . . . . . . . . . . . . . . . . . . . . . . . . . . . . . . . . . . . . . . . . . . . . . . . . . . . . . . . . . . . . . . . . . . . . . . . . . . . . . . . . . . . . . . . . . . . . . . . . . . . . . . . . . . . . . . . . . . . . . . . . . . ... ... . . . . .. .. . . . .. ... . . .... ... . ..... ... . . . ...... ... . .. ..... .... . . . .......... . .. .. . . . . . ... .................. .................. .................. ................... ..................... . ............. ..................... . ............. ... ...................... . ............. ........................ . ... . . .. ............ ..... ... .. ..... ............ ..... ...... ..... ............ ..... .... .. ............ ...... ..... ....... .... . . .... .. . .. ...... ....... ..... . ..... .. .. .... .. . ........ ..... ... ... .... . .... .... . ... ... .. . ..... ...... ..... ...... .. . .. ... . . . . .. .. . . . . . . . . . . . . . . . . . . . . . . . . . . . . . . . . . . . . . . . . . . . . . . . . . . . . . . . . . . . . . . . . . . .. . . . . . .. . . . .. . . . .. . .. .. . . . . .. . . ... .. . . . . ....... ...... ....... ...... ..... . ........ ........ .... ... .... ... ... ..... .... ..... ..... ..... .. .... ...... .. .... ...... .. .......... ..... ..... .. ........... ............ . .............................. ........... . .............................. ........... . .............................. ........... . .............................. ...................... ...................... ...................... .......... .......... .................... ......... .........

0, 1

1, 1

2, 1

3, 1

0, 0

1, 0

2, 0

3, 0

Question 2: The state of the server is a two-state Markov process. The server switches independently of the customers between in order and under repair, with mean value 1/, respectively 1/, and we get by considering one cycle: p(able to work) = p(under repair) = p(working) = , + , + p(0, 0) . +

Question 3: The system corresponds to an M/M/1 queueing system which is frozen during periods where it is under repair (no arrivals and no service). So if we look away from these frozen periods, it is an ordinary M/M/1 system. From the state transition diagram we also notice that the state probabilities must be similar to the probabilities of M/M/1 apart from that every state

2 is split up into two states according to the ratio found in Question 2: p(i, 0) =
i

p(0, 0) ,
i

i = 0, 1, 2 . . . ,

+
i

p(i, 1) = =

p(0, 0) ,
i

i = 0, 1, 2 . . . ,

Alternatively, we may express all states by e.g. state p(0, 0) and normalize. Question 4: p(immediate service) = p(0, 0) = (1 A)

, +

p(delay before service) =


i=1

p(i, 0) = A p(i, 1) .
i=0

, +

p(loss)

The sum of the two rst is /( + ), and the third is equal to /( + ) (cf. Question 2). Question 5: Customers are only accepted for service and served when the server is working. So we only have to require < . The parameters and may take all non-negative values. This is also obvious from the state transition diagram and from the state probabilities expressed by state (0, 0). Question 6: When service of a customer starts, next event is either termination of service (rate ) or break down of server (rate ). This corresponds to the rst phase with total rate + . The service terminates with probability /( + ) and we leave by the rst branch downward. The server breaks down with probability /( + ) and we go to the second box, which corresponds to the repair time, which is exponentially distributed with rate .

3 When the server is repaired it resumes service of the interrupted customer. As the exponential service time has no memory, the state is the same as at the start of the rst box, and we repeat the same pattern again. So the nal result is a Cox-distribution with innitely many phases. (To nd the total service time of a customer we may look away from the boxes with (repair time), and then we get a Coxdistribution which is equivalent to a single exponential distribution with rate (Fig. 4.9). So the total service time is independent of the break downs).
Updated: 2003-03-17

1 Technical University of Denmark COM Center, Networks Teletrac Engineering & Network Planning Course 34 340

Solution to exercise 8.12 Question 1:

(exam 2005)

If there is no congestion, then on the average a source is on one time unit and o two time units. So the oered trac per source is = 1/3 (8.8). The total oered trac becomes (8.9): 4 A=S= [erlang]. 3 Question 2: The state probabilities of a single source is (Binomial distribution for a single source = Bernoulli distribution): 2 1 p(0) = , p(1) = . 3 3 By using the relative state probabilities 2 : 1 we get by convolving 4 sources:

State S1 0 1 2 3 4 2 1 0 0 0

S2 2 1 0 0 0

S12 4 4 1 0 0

S3 2 1 0 0 0

S123 8 12 6 1 0

S4 2 1 0 0 0

S1234 16 32 24 8 1

Truncated 16 32 24 8

Normalised
2 10 4 10 3 10 1 10

To the right we have the state probabilities. As a control we may of course use the formul in the textbook (8.24) or a state transition diagram:
#
4 2

# j

3 2

# j

2 2

# j

0 1

1 2

2 3

"! "! "! "! Y Y Y

We see that the above state probabilities fulll the node balance equations and add to one. Therefore, this is the unique solution.

2 Question 3: From the state probabilities we get: E = p(3) = 1 = 0.1000 . 10

Using the arrival theorem, the call congestion is the time congestion with one source less, i.e. three sources. From the table for 3 sources (S123 ) we get for this inhomogeneous system: E= The carried trac is:
3

1 1 = = 0.0370 . 8 + 12 + 6 + 1 27

=
i=0

i p(i) 2 4 3 1 +1 +2 +3 10 10 10 10

= 0 =

13 . 10

As the oered trac is A = 4/3 (Question 1) we get: C = AY = A


4 3

4 3

13 10

C =

1 = 0.0250 . 40

We may of course also use the formul of the textbook, e.g. (8.32): C= Question 4: S n 43 1 1 E = = , S 4 10 40 q.e.d.

State S1234 0 1 2 3 2 4 3 1

S5 1 1 0 0 21 21 + 20 + 20 +

Convolution 41 41 + 40 + 31 31 + 11

S12345 =2 =6 =7 =4
2 19 6 19 7 19 4 19

Normalised = 0.1052 = 0.3158 = 0.3684 = 0.2105

3 We still consider a system with 3 channels. The time congestion for both types of sources (both requiring one channel) becomes: E1 = E2 = Question 5: The arrival theorem tells that the call congestion of a source is equal to the time congestion of the same system without this source. Thus we get from Question 3: 1 = 0.1000 . 10 The call congestion of type 1 is the time congestion of a system with 3 type 1 (S123 ) sources and the type 2 source (S5 ): B2 = State 0 1 2 3 S123 8 12 6 1 S5 1 1 0 0 S1235 8 20 18 7
8 53 20 53 18 53 7 53

4 = 0.2105 19

Normalised = = = = 0.1509 0.3774 0.3396 0.1321

Thus the call congestion of type 1 sources becomes: B1 = Question 6: From the convolution scheme in the table in Question 4 we see that all the terms in the rst column corresponds to zero erlang of type 1 sources, the terms in the second column corresponds to one erlang, the terms in the third column corresponds to 2 erlang, and the term in the last column corresponds to 3 erlang. Thus we get the carried trac, remembering that the normalisation factor is 1/19 and looking away from zero terms: Y1 = {1 (4 1 + 4 1) + 2 (3 1 + 3 1) + 3 (1 1)}/19 Y1 = 23 19
4 3

7 = 0.1321 . 53

As the oered trac is A1 =

we get the following trac congestion for sources of type 1: C1 =


4 3

4 3

23 19

C1 =

7 = 0.0921 76

4 In a similar way we identify the terms contribution to trac of type 2 (all by 1 erlang) as terms on the second diagonal in the table of Question 4: Y2 = {2 1 + 4 1 + 3 1}/19 = 9 19
1 2

C2

A2 Y2 = = A2

1 2

9 19

19 18 19

1 = 0.0526 . 19 This question may also be solved by considering the two-dimensional state transition diagram of two streams: one with the four identical sources (horizontally), and one with the additional source (vertically): C2 =
4 3    2 2 01  11  21    1 2 6 6 6

    ? 4 ? 3 ? 2 2 2 2 -

00  10  20  30     1 2 3
We get the following relative state probabilities as the state transition diagram is reversible (Chapter 10): 9 10 19 2 2 4 4 4 8 3 3 6 1 1

The normalisation constant is 19. From the marginal state probabilities we get the carried trac of each type: Y1 = (0 4 + 1 8 + 2 6 + 3 1) /19 = 23 , 19

Y2 = (0 10 + 1 9) /19 9 , 19 which are the values obtained above. =


Updated: 2006-02-27

1 Technical University of Denmark COM Center, Networks Teletrac Engineering & Network Planning Course 34 340

Solution to exercise 13.23 Question 1:

(exam 2005)

From the formula A = s we get: = A 3 = s 60 [sec] 1 [sec1 ] = 3 [min1 ] 20

Thus the arrival rate is 3 arrivals (call attempts) per minute or 1/20 arrivals per second. Question 2: Using the recurrence formula for Erlang-B (7.29): Ex (A) = we get: E0 = 1 , E1 = 3 31 = = 0.7500 , 1+31 4 = 9 = 0.5294 , 17 A Ex1 (A) , x + A Ex1 (A) E0 (A) = 1 .

E2

3 3 4 = 2+3

3 4

E3 =

9 3 17 9 = 0.3462 , 9 = 26 3 + 3 17 9 3 26 27 = 0.2061 , 9 = 131 4 + 3 26

E4 =

E5

27 3 131 81 = 0.1101 . = 27 = 736 5 + 3 131

2 Thus we have for Erlangs B-formula E1,5 (3) = 81 = 0.1101 . 736

From the relationship (12.9) we obtain Erlang-C from Erlang-B: E2,n (A) = E1,n (A) 1 A {1 E1,n (A)} /n
81 736 3(181/736) 5

405 3680 1965

E2,n (A) = We may also use 12.11:

81 = 0.2362 . 343

1 1 1 = E2,n (A) E1,n (A) E1,n1 (A) = 736 131 81 27 343 81 81 343 q.e.d.

E2,n (A) =

Question 3: The mean waiting time for all customers is (12.15): Wn = E2,n (A) Wn = = 81 60 s = nA 343 5 3

2430 = 7.0845[ sec] 343 81 = 0.1181 [min] . 686

3 Question 4: The non-preemptive model is dealt with in section 13.4.5. We nd the following mean waiting times using (13.33) = (12.15), respectively (13.34): W1 = E2,n (A) = W1 = = W2 = W2 = = 81 343 s n A1

60 51

1215 = 3.5423 [sec] 343 81 = 0.05904 [min] , 1372 n s E2,n (A) 5 n W1 = W1 , = {n A1 } {n (A1 + A2 )} n A1 A2 2 6075 [sec] = 8.8557 [sec] 686 405 = 0.1476 [min] . 2744

Question 5: Using the conservation law (13.23) we nd the constant for the system without priority: C =AW = 3 = For the non-preemptive strategy we get: Cnp = A1 W1 + A2 W2 = 1 6075 1215 +2 343 686 q.e.d. 7290 2430 = [sec] 343 343

243 = 0.3542 min] . 686

7290 = 21.2536 [sec] 343

We thus conrm that the conservation law is valid.

4 In the textbook the conservation law is only used for single-server queues, but it is also valid for multi-server systems, but with dierent formul. In this qyestion we can of course not use the single-server formul. Question 6: For the preemptive-resume discipline the high priority customers can be considered as being alone. so using Erlang-C with n = 5 channels and A = 1 erlang we nd in the same way as above:

E0 = 1 , E1 = 1 11 = = 0.5000 , 1+11 2 = 1 = 0.2000 , 5 1 = 0.0625 , 16

E2

1 1 2 = 2+1 1 1 5 = 3+1

1 2

E3

1 5

E4 =

1 1 16 1 = 0.0154 , 1 = 65 4 + 1 16 1 1 65 1 = 0.0031 , 1 = 326 5 + 1 65

E5 =

E1,5 (1) =

1 = 0.003067 326 1/326 1(1 1/326) 1 5 1 = 0.003831 261

E2,5 (1) =

5 Thus we get (12.15): W1 = E2,n (A) 1 60 s = nA 261 5 1

= =

15 = 0.05747 [sec] 261 1 = 0.0009579 [min] . 1044

Using the conservation law we have: Cpr = 7290 15 = A1 W1 + A2 W2 = 1 + 2 W2 , 343 261 1897545 = 10.5981 [sec] = 0.1766 [min] . 179046

W2 =

Comments: This model may be used for modelling voice/data in GSM/GPRS, where voice has the highest priority. When the blocking/delay probability of voice is low we can consider the oered trac being equal to the carried trac. Remaining capacity is used by data calls waiting in a buer. Furthermore, the mean holding time of data calls can be chosen independently of the mean holding time of voice calls.
Updated: 2005-07-05

Вам также может понравиться